Anesthesiology: A Problem-Based Learning Approach (Anaesthesiology: A Problem-Based Learning Approach) [1 ed.] 0190850698, 9780190850692

Linking existing knowledge to new knowledge by presenting it in the form of a case or a problem is a popular and effecti

192 62 53MB

English Pages 480 [481] Year 2018

Report DMCA / Copyright

DOWNLOAD FILE

Polecaj historie

Anesthesiology: A Problem-Based Learning Approach (Anaesthesiology: A Problem-Based Learning Approach) [1 ed.]
 0190850698, 9780190850692

Citation preview

ANESTHESIOLOGY

ANESTHESIOLOGY A P R O B L E M- ​B A S E D L E A R N I N G A P P R O A C H

EDITED BY

Tracey Straker

P R O F E S S O R O F A N E S T H E S I O L O G Y, DIRECTOR OF GENERAL ANESTHESIA, D I R E C T O R O F A D VA N C E D A I R WAY F E L L O W S H I P A N D R O TAT I O N , D E PA RT M E N T O F A N E S T H E S I O L O G Y, M O N T E F I O R E M E D I C A L C E N T E R , ALBERT EINSTEIN COLLEGE OF MEDICINE, BRONX, NEW YORK

and Shobana Rajan

A S SI STA N T PR O F E S S O R A N D STA F F A N E ST HE SI O LO G I ST, G E N E R A L A N E S T H E S I O L O G Y, A S S O C I AT E P R O G R A M D I R E C T O R , N E U R O A N E S T H E S I A F E L L O W S H I P, A SSISTA N T DIR E CTOR , PACU, CLEVELAND CLINIC, CLEVELAND, OHIO

1

1 Oxford University Press is a department of the University of Oxford. It furthers the University’s objective of excellence in research, scholarship, and education by publishing worldwide. Oxford is a registered trade mark of Oxford University Press in the UK and certain other countries. Published in the United States of America by Oxford University Press 198 Madison Avenue, New York, NY 10016, United States of America. © Oxford University Press 2019 All rights reserved. No part of this publication may be reproduced, stored in a retrieval system, or transmitted, in any form or by any means, without the prior permission in writing of Oxford University Press, or as expressly permitted by law, by license, or under terms agreed with the appropriate reproduction rights organization. Inquiries concerning reproduction outside the scope of the above should be sent to the Rights Department, Oxford University Press, at the address above. You must not circulate this work in any other form and you must impose this same condition on any acquirer. CIP data is on file at the Library of Congress ISBN 978–​0–​19–​085069–​2 This material is not intended to be, and should not be considered, a substitute for medical or other professional advice. Treatment for the conditions described in this material is highly dependent on the individual circumstances. And, while this material is designed to offer accurate information with respect to the subject matter covered and to be current as of the time it was written, research and knowledge about medical and health issues is constantly evolving and dose schedules for medications are being revised continually, with new side effects recognized and accounted for regularly. Readers must therefore always check the product information and clinical procedures with the most up-​to-​date published product information and data sheets provided by the manufacturers and the most recent codes of conduct and safety regulation. The publisher and the authors make no representations or warranties to readers, express or implied, as to the accuracy or completeness of this material. Without limiting the foregoing, the publisher and the authors make no representations or warranties as to the accuracy or efficacy of the drug dosages mentioned in the material. The authors and the publisher do not accept, and expressly disclaim, any responsibility for any liability, loss or risk that may be claimed or incurred as a consequence of the use and/or application of any of the contents of this material. 1 3 5 7 9 8 6 4 2 Printed by LSC Communications, United States of America

D I G I TA L M E D I A AC C O M PA N Y I N G T H E B O O K Individual purchasers of this book are entitled to free personal access to accompanying digital media in the online edition. Please refer to the access token card for instructions on token redemption and access. These online ancillary materials, where available, are noted with iconography throughout the book. Test The corresponding media can be found on Oxford Medicine Online at: http://oxfordmedicine.com/anesthesiologyPBL If you are interested in access to the complete online edition, please consult with your librarian.

v

CONTENTS

Contributors

ix

S E C T I O N I : A I RWAY 1. Management of the Difficult Airway in the MRI Suite Letha Mathews and Jason Lane

3

2. Hip Dislocation in a Patient with Scleroderma Lisa Mouzi Wofford and Yi Deng S E C T I O N I I : C A R D I AC F O R T H E GENERALIST 3. Atrial Fibrillation Christopher Sheerer and Kurt Ruetzler 4. The LVAD Patient Presenting for Noncardiac Surgery Christopher Y. Tanaka and Elizabeth M. Vue

11

6. Perioperative Anesthetic Management of Anterior Mediastinal Mass Alexandra Bastien

53

7. Gastric Ulcer Bleed with a History of Coronary Artery Disease, Hypertension, and Hyperlipidemia Sean P. Kelly and Radhika P. Grandhe

59

8. Percutaneous Coronary Intervention with Concomitant Atrial Fibrillation Joseph W. Woo and Gregory K. Kim 

67

11. TEVAR—​Thoracic Endovascular Surgery—​Cerebral Spinal Fluid Drainage Catheters Oran Kremen and Kumar Vivek S E C T I O N I V: A B D O M I NA L 1 2. Active Pheochromacytoma Vilma Joseph and Chukwuemeka Okafor

S E C T I O N V I I : O RT H O P E D I C S 15. Anesthetic Management for Patients with Severe Cervical Myelopathy Mark Teen, Michael Ting, and Ehab Farag

29

43

10. Lower Extremity Claudication Perin Kothari, Rachel Diehl, and Maged Argalious

SECTION VI: PLASTICS 14. Anesthesia for Mastectomy with Autologous Flap Eman Nada

21

5. Preoperative Chest Pain in a Patient with Colon Obstruction A. Elisabeth Abramowicz and Irim Salik

S E C T I O N I I I : VA S C U L A R 9. Ruptured Abdominal Aortic Aneurysm—​Open Abdominal Aortic Aneurysm Revisited Alexandra Bastien and Edwin Tavarez

S E C T I O N V: G A S T R O I N T E S T I NA L 13. Pulmonary Comorbidities in Endoscopic Retrograde Cholangio-Pancreatography Procedure and Ambulatory Anesthesia Ali R. Abdullah and Lu Zheng

115

125

133

16. A Patient with Morbid Obesity and OSA Presenting for Shoulder Arthroscopy Iyabo Muse and Nii Amon Robertson

139

S E C T I O N V I I I : G E N I TO U R I NA RY 17. Robotic Prostatectomy Terry Ann Chambers

149

18. End-​Stage Renal Disease—​Patient for Kidney Transplant Ferenc Gyulai

155

SECTION IX : BLOOD PRODUCTS

81

19. Massive Transfusion Jay Berger

169

20. Update on Blood Management in Liver Transplant Lovkesh Arora, Sung Kim, and Surangama Sharma

181

21. Acute Intraoperative Blood Loss in a Jehovah’s Witness—​Medical, Ethical, and Legal Issues Myrna Kcomt

89

SECTION X : ALLERGY 22. Anaphylaxis in the OR—​Causes, Recognition, and Treatment Surangama Sharma and Lovkesh Arora

95

SECTION XI: TRANSPLANT 23. Anesthetic Considerations in a Liver Transplant Recipient Presenting for Nontransplant Surgery Roshni Sreedharan and Sandeep Khanna

105

vii

189

199

209

24. Patient with Liver Failure Presenting for Transjugular Intrahepatic Portosystemic Shunt Marina Moguilevitch

215

S E C T I O N X I I : P O S TA N E S T H E S I A CARE UNIT 25. Waking Up My Outpatient, Who Looks to Now Be Seizing: What to Do Next in the PACU? Miguel A. Cruz

225

26. Obstructive Sleep Apnea: Perianesthetic Concerns and Management Scott Chamberlain and Melissa Ehlers

229

27. Rheumatoid Arthritis Patient Presenting for a Total Knee Replacement Perin Kothari and Sree Kolli

237

S E C T I O N X I I I : O B S T ET R I C S 28. Anesthesia in a Pregnant Patient Presenting for Urgent Nonobstetric Surgery 243 Shobana Bharadwaj and Jessica Galey 29. Preeclampsia Problem-​Based Learning Discussion Shamantha Reddy and Yelena Spitzer

249

30. Pregnancy Management in a Parturient with Tetralogy of Fallot Yi Deng and Shobana Murugan

257

S E C T I O N X I V: R E NA L 31. Intraoperative Hyperkalemia M. Angele Theard and Alexandra Bastien

269

S E C T I O N X V: E N D O C R I N E 32. Diabetic Ketoacidosis Management Guensley Delva and Tracey Straker

281

38. Methemoglobinemia Melinda Aquino

335

39. Anesthesia for Functional Endoscopic Sinus Surgery Jagan Devarajan

341

4 0. Anesthetic Management of a Patient with Multiple Laryngeal Papillomatosis Shilpa Rao 41. Anesthetic Management of the Patient with Retropharyngeal Abscess with Emphasis on Perioperative and Airway Management Mazen A. Maktabi S E C T I O N X V I I I : A M B U L ATO RY 42. Anesthesia for Carpal Tunnel Surgery Curtis Choice S E C T I O N X I X : B A R I AT R I C ANESTHESIA 43. Perioperative Care of the Bariatric Surgery Patient John N. Afthinos and Karen E. Gibbs SECTION XX : MISCELLANEOUS 4 4. Anesthetic Considerations in Patients with Automatic Implantable Cardio Defibrillator Presenting for Noncardiac Surgery Sandeep Khanna and Roshni Sreedharan

34. Patient with a Large Goiter Presenting for Thyroid Surgery Cheryl K. Gooden and Walter Ralph

295

S E C T I O N X V I : H E M ATO L O G Y 35. Sickle Cell Disease and Cholecystectomy Thomas Powell and Yi Deng

305

36. Anesthesia in a Patient on Anticoagulant Dabigatran Presenting for Emergency Surgery Indu Kapoor, Charu Mahajan, and Hemanshu Prabhakar

315

S E C T I O N X V I I : E A R , N O S E , A N D T H R O AT 37. Tracheal Stenosis 325 Nichelle Broderick and Tracey Straker

359

369

383

393

45. Anesthetic Considerations in Autonomic Hypereflexia 401 Roshan Patel and Suzette Singh 4 6. NPO Guidelines Glenn Mann and Dustin Liebling

33. Myasthenia Gravis Patient Presenting for Ureteroscopy 287 Theresa J. Barnes, Amanda Moraska Benson, and Ashish K. Khanna

351

47. Severe COPD Patient Presenting for Retinal Detachment Surgery Komal Gandhi and Nidhi Panda

407

415

S E C T I O N X X I : P R AC T I C E M A NAG E M E N T I S S U E S 48. Utilization of Practice Improvement Methodologies to Enhance OR Efficiency 423 Alexandra Bastien 49. Operating Room Fire Michael Jon Williams and Valerie Armstead

435

50. Intraoperative Awareness Jennifer Knuth, Lindsay Gennari, and John Cagino

443

51. Postoperative Cognitive Dysfunction Wael Saasouh and Shobana Rajan

449

Index

viii  •   C ontents

455

CONTRIBUTOR S

Ali R Abdullah, MD

Theresa J. Barnes, MD, MPH

Anesthesiologist Department of Anesthesiology and Critical Care Medicine Allegheny Healthy Network Allegheny General Hospital Pittsburgh, PA

Department of General Anesthesia Cleveland Clinic Cleveland, OH

Alexandra Bastien, MD

Assistant Professor Department of Anesthesiology Albert Einstein College of Medicine Montefiore Medical Center Bronx, NY

A. Elisabeth Abramowicz, MD

Department of Anesthesiology Program Director Westchester Medical Center/ New York Medical College Valhalla, NY

Amanda Moraska Benson, MD

John N. Afthinos, MD, FACS Surgeon Department of Surgery Northwell Health Forest Hills, NY

Staff Anesthesiologist and Intensivist Surgical Intensive Care Unit Center for Critical Care Cleveland Clinic Cleveland, OH

Melinda Aquino, MD

Jay Berger, MD

Assistant Professor Anesthesiology, Pain Management and Regional Anesthesiology Albert Einstein College of Medicine Montefiore Medical Center Bronx, NY

Assistant Professor Department of Anesthesiology Montefiore Medical Center Bronx, NY

Shobana Bharadwaj, MD

Program Director Obstetric Anesthesiology Fellowship Interim Obstetric Division Chief University of Maryland Medical Center Baltimore, MD

Maged Argalious, MD, MSc, MBA, MEd, FASE Professor of Anesthesiology CCLCM Program Director Anesthesiology Residency Cleveland Clinic Vice Chair of Education Anesthesiology Institute Cleveland, OH

Nichelle Broderick, MD Anesthesiology Resident Montefiore Medical Center Bronx, NY

Valerie Armstead, MD

John Cagino, MD

Professor Lewis Katz School of Medicine Temple University Philadelphia, PA

Anesthesiologist Albany Medical Center Albany, NY

Scott Chamberlain, DO

Lovkesh Arora, MBBS, MD

Albany Medical Center Albany, NY

Clinical Assistant Professor Roy J and Lucille A Carver College of Medicine University of Iowa Iowa City, IA

ix

Terry Ann Chambers, MD

Assistant Professor Department of Anesthesiology Albert Einstein College of Medicine Montefiore Medical Center Bronx, NY

Curtis Choice, MD, MS

Assistant Professor of Anesthesiology Site Director of Anesthesiology Hutch Ambulatory Surgery Center Preadmission Testing Hutch Tower II Department of Anesthesiology Bronx, NY

Miguel A. Cruz, MD Staff Anesthesiologist Cleveland Clinic Cleveland, OH

Guensley Delva, MD

CA-2 Resident Anesthesiology Montefiore Medical Center Bronx, NY

Yi Deng, MD

Department of Anesthesiology Baylor College of Medicine Assistant Professor of Cardiothoracic Anesthesiology and Critical Care Medicine Associate Director of Cardiothoracic Anesthesiology Ben Taub Hospital Baylor College of Medicine Houston, TX

Jagan Devarajan, MD Staff Anesthesiologist Cleveland Clinic Cleveland, OH

Rachel Diehl, MD,

Chief Resident Anesthesiology Institute Cleveland Clinic Cleveland, OH

Ehab Farag, MD, FRCA

Staff Anesthesiologist Cleveland Clinic Director of Research General Anesthesiology Associate Professor of Anesthesiology Case Western Lerner College of Medicine Cleveland, OH

Jessica Galey, MD

Melissa Ehlers, MD

Professor Albany Medical Center Albany, NY

Komal Gandhi, MD Assistant Professor Marshall Health Huntington, WV

Lindsay Gennari, MD Assistant Professor Albany Medical Center Albany, NY

Karen E. Gibbs, MD, FACS, FASMBS

Department of Surgery Division of Minimally Invasive and Bariatric Surgery Staten Island University Hospital Northwell Health Staten Island, NY

Cheryl K. Gooden, MD, FASA

Associate Professor of Anesthesiology Department of Anesthesiology Yale University School of Medicine Yale-New Haven Hospital New Haven, CT

Radhika P. Grandhe, MD

Assistant Professor Department of Anesthesiology University of New Mexico Albuquerque, NM

Ferenc Gyulai, MD

Professor of Anesthesiology Vice Chair Department of Anesthesiology and Perioperative Medicine University of Missouri Columbia, MO

Vilma Joseph, MD, MPH, FASA

Associate Professor of Clinical Anesthesiology, AECOM Department of Anesthesiology Montefiore Medical Center Bronx, NY

Indu Kapoor, MD

Assistant Professor All India Institute of Medical Sciences New Delhi, India

Myrna Kcomt, MD

Anesthesiologist Department of Anesthesiology Montefiore Medical Center Bronx, NY

Assistant Professor University of Maryland School of Medicine Baltimore, MD

x  •   C ontrib u tors

Sean P. Kelly, MD

Charu Mahajan, MD

Ashish K. Khanna, MD, FCCP

Mazen A. Maktabi, MBBCH

Sandeep Khanna, MD

Glenn Mann, MD

Resident Department of Anesthesiology University of New Mexico Albuquerque, NM Staff Anesthesiologist and Intensivist Surgical Intensive Care Unit Center for Critical Care Cleveland Clinic Cleveland, OH Staff Anesthesiologist Cleveland Clinic Associate Program Director Anesthesiology Residency Anesthesiology Institute Clinical Assistant Professor Lerner College of Medicine Cleveland, OH

Sung Kim, MD

University of Iowa Roy J and Lucille A Carver College of Medicine Iowa City, IA

Jennifer Knuth, MD Assistant Professor Albany Medical Center Albany, NY

Sree Kolli, MD

Staff Anestheisologist Cleveland Clinic Anesthesiology Institute Cleveland, OH

Perin Kothari, DO,

Chief Resident Anesthesiology Institute Cleveland Clinic Cleveland, OH

Oran Kremen, MD

Anesthesiologist North Shore University Hospital Manhasset, NY

Jason Lane

Assistant Professor Neuro Anesthesiology All India Institute of Medical Sciences New Delhi, India Associate Professor Department of Anesthesia, Critical Care, and Pain Medicine Massachusetts General Hospital Harvard Medical School Boston, MA Assistant Professor of Anesthesiology, AECOM Department of Anesthesiology Montefiore Medical Center Bronx, NY

Letha Mathews

Department of Anesthesiology Vanderbilt University Medical Center Nashville TN

Marina Moguilevitch, MD

Director, Division of Transplant Anesthesia Department of Anesthesiology Montefiore Medical Center Bronx, NY

Shobana Murugan, MD

US Anesthesia Partners Texas Childrens Hospital Pavilion for Women Houston, TX

Iyabo Muse, MD

Assistant Professor Regional Anesthesiologist Department of Anesthesiology Montefiore Medical Center Hospital of Albert Einstein Medical School Bronx, NY

Eman Nada, MD, PhD

Assistant Professor Department of Anesthesiology and Perioperative Medicine University of Massachusetts Worcester, MA

Chukwuemeka Okafor, MD

Department of Anesthesiology Vanderbilt University Medical Center Nashville TN

Resident Anesthesiology Montefiore Medical Center Bronx, NY

Dustin Liebling, MD

Nidhi Panda, MD

Anesthesiology Resident Monte Fiore Medical Center Bronx, NY

Professor, Department of Anaesthesia and Intensive Care Post-Graduate Institute of Medical Education and Research Chandigarh, India

 C ontrib u tors 

• 

xi

Roshan Patel, MD

Christopher Sheerer, DO

Thomas Powell, MD

Yelena Spitzer, MD

Assistant Professor Department of Anesthesiology Albert Einstein College of Medicine Bronx, NY

Anesthesiology Resident Anesthesiology Institute Cleveland Clinic Foundation Cleveland, OH

Department of Anesthesiology Baylor College of Medicine Houston, TX

Hemanshu Prabhakar, MD

Professor, Department of Neuroanaesthesiology and Critical Care All India Institute of Medical Sciences New Delhi, India

Shilpa Rao, MD

Assistant Professor of Anesthesiology Department of Anesthesiology Yale New Haven Hospital Yale School of Medicine New Haven, CT

Nii Amon Robertson, MD

Edwin Tavarez, MD

Resident Department of Anesthesiology Montefiore Medical Center Hospital of Albert Einstein Medical School Bronx, NY

Assistant Professor of Anesthesiology, AECOM Department of Anesthesiology Montefiore Medical Center Bronx, NY

Mark Teen, MD

Kurt Ruetzler, MD

Pediatric Anesthesiology Fellow Cincinnati Children’s Hospital Medical Center Cincinnati, OH

Staff Anesthesiologist Anesthesiology Institute Department of Outcomes Research Cleveland Clinic Foundation Cleveland, OH

M. Angele Theard, MD

Associate Anesthesiologist Legacy Emanuel Medical Center Co-Chair, Quality Section of Anesthesiology Portland, OR

Wael Saasouh, MD

Neuroanesthesiology Fellow Department of General Anesthesiology Cleveland Clinic Cleveland, OH

Michael Ting, MD

Department of Anesthesiology Associate Program Director Faculty Pediatric Anesthesiologist Maria Fareri Children’s Hospital/ Westchester Medical Center Valhalla, NY Clinical Assistant Professor Roy J and Lucille A Carver College of Medicine University of Iowa Iowa City, IA

Assistant Professor of Anesthesiology Cleveland Clinic Lerner College of Medicine Staff Anesthesiologist and Intensivist Department of General Anesthesiology and Center for Critical Care Cleveland Clinic Foundation Cleveland, OH Attending Anesthesiologist Assistant Professor of Anesthesiology Montefiore Medical Center Albert Einstein College of Medicine Department of Anesthesiology Bronx, NY

Assistant Professor of Anesthesiology Department of Anesthesiology Montefiore Medical Center Albert Einstein College of Medicine Bronx, NY

Surangama Sharma, MBBS, MD

Roshni Sreedharan, MD

Christopher Y. Tanaka, MD

Shamantha Reddy, MD

Irim Salik, MD

Assistant Professor of Anesthesiology Department of Anesthesiology Montefiore Medical Center Albert Einstein College of Medicine Bronx, NY

CA 3 Resident, Cleveland Clinic Cleveland, OH

Kumar Vivek, MD

Anesthesiologist Montefiore Medical Center Bronx, NY

Elizabeth M. Vue, MD

Anesthesiology Resident Department of Anesthesiology Bronx, NY

xii  •   C ontrib u tors

Michael Jon Williams, MD

Anesthesiologist Thomas Jefferson University Hospital Philadelphia, PA

Lisa Mouzi Wofford, MD

Lu Zheng, MD

Department of Anesthesiology and Critical Care Medicine Allegheny Healthy Network Allegheny General Hospital Pittsburgh, PA

Department of Anesthesiology Baylor College of Medicine Houston, TX

 C ontrib u tors 

• 

xiii

SECTION I AIRWAY

1. MANAGEMENT OF THE DIFFICULT AIRWAY IN THE MRI SUITE Letha Mathews and Jason Lane

Anesthesia equipment availability is also a chief concern (MRI-​ compatible anesthesia machines, infusion pumps, monitors, and airway equipment).

S T E M C A S E A N D K EY Q U E S T I O N S A 67-​year-​old morbidly obese male (body mass index [BMI] 45) with obstructive sleep apnea (OSA), claustrophobia, and known difficult airway is scheduled to undergo magnetic resonance imaging (MRI) for evaluation of new-​onset seizures. He had one seizure 3 days ago and described it as “my whole body was shaking.” When asked about his previous anesthetic history, he indicates, “I was told it was very difficult to place a breathing tube in me during my last anesthetic. My throat was very sore for several days afterwards.” The patient reports severe anxiety and claustrophobia with previous MRI experiences which he attributes to his history of posttraumatic stress disorder from military service during the Vietnam War. The MRI has been scheduled with “conscious sedation vs. general anesthesia” in the electronic ordering system at your large tertiary academic medical center hospital. He also has a past medical history significant for:

WH AT A R E T H E VA R I O US O P T I O NS F O R P RO V I D I N G S E DAT I O N I N T H E M R I S U IT E?

Depending on the severity of this patient’s claustrophobia, existing medication tolerances, and predicted difficulty in managing his airway, there are several sedation/​anesthetic techniques to choose from. Options include oral sedation with anxiolytics (oral diazepam or lorazepam), conscious sedation (small doses of midazolam and fentanyl), deep sedation/​general anesthesia (propofol) without a secure airway, and general anesthesia via laryngeal mask airway or endotracheal tube (ETT). WO U L D YO U C O NS I D E R I N T R AVE N O US ( I V ) S E DAT I O N F O R T H I S PAT I E N T G I V E N H I S H I S TO RY O F C L AUS T RO P H O B I A A N D P R EVI O US D I FFI CU LT A I RWAY M A NAG E M E N T ? I F Y E S , WH Y ? I F N O, WH Y N OT ?

Hypertension, type 2 diabetes, OSA (on continuous positive air pressure [CPAP] 18 cmH20), rheumatoid arthritis (RA), gastroesophageal reflux disease He has no known drug allergies and takes the following medications:

Given this patient’s comorbidities of morbid obesity, OSA, and known difficult airway management, I  would avoid administering conscious sedation without a secured airway. Concerns include extreme sensitivity to conscious sedation agents in the morbidly obese patient. These agents can rapidly lead to airway obstruction and respiratory distress/​failure. In addition, the known history of difficult endotracheal intubation in an elective setting (previous surgery) is a warning sign that emergent intubation in the MRI scanner would be even more difficult—​and dangerous. I  would electively intubate this patient in an effort to safely secure his airway prior to the start of the MRI.

Metoprolol, hydrochlorothiazide, metformin, prednisone, ibuprofen, pantoprazole His vital signs are: weight 127 kg, height 5’7”, BMI 45.4, blood pressure 160/​105, heart rate 98 WH AT A R E YO U R C O N C E R N S A B O U T P RO VI D I N G S E DAT I O N/​A N E S T H E S I A C A R E F O R T H I S PAT I E N T I N T H E M R I S U I T E?

H OW D O E S T H I S PAT I E N T ’S H I S TO RY O F O S A A FFEC T YO U R M A NAG E M E N T O F S E DAT I O N/​ A N E S T H E S I A A N D A I RWAY M A NAG E M E N T ? WH AT A R E T H E A N E S T H ET I C C H A L L E N G E S A S S O C I AT E D WI T H O S A ?

Several concerns exist for caring for this patient in the MRI suite. Inability to quickly access the patient in the MRI magnet in case of an emergency is the top concern. Presence of a very strong magnetic field is also concerning for airway management. The patient’s comorbidities of obesity and OSA create potential dangers of respiratory distress if the appropriate sedation/​ anesthetic technique is not chosen.

Patients with OSA are more likely to have other comorbidities such as obesity, hypertension coronary artery disease and are 3

at increased risk for perioperative complications. The incidence of difficult intubation is significantly higher in patients with OSA, so all preparations to manage a difficult airway should be immediately available along with experienced anesthesia personnel. Additionally, these patients are more sensitive to anesthetic drugs such as hypnotics and narcotics. The residual effects of these drugs can potentially cause postoperative respiratory depression and even respiratory arrest. Patients undergoing outpatient surgery/​procedures pose special risks, and plans should be discussed in advance with the patient and family. Patients who have been diagnosed with moderate or severe OSA may need to be admitted for observation after the procedure. A F T E R O BTA I N I N G A D ETA I L E D H I S TO RY F RO M T H E PAT I E N T A N D WE I G H I N G T H E R I S K S A N D B E N E F IT S O F VA R I O US S E DAT I O N/​A N E S T H E S I A T Y P E S A N D P RO C E D U R A L G OA L S , YO U D EC I D E TO P RO V I D E G E N E R A L E N D OT R AC H E A L A N E S T H E S I A . H OW WO U L D YO U I N D U C E G E N E R A L A N E S T H E S I A A N D S EC U R E T H E A I RWAY I N T H I S PAT I E N T ? WO U L D YO U I N D U C E A N E S T H E S I A I N T H E M R I S U IT E , WH I C H I S S EV E R A L F L O O R S AWAY FRO M T H E M A I N O P E R AT I N G S U I T E WI T H L I M IT E D R E S O U RC E S , O R I N T H E M A I N O P E R AT I N G RO O M (O R )  S U IT E?

This patient has multiple risk factors that predict difficult intubation, which is backed up by his history of difficult intubation. The MRI suite away from the main operating suite with limited supplies and personnel is not the ideal place to secure a known difficult airway. So, this patient should be prepared for anesthesia in the main suite, where experienced anesthesia personnel are available. Preoperative medications should include his dose of leviteracetam (if he has not taken his morning dose), prednisone, and esomeprazole. Avoid the use of preoperative sedatives such as midazolam unless absolutely necessary to allay anxiety. The patient should be reassured, and the details of anesthetic management explained in detail. WH AT A R E T H E I M P L I C AT I O N S O F R A O N A I RWAY M A NAG E M E N T A N D G E N E R A L ANESTHESIA?

RA affects multiple organ systems and has significant implications for perioperative management, particularly airway management. The atlanto-​axial joint is commonly involved especially in patients with longstanding RA. Anterior atlanto-​axial subluxation is the most common deformity, and acute subluxation can cause spinal cord compression and quadriparesis, respiratory insufficiency, and possible death. Cricoarytenoid involvement can lead to stridor and vocal cord dysfunction. Displacement of the Cricoarytenoid joint could happen during intubation. Often these patients would require a smaller endotracheal tube.

D E S C R I B E H OW YO U WO U L D S ECU R E T H E A I RWAY O F T H I S PAT I E N T. D E S C R I B E VA R I O US A I RWAY M A NAG E M E N T T EC H N I Q U E S F O R T H I S PAT I E N T I N T H E M R I S U IT E

There are several options for securing the airway in this patient. Fiber optic bronchoscopic (FOI) intubation may be warranted in this patient given his history of RA with limited range of motion of neck, obesity, and potentially difficult bag mask ventilation (BMV) because of his beard. However, video laryngoscopic intubation could be considered which may be a better option for anesthesia providers with limited experience with FOI. The experience and comfort level of the anesthesia provider as well as the severity of neck instability should influence the decision about awake versus asleep intubation. In addition, the patient should be able to cooperate well for an awake intubation. Mask ventilation can also cause significant c-​spine movement (If the decision is to intubate with video laryngoscope, it is important to stabilize the neck with manual in line stabilization [MILS]. MILS has been shown to result in poorer glottic views). Whatever the mode chosen for intubation, the endoscopist should be experienced, and back-​up airway devices such as a laryngeal mask airway and difficult airway cart should be readily available. Ideally, in this patient with multiple risk factors for difficult intubation and a known history of difficult intubation, FOI may be indicated. This can be done in the MRI suite or in the main OR suite based on the institutional practice and availability of equipment and trained help. At our institution we would induce anesthesia in the main OR suite. WH AT A R E A D D IT I O NA L C O NS I D E R AT I O NS I N T H I S PAT I E N T WIT H N EW- ​O NS ET S E I ZU R E S ?

A patient with new-​onset seizures raises several important considerations. The possibility of a newly developing intracranial mass is of serious concern for this patient. Raising intracranial pressure, cerebral edema, possible impending brain herniation, and cerebrovascular autoregulatory mechanisms (i.e., auto-​pressing) to preserve blood flow to the brain are all factors that must be taken into account. Furthermore, the effects of the sedation versus general anesthesia on all of these factors create additional danger for this patient when undergoing MRI. H OW WO U L D YO U M A I N TA I N A N E S T H E S I A I N T H I S PAT I E N T ?

Once the patient’s airway is secured, he can be transported to the MRI suite with continuous IV infusion of propofol along with other adjuvants as needed. Muscle relaxants can be used as necessary. In this patient with OSA undergoing MRI, narcotics and sedatives such as benzodiazepines should be avoided. Narcotics and midazolam can affect pharyngeal function and airway protection in patients. Anesthesia can be maintained with total IV anesthesia and or inhalational agents. Controlled ventilation would provide better conditions for imaging by avoiding movements

4  •  A irway

associated with respiratory excursions in a spontaneously breathing morbidly obese patient. Standard American Society of Anesthesiologists monitoring compatible with an MRI suite is recommended. WH AT A R E T H E C H A L L E N G E S WIT H P O S TO P E R AT I VE M A NAG E M E N T I N PAT I E N T S WI T H O S A ? WO U L D YO U D I S C H A RG E T H I S PAT I E N T TO H O M E?

A postoperative respiratory complication is the most common perioperative complication in patents with OSA. Neuromuscular blockade should be fully reversed with train of four ration >0.9. So, it is critical that the patient is extubated only when he is fully awake and following commands. The patient should be maintained in the head-​up position and should be very closely monitored for hypoxic events. Avoid any postoperative narcotics. If the patient can maintain normal vital signs and normal SpO2 he could be discharged home, provided he has a responsible adult to stay with him overnight. The patient and family member should be given clear instructions about when to seek medical help. If the patient experiences multiple episodes of hypoxic events in the postanesthesia care unit, he would need to be monitored for an extended period. DISCUSSION C H A L L E N G E S A S S O C I AT E D WIT H P ROVI D I N G C A R E I N T H E M R I S U I T E

Sedation/​anesthesia care in the MRI suite is challenging and often fraught with danger for any patient, let alone a morbidly obese patient with OSA and a potentially difficult airway. Concern for basic MRI safety surrounds the inability to utilize traditional anesthesia equipment like laryngoscopes and fiber-​optic bronchoscopes due to the presence of an “always

on” magnetic field.1 In addition, ability to monitor the patient (electrocardiogram [EKG], pulse oximetry, end-​tidal carbon dioxide) is also a concern as not all medical centers have readily available MRI-​compatible anesthesia/​sedation monitors. In the early days of providing sedation/​anesthesia in the MRI suite, makeshift techniques such as securing a colored ball atop a Styrofoam cup placed on the chest as a surrogate for chest excursion (respirations/​min) sometimes was the only “monitor” available. Modern-​day technology now allows us to have wireless full functional MRI-​compatible monitors including electrocardiogram, pulse oximetry, end-​tidal gas analysis, and blood pressure (noninvasive and invasive). The MRI suite is divided into 4 zones. Zone 1 is considered freely accessible to the general public. Magnetic fields in Zone 1 are considered minimal. Zone 2 is still a public area but considered an interface between Zone 1 and the more dangerous Zone 3. Zone 2 is typically where MRI screening occurs of patients and for staff desiring entrance into Zone 3. Zone 3 is an area near the magnet room (Zone 4). Figure 1.1 shows a computer control panel desk in Zone 3. The magnetic fields in Zone 3 are sufficiently strong to be considered a hazard to unscreened individuals. Finally, Zone 4 is what is considered the magnet room. Here the MRI magnetic fields are highest. All ferromagnetic items should be kept out of Zone 4 (i.e., non-​ MRI-​compatible oxygen tanks, IV poles, laryngoscopes, fiber-​ optic bronchoscopes, etc.). Table 1.1 shows the various MRI zones and associated levels of danger as one moves closer to the magnet. Knowledge of these MRI zones is vital for the anesthesiologist and anesthesia care team personnel. When caring for patients in MRI, it is vitally important to partner with the MRI technician to ensure safety of not only the patient in the MRI scanner but also all people and equipment in the area. When dealing with difficult-​to-​sedate patients or patients with predicted difficult airway management, it is important to always think several steps ahead. You might not be able to bring in that favorite fiber-​optic bronchoscope you must have when your patient is experiencing hypoxia and airway obstruction in Zone 4. Communication and preparation are the most crucial aspects caring for sedated and anesthetized patients in the MRI suite. T Y P E S O F S E DAT I O N/​A N E S T H E S I A S U P P O RT U T I L I Z E D I N R A D I O L O G I C P RO C E D U R E S ( I N C LU D I N G  M R I )

Figure 1.1 

Image of a modern MRI suite.

There is a wide range of options when considering sedation and anesthesia support for patients in MRI. The first question the anesthesiologist and anesthesia care provider should always ask themselves is “Why am I being asked to sedate or provide anesthesia to this patient?” Often patients request to be completely anesthetized for an MRI when they can actually tolerate the scan with some verbal reassurance or oral anxiolytics. Second, the anesthesia care team must have in-​ depth knowledge of the patient’s comorbidities. Providing conscious sedation for the 70-​kg college athlete is very different than caring for the morbidly obese patient with OSA. The anesthesiologist must balance risk and benefit of each

1. M anag ement o f t h e D i f f icu lt A irway in t h e M R I   S uite  •   5

Table 1.1  MRI ZONES OF SAFETY AND DANGER Zone 1

Area outside MRI environment that is accessible to the general public

Zone 2

Area between Zone 1 and Zone 3. This usually includes MRI reception area and waiting room

Zone 3

Area is restricted to people who have not been screened for ferromagnetic objects. Zone 3 access is supervised directly by MR technicians. Zone 3 access should be restricted from general public by a locking door

Zone 4

The MRI magnet/​scanner is in this zone. Potentially hazardous area due to strong magnetic field that is always on.

available sedation and anesthetic technique in conjunction with patient’s comorbidities. In the case example here, the patient’s morbid obesity, OSA, and history of known difficult airway should all be considered when devising an anesthetic plan. In the MRI suite, a conservative and preventative approach to managing patient’s sedation/​anesthesia and airway is the safest way to deliver care. The Anesthesia Patient Safety Foundation (APSF) data clearly shows increased mortality and morbidity in the nonoperating room anesthesia (NORA) areas compared to OR suites.2 On root cause analysis, this increase mortality and morbidity found in the APSF data set was focused on respiratory events, inadequate oxygenation/​ ventilation, and preventable by better monitoring.3 The spectrum of sedation the anesthesia care team can deliver in the MRI suite ranges from oral sedation, IV conscious sedation, deep sedation, to general anesthesia. There is a wide array of drugs in the anesthesiologist’s armamentarium to accomplish the goal of sedating the patient. Typical drugs utilized include opioids, benzodiazepines, ketamine, propofol, dexmedotomidine, and inhalational anesthetic agents. Choice of sedation/​ anesthesia technique and drug choice often depends on why the MRI scan is being obtained. For example, if the patient is having a preoperative stealth MRI for brain tumor resection, the ordering physician’s tolerance for inferior image quality from patient movement while in the scanner may be much lower than if the MRI was ordered by an orthopedic surgeon to assess a ligamentous injury of the patient’s knee. Once again, communication is key to devising a safe and appropriate sedation/​anesthetic plan for the patient.

with decreased functional residual capacity, atelectasis and increased ventilation perfusion mismatch, and therefore increased shunting. All these changes lead to decreased time to desaturation and development of hypoxia. Cardiovascular physiology is also altered in these patients with increased stroke volume and cardiac output, which can lead to left ventricular failure. Hypertension is prevalent in obese patients, which also can lead to left ventricular failure. Additionally, patients with OSA have a higher incidence of hypoxia and hypercarbia which can lead to right heart failure. Respiratory complications are the most common complication in the postoperative period in patients with OSA. Sleep apnea has been identified as an independent risk factor for perioperative pulmonary complications and mechanical ventilation.4 Patients with OSA undergoing outpatient surgery/​ procedures poses special risks and plans should be discussed in advance with the patient and family. Patients who have been diagnosed with moderate or severe OSA may need to be admitted for observation after the procedure. Patients with OSA are also more likely to have other comorbidities such as obesity and cardiovascular risk factors such as systemic and pulmonary hypertension, coronary artery disease, heart failure, and arrhythmias.5,6 The risk of developing cardiovascular complications is twice as high (OR 2.1).7 Additionally, these patients often have associated metabolic syndrome with glucose intolerance or diabetes and obesity. This patient is morbidly obese with a BMI of 47, which makes the management more challenging including the feasibility of being placed in the MRI scanner due to his body habitus. C H A L L E N G E S O F A I RWAY M A NAG E M E N T Airway management can be challenging in patients with I N N O R A , R A D I O L O GY, A N D  M R I OSA. BMV has been reported to be more difficult in patients The challenges of airway management in the MRI suite re- with a history of snoring.8 Sleep apnea has also been implicated volve around the presence of a large, “always-​on” powerful as a contributory factor for difficult intubation. However, the magnetic field. The anesthesiologist’s typical tools of airway association between OSA as an independent factor for diffimanagement (laryngoscopes, laryngoscope blades, video cult intubation is disputed.9 Patients with a history of difficult laryngoscopes, fiber-​optic bronchoscopes) are all ferromag- intubation have a higher incidence of OSA.10 The choice of anesthetic agents also has an impact on netic. There are specialized “MRI-​compatible” laryngoscope blades that can be brought into Zone 4 to manage a patient’s patient outcomes following anesthesia. Sedatives and airway. Some video laryngoscope (McGrath MAC) function opioids cause central respiratory depression which may be well within Zone 4.  If your airway plan requires fiber-​optic prolonged in patients with OSA. So, when possible, short-​ intubation, you may want to consider performing the fiber-​ acting agents are recommended, especially for outpatients. optic intubation in Zone 3, or even starting out in the OR to Reversal of neuromuscular blockers should be ensured prior secure the patient’s airway and then transfer to MRI Zone 4 to extubation. Patients with a longstanding history of RA often have for the scan. If a patient ever needed a surgical airway (emergent cricothyrotomy), the anesthesiologist would want to multiorgan involvement. These patients could have myocarmake sure an MRI-​compatible cricothyrotomy kit or plastic ditis, pericardial effusions, endocarditis, restrictive lung dihandled #11 scalpel was available to perform open emergent sease, pleural effusions, anemia, chronic renal, and hepatic cricothyrotomy. The challenges of airway management in the impairment from drug treatment.11 Airway management can be challenging due to involvement of upper cervical vertebrae. MRI suite are omnipresent. Anterior atlanto-​axial subluxation is the most common deformity, and acute subluxation can cause spinal cord comD E S C R I B E T H E I M PAC T O F MO R B I D O B E S IT Y, pression and quadriparesis and respiratory insufficiency O S A , A N D R A O N A N E S T H E S I A M A NAG E M E N T and possible death. Cricoarytenoid involvement can lead to Morbid obesity adds significant challenges in the perioperative stridor and vocal cord dysfunction.12 Displacement of the management of patients. Respiratory physiology is altered Cricoarytenoid joint could happen during intubation.

1. M anag ement o f t h e D i f f icu lt A irway in t h e M R I   S uite  •   7

P OT E N T I A L S E R I O US C O M P L I C AT I O NS F RO M I M P RO P E R A N E S T H ET I C M A NAG E M E N T I N M R I

into Zone 3 is under the control of the MRI technologist and not allowed without the individual going through an “MRI screening” process administered by an MRI technologist.

It is clear that providing sedation and anesthesia support in the MRI suite is challenging for a multitude of reasons. There are 2 very important dangers that need to be emphasized because they can have dire consequences for patient and anesthesia team. The first is the ability of the magnetic field to generate enough heat to actually burn the patient’s skin. This necessitates the use of only MRI-​compatible EKG leads. EKG pads will reduce the risk of the magnetic field generating enough heat to burn the patient. Also take care that there is not any standing liquid touching the patient, which could heat up in the magnetic field and burn the patient. The second issue is the MRI magnet quench and subsequent inadequate venting of gaseous helium. An MRI quench (magnet shut-​down) would be necessary when a ferromagnetic piece of equipment is brought into Zone 4 and subsequently pulled into the MRI magnet bore. This situation would require shutting down or quenching the magnet, and transfer of heat energy into a vat of liquid helium which sits above the MRI coil. This liquid helium is that converted to gaseous helium and should ventilate outside the building. There is inherent danger to the patient on the MRI scanner table and surrounding staff when a ferromagnetic piece of equipment (anesthesia supply cart) is brought into Zone 4.  This ferromagnetic equipment quickly becomes a projectile and can cause fatal injuries to anyone in its way. There is a secondary danger to everyone in Zone 4 if the gaseous helium vents into Zone 4 as opposed to outside the building. If gaseous helium is ventilated into Zone 4, immediately the room becomes highly pressurized and hypoxic. This micro-​environment thus created can threaten everyone’s life within Zone 4. Quick thinking and utilization of a MRI pressure equalization kit (protective eye goggles, thick padded gloves, and a hammer/​blade to break the Faraday cage) is usually what stands between life and death of everyone in Zone 4 during a quench with inadequate ventilation of gaseous helium.

2. Which ACR MRI zone represents the interface between the unregulated zone and the strictly controlled zones under control of the MRI technologist?

R E VI EW Q U E S T I O N S 1. Which American College of Radiology (ACR) MRI zone represents an area where there are sufficiently strong magnetic fields to present physical hazard to patient or personnel but is not in the magnet room? A. Zone  1 B. Zone  2 C. Zone  3 D. Zone  4 Answer: C Zone 3 represents an area adjacent to the MRI magnet room (Zone 4). Serious injury or death can result in Zone 3 as a result of interaction between objects, individuals, and the magnetic fields which are present. Zone 3 is a restricted area, typically under an access control (electronic door key code). Admission

A. Zone  1 B. Zone  2 C. Zone  3 D. Zone  4 Answer: B Zone 2 is the interface zone between the unscreened public (Zone 2) and the screened/​controlled Zone 2 (where effects of magnetic field can cause danger). 3. Your patient is in the MRI scanner when a nurse brings a ferromagnetic IV pole into the scanner room. Immediately the IV pole is pulled into the MRI magnetic bore. The patient is unhurt but needs to be extricated from the MRI magnet. Your staff is unable to pry the IV pole from the magnet. What is the name of the process that must be initiated to remove the IV pole from the magnet? A. Magnet shut-​down B. Code  Red C. Code  Blue D. Quench Answer: D Quench or quenching the MRI magnet is the process of shutting down the MRI magnetic field. This process releases energy into a helium cryogen bath, resulting in formation of gaseous helium exhaust. A quench can be an extremely dangerous and life-​threatening event for the patient and any staff in Zone 4 if the gaseous helium does not vent outside the building. There have been reported cases of a quench releasing gaseous helium into Zone 4, immediately causing a high-​pressure, hypoxic environment in the MRI scanner room. 4. Your patient is in the MRI scanner under conscious sedation. The patient becomes hypoxic and your medical student rushes in with a small E-​cylinder (non-​MRI-​compatible) that is catapulted into the MRI bore. Luckily the patient is not injured. However, the MRI technician sees this all occurring and hits the “red button” above his control panel which quenches the magnet. Immediately you see a rush of white gas fill the room as your medical student starts to panic. The room is now filled with a gaseous white cloud and you can’t open the door to get to your medical student or patient. What is the appropriate next step? A. Call a Code Blue B. Call a Code Red C. Put on eye goggles D. Put on thick padded gloves E. Break the MRI glass and Faraday cage with hammer/​ blade F. C, D, and E

8  •  A irway

Answer: F An MRI magnet quench is an extremely dangerous situation. The event described in the question stem is prototypical of helium gas expunge into the actual MRI room. The room door can’t be opened due to pressure differences and the white gaseous cloud is helium, thus making the room a hypoxic environment. Time is of the essence at this point in time. Someone must don protective eye googles and padded gloves and immediately break the MRI glass and cut the Faraday cage to relieve the pressure build-​up and hypoxic environment for the trapped patient and medical student. 5. All of the following statements about OSA are true  EXCEPT A. the incidence of OSA increases with age in men and women. B. OSA is an independent predictor of increased postoperative respiratory complications. C. if OSA is suspected, surgery should be postponed for a formal diagnosis. D. patients with OSA have a higher incidence of difficult endotracheal intubation. Answer: C There is insufficient evidence to support postponing surgery unless patients have uncontrolled systemic disease.13 The incidence of OSA in 9% is in women and 26% in men in the 30-​to 49-​year age group compared to 27% and 43% in those 50 to 70 years old.13 The prevalence of difficult intubations is 15% to 20% for OSA patients.14,15 6. The tools commonly used to identify surgical patients with suspected OSA in the perioperative period are A. STOP-​Bang questionnaire B. Berlin Questionnaire. C. P-​SAP  score. D. polysomnography. E. All of the above Answer: E A STOP-​Bang score of 4 has a sensitivity of 88% for identifying patients with OSA.13 6. All of the following are common manifestation of RA EXCEPT A. anterior atlanto-​axial subluxation. B. cricoarytenoid joint involvement. C. pericardial effusion, myocarditis, and atherosclerosis. D. obstructive lung disease. E. chronic renal failure. Answer: D The atlanto axial-​joint is commonly affected in RA because of attenuation of the transverse ligament and erosion of the odontoid peg.11 Restrictive lung disease is more common in RA.

QUESTIONS AND ANSWER S This chapter has accompanying questions and answers which are available to subscribers as part of the Oxford eLearning platform. To access the questions, go to http://​ oxfordmedicine.com/​anesthesiologyPBL REFERENCES 1. Martin R. Anesthesia concerns in the magnetic resonance imaging environment In:  Weiss M, Fleisher LA, ed. Non-​Operating Room Anesthesia. Philadelphia: Elsevier;2014:171–​175. 2. Metzner J, Domino K. Risk of anesthesia care in remote locations. ÄPSF Newslett; 2011. 3. Metzner J, Posner KL, Domino KB. The risk and safety of anesthesia at remote locations: the US closed claims analysis. Curr Opin Anesthesiol. 2009;22(4):502–​508. 4. Memtsoudis S, Liu SS, Ma Y, et al. Perioperative pulmonary outcomes in patients with sleep apnea after noncardiac surgery. Anesthes Analg. 2011;112(1):113–​121. 5. Gottlieb DJ, Yenokyan G, Newman AB, et  al. Prospective study of obstructive sleep apnea and incident coronary heart disease and heart failure:  the Sleep Heart Health Study. Circulation. 2010;122(4):352–​360. 6. Cadby G, McArdle N, Briffa T, et  al. Severity of OSA is an independent predictor of incident atrial fibrillation hospitalization in a large sleep-​clinic cohort. Chest. 2015;148(4):945–​952. 7. Kaw R, Chung F, Pasupuleti V, Mehta J, Gay PC, Hernandez AV. Meta-​analysis of the association between obstructive sleep apnoea and postoperative outcome. Br J Anaesth. 2012;109(6):897–​906. 8. Langeron O, Masso E, Huraux C, et al. Prediction of difficult mask ventilation. Anesthesiology. 2000;92(5):1229–​1236. 9. Neligan PJ, Porter S, Max B, Malhotra G, Greenblatt EP, Ochroch EA. Obstructive sleep apnea is not a risk factor for difficult intubation in morbidly obese patients. Anesth Analg. 2009;109(4):1182–​1186. 10. Chung F, Yegneswaran B, Herrera F, Shenderey A, Shapiro CM. Patients with difficult intubation may need referral to sleep clinics. Anesth Analg. 2008;107(3):915–​920. 11. Fombon F, Thompson, Jonathan P. Anaesthesia for the adult patient with rheumatoid arthritis. Contin Educ Anesth Crit Care Pain. 2006;6(6):235–​239. 12. Kolman J, Morris I. Cricoarytenoid arthritis:  a cause of acute upper airway obstruction in rheumatoid arthritis. Can J Anaesth. 2002;49(7):729–​732. 13. Chung F, Memtsoudis SG, Ramachandran SK, et  al. Society of Anesthesia and Sleep Medicine guidelines on preoperative screening and assessment of adult patients with obstructive sleep apnea. Anesth Analg. 2016;123(2):452–​473. 14. Kim JA, Lee JJ. Preoperative predictors of difficult intubation in patients with obstructive sleep apnea syndrome. Can J Anaesth. 2006;53(4):393–​397. 15. Acar HV, Yarkan Uysal H, Kaya A, Ceyhan A, Dikmen B. Does the STOP-​ Bang, an obstructive sleep apnea screening tool, predict difficult intubation? Eur Rev Med Pharmacol Sci. 2014;18(13):1869–​1874.

1. M anag ement o f t h e D i f f icu lt A irway in t h e M R I   S uite  •   9

2. HIP DISLOCATION IN A PATIENT WITH SCLERODERMA Lisa Mouzi Wofford and Yi Deng

procedure is urgent as delay will increase the risk of avascular necrosis of the hip.

S T E M C A S E A N D K EY Q U E S T I O N S A 43-​year-​old African American female presents to the emergency department by ambulance after being involved in a motor vehicle collision. She was the passenger and was wearing her seatbelt when the driver lost control of the vehicle and hit a tree. She never lost consciousness and was hemodynamically stable during transport to the hospital. She is awake and alert, and her primary complaint is severe left-​sided hip pain.

WH AT E L S E WO U L D YO U L I K E TO K N OW A B O U T T H I S PAT I E N T ’S H I S TO RY ? WH AT C O N C E R NS D O YO U H AVE A B O U T T H I S PAT I E N T U N D E RG O I N G A N E S T H E S I A ?

Upon further questioning of her medical history, the patient says that she takes medication daily for her lungs, her kidneys, and her stomach but cannot remember their names. She can walk a block or two slowly but gets short of breath with any additional physical exertion. She denies any heart conditions and does not see a cardiologist. She complains of muscle aches, skin tightness, and cold hands but has never had ulcers on her extremities. She did have symptoms of gastroesophageal reflux but they are currently controlled on medication. Her laboratory tests sent in the emergency room are significant for an elevated creatinine of 1.6 mg/​dl and mild anemia. She has a small mouth opening with a relatively large tongue. The skin around her mouth and neck is very tight, limiting mandibular and neck mobility. She has some face lacerations and two loose front teeth from the car accident.

WH AT A D D I T I O NA L I N F O R M AT I O N I S I M P O RTA N T TO G AT H E R AT T H I S T I M E? WH AT I S I N VO LV E D I N T H E P R I M A RY A N D S E C O N DA RY S U RV EY S O F A D VA N C E D T R AU M A L I F E S U P P O RT ?

Upon primary survey of the patient, she is noted to be awake and alert and breathing comfortably. The patient is tachycardic but normotensive. Her respiratory rate is 20 breaths per minute and her oxygen saturation is 95% on room air. She has no pain in her neck and can move all extremities except her left leg. Examination of her chest and abdomen are negative for any overt abnormalities. Her left leg is immobile, appears shorter than the right leg, and is internally rotated. Her review of systems is pertinent for hypertension, poor exercise tolerance, and an 8-​year history of scleroderma.

WO U L D YO U P RO C E E D WIT H G E N E R A L A N E S T H E S I A O R N EU R AX I A L A N E S T H E S I A ? WH AT A R E T H E R I S K S A N D B E N E FIT S O F E AC H ?

WH AT L A B O R ATO RY A N D I M AG I N G T E S TS WO U L D YO U O R D E R AT T H I S T I M E? WH AT I S S C L E RO D E R M A ? WH AT A R E S O M E O F T H E C L I N I C A L M A N I F E S TAT I O NS O F S C L E RO D E R M A ?

After hearing about the risks and benefits of general anesthesia and neuraxial anesthesia, the patient adamantly denies a spinal. She says that obtaining intravenous access on her is always difficult and that she does not want to undergo a neuraxial technique that may require multiple attempts. Despite the practitioner explaining that the spine is typically spared in scleroderma and that the procedure would likely be uncomplicated, she continues to refuse.

The patient undergoes a computed tomography (CT) scan of her head, chest, abdomen, and pelvis and a full series of radiographs of her pelvis. The CT scan shows some pulmonary interstitial fibrosis but no acute thoracic or abdominal injury. The radiographs confirm a posterior dislocation of the left hip. Orthopedic surgery is immediately consulted and the surgeons attempt a closed reduction of the hip under conscious sedation in the emergency department but the patient cannot tolerate the procedure. The patient will need to proceed to the operating room for another attempt at closed reduction with the possibility of an open reduction of the hip dislocation. The orthopedic surgeon explains that the

H OW WO U L D YO U S ECU R E T H E A I RWAY I N T H I S PAT I E N T ? D I S CUS S T H E S T E P S I N T H E D I FFI CU LT A I RWAY A L G O R IT H M

Given the unfavorable airway exam, the patient is prepped for an awake fiberoptic intubation. Light sedation and an anti-​ sialogogue are administered. The airway is topicalized with 11

nebulized 4% lidocaine. The patient is verbally reassured during the procedure and mild sedation is carefully administered to ensure cooperation and maintain spontaneous ventilation. The patient is intubated orally to avoid the nasal telangiectasias that commonly develop in patients with scleroderma. Care is taken to prevent hypothermia by increasing the temperature of the operating room, warming intravenous fluids, and covering exposed skin with warm blankets. The eyes are lubricated and covered and special attention is paid to the padding of the extremities and pressure points. The surgeons achieve closed reduction of the hip with muscle relaxation and the patient is extubated in the operating room fully awake after return of spontaneous ventilation and protective airway reflexes. Her postoperative course is uncomplicated. Her hip will take 2 to 3 months to heal and she will require physical therapy. DISCUSSION Systemic sclerosis (SSc) is a chronic autoimmune disorder that affects multiple organ systems, with interstitial and vascular fibrosis being the hallmark of this disease. It is varied in its presentations from limited cutaneous manifestations to fulminant fibrosis of multiple organs. The term “scleroderma” refers to the thickening of skin and is a prominent finding; however, it should not be confused with the classification of the disease itself. As seen in Table 2.1, SSc encompasses several clinical subsets, the most common being diffuse cutaneous SSc (dcSSc) and limited cutaneous SSc (lcSSc).1

propagation given the relative risk of 13 in SSc patients having affected first-​degree relatives; however, the exact pathway of inheritance is still poorly understood. Human leukocyte antigen association is weak, cytomegalovirus has been proposed as a possible trigger, and a variety of environmental/​pharmacological agents such as silica, polyvinyl chloride, bleomycin, and cocaine may play a role in the development of scleroderma.5,6 PAT H O P H YS I O L O GY

In the initial stages of the disease, vascular damage occurs in a reversible but widespread manner. This microangiopathy is noninflammatory in nature, hence the lack of typical vasculitic findings. Nevertheless, vascular endothelial injury results in intimal thickening, luminal narrowing, and upregulation of both immune response as well as coagulation/​thrombotic cascades. This leads to decreased small vessel blood flow and tissue hypoxia, as evident in Raynaud’s phenomenon, which is nearly universal across the disease spectrum. The major internal organs are affected as well. The amplified cytokine burst from immune cells, along with multiple other mediators, activates fibroblasts in an unregulated manner, resulting in exaggerated synthesis of collagen and extracellular matrix molecules. These displace normal tissue architecture, resulting in prominent fibrotic changes in multiple organs and functional impairment. Most patients with SSc also exhibit autoantibodies specific to their subset of disease, likely produced from hyperresponsive B cells.7 The combination of vascular damage, dysregulated immune response, and fibrotic changes are the hallmark of this disease, as depicted in Figure 2.1.

E P I D E M I O L O GY

SSc is a rare autoimmune disease, with prevalence ranging from 30 to 286 cases per million.2,3 It is more prevalent in females at approximately 3 to 4:1 ratio and occurs in the 4th to 6th decade of life. Disease strikes African Americans at an earlier age along with more systemic involvement. The 10-​year survival rate ranges from 60% to 90% depending on the clinical subset of disease.4 There is likely a genetic linkage to disease

C L I N I C A L SY M P TO M S

SSc is highly varied in presentation but can be generally grouped into dcSSc and lcSSc. Most individuals will have thickening of the skin, which distinguishes this from other autoimmune diseases. In dcSSc, fibrosis of the skin is widespread and visceral organ pathologies can progress rapidly, leading to increased mortality. LcSSc, in contrast, has limited

Table 2.1  CLASSIFICATION OF SYSTEMIC SCLEROSIS Diffuse cutaneous SSc

Topoisomerase I (Scl-​70), RNA polymerase III, U3-​RNP, B23

Widespread skin involvement, major visceral organ damage including renal crisis, interstitial lung disease, and cardiac abnormalities. Rapidly progressive.

Limited cutaneous SSc

Centromere, U3-​RNP, B23, Th-​To-​RNP

Limited skin involvement usually to distal extremities, fewer visceral organ damage other than GI, telangiectasia. Slowly progressive.

CREST

Centromere

Calcinosis, Raynaud, esophageal dysmotility, sclerodactyly, telangiectasia.

Sine scleroderma Overlap syndrome Mixed connective tissue

Characteristic visceral organ involvement without skin findings. PM/​Sci, U1-​RNP

Coexisting with other AI diseases such as SLE, RA. Similar to above but has U1-​RNP positive only.

SSc = systemic sclerosis; GI = gastrointestinal; AI = autoimmune; SLE = systemic lupus erythematosus; RA = rheumatoid arthritis. Source. Choi M, Fritzler M. Progress in understanding the diagnostic and pathogenic role of autoantibodies associated with systemic sclerosis. Curr Opin Rheumatol. 2016;28(6):586–​594.

12  •  A irway

Vessel injury

Immune upregulation

Inflammatory response

B cell activation

Cytokine burst

Vessel remodeling

Antibody production

Fibroblast activation

Tissue hypoxia

Production of ECM

Ischemic damage

Tissue fibrosis

Interaction of vascular damage and dysregulated immune response causing activation of fibroblasts, which then increases production of extracellular matrix and leads to fibrotic changes in a number of organs. Figure 2.1 

skin involvement confined to distal extremities, and disease typically follows an indolent course. CREST syndrome is a subset of lcSSc that manifests with subcutaneous calcinosis, Raynaud’s phenomenon, esophageal dysmotility, sclerodactyly, and telangiectasias. Some patients exhibit the characteristic visceral organ involvement with no skin findings, hence the term “sine scleroderma.” Finally, the term “scleroderma” now refers to localized skin disorders such as morphea, which are related but separate from SSc. Early symptoms of SSc are nonspecific. Raynaud’s phenomenon, sensitivity to cold, and nailbed capillary abnormalities can be evident early in the disease process.8 In dcSSc, this can rapidly progress to the inflammatory phase with soft tissue swelling (puffy fingers), intense pruritis, and edema. Skin pigmentation can both increase and decrease, creating a vitiligo-​ like appearance. After a period of months, the inflammatory phase evolves into the fibrotic phase, with skin thickening most apparent on the fingers, hands, and face. Dermal sclerosis can lead to joint contractures, ulcerations, pain, and profound weight loss. Characteristic facial changes include narrow oral aperture, thinning of lips, masked expression, and protruding teeth. Neck mobility can be limited due to densely adherent skin to the underlying platysma muscle. Maximal skin involvement tends to peak at 3 years and will regress over time in many patients. Visceral organ damage occurs early in the inflammatory phase and is discussed in detail later. Figure 2.2 details the various organ specific manifestations of this disease. LcSSc tends to have a much slower progression of symptoms, with skin thickening typically following years after the onset of Raynaud’s phenomenon. Telangiectasias are more frequent. Visceral organ involvement is less prominent, often limited to the gastrointestinal system with dysphagia and dysmotility as primary complaints. O RG A N-​S P E C I F I C PAT H O L O G I E S

Gastrointestinal symptoms are common in both the dcSSc and lcSSc subsets of the disease. Esophageal sphincter tone

and motility are affected, leading to acid reflux, dyspepsia, and dysphagia. Gastroparesis is frequently associated with esophageal disease and can increase the risk of aspiration. Bleeding and anemia can result from both esophageal ulceration and gastric antral vascular ectasia syndrome. Diarrhea is common from bacterial overgrowth of small intestines, leading to malnutrition and weight loss. Finally, diverticuli can form in both small and large bowels, causing pain and risk of perforation. Pulmonary arterial hypertension (PAH) and interstitial lung disease (ILD) are the two major causes of mortality in patients with SSc. Patients initially present with dyspnea on exertion and increased fatigue, whose deterioration can be rapid and unrelenting in dcSSc. The pulmonary function test is the most sensitive method for detection of early subtle changes suggestive of restrictive ventilatory and diffusion defects. Forced vital capacity (FVC) of less than 70% and high-​resolution computed tomography (HRCT) calculated disease involvement of more than 20% of lung are associated with very poor outcomes. PAH can occur either in isolation or together with ILD. Like other causes of PAH, dcSSc can ultimately result in cor pulmonale, right heart failure, and death if left untreated. In patients with lcSSc, ILD is uncommon, although isolated PAH can develop in a small percentage of the population.9 SSc can cause multiple etiologies of renal dysfunction, including glomerulonephritis and interstitial nephritis, but the most devastating is scleroderma renal crisis. It can acutely present early in the disease process, rapidly progressing to oliguric renal failure with high mortality. Those who do not recover will require renal transplant. Interestingly, the pathogenesis of renal dysfunction in SSc is predominantly vascular in etiology, with very little fibrotic changes of the renal parenchyma. SSc can cause fibrotic changes and dysfunction in virtually any part of the cardiac system, from conduction abnormalities to ventricular dysfunction to restrictive pericarditis. Patients are usually asymptomatic until the disease has reached advanced stages. Clinicians should monitor for signs of congestive heart failure and arrhythmias. Routine

2. Hip D is l ocation in a Patient wit h S c l eroderma  •   13

Digital vasculopathy (a)

Gastrointestinal

Cardiac (j)

(e)

Pulmonary hypertension

Lung fibrosis (f )

(d)

(c)

(b)

(g)

(h)

Renal

(i)

Musculoskeletal (l)

(k)

Calcinosis (m)

Acro-osteolysis (n)

Overview of major clinical complications of systemic sclerosis (A) Digital ulceration. (B) Dry gangrene. (C) Autoamputation. (D) Intestinal pseudo-obstruction. (E) Gastric antral vascular ectasia. (F) Lung fibrosis with a usual interstitial pneumonia appearance on high-resolution chest CT. (G) Lung fibrosis with an non-specific interstitial pneumonia appearance on high-resolution chest CT. (H) Pulmonary arterial hypertension shown histologically. (I) Pulmonary arterial hypertension shown on CT. ( J) Cardiac fibrosis. (K) Scleroderma renal crisis. (L) Digital contractures. (M) Calcinosis. (N) Acro-osteolysis. Denton CP, Khanna D. Systemic sclerosis. The Lancet. April 13, 2017. Figure 2.2 

surveillance with Holter monitors and echocardiography may be warranted. D I AG N O S I S

A combination of skin sclerosis, capillary abnormalities, presence of autoantibodies, and visceral organ dysfunction all contribute to a diagnosis of SSc. As mentioned, Raynaud’s phenomenon and nailfold capillary abnormalities are usually the first signs, but diagnosis may not be established for months. A skin biopsy can sometimes be of value in distinguishing SSc from other dermato-​fibrotic syndromes such as eosinophilic fasciitis, scleromyxedema, and nephrogenic systemic fibrosis. Antinuclear antibody is almost universally positive in SSc patients, and other specific autoantibodies can further narrow the subset of SSc disorder. A small percentage of patients will endorse other autoimmune disorders such as systemic lupus erythematosus and rheumatoid arthritis, creating an overlap syndrome with SSc. T R E AT M E N T

To date, there is no truly effective treatment for SSc, and most therapies are aimed at ameliorating symptoms. Cold

avoidance and calcium channel blockers are first-​line therapy for Raynaud’s. Proton pump inhibitors and histamine receptor blockers may alleviate reflux and dyspepsia. Angiotensin converting enzyme (ACE) inhibitor should be started early in the disease course for both hypertension control and prevention of renal crisis. Inhaled prostaglandins (i.e., iloprost), endothelin-​1 receptor inhibitors (i.e., bosentan), and phosphodiesterase type 5 inhibitors (i.e., sildenafil, tadalafil) have all been used to treat PAH. Immunosuppressants such as cyclophosphamide, methotrexates, and mycophenolate mofetil may be beneficial in the early inflammatory stage of the disease. Novel agents targeting transforming growth factor beta and other growth factors and immunoablatives are currently undergoing clinical trials. A N E S T H E S I A C O N C E R NS : P R EO P E R AT I V E M A NAG E M E N T

As mentioned, SSc is a chronic disease characterized by widespread inflammation, vascular sclerosis, and fibrosis of the skin and internal organs. Because scleroderma has the potential to dramatically affect all major organ systems and impact every aspect of anesthesia care, a thorough understanding of its manifestations is essential for all anesthesia providers.

14  •  A irway

Preoperative management of patients with scleroderma involves close scrutiny of all of the organ systems involved by this disease:  skin and musculoskeletal, gastrointestinal, cardiac, pulmonary, and renal. The anesthesia provider should perform a thorough examination of the face, neck, and mouth opening to determine the degree of microstomia resulting from skin tightening. Sometimes, neck mobility can be affected leading to poor neck extension. All of these changes will determine the ease or difficulty of endotracheal intubation. The extremities should be closely examined for skin changes and flexion contractures that may make intravenous access and positioning difficult. Pallor of the fingers and toes or ulcerations in the digits are signs of vascular compromise and may lead to unreliable noninvasive blood pressure measurements and may preclude radial arterial line monitoring. These patients may be on calcium channel blockers or phosphodiesterase inhibitors for treatment of Raynaud’s phenomenon. Patients with scleroderma should be assessed for gastrointestinal symptoms like dryness of the mouth and throat, difficulty swallowing, weight loss, reflux, and regurgitation. Fibrosis of the gastrointestinal tract can delay motility through the esophagus, stomach, and intestines. This, along with decreased lower esophageal sphincter tone, increases the risk for aspiration during induction of anesthesia. Patients may be on antacid therapy with proton-​pump inhibitors or histamine receptor blockers for symptomatic relief. Delayed intestinal motility can also lead to bacterial overgrowth, which can affect absorption of vitamin K, leading to a decrease in vitamin-​ K-​dependent clotting factors. A  complete blood count and coagulation panel should be assessed in patients who have a history of intestinal malabsorption or easy bruising. Sclerosis of the coronary arteries and fibrosis of cardiac muscle can manifest as conduction system abnormalities, ventricular hypertrophy, and systolic and diastolic dysfunction. Pulmonary hypertension is often present, even in asymptomatic patients, due to fibrosis of pulmonary arteries. Patients with hemodynamically significant cardiac disease may be on ACE inhibitors for systolic dysfunction, diuretics for diastolic dysfunction, or immunosuppressants for myocarditis, or they may have implantable defibrillators for conduction abnormalities or impaired ventricular function. Patients with cardiac involvement should be evaluated with a good history and physical, an electrocardiogram, and a chest X-​ray. An echocardiogram or stress test may be also indicated depending on the severity of symptoms. Lung disease from interstitial and peribronchial fibrosis is an early complication of scleroderma with severe restrictive lung disease developing in the first 5 years of symptom onset. Assessment of pulmonary involvement should include an electrocardiogram, chest X-​ray, and arterial blood gas measurement. Pulmonary function tests may be helpful to assess the degree of restrictive lung disease. The renal system is commonly involved because of glomerulosclerosis and renal artery intimal hyperplasia. This can lead to hypertension, chronic renal insufficiency, and decreased renal clearance of medications. Electrolytes, blood urea nitrogen, and creatinine should be assessed preoperatively. Many patients are treated with ACE inhibitors for

blood pressure control and to improve compromised renal function. A N E S T H E S I A C O N C E R NS : I N T R AO P E R AT I VE M A NAG E M E N T

A thorough understanding of the pathophysiology of scleroderma along with a comprehensive preoperative evaluation are vital for determining the anesthesia plan of patients affected with this disease. It affects multiple organ systems, which can make management difficult for anesthesiologists. Dermal thickening and contractures of the extremities can make intravenous access and noninvasive blood pressure measurement challenging. Central line access and arterial line monitoring may be required in patients with advanced disease. Arterial line access is not without complication, however. Distal arterial monitoring can precipitate Raynaud’s phenomenon and can lead to vasospastic ischemia of the digits.10 The decreased range of motion of mandible and neck, coupled with poor mouth opening, can impede mask ventilation and direct laryngoscopy after induction of general anesthesia (Figure 2.3). Asleep and, in some cases, awake fiberoptic laryngoscopy may be required for safe endotracheal intubation. Even though these patients are at high risk for aspiration, rapid-​sequence induction is probably best avoided given the likelihood of difficult airway management. It is recommended to pretreat patients with antacids and histamine receptor blockers before the initiation of anesthesia to increase gastric pH. Care must be taken to avoid traumatizing nasal and oral

Maximum active opening: 19 mm, negative end-​feel. Pronounced labial wrinkles, sharp nose. Crincoli V, Fatone L, Fanelli M, Rotolo RP, Chialà A, Favia G, Lapadula G. Orofacial manifestations and temporomandibular disorders of systemic scleroderma: an observational study. Int J Mol Sci. 2016;Jul 22;17(7). Figure 2.3 

2. Hip D is l ocation in a Patient wit h S c l eroderma  •   15

telangiectasias that may be present in patients with this disease, thus nasal temperature probes and nasogastric tubes should be well lubricated and gently placed or avoided altogether. Patients should be monitored with standard American Society of Anesthesiologists monitors; however, it may be beneficial to additionally monitor cardiac performance if the patient suffers from severe cardiac disease or pulmonary hypertension. Many patients have a chronically contracted intravascular state because of hypertension and because of medical therapy for the cardiac and renal conditions associated with scleroderma. Thus patients can exhibit significant hypotension under anesthesia. The location of the pulse oximeter probe should be changed often during the case to avoid causing ischemic damage to the digits.11 The interstitial lung disease associated with scleroderma can manifest with increased airway pressures requiring high oxygen levels to maintain adequate oxygenation and ventilation.12 Hypercapnea, acidosis, and hypoxia need to be avoided so that pulmonary hypertension is not exacerbated. Depending on the severity of the pulmonary symptoms and the type of surgery, postoperative ventilatory support may be required. Because scleroderma also affects the kidneys, the degree of renal impairment should be assessed before surgery and careful consideration should be made when selecting drugs that depend on renal elimination. Patient temperature should be closely monitored. To avoid vasoconstriction, the patient should be kept warm by increasing the temperature of the operating room, administering warm intravenous fluids, and using warming blankets. The patient should be carefully positioned with ample padding to avoid pressure necrosis and the eyes should be lubricated and covered to avoid dryness and corneal abrasions.13 In many cases, regional anesthesia can be offered as a safe alternative to general anesthesia in patients with scleroderma. It is highly effective for postoperative pain management and can improve tissue perfusion by the vasodilatory effects of sympathetic blockade. However, regional anesthesia is not without its limitations. Peripheral nerve blocks and neuraxial anesthesia can be technically challenging to perform because of thickened skin, contractures, altered fascial planes, and distorted anatomy. Prolonged sensory block is the most common complication of regional anesthesia in patients with scleroderma.14 There are multiple theories to explain this phenomenon. One idea is that the effect of local anesthetics is prolonged because of poor tissue perfusion from vasoconstriction.15 Other explanations include abnormal tissue pH delaying diffusion of local anesthetic into the nerve and compression of the nerve and its blood supply from injection of local anesthetic into inelastic fascial planes.16,17 Neuraxial anesthesia can also be a good alternative for patients with scleroderma. It can be less technically challenging than general anesthesia and peripheral nerve blocks because the disease does not typically affect the spine. Spinals, combined spinal epidurals, epidurals, and continuous intrathecal catheters have all been successfully performed with little complication.14 The anesthesia provider must be aware that scleroderma patients may exhibit more significant hypotension refractory to treatment because of their chronically contracted intravascular state.

R E VI EW Q U E S T I O N S 1. A  40-​ year-​ old African American female is recently diagnosed with systemic sclerosis. She has scleroderma and Raynaud phenomenon of her bilateral elbows and hands. She also has a history of hypertension. In addition to starting a calcium channel blocker, she should be started on which class of medication as well for her blood pressure control? A. Angiotensin converting enzyme inhibitor B. Beta-​blocker C. Selective alpha blocker D. Thiazide diuretic Answer: A Systemic sclerosis in African American females is often rapidly progressive and can involve multiple organ systems, even if the initial symptoms are limited to skin findings. One of the most feared complications of systemic sclerosis is scleroderma renal crisis, which is caused by underlying vasculopathy and can rapidly progress to oliguric renal failure and dialysis dependence. ACE inhibitors have been shown to decrease the incidence of renal crisis so should be started early in the disease process for both preventive reasons as well as hypertension control. None of the other classes of antihypertensives have been shown to decrease the incidence of renal failure in systemic sclerosis. A calcium channel blocker is primarily used here to alleviate symptomatic Raynaud’s phenomenon and not for blood pressure control. 2. A 58-​year-​old female presents to the anesthesia preoperative testing clinic for clearance for her upcoming surgery. She recently had a minor rotator cuff tear and is planning to undergo elective rotator cuff repair. She has been diagnosed with scleroderma within this past year because of thickening of skin, Raynaud’s phenomenon, and symptomatic gastroesophageal reflux with occasional dysphagia. She is otherwise asymptomatic with good exercise tolerance. What additional imaging workup would you require before clearing her for surgery? A. Transthoracic echocardiogram B. High resolution CT scan of the lung C. Renal ultrasound D. All of the above E. None of the above Answer: E The patient comes in for preoperative clearance for her routine surgery. Although systemic sclerosis can involve many internal organs and cause significant morbidity, in the absence of any symptoms with good exercise tolerance, it is unlikely that the patient has developed significant cardiopulmonary disease. Therefore routine preoperative imaging testing such as transthoracic echocardiogram and CT scan of the lung is not indicated. Patients with scleroderma can have rapidly progressive renal injury resulting in failure as a result of scleroderma renal crisis, and, in many cases, acute kidney injury can be asymptomatic until advanced disease has set in. It may be prudent to obtain a metabolic panel

16  •  A irway

with creatinine in the preoperative clinic; however, renal ultrasound is unlikely to be helpful because this patient has not suffered long-​standing medical renal disease and ultrasound will unlikely demonstrate atrophic kidneys or any obvious deformity. 3. A patient with prolonged history of limited cutaneous systemic sclerosis is undergoing a T6-​L1 laminectomy and fusion for spinal stenosis causing chronic back pain syndrome. In addition to careful padding and positioning, which other consideration should be taken into account during surgery in order to minimize complications? A. Routine changing of the pulse oximetry probe location B. Avoidance of forced-​air warming device C. Insertion pulmonary artery catheter with continuous cardiac output monitoring D. High tidal volume ventilation with low positive end expiratory pressure Answer: A Patients with lcSSc belong to a subset of the SSc (scleroderma) disease spectrum, which has more localized skin findings than those with dcSSc. These patients typically present with less systemic organ involvement and the disease usually follows an indolent course. However, Raynaud’s phenomenon is usually a prominent finding in these patients, and, during an attack, the hand and fingers become temporarily ischemic and notable distal cyanosis ensues. It has been suggested to change pulse oximetry probe location frequently during a prolonged operation in order to minimize incidence of ischemia to the digits and prevent the formation of ulcers.11 Cold temperature is one of the known triggers of Raynaud’s and therefore normothermia maintenance techniques such as use of a forced-​air warming device is recommended. Although patients with lcSSc can have visceral organ damage such as myocardial fibrosis causing heart failure, the incidence is rare and, in the absence of overt symptoms, routine pulmonary artery catheter is not recommended. Lastly, patients with SSc can have interstitial lung disease and pulmonary fibrosis which can result in high peak and plateau pressures under positive pressure ventilation, but even in its absence, a low tidal volume ventilation strategy with 6 to 8 mL/​kg of tidal volume based on ideal body weight should be used routinely. 4. In patients with SSc, pulmonary involvement is usually the biggest contributor to mortality. A. True B. False Answer: A Pulmonary arterial hypertension and ILD are the two major contributors of mortality in patients with scleroderma. ILC can be unrelenting, especially in those with dcSSc. The pulmonary function test is the most sensitive method to detect early subtle changes suggestive of restrictive ventilatory and diffusion defects. Both HRCT-​calculated disease involvement >20% of lung or FVC 300 beats per minute).4,5

S T E M C A S E A N D K EY Q U E S T I O N S A 57-​year-​old male presents for a catheter ablation due to long-​standing persistent atrial fibrillation. His past medical history includes diabetes mellitus type 2, hypertension, hyperlipidemia, gout, gastroesophageal reflux disease, peripheral vascular disease, and atrial fibrillation. His medications include insulin lispro, insulin glargine, lisinopril, metoprolol, atorvastatin, allopurinol, omeprazole, cilostazol, warfarin, and aspirin. The patient was seen in the preoperative clinic a week prior and cleared for surgery. He was instructed to stop warfarin 5 days before surgery and to take metoprolol, aspirin, and half the dose of his insulin regimen on the morning of his surgery. He is currently bridging his anticoagulation with subcutaneous enoxaparin once daily. While placing electrocardiogram monitors on the patient in the cardiac catheterization lab, the anesthesiologist notes atrial fibrillation in the patient with a heart rate of 85 beats per minute. The patient is currently asymptomatic and informs the anesthesiologist that he has been rate controlled for several years on his beta-​blocker.

WH AT A R E T H E D I FFE R E N T T Y P E S O F AT R I A L FI B R I L L AT I O N, A N D H OW A R E T H EY C L A S S I FI E D ?

1. Paroxysmal atrial fibrillation, also referred to as intermittent atrial fibrillation, is defined as an episode of atrial fibrillation that terminates spontaneously or with intervention in less than 7 days. 2. Persistent atrial fibrillation is defined as atrial fibrillation that fails to resolve within 7 days. This type of atrial fibrillation often requires pharmacologic or electrical cardioversion to restore sinus rhythm. Long-​ standing persistent atrial fibrillation occurs when atrial fibrillation lasts longer than 12 months.

WH AT I S T H E ET I O L O GY O F AT R I A L F I B R I L L AT I O N ?

3. Permanent atrial fibrillation occurs when a patient has atrial fibrillation greater than 12 months and the physician and patient jointly decide to no longer pursue rhythm control.

Atrial fibrillation is a disorder of electrical activity in the atrial myocytes, affects over 5  million patients globally, and is therefore the most common cardiac arrhythmia.1 The prevalence is about 1% in patients older than 60  years, 7.2% in patients older than 65 years, and 10% in patients older than 75  years.2 Atrial fibrillation must be recognized at an early stage and treated accordingly, as atrial fibrillation substantially contributes to morbidity and mortality. Atrial fibrillation can cause palpitations, dyspnea, dizziness, diaphoresis, lightheadedness, anxiety, syncope, respiratory distress, and/​or hypotension.3 However, many patients are asymptomatic and remain undiagnosed until a secondary complication occurs. Secondary complications include thromboembolic events, congestive heart failure, and even death.

WH AT A R E T H E R I S K FAC TO R S F O R AT R I A L FI B R I L L AT I O N ?

Predisposing factors can be cardiovascular and/​ or noncardiovascular in origin. 1. Cardiovascular origin: myocardial infarction, cardiomyopathy, hypertension, valvular disease, and heart failure 2. Non-cardiovascular origin: diabetes mellitus, obesity, excessive alcohol intake (also known as the “holiday heart syndrome”), hyper- or hypothyroidism, chronic obstructive pulmonary disease, obstructive sleep apnea, and other metabolic disorders

H OW D O E S O N E D E F I N E A N D D I AG N O S E AT R I A L F I B R I L L AT I O N ?

Atrial fibrillation is defined as rapid oscillation and/​or fibrillation of the atria due to irregular electrical activity, which varies in amplitude, shape, and timing, while the atrioventricular conduction remains intact.

An inflammatory reaction via leukocyte signaling may be a basic pivotal cause for atrial fibrillation, but this is still controversial and needs final judgment. Some genetic variants have 21

also been described to cause the development of atrial fibrillation; however, final clarification is still pending. WH AT A R E T H E FAC TO R S P R E D I S P O S I N G TO AT R I A L FI B R I L L AT I O N A F T E R S U RG E RY ?

Atrial fibrillation can be triggered by a previous surgical procedure. The causes are manifold and include:

WH AT I S T H E PAT H O P H YS I O L O GY O F AT R I A L FI B R I L L AT I O N ?

While there are multiple theories about the etiology of atrial fibrillation, a commonly accepted theory is that rapid electrical discharges are produced in the atria and from adjacent parts of the heart, particularly the pulmonary veins. Sources of these discharges can be either automatic foci, often localized to one of the pulmonary veins, or from localized sources from repetitive focal beats or reentrant electrical spiral waves.6

• Electrolyte imbalance (e.g., hypokalemia, hypomagnesemia)

H OW D O E S AT R I A L FI B R I L L AT I O N D I F F E R FRO M AT R I A L FLU T T E R , A N D H OW A R E T H EY D I S T I N GU I S H E D FRO M E AC H OT H E R C L I N I C A L LY ?

• Imbalance of the autonomic nervous system (“sympathetic overdrive”) • Volume overload (e.g., crystalloid products, blood transfusions)

Atrial fibrillation is a micro-​reentrant anomaly where regions of the myocardium, usually the left atrium, in addition to the pulmonary veins, produce rapid and irregular depolarizations.

• Hemodynamic instability (e.g., hypotension)

AF Mechanisms (b) svc

svc

(a)

LA

RA

LA

RA

svc

(c) RA

LA

* PVs

PVs

Ectopic focus IVC

IVC (d)

PVs

Single-circuit reentry

Multiple-circuit IVC reentry

Relationship to Clinical Forms Paroxysmal

Persistent

Permanent

Time course and progression of undelying (heart) disease Pulmonary Vein Atria

Triggers/Drivers Functional reentry substrate

Structural reentry substrate Reproduced with permission from Chen SA, Hsieh MH, Tai CT, et al. Initiation of atrial fibrillation by ectopic beats originating from the pulmonary veins: electrophysiological characteristics, pharmacological responses, and effects of radiofrequency ablation. Principal atrial fibrillation (AF)-​maintaining mechanisms. (A) Local ectopic firing. (B) Single-​circuit-​reentry. (C) Multiple-​circuit reentry. (D) Clinical AF forms and relation to mechanisms. Paroxysmal forms show a predominance of local triggers/​drivers, particularly from pulmonary veins (PVs). As AF becomes more persistent and eventually permanent, reentry substrates (initially functional and then structural) predominate. RA = right atrium; SVC = superior vena cava; LA = left atrium; IVC = inferior vena cava. Figure 3.1 

22  •   C ardiac f or t h e Genera l ist

On electrocardiogram, the main distinguishing feature between the two arrhythmias is the presence of p waves. In atrial fibrillation, there are lack of p waves in addition to unorganized atrial activity resulting in an irregular appearance. In atrial flutter, electrocardiogram displays the classic “saw tooth” wave pattern, usually in a 2:1 or 3:1 conduction from the flutter waves, and the atrial rate usually ranges from 250 to 350 beats per minute. Clinically, patients may be asymptomatic or have various symptoms. Generally, the degree of symptoms depends on the ventricular rate associated with the atrial arrhythmia. Symptoms include, but are not limited to, generalized fatigue, dyspnea at rest or on exertion, feelings of rapid palpitations in the chest, and exercise intolerance.7,8

(a)

WH AT A R E T H E I N D I C AT I O NS F O R C A R D I O V E R S I O N I N A PAT I E N T WIT H AT R I A L FI B R I L L AT I O N ?

In any patient with unstable atrial fibrillation, such as hypotension and respiratory distress, immediate direct cardioversion should be performed. When a patient presents in the postoperative period with new-​onset atrial fibrillation, the first step is to determine at what time the patient went into the arrhythmia. If the clinician can be precisely determine that a patient has been in atrial fibrillation for less than 48 hours and pharmacologic methods have failed to rate control or have failed to convert to sinus rhythm, direct cardioversion with sedation may be safely attempted. If more than 72 hours has passed since the onset of atrial fibrillation, rate control should be the goal, and a transesophageal echocardiogram (TEE) should be performed to look for any atrial clots prior to attempting direct cardioversion. Again, without TEE prior to cardioversion, there is a risk of dislodging an atrial clot causing embolization to the brain and ultimately a stroke.

(b)

WH E N S H O U L D A PAT I E N T WI T H AT R I A L FI B R I L L AT I O N B E S TA RT E D O N O R A L A N T I C OAGU L A N T T H E R A P Y ?

Figure 3.2  (A) Electrocardiogram of atrial fibrillation. (B) Electrocardiogram

of typical atrial flutter. Note the regular uniform P waves. Reproduced with permission from Gupta S, Oral, H. Atrial fibrillation and atrial flutter, in inpatient cardiovascular medicine. 2014.

Atrial flutter is a macro-​reentrant anomaly that originates from the cavo-​tricuspid isthmus of the left or right atria. Atrial flutter creates a circular tract within the atria that continually undergoes sequential depolarization and repolarization.

If a patient experiences an episode of paroxysmal atrial fibrillation for the first time and spontaneously or pharmacologically converts to normal sinus rhythm in less than 48 hours, future anticoagulation is generally not indicated. In a patient who has recurrent paroxysmal atrial fibrillation (i.e., more than one lifetime episode) for greater than 48 hours who fails to spontaneously convert to normal sinus rhythm, indefinite anticoagulation should generally be pursued. Anticoagulation with warfarin, a direct thrombin inhibitor (e.g., dabigatran), or a factor Xa inhibitor (e.g., apixaban, rivaroxaban) can reduce the risk of systemic embolization by 70% in most cases of nonvalvular atrial fibrillation.9 It is important to evaluate each patient based on his or her risk factors as sometimes the risk of bleeding outweighs the benefit of anticoagulation prophylaxis. One tool to risk stratify for anticoagulation prophylaxis is the CHA2DS2VASc score (see Table 3.1). If the CHA2DS2VASc is greater than 2, chronic anticoagulation is recommended. For a CHA2DS2VASc score of 1, there are conflicting opinions from

3. Atria l Fibri l l ation  •   23

Table 3.1 CHA 2DS 2VASC SCORE RISK FACTORS

SCORE

CHF/​LVEF ≤40%

1

HTN

1

Age ≥75

2

DM

1

Stroke/​TIA/​embolism

2

Vascular disease (prior MI, PAD, or aortic plaque)

1

Age 65–​74 years

1

Sex category (Female)

1 Possible points: 9

CHF = congestive heart failure; LVEF = left ventricular ejection fraction; HTN = hypertension; DM = diabetes mellitus; TIA = transient ischemic attack; MI = myocardial infarction; PAD = peripheral arterial disease.

clinicians as to whether anticoagulation should be initiated. Generally, the patient’s risk factor and severity of that risk factor is weighed against the risk of bleeding and embolization if anticoagulation is not started. H OW WO U L D YO U M A NAG E A PAT I E N T WH O D EV E L O P S N EW- ​O N S ET AT R I A L F I B R I L L AT I O N I N T H E O P E R AT I N G RO O M P R I O R TO I N D U C I N G A N E S T H E S I A ?

The first step in determining how to manage a patient with atrial fibrillation in the operating room is to determine whether the surgery is emergent, urgent, or elective. For surgeries that are emergent or urgent, it is generally advised to proceed with the surgery and attempt to maintain heart rate control and hemodynamic stability (e.g., strict fluid management to maintain euvolemia, tight blood pressure control) throughout the perioperative period. For our patient as previously described undergoing an elective procedure, the anesthesiologist should determine if the patient is symptomatic or asymptomatic. If the patient elicits a history of paroxysmal atrial fibrillation without symptoms, it is generally safe to proceed with the surgery and utilize the aforementioned techniques to maintain hemodynamic stability throughout the surgery. If the atrial fibrillation is new for the patient or the patient is currently symptomatic, it is advisable to cancel the surgery and to pursue additional workup including a cardiology consult prior to proceeding with the surgery. A PAT I E N T G O E S I N TO AT R I A L F I B R I L L AT I O N WI T H A R A P I D V E N T R I C U L A R R E S P O NS E WH I L E I N T H E P O S T A N E S T H E S I A C A R E U N I T. WH AT S H O U L D B E T H E F I R S T S T E P S I N M A NAG E M E N T ?

The first step when evaluating a patient with any arrhythmia is to assess the patient’s hemodynamic status, which includes

airway, breathing, and circulation (ABCs). Next is to determine if the patient is stable or unstable. In an unstable patient with hemodynamic compromise, immediate direct cardioversion should take place, generally starting at 75 Joules. If the patient is stable, which includes normal blood pressure, respiratory rate, and oxygenation greater than 90%, rate control over cardioversion should be the goal. Generally, a beta-​adrenergic antagonist (e.g., metoprolol) should be given with a goal heart rate of under 100 beats per minute. If heart rate does not decrease with the first dose of a beta-​ blocker, additional doses of beta-​blocker may be given. If heart rate continues to remain elevated despite multiple doses of beta-​blocker, a nondihydropyridine calcium channel blocker (e.g., diltiazem) may be used. If the patient is profoundly hypotensive or in acute heart failure, digoxin is an acceptable alternative for acute rate control. If the aforementioned pharmacologic methods fail to rate control, it is generally advisable to consult a cardiologist before additional medications are given. Some clinicians acknowledge it is acceptable to administer a bolus dose of amiodarone and to start the patient on an amiodarone infusion; however, when the timeline of atrial fibrillation is unknown, there is a theoretical risk that giving amiodarone may cause a pharmacologic cardioversion, which could lead to subsequent embolization of an atrial clot and ultimately result in a stroke. WH AT A R E T H E A N E S T H ET I C C O NS I D E R AT I O NS I N A PAT I E N T U N D E RG O I N G A N A B L AT I O N P RO C E D U R E F O R AT R I A L FI B R I L L AT I O N ?

It is first important to consider the age of the patient as well as the patient’s preexisting comorbidities. For children, adolescents, and adults who have high levels of anxiety, general anesthesia is preferred. In some cases where general anesthesia may suppress the arrhythmia, sedation with local injection is preferred. It is important to have a thorough discussion with the interventional cardiologist and the patient to weigh the risks and benefits for each type of anesthetic. Standard American Society of Anesthesiologists (ASA) monitors should be placed on the patient, which include electrocardiogram leads, pulse oximeter, blood pressure cuff, and a temperature probe. An arterial line may be considered depending on the length and intervention of the procedure as well as the patient’s medical comorbidities. Positioning of the patient is key with any type of surgery but becomes especially important in the cardiac electrophysiology suite where the room set-​up differs than a standard operating room. The anesthesiologist needs to consider location of the anesthesia machine and monitors, the procedure table, and positioning of the patient’s limbs during the procedure. Special attention should be made to avoid pressure sores by using foam padding under the arms and legs. Depending on whether the interventional cardiologist wants to utilize lateral imaging during the procedure, the arms may need to be placed above the patient’s head. Careful placement of the arms and hands should be considered to avoid brachial plexus injury and peripheral nerve damage. The patient’s limbs should

24  •   C ardiac f or t h e Genera l ist

be thoroughly wrapped and protected as defibrillation used during the procedure and could lead to serious burns. Because these procedures may last anywhere from 1 to 6 hours, maintenance of normothermia is essential, especially if the patient is under general anesthesia. The use of intraoperative forced-air warming blankets, warm fluids, and heat moisture exchangers are often used to help maintain adequate temperatures throughout the procedure. The interventional cardiologist may ask the anesthesiologist to monitor temperature continuously or periodically throughout the case (i.e., via esophageal, nasopharyngeal, or bladder probes). One important monitor that is often used in procedures requiring long radiofrequency ablation times is the luminal esophageal temperature (LET) monitor. Because the interventional cardiologist often works around or near the posterior wall of the left atrium, esophageal burn injuries are a real risk. Limiting energy delivery to the posterior wall of the left atrium, mechanical deflection of the esophagus during catheter ablation, and esophageal cooling/ insulating techniques are preventative methods to reduce esophageal injury. Some anesthesiologists may prefer general anesthesia over sedation, depending on the length of the procedure, if a patient has a difficult airway, or if invasive monitoring is required for the procedure (e.g., TEE). New evidence suggests that general anesthesia reduces the prevalence of pulmonary vein reconnection and thus recurrence of atrial fibrillation compared with sedation.10 Fluid management should be carefully considered, as fluid overload and pulmonary edema can occur, especially in a patient with congestive heart failure, or if excessive “cooling” fluids are used to counteract the heat dispersed from the catheter tip used to ablate areas of the myocardium.11 Neuromuscular blockade should be monitored closely as to prevent coughing and movement. Tidal volumes should be decreased while respiratory rate should be increased to maintain minute ventilation while also decreasing chest excursion. The activated clotting time should be monitored as heparin is administered prior to transseptal puncture. Activated clotting time (ACT) should be kept between 250 and 300 seconds. The worldwide mortality rate for catheter ablation for atrial fibrillation is 1:1,000.12 Surgical complications include, but are not limited to, puncture site infection and bleeding, transseptal puncture, arrhythmias, valvular damage, esophageal damage due to thermal injury, thoracic nerve and phrenic nerve injury, endocarditis, and skin burns. Anesthetic complications include, but are not limited to, hypotension, suppression of the arrhythmia, airway complications, and complications from positioning. DISCUSSION T R E AT M E N T

Various treatment options exist for atrial fibrillation depending on the patient’s other medical conditions and comorbidities. While atrial fibrillation can be due to valvular or non-valvular etiologies, the focus of this discussion

will be on treatment for nonvalvular atrial fibrillation, as this is the most common type of atrial fibrillation encountered in the perioperative period.

Rate control The first step in managing any patient with new-​onset atrial fibrillation should be rate control with pharmacologic methods. Generally, beta-​blockers (e.g., metoprolol) and nondihydropyridine calcium channel blockers (e.g., diltiazem) should be used as first line medications to keep the heart rate under 100 beats per minute. Numerous studies suggest that rate control over rhythm control is advantageous in the acute period of atrial fibrillation as both myocardial oxygen supply and demand can be compromised if heart rate is not adequately controlled. Some clinicians who are unable to control a patient’s heart rate with beta-​blockers or calcium channel blockers may reach for amiodarone. It is important to note that giving a bolus dose of amiodarone and starting an amiodarone infusion can break the arrhythmia and convert to normal sinus rhythm, and if the patient is not properly anticoagulated, there is a theoretical risk for atrial clot embolization leading to a stroke.

Synchronized cardioversion In any patient who is hemodynamically unstable, direct synchronized cardioversion should be implemented immediately to stabilize the rhythm and the patient. When a patient experiences new-​onset atrial fibrillation and less than 48 hours have passed since the onset of the arrhythmia, synchronized cardioversion can be safely attempted. In patients where the timeline of when the arrhythmia started is not known or a patient is chronically in atrial fibrillation, the patient should undergo TEE to look for atrial clots prior to attempting any cardioversion. The risk of stroke embolization in patients undergoing cardioversion without first having a TEE or anticoagulation is around 1%. Generally, if a patient is chronically in atrial fibrillation and synchronized cardioversion is being considered, the patient should be started on warfarin and bridged with heparin to safely anticoagulate the patient prior to attempting any cardioversion.

Radiofrequency ablation When pharmacologic methods and electrical cardioversion fail to convert a patient to normal sinus rhythm, a procedure known as radiofrequency ablation may be attempted. With radiofrequency ablation, a medium frequency alternating current between 350 to 500 kHz is applied through a catheter.13 Prior to applying the current, a qualified clinician “maps” the areas of the heart that are believed to be contributing to the arrhythmia. Once the areas are identified, the current is applied through the catheter into a cardiac vein, and the current destroys the abnormal electrical pathways.

3. Atria l Fibri l l ation  •   25

A N A N E S T H E S I O L O G I S T ’S P E R S P EC T I VE O N N EW- ​O N S ET AT R I A L F I B R I L L AT I O N

All these considerations are important and have general validity. However, adequate treatment of new-​ onset atrial fibrillation during surgery may sometimes be difficult. The major goals of the anesthesiologist are symptom relief (if the patient is awake), maintenance of hemodynamic stability, and prevention of immediate cardiopulmonary complications such as acute heart failure or pulmonary edema. Close coordination with the surgeon or proceduralist is indispensable, and a decision must be made whether to continue with the procedure. Discontinuation of the surgery or procedure should be discussed as the most extensive option, but this should be chosen carefully and should ultimately only be due to hemodynamic instability and/​or refractory to treatment. C O N C LUS I O N

Atrial fibrillation is a very common arrhythmia that can present in the perioperative period. Various etiologies contribute to atrial fibrillation including, but not limited, to hyper-​or hypotension, coronary artery disease, congestive heart failure, surgical stress, and metabolic abnormalities. The first step in treating any patient with atrial fibrillation is rate control and hemodynamic stabilization (e.g., blood pressure control, volume optimization). Direct synchronized cardioversion should only be attempted if the onset of atrial fibrillation is less than 48 hours or after anticoagulation has properly been initiated and TEE has been performed. Catheter ablation should be considered in a patient who fails to convert to sinus rhythm after pharmacologic methods and synchronized cardioversion have been tried. Anesthetic considerations for catheter ablation of atrial fibrillation include factoring in the patient’s medical comorbidities, the length of the procedure, temperature regulation, intraoperative monitoring techniques (e.g., TEE, LET), and carefully considering and acting on complications that may arise from the procedure. An anesthesiologist should be adept at caring for patients in the postoperative period who develop atrial fibrillation and should understand the various treatment options. R E VI EW Q U E S T I O N S 1. A 73 year-old male with a history significant for coronary artery disease and congestive heart failure is postoperative day 0 from a left heart catheterization. While in the postanesthesia care unit (PACU), his heart rate suddenly spikes to 168 bpm. The electrocardiogram reveals atrial fibrillation with a rapid ventricular response. Blood pressure is 83/56 mm Hg, respirations 26 breaths/minute, and pulse oximetry 89%. The patient appears to be in distress and states he cannot breathe. What is the MOST appropriate next step in management? A. Administer 5 mg metoprolol IV and place a nasal cannula

B. Administer a large fluid bolus of lactated ringers, support the blood pressure with a phenylephrine infusion, and place the patient on a non-rebreather face mask C. Perform synchronized cardioversion immediately starting at 75 Joules D. Perform unsynchronized cardioversion immediately using defibrillator pads Answer: C This patient is hemodynamically unstable and requires immediate synchronized cardioversion. If the patient went into atrial fibrillation with RVR and had stable vital signs, then it would be appropriate to administer a beta blocker medication and provide supplemental oxygen support. Administering a large fluid bolus in a patient with congestive heart failure is not an appropriate choice as large amounts of fluid can exacerbate heart failure and trigger pulmonary edema. While supporting the blood pressure with a vasopressor medication such as phenylephrine may be warranted, it is not the most appropriate next step in management. Performing unsynchronized cardioversion is not an appropriate treatment option as this patient is experiencing atrial fibrillation. If one were to perform unsynchronized cardioversion, there is a risk of delivering the shock during ventricular repolarization, which may result in an R-on-T phenomenon, resulting in ventricular fibrillation. Unsynchronized cardioversion is reserved for arrhythmias such as ventricular fibrillation where there is no distinct or coordinated electrical activity coming from the heart. 2. All of the following are potential complications from radiofrequency catheter ablation procedures EXCEPT: A. Esophageal injuries B. Skin burns C. Endocarditis D. Phrenic nerve injury E. Anterior ischemic optic neuropathy (AION) Answer: E Esophageal burn injuries are a concern when radiofrequency ablation is performed for a prolonged period of time. Skin burns can occur if the skin is not properly protected or insulated. Endocarditis is a potential risk, especially in patients with mechanical valves or underlying congenital heart disease. Phrenic nerve injury presents often as unilateral diaphragm paralysis and can be significant in a patient with severe chronic obstructive pulmonary disease. Anterior ischemic optic neuropathy, while a known complication of open heart surgery, is not a known complication of radiofrequency catheter ablations. 3. A 63 year-old female with a history significant for diabetes mellitus, coronary artery disease, hypertension, and prior history of TIA is post-operative day two following a total knee replacement. While on the nursing floor, the nurse notes that the patient appears to be in atrial fibrillation. Electrocardiogram confirms the absence of P-waves. The nurse is not aware of when the patient went into atrial fibrillation. The patient states she has had intermittent atrial fibrillation in the past with prior surgeries. Currently her heart rate is 103 bpm, and

26  •   C ardiac f or t h e Genera l ist

her vital signs are otherwise stable. What is the MOST appropriate next step in management? A. Attempt rhythm control with a beta blocker or calcium channel blocker, initiate anticoagulation, and obtain a transesophageal echocardiogram (TEE) B. Synchronized cardioversion C. Amiodarone bolus followed by amiodarone infusion D. Begin digoxin and obtain a digoxin level three days later Answer: A This patient is otherwise hemodynamically stable and therefore does not require immediate synchronized cardioversion. Because we do not know the timing of when the patient went into atrial fibrillation, it is unsafe to perform elective cardioversion or begin amiodarone as conversion to sinus rhythm could dislodge atrial clots that could embolize to the brain and cause a stroke. In this scenario, where the timing of atrial fibrillation is unknown, it is safest to begin rhythm control with a beta blocker or calcium channel blocker, initiate anticoagulation, and obtain a TEE prior to attempt any mechanical or pharmacologic cardioversion. It is also advisable to obtain a cardiology consult for long-term anticoagulation planning since this patient elicits a history of prior episodes of atrial fibrillation. Starting digoxin is generally reserved for patients with atrial fibrillation who are hypotensive but are otherwise hemodynamically stable who do not require immediate cardioversion. 4. A patient has been in atrial fibrillation for 10 months. What is the definition of this type of atrial fibrillation? A. Paroxysmal atrial fibrillation B. Long-standing persistent atrial fibrillation C. Permanent atrial fibrillation D. Persistent atrial fibrillation Answer: D This patient by definition has persistent atrial fibrillation, which fails to resolve after 7 days and lasts up to 12 months after initial diagnosis. Paroxysmal atrial fibrillation is defined as an episode of atrial fibrillation that lasts for less than 7 days and resolves with or without intervention. Long-standing persistent atrial fibrillation occurs when the patient experiences atrial fibrillation for greater than 12 months while still actively receiving pharmacologic medications for rhythm control. Permanent atrial fibrillation occurs when a patient experiences the arrhythmia for greater than 12 months and the decision between the doctor and patient is to no longer pursue rhythm control. 5. All of the following are true regarding the anesthetic management of patients undergoing radiofrequency catheter ablation EXCEPT: A. Controlled ventilation with muscle paralysis should be employed along with low tidal volumes and increased respiratory rate to preserve minute ventilation B. Heparin dosing should be used for an activated clotting time (ACT) goal of greater than 450 seconds

C. Monitored anestheisa care (MAC) may be necessary if the interventional cardiologist believes general anesthesia will suppress the arrhythmia D. An arterial line may or may not be used depending on the patient’s medical comorbidities and the length of the procedure Answer: B Heparin dosing should be directed to a goal ACT of usually 200 to 300 seconds. An ACT greater than 450 seconds is usually what is required when placing a patient on cardiopulmonary bypass. Controlled ventilation with muscle relaxation is usually required for a patient under general anesthesia as the goal is to minimize chest excursion while also preserving minute ventilation. This can be accomplished by decreasing tidal volumes and increasing respiratory rate. Monitored anesthesia care is an alternative to general anesthesia if the interventional cardiologist believes general anesthesia may suppress the arrhythmia. An arterial line is not necessary for all ablation procedures and should be tailored to the patient’s medical comorbidities and the anaesthesiologist’s clinical judgement.

QUESTIONS AND ANSWER S This chapter has accompanying questions and answers which are available to subscribers as part of the Oxford eLearning platform. To access the questions, go to http://​ oxfordmedicine.com/​anesthesiologyPBL

REFERENCES 1. Liu X, Wang F, Knight AC, Zhao J, Xiao J. Common variants for atrial fibrillation: results from genome-​wide association studies. Hum Genet. 2012;131(1):33–​39. 2. Lip GY, Tse HF, Lane DA. Atrial fibrillation. Lancet. 2012;379(9816):648–​661. 3. Amar D. Perioperative atrial tachyarrhythmias. Anesthesiology. 2002;97(6):1618–​1623. 4. European Heart Rhythm Association, European Association for Cardio-​Thoracic Surgery, Camm AJ, et  al. Guidelines for the management of atrial fibrillation: the Task Force for the Management of Atrial Fibrillation of the European Society of Cardiology (ESC). Eur Heart J. 2010;31(19):2369–​2429. 5. Fuster V, Ryden LE, Cannom DS, et  al. ACC/​AHA/​ESC 2006 guidelines for the management of patients with atrial fibrillation: full text: a report of the American College of Cardiology/​American Heart Association Task Force on Practice Guidelines and the European Society of Cardiology Committee for Practice Guidelines (Writing Committee to Revise the 2001 Guidelines for the Management of Patients with Atrial Fibrillation) developed in collaboration with the European Heart Rhythm Association and the Heart Rhythm Society. Europace. 2006;8(9):651–​745. 6. Chen SA, Hsieh MH, Tai CT, et al. Initiation of atrial fibrillation by ectopic beats originating from the pulmonary veins: electrophysiological characteristics, pharmacological responses, and effects of radiofrequency ablation. Circulation. 1999;100(18):1879–​1886. 7. Gupta S, Oral H. Atrial fibrillation and atrial flutter. In: Nallamothu BK, Baman TS, eds. Inpatient Cardiovascular Medicine. Oxford: John Wiley; 2014.

3. Atria l Fibri l l ation  •   27

8. Iwasaki YK, Nishida K, Kato T, Nattel S. Atrial fibrillation pathophysiology:  implications for management. Circulation. 2011;124(20):2264–​2274. 9. Palomäki A, Mustonen P, Hartikainen JE, et  al. Strokes after cardioversion of atrial fibrillation—​the FibStroke Study. Int J Cardiol. 2016;203:269–​273. 10. Di Biase L, Conti S, Mohanty P, et al. General anaesthesia reduces the prevalence of pulmonary vein reconnection during repeat ablation when compared with conscious sedation: results from a randomised study, Heart Rhythm. 2011;8:368–​372.

11. Eick OJ. Temperature controlled radiofrequency ablation. Indian Pacing Electrophysiology J. 2002;2(3):66–​73. 12. Ashley E. Anaesthesia for electrophysiology procedures in the cardiac catheter laboratory, Cont Educ Anaesth Crit Care Pain. 2012;12(5):230–​236. 13. Jais P, Haissaguerre M, Shah DC, et  al. A focal source of atrial fibrillation treated by discrete radiofrequency ablation. Circulation. 1997;95(3):572–​576.

28  •   C ardiac f or t h e Genera l ist

4. THE LVAD PATIENT PRESENTING FOR NONCARDIAC SURGERY Christopher Y. Tanaka and Elizabeth M. Vue

S T E M C A S E A N D K EY Q U E S T I O N S

WH AT A R E T H E P E R I O P E R AT I V E C O N C E R NS F O R LVA D PAT I E N TS P R E S E N T I N G F O R N C S ? H OW S H O U L D T H I S PAT I E N T B E M A NAG E D I N T H E I M M E D I AT E P R EO P E R AT I VE P E R I O D ? WH AT I S S U E S S H O U L D B E D I S CUS S E D D U R I N G MU LT I D I S C I P L I NA RY M E ET I N G S ? C A N E L EC T RO C AU T E RY B E S A FE LY US E D I N LVA D PAT I E N TS ?

A 47-​year-​old female with a left ventricular assist device (LVAD) presents for preoperative evaluation prior to an elective laparoscopic cholecystectomy. WH AT A R E LVA D S , A N D WH Y A R E T H EY I M P L A N T E D ? WH AT A R E T H E D I FF E R E N T T Y P E S O F LVA D D EV I C E S , A N D H OW D O T H EY F U N C T I O N ? H OW C O M MO N I S I T F O R LVA D PAT I E N T S TO P R E S E N T F O R N O N C A R D I AC S U RG E RY ( N C S )? WH AT Q U E S T I O N S WI L L YO U A S K T H E PAT I E N T ?

The patient is admitted 2 days prior to surgery to bridge warfarin to heparin infusion. All other home medications are continued up to the morning of surgery. She is brought to the operating room (OR) on the day of surgery.

The patient reports a history of advanced heart failure (HF) due to nonischemic cardiomyopathy and has been on the heart transplant waiting list for 3 years. She had a HeartMate II (Thoratec, Inc., Pleasanton, California) implanted 2 years ago without major complications. She is able to walk 2 to 3 city blocks now and denies orthopnea. She also had an implantable cardioverter defibrillator (ICD) placed for primary prevention several years ago. She otherwise reports being healthy. On physical exam, she is a thin, well-​appearing female with an unremarkable airway. She is on daily aspirin, warfarin, a beta-​blocker, and an angiotensin converting enzyme inhibitor.

WH AT A R E T H E A N E S T H ET I C A N D H E MO DY NA M I C G OA L S F O R LVA D PAT I E N TS ? H OW WI L L YO U MO N ITO R B L O O D P R E S S U R E ( B P) I N T R AO P E R AT I V E LY ? I S A N A RT E R I A L L I N E N EC E S S A RY ? I S A C E N T R A L L I N E N EC E S S A RY ? WH O S H O U L D B E AVA I L A B L E I N T H E O R TO M A NAG E T H E LVA D ?

The LVAD nurse connects the device console and reports parameters are normal for the patient. Standard monitors are placed, and an arterial line is inserted prior to induction. A magnet is placed on the ICD and external defibrillator pads are placed. The patient is carefully induced with etomidate, fentanyl, and rocuronium. Intubation is uncomplicated, the patient is placed on the ventilator, and sevoflurane is used for maintenance. Soon after abdominal insufflation, you notice the mean arterial pressure (MAP) is 20  mm Hg lower than baseline. The LVAD nurse tells you the flow, power, and pulsatility index (PI) are below baseline.

WH AT P R E O P E R AT I VE T E S T S A R E N E E D E D F O R T H I S PAT I E N T ? WH AT A R E YO U R C O N C E R N S I N P R EO P E R AT I VE P L A N N I N G S TAG E S ? A R E A N Y C O NS U LTAT I O NS R E Q U I R E D ? S H O U L D A C A R D I AC-​T R A I N E D A N E S T H E S I O L O G I S T C O VE R T H I S  C A S E?

H OW S H O U L D H Y P OT E NS I O N B E EVA LUAT E D I N LVA D PAT I E N TS ? WH AT I S T H E S I G N I FI C A N C E O F LVA D FL OW, P OWE R , A N D P I ? H OW S H O U L D H Y P OT E NS I O N B E T R E AT E D I N T H I S PAT I E N T ?

A multidisciplinary team involving anesthesiology, the HF team, and the cardiac surgeon and general surgeon review the case. A  recent echocardiogram demonstrates mild right ventricular (RV) dysfunction and normal LVAD function. Laboratory values are unremarkable with an appropriately elevated international normalized ratio. Electrocardiogram shows normal sinus rhythm. The HF team states the patient’s LVAD function and cardiovascular status are optimal. A recent ICD interrogation shows no arrhythmias.

The surgeon immediately stops abdominal insufflation, and a small fluid bolus is administered with a limited effect. A central line is quickly inserted and the central venous pressure (CVP) is 3  mm Hg. Additional fluid is administered with 29

resolution of hypotension. The LVAD flows, power, and PI return to baseline. The surgeon insufflates the abdomen gradually this time, and the surgery proceeds without complications. C A N T H I S PAT I E N T B E E X T U BAT E D I N T H E O R ? S H O U L D T H I S PAT I E N T R EC OVE R I N T H E P O S TA N E S T H E S I A C A R E U N I T ( PAC U ) O R I N T H E I N T E N S I V E C A R E U N I T ( I C U )?

The patient is extubated to nasal cannula in the OR and is taken to the PACU. The LVAD nurse remains to monitor LVAD function. Two hours later, you are called emergently to the bedside. The patient has become unresponsive, the LVAD console is alarming for low flow, and the patient is extremely hypotensive to a MAP of 27  mm Hg. The CVP now reads 16 mm Hg. WH AT A R E T H E P OT E N T I A L C AUS E S O F I N S TA B I L I T Y I N T H I S PAT I E N T ? WH AT I S A S U C T I O N EV E N T ? H OW WI L L YO U M A NAG E T H I S PAT I E N T ?

You discover the patient was given repeated doses of morphine and became progressively somnolent before becoming unresponsive. You immediately reintubate the patient for airway protection. Hemodynamics do not improve. The LVAD nurse turns down the LVAD speed, and you administer a small vasopressin bolus, and the MAP rises to 48 mm Hg. The LVAD alarm subsides, but the flow and MAP still remain low. A  cardiac-​trained colleague performs a transesophageal echocardiogram (TEE) demonstrating a severely dilated and hypokinetic RV, the interventricular septum bulging toward the left ventricle (LV), and a small LV. You begin a dobutamine infusion, which gradually improves the MAP and LVAD flow. WH AT C AUS E D T H E H E MO DY NA M I C C O L L A P S E I N T H I S PAT I E N T ? H OW C O U L D I T H AV E B E E N AVO I D E D ? H OW WO U L D YO U M A NAG E T H I S PAT I E N T I F S H E A R R E S T E D I N T H E PAC U ? WO U L D YO U P E R F O R M C H E S T C O M P R E S S I O N S O N A N LVA D PAT I E N T ?

The patient is transferred to the ICU and is extubated. She is weaned off of vasoactive medications the next day and discharged home 3  days later with return to her baseline function. DISCUSSION I N T RO D U C T I O N TO H E A RT FA I LU R E A N D  LVA D S

Epidemiology of heart failure Over 6.5 million people have HF in the United States, with over 960,000 new cases every year.1 While the vast majority of patients are managed medically, HF is a progressive disease and eventually patients become refractory to these therapies.1,2

Mortality for these end-​stage HF patients is as high as 50% after 1 year3 and 80% after 5 years.4 Aside from palliative care, heart transplantation was traditionally the only option for end-​stage HF patients. Heart transplantation is extremely effective, with 1-​year and 3-​year survival rates of 91% and 85.5%, respectively.5 While 200,000 to 250,000 patients could potentially benefit from heart transplantation (New  York Heart Association functional Class IIIB–​IV),6 only 2,700 heart transplants are performed annually in the United States.5 However, recent advancements in mechanical circulatory support (MCS) therapies have given many advanced HF patients new alternatives.

LVAD terminology MCS refers to a variety of devices that support cardiac function. MCS devices can be categorized by their intended duration of use: durable for long-​term use versus nondurable for short-​term use. Nondurable devices include intra-​aortic balloon pumps, extracorporeal membrane oxygenation (ECMO), and some ventricular assist devices. These nondurable ventricular assist devices can be placed percutaneously, such as the Impella (Abiomed, Inc., Danvers, Massachusetts) and TandemHeart (CardiacAssist, Inc., Pittsburgh, Pennsylvania), or centrally such as the CentriMag (Thoratec, Inc.). The nondurable ventricular assist devices are usually placed in emergency situations (i.e., rescue therapy) to allow time for recovery (i.e., bridge to recovery) or for clinical reevaluation (i.e., bridge to decision).7 Durable devices are designed with the intent of allowing survival to heart transplantation (i.e., bridge to transplant [BTT]) or as a permanent therapy (i.e. destination therapy [DT]).6 Compared to nondurable devices, durable devices are smaller and allow resumption of many daily activities. Examples are the Total Artificial Heart (SynCardia Systems, Inc., Tucson, Arizona) and the durable LVADs.7

Durable LVAD evolution and outcomes First-​ generation durable LVADs used a pulsatile pump. While superior to medical management in end-​stage HF,8 these devices had various disadvantages including mechanical failure after 1 year, large size, and patient discomfort.7 The first-​generation devices are no longer in clinical use. Current LVADs use continuous-​flow (CF) designs that are far smaller and more durable. The Thoratec HeartMate II was the first second-​generation device and is currently the most commonly implanted LVAD.9 In studies, it improved survival, functional status, and quality of life in BTT and DT patients with fewer complications compared to first-​ generation devices.10–​12 The HeartMate II is the only Food and Drug Administration–​approved device for both BTT and DT.7 Third-​generation devices modified the CF pump design to reduce size and complications. The HeartWare VAD (HVAD, Heartware, Inc., Framingham, Massachusetts) is approved for BTT only and the Thoratec HeartMate III is still awaiting approval.7,13,14 Future designs may feature smaller, more durable, and fully internalized device components.6,7

30  •   C ardiac f or t h e Genera l ist

LVAD indications and statistics Current guidelines consider MCS for BTT and DT as reasonable for select stage D HF patients.2,15 The Centers for Medicare and Medicaid Services (CMS) requires BTT patients be actively listed for transplantation before LVAD implantation.16 For DT, the Centers for Medicare and Medicaid Services requires New York Heart Association Class IV, transplant ineligibility, failed medical therapy, ejection fraction 90 mm Hg was an independent risk factor for ischemic and hemorrhagic stroke.29 Systemic hypertension may also worsen pulmonary artery pressures, RV function, and ventricular arrhythmias.28 Recommendations and guidelines vary but generally recommend MAP >70  mm Hg and 7 American Society of Anesthesiology (ASA) base units.40

A multidisciplinary team approach Although all types of NCS have been performed safely on LVAD patients, the population is complex and at high risk for complications. The 30-​day postoperative mortality for LVAD patients in NCS is between 5.3% and 12%.36,39,40 Data for morbidity are limited, but 1 study found acute kidney injury occurred in 8% of cases.40 An LVAD patient requires evaluation by multiple disciplines in a team-​ based approach.15,18,39,41,42 The LVAD team (HF cardiologist, LVAD coordinator), cardiac surgeon, anesthesiologist, and noncardiac surgeon should all review perioperative risks related to the LVAD and NCS itself. Management of the patient throughout the entire perioperative period should be discussed. Although cases are sometimes emergent (6.7%–​21% of NCS cases),39,40 every effort should be made to have a team discussion before every NCS case. The anesthesiologist has the unique advantage of being familiar with the various noncardiac surgeries and should be involved in all discussions. For example, when assessing perioperative risk of an LVAD patient for a knee replacement, the cardiologists may not be familiar with specific orthopedic surgery complications. However, most anesthesiologists are well aware of the physiologic impact of tourniquet use, which could have even more serious consequences in the LVAD population by precipitating pulmonary hypertension and RV failure during deflation.

Perioperative personnel and anesthesiology staffing Throughout the intraoperative and immediate postoperative period, a clinician fully trained in managing the LVAD and potential alarms must be present.15 This is commonly an LVAD registered nurse or perfusionist but may be an anesthesiologist or another appropriately trained clinician. A cardiac surgeon should at least be immediately available.15 There is a debate whether a cardiac-​trained anesthesiologist is required for LVAD patients presenting for NCS.43,44 Cardiac anesthesiologists have the advantage of being familiar with LVAD physiology and advanced monitoring techniques.

34  •   C ardiac f or t h e Genera l ist

However, a general anesthesiologist with appropriate training will often be sufficient.39 In 1 study, nurses sedated 63% of endoscopy patients without any anesthesiology provider present.45 Sheu et al.46 surveyed anesthesiologists about staffing patterns for LVAD patients presenting for NCS. Although a great majority of respondents stated that cardiac anesthesiologists “usually” staffed these cases (endoscopy:  79%; OR-​based:  84%), many respondents stated noncardiac anesthesiologists “sometimes” staffed these cases (endoscopy: 48%; OR-​based: 40%). Large retrospective studies found noncardiac anesthesiologists covered 43% to 53% of NCS cases.39,40 Moreover, over time, the trend was for noncardiac anesthesiologists to cover LVAD patients more frequently. The general consensus is that noncardiac trained anesthesiologists can safely manage LVAD patients in many situations.39,40,43 In a retrospective study, Mathis et  al.40 found that anesthesiologist training did not independently predict complications. We recommend that staffing should be decided on a case-​to-​case basis, with each patient’s perioperative risk and the individual anesthesiologist’s comfort level considered. The anesthesiologist should have some minimal LVAD education and experience and an LVAD nurse or perfusionist should be present in the OR. A  cardiac-​trained anesthesiologist should be readily available if more advanced consultation is required.

Preoperative evaluation As with any patient, the history and physical, laboratory results and imaging tests should be used to assess medical optimization and perioperative risk. The preoperative evaluation should also include LVAD function assessment by the HF cardiologist.18 However, anesthesiologists should be familiar with assessing LVAD patients, especially in emergency situations. Assessment of LVAD function begins with a history and physical focusing on cardiopulmonary function. Traditional assessments of functional status should still be used in this population. Any signs or symptoms of worsening HF (e.g., orthopnea, lung crackles, pedal edema) should prompt further evaluation and testing. A chest radiograph may be helpful.43 Patients are usually well educated about their LVAD device and should maintain a log of device function.15 Any recent alarms should be reviewed with the patient, especially to detect suction events. An LVAD clinician may also interrogate the device, which will provide clinicians varying amounts of historic information, depending on the model.18,42,43 LVAD patients should have regular echocardiogram exams, and any change in clinical status should prompt echocardiogram evaluation.15,24 The anesthesiologist should make note of baseline RV and valvular function.18,43 The HF team may consider adjusting LVAD speeds if LV size, AV opening, interventricular septum position, or RV function are not optimal.24 A ramp study should be considered if the LVAD speed is to be changed.19,24 Right heart catheterization may be necessary if RV function has worsened.15 A review of systems should also screen for end-​organ function,19,41 particularly renal and hepatic function.18 LVAD-​related

complications such as bleeding, neurologic events, and hemolysis should also be covered. Laboratory testing should be ordered to assess for end-​organ function and complications.15,18,41,43

Arrhythmia and implantable cardiac device management Current guidelines recommend ICDs for advanced HF patients2; thus, 68% to 80% of patients already have ICDs at the time of LVAD implantation.11–​14 Furthermore, guidelines recommend LVAD patients without ICDs should be considered for primary prevention.15 Indeed, 34% of LVAD patients have an appropriate ICD intervention within 1 year of LVAD implantation.47 Although some have raised concerns regarding electromagnetic interference and LVAD function,18,41 we have not encountered any LVAD dysfunction with electrocautery, even when operating near LVAD components. However, patients with concurrent ICDs or pacemakers should be managed according to the current ASA practice advisory.48 External defibrillator pads should be considered for all patients. A preoperative electrocardiogram should be standard,18 and interrogation of the ICD for recent events should be considered. Malignant arrhythmias such as ventricular tachycardia and even asystole may be surprisingly well tolerated in some LVAD patients.26 In these situations, a Fontan-​like physiology with passive pulmonary circulation has been described.26,49 Patients with malignant ventricular arrhythmias should be evaluated for ablation prior to elective NCS.15

Antibiotic prophylaxis In the perioperative setting, prophylaxis for endocarditis is reasonable given the high risk of bacteremia in LVAD patients.15 Some have advocated using prophylactic regimens similar to the multiantibiotic and antifungal combination used during initial LVAD implantation.41 It is our practice to consult an infectious disease specialist routinely before NCS, as each patient and surgery has specific risks.

Bleeding and anticoagulation management LVAD patients are maintained on aspirin and warfarin to prevent thrombotic events but are also at high risk for bleeding events. Current guidelines recommend holding anticoagulation and antiplatelet medications in the setting of significant GI bleeding and even reversal in hemorrhagic strokes.15 However, multidisciplinary team should weigh the risks of bleeding versus pump thrombosis. For LVAD patients presenting for NCS, the International Society for Heart and Lung Transplantation (ISHLT)15 made the following recommendations: • Decisions should be made based on collaboration between the LVAD and NCS teams. • For elective procedures, warfarin and antiplatelet therapy may be continued if the risk of bleeding is low.

4 . LVA D Patient P resentin g f or N oncardiac S ur g ery  •   35

• Warfarin and antiplatelet therapy should be held preoperatively if warranted by the risk of surgical bleeding. Bridging with heparin may be appropriate. • For emergency procedures, warfarin may need to be reversed with fresh frozen plasma or prothrombin protein concentrates. Vitamin K should be administered with caution. • Warfarin and antiplatelet therapy should resume postoperatively when the risk of surgical bleeding is acceptable.15 In retrospective reviews, approximately 20% of patients had received warfarin within 3 days of NCS, and 27% were on a heparin bridge preoperatively.37,40 LVAD patients have high rates of intraoperative blood transfusions in NCS, ranging from 12% to 17% in studies.34,39,40 Arrangements should be made in advance to have blood products available for the OR. If not discontinued or reversed preoperatively, appropriate anticoagulant reversal agents should be immediately available. Likewise, desmopressin should be available if acquired von Willebrand’s disease is suspected.

the PACU team’s training level. An ICU may be appropriate in many cases. Again, these decisions should be made on a case-​by-​case  basis. Recovery location practices will vary between institutions. Sheu et al.46 found that 54% of anesthesiologists surveyed typically recover their NCS patients in the PACU, while the remaining 46% admit their patients to the ICU postoperatively. However, only 29% of respondents said they recover their endoscopy patients in the ICU. Wherever patients recover, the multidisciplinary team should follow-​up with the patient postoperatively. I N T R AO P E R AT I V E M A NAG E M E N T

LVAD-​related equipment Once in the OR, the LVAD controller should be attached to the console and plugged into a power outlet.18,41 Extra batteries should be available in case of power failure.41 The LVAD controller and lines should be secured to the patient or operating table and protected from potential damage. The LVAD console should be accessible to the LVAD clinician with the screen visible to the anesthesiology team.

Surgical concerns Preoperatively, the multidisciplinary team should review all steps in the operation to identify potential issues. Ideally, the noncardiac surgeon should have experience operating on LVAD patients.15 This is particularly important when the operating site is close to the LVAD components. In such instances, a cardiac surgeon should be in the OR or immediately available. Patient positioning and its potential impact on the LVAD should also be reviewed. The percutaneous driveline must be protected at all times, especially in lateral decubitus and prone positions. Reverse Trendelenburg and lateral decubitus positioning may decrease venous return and LVAD flow.18,44 Trendelenburg may also be deleterious by acutely increasing venous return and precipitating RV failure18 or by increasing intrathoracic pressure thereby decreasing venous return.44 However, with gradual position changes while monitoring hemodynamics, adequate positioning for surgery should be possible, although fluid administration and vasoactive medications may be necessary.18,41,44 Laparoscopic procedures may cause hemodynamic instability by increasing intrathoracic pressures and decreasing venous return.18,41,44 In addition, SVR will typically increase, which may decrease LVAD flow.44 Insufflation may worsen pulmonary vascular resistance (PVR) and RV function by causing hypercarbia and increased airway pressures. As with position changes, insufflation can usually be performed safely if done gradually.18,41,44

Postoperative planning Multidisciplinary team discussions should include plans for postoperative location and staffing. The same intraoperative staffing considerations apply postoperatively: an LVAD nurse or perfusionist may need to be at the bedside depending on

Blood pressure monitoring ASA standard monitors should be used for every case. However, BP measurement is frequently an issue, as Mathis et  al.40 reported 55% of cases had a 20-​minute gap without a single reading. Due to decreased pulsatility, automatic cuff measurements are only possible in 50% of CF-​LVAD patients.50 Even when a reading is possible, the data are conflicted whether automatic cuff readings correlate well with simultaneous arterial line measurements.21,50 The ISHLT recommends that for minor procedures, Doppler measurement of BP is sufficient.15 This is performed by using an audible Doppler device over the brachial artery while slowly deflating a manual BP cuff.50 In patients with a wide pulse pressure, the Doppler pressure more closely resembles systolic pressures, while in patients with a low pulse pressure, it more closely reflects MAP. Due to this ambiguity, pressures should be simply recorded as “Doppler pressures.”19 For patients at risk for hemodynamic instability during NCS, the ISHLT recommends an arterial line.15 However, this recommendation is vague, and thus practices vary. In the survey by Sheu et al.,46 71% of respondents routinely placed arterial lines for OR cases, while 50% did so for endoscopies. Mathis et  al.40 found only 20% of cases used arterial lines. Interestingly, Stone et  al.39 found that overall, 41% of cases used arterial lines, but by the last year of the study this number decreased to 23%. The decision to place an arterial line should be made on an individual case basis. The patient’s baseline cardiac function, the risk of intraoperative bleeding and fluid shifts, and ability to obtain automatic BP measurements should be considered. While automatic BP measurements may be possible when awake, this may not be true once a patient is anesthetized and pulsatility diminishes.39 The routine use of ultrasound for

36  •   C ardiac f or t h e Genera l ist

arterial line placement is recommended, as the pulse is often not palpable.18

Oxygen saturation monitoring Although there is often sufficient pulsatility for accurate pulse oximetry,41 readings may be unreliable in some situations.19 Cerebral oximetry, which does not require pulsatile flow, may be used as a surrogate.18,19,51 However, cerebral oximetry readings are influenced by a number of factors besides arterial oxygen saturation.51 Decreasing LVAD speeds to promote pulsatility and improve pulse oximetry readings have been described,42 although we do not recommend doing this routinely. Frequent arterial blood gas sampling may be necessary in some situations.

Additional advanced monitoring The ISHLT recommends central venous catheters (CVCs) during NCS if hemodynamic instability is anticipated.15 CVP monitoring can help assess intravascular volume status and RV function.41 Pulmonary artery catheters (PACs) may be useful if there are concerns for pulmonary hypertension that may precipitate RV failure.18 PACs can also provide cardiac output measurements using thermodilution and/​or mixed venous saturation methods, which may be useful if calculated LVAD flows are questionable.41 TEE is extremely useful for assessing RV function and LV loading conditions.18,24,41 TEE may also become necessary if device malfunction (e.g., cannula obstruction, pump thrombus) is suspected or LVAD speeds need to be adjusted.24 As with arterial lines, use of these advanced monitoring techniques varies. Sheu et al.46 found that 12% of respondents routinely placed CVCs, 1.5% routinely placed PACs, and 14.6% routinely used TEE for NCS cases. In retrospective studies, CVCs were used in 4.6% to 12% of cases, PACs in 1.5% to 10% of cases, and TEE in 1.4% to 14.6% of cases.34,39,40 We recommend that CVCs and/​or TEE be used if the risk of perioperative RV dysfunction is high due to baseline RV dysfunction, a need for large volume resuscitation, or potential conditions that may precipitate pulmonary hypertension. PACs should be reserved for cases that may precipitate pulmonary hypertension. In some situations, it may be sufficient to have TEE on standby in case of instability. We have also had limited experience using intraoperative TTE for monitoring during NCS. Use of these advanced techniques may require consultation with cardiac-​trained anesthesiologists.

Monitored anesthesia care (MAC) can be performed safely in LVAD patients. Mathis et al.40 reported that over time, the trend was for more cases to be performed under MAC. When MAC is being considered, the baseline pulmonary function and airway should be carefully evaluated. Especially in patients with baseline RV dysfunction, hypoxia and hypoventilation could precipitate pulmonary hypertension and hemodynamic instability.18 In some cases, intubation may be warranted to better manage pulmonary function, although positive pressure ventilation may worsen pulmonary vascular resistance.18 For general anesthesia, the goals during induction and maintenance should be to maintain preload and RV function and to minimize swings in systemic pressures.18 A  balanced anesthetic technique is usually best to achieve these goals. Patients should be adequately volume resuscitated prior to induction.18 A  preinduction arterial line should be strongly considered, although is not necessary in all LVAD patients undergoing general anesthesia. With earlier LVAD designs, there was a concern for aspiration during induction due to the subdiaphragmatic location of the pump.18 However, current pumps are smaller and sometimes intrapericardial, and thus rapid sequence intubation is only necessary if otherwise indicated. Extubation criteria should not change for LVAD patients,18 although anesthesiologists should be especially careful to avoid hypoventilation and hypoxia postextubation. Nasopharyngeal airways should be avoided given the hypervascularity of the nasal mucosa with long term LVAD implantation.32

Managing hypotension and low LVAD flows

No prospective studies have determined ideal intraoperative blood pressure goals for LVAD patients undergoing NCS. However, intraoperative MAP 70 mm Hg and 7 ng/​L over 2 hours) may indicate reinfarction.3 Patients with cardiac troponin elevations are at high risk and benefit from intensive management and early revascularization. Creation kinase-​MB is much less sensitive for detection of myocardial injury than troponin, and substantially more tissue injury is required for its detection (Table 5.1). P E RCU TA N EO US C O RO NA RY I N T E RVE N T I O N

PCI is a nonsurgical technique for treating obstructive CAD including unstable angina, acute MI, and multivessel CAD. Clinical indications for PCI include the following: acute ST-​ elevation MI (STEMI), NSTE-​ACS, unstable angina, stable angina, angina equivalent (dyspnea, arrhythmia, or dizziness or syncope), or high-​risk stress test findings. In a patient who is asymptomatic or mildly symptomatic, evidence of moderate to severe ischemia on noninvasive testing is an indication for PCI. Angiographic indications include hemodynamically significant lesions in vessels serving viable myocardium with a vessel diameter greater than 1.5  mm. Clinical contraindications for PCI include intolerance of long-​term antiplatelet therapy or the presence of any comorbid conditions that severely limit life expect­ ancy. Relative angiographic contraindications include arteries less than 1.5 mm in diameter, diffusely diseased saphenous vein graft, or other coronary anatomy not amenable to PCI.4 Medical management is commonly recommended as first-​line therapy in patients with stable angina, unless 1 or more of the following indications for cardiac catheterization and PCI or coronary artery bypass grafting are present:  severe symptoms, a change in symptom severity, failed medical therapy, high-​risk coronary anatomy, or worsening left ventricular dysfunction. For patients with STEMI, immediate coronary angiography with PCI is recommended.5,6 For patients with NSTE-​ACS, the American College of Cardiology/​American Heart Association guidelines on the management of NSTE-​ACS recommend an early invasive strategy, with timing as follows: • Immediate (within 2 hours)—​Patients with refractory angina, recurrent angina after initial treatment, signs/​ symptoms of heart failure, new/​worsening mitral regurgitation, hemodynamic instability, sustained ventricular tachycardia, or ventricular fibrillation • Early (within 24 hours)—​None of the immediate characteristics, but new ST-​segment depression, a GRACE risk score >140, or temporal change in troponin

5. P reoperative C h est Pain in A Patient wit h C o l on O bstruction  •   45

Step 1

Urgent Surgery

Yes

Patient-or surgical specific factors dictate the strategy and do not allow further cardiac testing or treatment. The consultant provides recommendations on perioperative medical management, surveillance for cardiac events, and continuation of chronic cardiovascular medical theraphy.

Yes

Treatment options should be discussed in a multidisciplinary team, involving all perioperative care physicians as interventions might have implication on abaesthesiological and surgical care. For instance, in the presence of unstable angina, depending on the outcome of this discussion, patients can proceed for caronary artery intevention, with the initiation of dual anti-platelet therapy if the index surgical procedure can be delayed, or directly for operation if delay is impossible with optimal medical therapy.

No

Step 2

One of active or unstable cardiac conditions (table 9)

No

Step 3

Determine the risk of the surgical procedure (table 3)

The consultant can identify risk factors and provide recommendations on lifestyle and medical therapy, according to the ESC guidelines. In patients with one or more clinical risk factors, preoperative baseline ECG may be considered to monitor changes during the perioperative period.

Low

In patients with known IHD or myocardial ischaemia, initiation of a titrated low-dose beta-blocker regimen may be considered before surgery.a In patients with heart failure and systolic dysfunction, ACEI should be considered before surgery.

Intermediate or high

Step 4

Consider the functional capacity of the patient

>4 METs

In patients undergoing vascular surgery, initiation of statin therapy should be considered.

≤4 METs

Step 5

In patients with a poor functional capacity consider the risk of the surgical procedure

Intermediate risk surgery

In addition to suggestions above: In patients with one or more clinical risk factors, noninvasive stress testing may be considered.

High-risk surgery

Step 6

Cardiac risk factors (table 4)

In addition to suggestions above: Rest echocardiography and biomarkers may be considered for evaluation of LV function and obtaining prognostic information for perioperative and late cardiac events.

≤2

≥3

Step 7

Consider non-invasive testing. Noninvasive testing can also be considered prior to any surgical procedure for patient counselling, change of perioperative management in relation to type of surgery and anaesthesia technique.

No/mild/ moderate stress-induced ischaemia

Interpretation on non-invasive stress test results

Extensive stress-induced inschaemia

Balloon angioplasty: surgery can be performed >2 weeks after intervention with continuation of aspirin treatment.

Bare-metal stent surgery can be performed >4 weeks after intervention. Dual antiplatelet therapy shoud be continued for at least 4 weeks.

Proceed with the planned surgeryb.

Individualized perioperative management is recommended considering the potential benefit of the proposed surgical procedure compared with the predicted adverse outcome, and the effect of medical therapy and/or coronary revascularization.

Surgery can be performed within 12 months after intervention for old-generation DES and within 6 months for new-generation DES.

CABG

Continuation or discontinuation of aspirin in patients previously treated with aspirin may be considered in the perioperative period and should be based an individual decision that depends on the perioperative bleeding risk weighed against the risk of thrombotic complications (see also table 8). Surgery aTreatment should be initiated optimally between 30 days and at least 2 days before surgery and should be continued postoperatively aiming at target resting heart rate of 60–70 beats per minute and systolic blood pressure >100 mmHg. bFor strategy of anaesthesia and perioperative monitoring see appropriate sections. ACEI = angiotension converting enzyme inhibitor; CABG = coronary artery bypass graft; DES = drug-eluting stent; ECG - electrocardiogram; IHD = ischaemic heart disease; MET = metabolic equivalent.

Figure 5.1 

Algorithm for treatment for cardiac vessel occlusion.

Table 5.1  TROPONIN EVIDENCE Cardiac-​specific troponin (troponin I or T when a contemporary assay is used) levels should be measured at presentation and 3 to 6 hours after symptom onset in all patients who present with symptoms consistent with ACS to identify a rising and/​or falling pattern

Level of Evidence: A

Additional troponin levels should be obtained beyond 6 hours after symptom onset in patients with normal troponins on serial examination when electrocardiographic changes and/​ or clinical presentation confer an intermediate or high index of suspicion for ACS

Level of Evidence: A

If the time of symptom onset is ambiguous, the time of presentation should be considered the time of onset for assessing troponin values

Level of Evidence: A

With contemporary troponin assays, creatine kinase myocardial isoenzyme and myoglobin are not useful for diagnosis of ACS

Level of Evidence: A

Remeasure troponin once on day 3 or day 4 in patients with MI as an index of infarct size and dynamics of necrosis

Level of Evidence: B

Use of selected newer biomarkers, especially Level of Evidence: B B-​type natriuretic peptide, may be reasonable to provide additional prognostic information

or NSTEMI. Each of the following criteria constitutes 1 point for TIMI scoring:  age ≥65  years, 3 or more risk factors for CAD (family history of CAD, hypertension, hypercholesterolemia, diabetes mellitus, tobacco use), known CAD (stenosis >50%), aspirin use in the past 7 days, severe angina (≥2 episodes in 24 hours), ST deviation ≥0.5  mm, and elevated cardiac marker level23 (Table 5.2). The 2007 focused update of the ACC/​AHA/​Society for Cardiovascular Angiography and Interventions 2005 guideline update for PCI contains the following recommendations related to the selection of early invasive PCI versus conservative strategies. 1. An early invasive PCI strategy is indicated for patients with NSTE-​ACS who have no serious comorbidity and who have coronary lesions amenable to PCI and an elevated risk for clinical events. 2. An early invasive strategy (i.e., diagnostic angiography with intent to perform revascularization) is indicated in NSTE-​ACS patients who have refractory angina or hemodynamic or electric instability (without serious comorbidities or contraindications to such procedures). 3. In stabilized patients, an initially conservative or selectively invasive strategy may be considered as a treatment strategy for NSTE-​ACS patients (without serious comorbidities or contraindications to such procedures) who have an elevated risk for clinical events including those with abnormal troponin elevations.

Following a PCI procedure, patients must be placed on antithrombotic therapy and antiplatelet therapy. On the day of PCI, aspirin 162 to 325 mg is given to all patients. Unfractionated • Delayed invasive (within 25–​72 hours)—​None of the heparin, low-​molecular-​weight heparin, or bilirudin may be used immediate or early characteristics but renal insufficiency at the time of balloon angioplasty or PCI. Patients receiving (glomerular filtration rate 1 y may be reasonable

8. P ercutaneous C oronary I ntervention  •   75

Class IIb: discvontinuation after 6 mo may be reasonable

B. You need to be on dual antiplatelet therapy for 12 months. C. You need to be on dual antiplatelet therapy for the rest of your life. D. Proceed to surgery Answer: D Patients who receive a bare metal stent require dual antiplatelet therapy for 1  month. As seen from the algorithm from the AHA/​ACC guidelines, patients with bare metal stents may stop their dual antiplatelet therapy after 1 month and receive elective surgical procedures. 7. You are interviewing a 72-​year-​old male in the preoperative clinic for elective hip surgery. Seven months prior, he was found to have 2 vessel coronary artery stenosis while getting an emergent angiography for a ST-​elevation myocardial infarction (STEMI). The interventional cardiologists placed 2 drug-​eluting stents and started him on aspirin and clopidogrel. When you see him in clinic, he had stopped his Plavix but was still on aspirin. What do you tell him? A. You need to be on dual antiplatelet therapy for 6 months. B. You need to be on dual antiplatelet therapy for 12 months. C. You need to be on dual antiplatelet therapy for the rest of your life. D. Proceed to surgery Answer: B Patients who receive drug eluting stents in the setting of ACS require dual antiplatelet therapy for at least 12  months (see flow chart answer 6). 8. You are interviewing a 72-​year-​old male in the preoperative clinic for elective hip surgery. Seven months prior, he was found to have 2 vessel coronary artery stenosis while getting an emergent angiography for a STEMI. The interventional cardiologists placed 2 bare metal stents and started him on aspirin and clopidogrel. When you see him in clinic, he had stopped his Plavix but was still on aspirin. What do you tell him? A. You need to be on dual antiplatelet therapy for 6 months. B. You need to be on dual antiplatelet therapy for 12 months. D. You need to be on dual antiplatelet therapy for the rest of your life. D. Proceed to surgery Answer: B Patients with bare metal stents in the setting of stable ischemic heart disease can discontinue dual antiplatelet therapy after 30  days. However, patients who receive bare metal stents in the setting of ACS require dual antiplatelet therapy for at least 12 months (see flow chart in answer 6) just like drug-​eluting stents.

9. You are interviewing a 72-​year-​old male in the preoperative clinic for elective hip surgery. Seven months prior, he was found to have 2 vessel coronary artery stenosis while getting his routine cardiac clearance for hip surgery. The interventional cardiologists placed 2 drug-​eluting stents and started him on aspirin and clopidogrel. When you see him in clinic, he had stopped his clopidogrel but was still on aspirin. What do you tell him? A. You need to be on dual antiplatelet therapy for 6 months B. You need to be on dual antiplatelet therapy for 12 months C. You need to be on dual antiplatelet therapy for the rest of your life D. Proceed to surgery Answer: D Patients who receive drug-​eluting stents during a routine cardiac workup (stable ischemic heart disease) may stop their dual antiplatelet therapy after 6  months (see flow chart in answer 6). 10. You are interviewing a 72-​year-​old male in the preoperative clinic for a vascular surgery. Seven months prior, he was found to have 2 vessel coronary artery stenosis while getting an emergent angiography for a STEMI. The interventional cardiologists placed 2 drug-​ eluting stents and started him on aspirin and clopidogrel. The surgeons deemed this procedure would have a high risk of surgical bleeding and that the patient cannot wait the full year to proceed with the surgery. What recommendations do you tell the patient? A. Stop both the aspirin and clopidogrel and proceed to surgery. B. Stop the aspirin and continue the clopidogrel and proceed to surgery. C. Stop the clopidogrel but continue the aspirin and proceed to surgery. D. Tell the patient he cannot proceed to surgery for 5 more months. Answer: C In patients treated with dual antiplatelet therapy after drug-​ eluting stent implantation who must undergo the surgical procedure, the AHA/​ACC guidelines recommend discontinuing the clopidogrel prior to surgery and continuing the aspirin. After the surgery, the clopidogrel must be started as soon as possible.

QUESTIONS AND ANSWER S This chapter has accompanying questions and answers which are available to subscribers as part of the Oxford eLearning platform. To access the questions, go to http://​ oxfordmedicine.com/​anesthesiologyPBL

76  •   C ardiac f or t h e Genera l ist

REFERENCES

5. Collins LJ, et al. Cardioversion of nonrheumatic atrial fibrillation. Reduced thromboembolic complications with 4 weeks of precardioversion anticoagulation are related to atrial thrombus reso 1. Fleisher LA, Fleischmann KE, Auerbach AD, et al. 2014 ACC/​AHA lution. Circulation. 1995;92(2):160–163. guideline on perioperative cardiovascular evaluation and manage 6. Pollack CV, Reilly PA, Eikelboom J, et al. Idarucizumab for dabigatran ment of patients undergoing noncardiac surgery. J Am Coll Cardiol. reversal. N Engl J Med. 2015;373(6):511–​520. 2014;64(22):90–​99; 103–​105. 7. Benjamin EJ, Blahah MJ, Chiuve SE, et al. Heart disease and stroke 2. Hines RL, Marschall KE. Abnormalities of cardiac conduction and statistics—​2017 update. Circulation. 2017;135:146–​603. cardiac rhythm. In:  Watson KT, ed. Stoelting’s Anesthesia and Co-​ 8. Hines RL, Marschall KE. Ischemic heart disease. In:  Akhtar S, ed. existing Disease. Philadelphia: Elsevier Saunders; 2012:73–​103. Stoelting’s Anesthesia and Co-​existing Disease. Philadelphia:  Elsevier 3. The Atrial Fibrillation Follow-​ Up Investigation of Rhythm Saunders; 2012:1–​30. Management (AFFIRM) Investigators. A comparison of rate control 9. Levine GN, Bates ER, Bitti JA, et al. 2016 ACC/​AHA guideline foand rhythm control in patients with atrial fibrillation. N Engl J Med. cused update on duration of dual antiplatelet therapy in patients with 2002;347(23):1825–​1833. coronary artery disease. J Am Coll Cardiol. 2016;68(10):1082–​1104. 4. Van Gelder IC, Groenveld HS, Crijns HJGM, et al. Lenient versus strict rate control in patients with atrial fibrillation. N Engl J Med. 2010;362(15):1363–​1371.

8. P ercutaneous C oronary I ntervention  •   77

SECTION III VASCULAR

9. RUPTURED ABDOMINAL AORTIC ANEURYSM—​O PEN ABDOMINAL AORTIC ANEURYSM REVISITED Alexandra Bastien and Edwin Tavarez

severely hypotensive. The end-​tidal CO2 is not registering, wide complex QRS appear on the electrocardiogram, and the pulse oximeter appears to be malfunctioning. The anesthesiologist tells the blood bank to initiate the massive transfusion protocol.

S T E M C A S E A N D K EY Q U E S T I O N S Patient is a 75-​year-​old male who presents to the emergency room complaining of abdominal and left groin pain for 3 days. He has a past medical history of hypertension, chronic obstructive pulmonary disease, and peripheral vascular dis­ ease. The emergency room physician feels a pulsatile mass in the patient’s abdomen that he states has been getting larger. His blood pressure on arrival is 85/​63  mmHg; heart rate is 115 beats per minute. After getting a computed tomography (CAT) scan, he becomes unresponsive and is rushed to the operating room for an exploratory laparotomy. The diagnosis is a ruptured abdominal aortic aneurysm (AAA).

WH Y I S A N E N D O VA S CU L A R A P P ROAC H F I R S T AT T E M P T E D I N T H I S S ET T I N G O F K N OWN RU P T U R E D A A A ? WH AT A R E T H E C R IT E R I A I N VO LVE D I N E N D OVA S CU L A R A N EU RYS M R E PA I R VE R S US O P E N E M E RG E N T R E PA I R ? WH AT MO N ITO R I S I N D I C AT I VE O F C A R D I AC O U T P U T A N D C A R D I AC A R R E S T ? WH AT MO N ITO R S A R E T H E MO S T I M P O RTA N T MO N ITO R S YO U WO U L D C O NS I D E R I N T H I S PAT I E N T ?

WH AT I N F O R M AT I O N D O E S A C AT S C A N P RO V I D E? WH AT I S T H E M O S T C O M M O N L O C AT I O N F O R A A A RU P T U R E? WH AT I S T H E D E F I N I T I O N O F S H O C K ?

The surgeon cross-​clamps the aorta and hypertension is noted. Four units of O negative blood arrive. A  central venous The patient arrives to the operating room with a tense ab- catheter is inserted, a perfusionist arrives, and cell saver is domen and unresponsive. His vitals are blood pressure 70/​ set up. 45, heart rate 125, SpO2 86. He has labored breathing and is in pain. WH AT M A N EU VE R S A R E D O N E S U RG I C A L LY The blood bank is called and told to send type-​specific TO O BTA I N P ROX I M A L C O N T RO L ? WH AT I S blood. The response from the blood bank is that type-​specific T H E G OA L O F A N E S T H ET I C M A NAG E M E N T blood is not available. In the meantime, an 18 g intravenous F O R AO RT I C C RO S S - ​C L A M P I N G I N T H I S (IV) line is placed bilaterally placed in the antecubital region. S ET T I N G ? WH AT I S I N VO LV E D I N A M A S S I V E Fluid resuscitation and vasopressors are given. The patient is T R A NS F US I O N P ROTO C O L ? placed on 100% oxygen and an arterial line is being attempted. During cross-​clamping, an arterial blood gas result indicates the following:  pH 7.21; PCO2 36; PO2 136; base excess  –​6 WH AT I S T Y P E -​S P E C I F I C B L O O D ? WH AT and potassium of 7, hematocrit 28, Na 130, and calcium 3.1. A R E T H E O P T I O NS F O R M A S S I V E B L O O D The anesthesiologist increases minute ventilation and begins T R A N S F US I O N I N A PAT I E N T WH O I S N OT treatment for hyperkalemia. C RO S S -​M ATC H E D F O R B L O O D T R A NS F US I O N ? WH AT T Y P E O F AC C E S S S H O U L D B E AT T E M P T E D A N D WH Y ? H OW FA S T C A N A N 18 G I V T R A N S F US E? H OW L OW WO U L D YO U L ET T H E B L O O D P R E S S U R E G O ? WH AT I S P E R M I S S I VE H Y P OT E N S I O N ?

H OW D O YO U T R E AT ACU T E H Y P E R K A L E M I A ? WH AT A R E T H E OT H E R FAC TO R S I N FLU E N C I N G T H E H Y P E R K A L E M I A E F FE C TS O N T H E H E A RT ?

The surgeon decides to perform an emergent endovascular AAA approach. The patient’s vital signs continue to deteriorate, and an emergent laparotomy is done. Upon incision, massive hemorrhaging is noted, and the patient becomes

The patient’s temperature is 31.6°C. Urine output is 20 cc for an hour and a half. A transesophageal echo is placed that shows low filling noted with left ventricular wall hypokinesis in a short transgastric view. 81

WH AT A R E T H E C L I N I C A L I M P L I C AT I O NS O F H Y P OT H E R M I A ? WH AT T H E M A J O R C A R D I AC C H A N G E S A R E D U R I N G C RO S S -​ C L A M P I N G ? WH AT A N E S T H ET I C M A NAG E M E N T I S H E L P F U L TO I M P RO V E R E NA L P E R F US I O N D U R I N G C RO S S -​ C L A M P I N G ? WH AT OT H E R O RG A N S YS T E M A R E E F F EC T E D D U R I N G C RO S S -​C L A M P I N G ?

After an hour of cross-​clamping, no urine is noted, and the anesthesiologist notices that very little volatile agent has been used secondary to the initial hypotension. The surgeon notifies you that he will begin serial unclamping of the aortic to help with renal perfusion. WH AT I S T H E I N C I D E N C E O F R E C A L L U N D E R A N E S T H E S I A ? WH AT H E MO DY NA M I C C H A N G E S A R E S E E N A F T E R U N C L A M P I N G O F T H E AO RTA ? H OW WO U L D YO U M A NAG E F LU I D A N D B L O O D P RO D U C TS P R I O R TO U N C L A M P I N G T H E AO RTA ?

During the surgery, the lung mechanics appear to have changed requiring higher peak pressures for ventilation. Blood gases return showing severe hypocalcemia. H OW D O YO U WO R K U P A T R A N S F US I O N-​ R E L AT E D AC U T E LU N G I N J U RY ( T R A L I )? WH AT I S C I T R AT E TOX I C I T Y, A N D H OW I S IT T R E AT E D ?

The surgeon begins to close the abdomen, and protamine has been started. He notices poor clotting. The urine is noted to be blood tinged. WH AT I S T H E D I F F E R E N T I A L F O R C OAGU L O PAT H Y I N T H I S S ET T I N G ? WH AT I N F O R M AT I O N C A N A T H RO M B O E L A S TO G R A P H Y ( T EG) P ROVI D E? WH AT A R E T H E C O N C E R N S WI T H P ROTA M I N E I N F US I O N ?

The aortic cross-​ clamp time has exceeded 2 hours. The patient’s abdomen is closed, and the patient is transferred to the intensive care unit. Within 48 hours, the nurses caring for the patient notice a marked tone difference between upper and lower extremities. WH AT I S YO U R D I F F E R E N T I A L ? T R E AT M E N T ? O U TC O M E E X P E C T E D F O R T H I S D I AG N O S I S ?

Differential diagnosis for ruptured AAA include GI bleed, Ischemic Bowel,Perforated gastric ulcer, Nephrolithiasis, and myocardial infarction. Treatment depend on acute shock, and wether the patient can receive imaging to attempt endovascular repair as apposed to open repair. Ruptured abdominal aneurysm have a mortality rate of 90%.

DISCUSSION AAA is an enlargement of the aorta where the diameter is greater than 3  cm or more. Patients usually exhibit no symptoms except when they rupture. It is the 15th leading cause of death overall in the United States. The 10th leading cause of death in men over 55 years old. It results in over 8,800 hospital deaths annually. From 30% to 40% of patients with ruptured aortic aneurysms die before surgery. It carries with it a 40% to 50% operative mortality rate, and an overall mortality rate of 80% to 90%. The most common location of an AAA is infrarenal, and rupture occurs most often in the retroperitoneal cavity. The rupture rate is universally accepted as related to size.3 C L I N I C A L P R E S E N TAT I O N

Most aneurysms of the aorta are asymptomatic by the time of rupture. As many as 65% of patients with ruptured AAAs die of sudden cardiovascular collapse before arriving at a hospital. Patients at risk for AAA are usually 65 years and older. They usually have a history of hypertension, peripheral vascular disease, chronic obstructive pulmonary disease, and smoking. Genetic conditions associated with AAA include Marfans and Ehlers-​Danlos syndromes. At-​risk patients may benefit from routine screening. Abdominal ultrasound and CT serially follow for size changes. As previously mentioned, the risk of rupture increases with increasing size, such that elective repair is planned for aneurysms measuring 4 cm or greater. Patients with ruptured AAA can present with acute abdominal pain, as well as back or groin pain. Isolated groin pain is associated with retroperitoneal expansion and pressure on either the right or left femoral nerve. Patients can also present with hypotension, tachycardia, and altered mental status. The abrupt onset of pain due to rupture of an AAA may be quite dramatic, but due to retroperitoneal containment, the patient may have normal vital signs. The presence of a pulsatile abdominal mass is diagnostic of a AAA, but only found in 50% of cases.1,2 Patients with a ruptured AAA present with varying degrees of shock due to the location of rupture and the possibility of tamponade. Shock is defined as a state of cellular and tissue hypoxia due to reduced oxygen delivery and/​or increased oxygen consumption or inadequate oxygen utilization. There are 3 stages of shock are as follows: Stage 1 (also called compensated) is the stage where changes in hemodynamics such as low blood pressure will result in tachycardia and constriction of blood vessels to maintain adequate blood flow to vital organs. The kidney will also try to retain fluid to maximize blood volume. Stage 2: This stage begins when the aforementioned compensations begin to fail and the body can no longer compensate to improve blood flow. Stage 3: Complete shut down, and you begin to see multiple organ system failure.

82  •   Vascu l ar

Four types of shock are recognized:  distributive, cardiogenic, hypovolemic, and obstructive. The following 2 types are most relevant during the management of AAA rupture. Hypovolemic shock is due to reduced intravascular volume (i.e., reduced preload), which, in turn, reduces cardiac output. Hypovolemic shock can be divided into 2 categories—​hemorrhagic and nonhemorrhagic.

hemodynamic data may be difficult to obtain. Indirect blood pressure monitoring may be unreliable, and the insertion of an arterial line is essential for beat to beat readings. Central line venous access can be used to monitor central venous pressure and give an indication of preload. However in hemorrhagic shock, these readings may not be as accurate secondary to fast changes in preload, afterload, and stroke volume

Hemorrhagic shock is the presentation of most ruptured AAA in stage 3 with the highest mortality rate.4

6. Foley urometer catheter for 30-​minute assessment of renal perfusion and evidence of hemoglobinuria.

S U RG I C A L M A NAG E M E N T O F RU P T U R E D A A A

The most common sites of rupture of AAA are in the retroperitoneal cavity (80%), intraperitoneal (20%), and aortocaval fistula (3%–​4%).3 If a patient arrives with the diagnosis of ruptured AAA, immediate surgical intervention is done if the patient has severe hemodynamic instability. A  perioperative CT is done if there is time, which allows location of the aneurysm and endovascular repair options. Recently, endovascular aneurysm repair has been successfully used to treat ruptured AAA. Recent studies have shown that emergency endovascular repair of ruptured AAA versus open repair has demonstrated a threefold reduction of perioperative mortality. In the scenario of unstable ruptured AAA, proximal control is the main goal after laparotomy. This can be achieved by occluding the aorta at the infra renal supra renal, supraceliac, and thoracic levels depending on the suspected level of hemorrhage. A N E S T H ET I C M A NAG E M E N T O F RU P T U R E D A B D O M I NA L A N EU RYS M

When the perioperative arena is notified of the arrival of a patient with a potential ruptured aortic aneurysm in the emergency room, the blood bank is alerted of potential massive transfusion protocol activation, and the specimen is verified as received. The perfusionist is activated for set up of acute normovolemic hemodilution. The anesthesiology set up includes: 1. Routine American Society for Anesthesiology monitors, airway devices for possible difficult airway. 2. Large bore IV (French lines with rapid infuser). In situations where massive blood transfusion is needed and blood must be given quickly, it is important to have large bore access to prevent hemolysis of red blood cells; 18 g angiocaths have a maximal flow rate of 100 cc/​min (Table 9.1). 3. Perfusionist with acute normovolemic hemodilution prepared. 4. Dedicated personnel for checking blood products and giving them on direction. 5. Arterial line (arterial blood gas), central line, transesophageal echocardiogram. In a ruptured AAA,

7. Viscoelastic essays (thromboelastogram, activated clotting time), if available, and point of care to help with rapid assessment of coagulation status. International normalized ratio (INR), prothrombin time (PT), partial thromboplastin time (PTT), and fibrinogen may be outdated by the time results return. 8. Cerebral oximetery monitors to indicating areas of global hypoxemia. Anesthesia is usually induced with cardiac supportive agents, avoiding inhalational or other depressant anesthetics. Note that muscle relaxants can further reduce the tamponade effect of the abdomen, in addition to anesthetic reduction of sympathetic tone. Maintenance with a balanced technique using volatile agents/​opioids and neuromuscular blockade is suggested. High-​dose opioids (e.g., fentanyl 5–​20 µg kg−1, 1–​5 ug/​kg of sufentanil) are often used. When treating a patient in an acute trauma situation, it is possible to have recall because of the major concern is hemodynamics. We should consider administering amnestic drugs such as midazolam that may help prevent recall but causes little changes in blood pressure. H E MO DY NA M I C S

The goals for the initial management of a ruptured AAA is to restore intravascular volume, maintain adequate oxygenation of vital organs, and limit ongoing blood loss. The patient is immediately assessed for evidence of shock. Shock occurs as a result of cellular hypoxia from reduced tissue perfusion, oxygen delivery, and/​or inadequate oxygen utilization. This situation causes cell membrane dysfunction, intracellular edema, and leakage of contents. Subsequently, this leads to acidosis, impairing regional blood flow and causing a rise in serum lactate levels. The major physiologic determinants of tissue perfusion are blood pressure, oxygen carrying capacity (hemoglobin), and cellular conditions. The initial management of ruptured aneurysm patients may require permissive hypotension. This technique allows resuscitation to maintain vital organ perfusion without overshooting the blood pressure until aortic cross-​clamping. This technique is done to reduce hemorrhage effect. In hemodynamically unstable patients with ruptured AAA, indirect evidence from the trauma population and 1 observational study in patients with AAA suggest that allowing a relatively low systolic blood

9. Ruptured A bdomina l Aortic A neurysm  •   83

pressure of 80 to 100  mmHg (permissive hypotension) may prevent further tearing of the aorta and limit blood loss.3 Vasoconstrictors further reduce organ perfusion and, in general, should be avoided with volume repletion attempts if possible. Large volumes of IV fluids may be required rapidly; therefore, warmed transfusion tubing should be primed with fluids. It should be noted that excessive infusion of crystalloids has been associated with worse outcomes in patients with severe hemorrhage.4 When the aorta is clamped, we can see a dramatic increase in blood pressure. These changes can be treated with anesthetic gas or vasodilatory drugs such as nitroglycerin. AO RT I C C RO S S -​C L A M P I N G

The physiological response to aortic cross-​clamping depends on a number of variables, including preoperative left ventricular function, collateral circulation, and the level of the cross-​ clamp (Fig. 9.1). Once the aorta is cross-​clamped, increased afterload may cause hypertension proximal to the clamp. The greatest level of afterload increase is directly related to the altitude of aortic occlusion. Increased afterload and hypertension may be attenuated by increasing the depth of anesthesia or the administration of vasodilators. Intravascular fluid loading, transfusion protocols, and anesthetic agents can be administered if tolerated. On echocardiography, one can see segmental wall motion abnormalities from ventricular wall tension. Increase in systolic pressure and systemic vascular resistance are the most consistent findings while cardiac output results can be variable (Fig. 9.2).8 The differences in responses seen at different levels of cross clamping can be attributed to the ability to redistribute blood to splanchnic and nonsplanchnic vasculature. Unclamping of the aorta is associated with substantial decrease in vascular resistance and arterial blood pressure. Left ventricular end diastolic pressures decrease while cardiac Coeliac trunk Hepatic artery

Splenic artery

Right renal artery

Left renal artery

Superior mesenteric artery

Gonadal arteries Inferior mesenteric artery

Figure 9.1 

Aortic cross-​clamping.

output may be variable due to previous aorto-​occlusive disease, volume status, and acid base conditions. Reactive hyperemia is expected after period of clamping, owing to a decrease in vasomotor tone from cellular hypoxia below the clamp, central hypovolemia from blood sequestration, and active metabolites returning to central myocardial perfusion. These physiologic mechanisms can all contribute to the hypovolemia seen. These mechanisms may be attenuated by maintaining mean arterial pressure and expanding the circulating volume, facilitated by administration of vasodilators during cross-​clamp application. Vasoconstrictors and/​or inotropic drugs may usually be required. Surgeons will often release the clamps in stages to allow for adequate resuscitation.8 T R A NS F US I O N

In a hemodynamically unstable patient, proximal aortic control is the surgical goal, so delay in waiting for type-​specific blood is unnecessary. Type-​ specific blood occurs when a patient’s blood type is cross-​matched with specific antigens in his or her red blood cells. In emergent situations where there is no time for blood cross-​matching O Rh negative blood can be given in emergency situations. Activating a massive transfusion protocol to maximize oxygen-​ carrying capacity and minimize hypovolemic and dilutional coagulopathy to maintain vital organ perfusion is the anesthetic goal. Transfusion ratios of blood to platelets to fresh frozen plasma can be as high as 1:1:1. This ratio has been shown in massive trauma hemorrhage scenarios to improve survival. During the procedure, attention to ongoing blood loss is necessary through observation of sponges, suction canister contents, and possible hidden loss beneath surgical drapes. Surgical field assessments also alert the anesthesiologist for evidence of coagulopathy. A  restrictive transfusion strategy should be adopted once hemorrhage is controlled to minimize unnecessary exposure to blood. Hemoglobin less than 8 g/​dl and hematocrit values less than 25% are typically reported as restrictive transfusion strategy.7 Intraoperative red blood cell recovery methods can help remove activated cytokines, anaphylatoxins, and other waste substances that may have been collected in the reservoir suctioned from the surgical field. These devices offer only red cells for reinfusion; therefore, the patient still remains at risk for coagulopathy. In ruptured AAAs, these devices reduce autologous transfusion and are available in the operating room for quick salvage techniques. Citrate is contained in the collecting bags of blood (sodium citrate and citric acid) and is converted upon transfusion to bicarbonate. Metabolic alkalosis can result because of this. In turn, the resulting alkalosis can occur if renal ischemia or underlying renal disease prevents excretion of bicarbonate load. There is 23 meq of bicarbonate for each unit of blood causing shifting of potassium into cells in exchange for hydrogen ions. Citrate also binds to ionized calcium in the blood recipient and can lead to hypocalcemia. For patients with preexisting liver disease or ischemic hepatic dysfunction, citrate

84  •   Vascu l ar

Figure 9.2 

Progressive EKG changes in hyperkalemia.

induced hypocalcemia can be catastrophic and monitoring during transfusion imperative.5,7 TRALI

During massive blood transfusion, adverse reactions such as ABO antigen incompatibility may occur when hemoglobinuria or microvascular bleeding occurs. Lung injury occurring within 6 hours of transfusion should alert critical care providers to transfusion-​related lung injury. This phenomenon occurs in 1:5,000 transfusions. It is manifested as circulatory overload, hypoxemia, increased peak airway pressures, and acute respiratory distress. TRALI is now the leading cause of transfusion-​ related fatalities. Donor antibodies in the plasma react with antigen on the patient’s granulocytes.7,9 AC I D -​BA S E D I S T U R BA N C E

Metabolic acidosis and an increase in lactate occur. Blood is drained from ischemic tissues, causing reinjury to cells with electrolyte and pH disturbances. The degree of acidosis is reduced in patients with preexisting aorto-​occlusive disease. This occurs secondary to preexisting collateral circulation.6 HYPERKALEMIA

When long-​standing stored, packed red blood cells (PRBC) infuse the central circulation at high concentrations, hyperkalemia may develop. Hyperkalemia is demonstrated by peaked T waves, widening QRS, widening T waves with

a biphasic tracing, ventricular fibrillation, arrhythmias, and asystole. Rapid changes in this electrolyte are immediately treated with calcium, glucose, and insulin to stabilize hyperexcitable cells such as those in the myocardium. In addition to raising the pH, treating hyponatremia and hypocalcemia, as well as maintaining renal perfusion, will help maintain stability.6 R E NA L P E R F US I O N

Patients are at risk of developing renal impairment secondary to preoperative hypotension and hypovolemia. Renal hypoperfusion occurring during aortic clamping involves activation of the renin angiotensin system. Aortic clamping compromises the kidneys further by causing direct renal ischemia, possible large embolic load, and postoperative blood loss. To avoid postoperative renal impairment, every effort should be made to maintain adequate perfusion pressure and limit the duration of suprarenal clamping. Many anesthesiologists administer drugs such as mannitol, furosemide, or dopamine to prevent renal failure, but there is no convincing evidence that they improve outcome. The priority is to maintain an adequate extracellular fluid volume intraoperatively and postoperatively.6 H Y P OT H E R M I A

Environmental exposure, initial resuscitation with cold fluid, and deranged thermoregulation result in hypothermia. Progressive hypothermia and persistent metabolic acidosis may

9. Ruptured A bdomina l Aortic A neurysm  •   85

Table 9.1  INFUSION FLOW RATES FOR IV CATHETERS INFUSION RATE FOR 1 LITER

CATHETER SIZE

20 g

IV catheter

6.5 min

18 g

IV catheter

4.5 min

16 g

IV catheter

2.5 min

14 g

IV catheter

1.5 min

14 g

Triple lumen port

5.5 min

6 French

Sheath introducer

2.1 min

7 French

RIC line

1 min

Maximum flow rates of catheters (Emergency medicine updates November 2009 Rueben et al: Vascular access. International Journal of Critical Illness and Injury Science Vol.2 Issue 3, Sep-Dec 2012) IV = intravenous; g = gauge; RIC = rapid infusion catheter.

lead to end-​stage shock and death. Hypothermia reduces the enzymatic activity of plasma proteins. At core temperatures of 34° or below, platelet activation is prevented. This has a profound effect on platelet-​mediated primary hemostasis. Rapid transfusion of multiple units of blood products, which are chilled, can lead to cardiac arrhythmias. The open abdomen and operating room temperature with the rapid PRBC and crystalloid infusion with high flow of gases combine to create a greater than 3° to 4° drop in temperature. Six units of PRBCs at 4°C will reduce the body temperature of a 70 kg adult by 1°C. Therefore, high-​capacity fluid and blood warmers, warming convection devices, and thermovents, as well as warming the operating room, should all be employed to avert hypothermia.4 C OAGU L O PAT H Y

Aggressive fluid resuscitation can cause hemodilution, with decreased oxygen carrying capacity, decreased oxygen delivery, and dilution of substrate and enzymatic coagulation factors. Progressive coagulopathy leads to further hemorrhage and shock. There is a 10% decrease in the concentration of clotting proteins for each 500 mL of blood loss that is replaced. When individual coagulation proteins fall to 25% of normal, bleeding based solely on dilution can occur. Between 8 and 10 units of red cells are needed to replete the clotting proteins. Dilutional thrombocytopenia is seen with massive blood transfusions. Between 10 and 20 units of PRBC are associated with a 50% fall in platelet count. Six units of whole blood derived platelets or the equivalent of 1 apheresis concentrate can increase the platelet count by 30,000. Patients with coagulopathy should have INR, PT, PTT fibrinogen, and platelet studies sent. Viscoelastic essays such as TEG and rotational institutional thromboelastometry may not be available at all institutions, so large volumes of plasma are required to correct coagulopathy. Early administration of plasma is more efficacious. TEG reliably demonstrates both hypercoagulability and fibrinolysis,

both of which are frequently underestimated with conventional coagulation tests. Standard coagulation tests such as PT, PTT, INR, platelet count, and fibrinogen usually require 30 to 60 minutes for results to be available. For massive transfusion patients, point-​of-​care testing with thromboelastography, thromboelastometry, and sonoclot would yield results beneficial in acute interventions. Measurement of platelet count and fibrinogen only provide absolute amounts, not functional activity and may overestimate the levels. Prothrombin complex concentrates (PCC) are available for treating excessive bleeding. The 4-​factor prothrombin complex concentrates (factor II, VII, IX, and X) have been shown to reduce blood loss and normalize INR values. When traditional options for treating excessive bleeding due to coagulopathy have been exhausted consider administering single factor prothrombin complex concentrates such as recombinant-​activated factor VII.4,5 Hypofibrinogenemia 2 mg/​dL.3 This patient has a history of ischemic heart disease, cerebrovascular disease, and preoperative treatment with insulin. This places him at a ≥ 11% risk for a MACE perioperatively. Note that this surgery does not classify as a high-​risk surgery as femoropopliteal bypass is an infrainguinal procedure. WH AT P R EO P E R AT I VE C A R D I AC S T U D I E S A R E I N D I C AT E D I N T H I S PAT I E N T ?

According to the 2014 American College of Cardiologists (ACC)/​ American Heart Association (AHA) guidelines, this patient has an elevated risk per RCRI and a poor functional capacity at less than 4 METs; therefore, preoperative testing depends on whether the results will impact decision-​ making or perioperative care.2 If so, and if he patient is willing to undergo coronary artery bypass grafting or percutaneous coronary intervention, it is reasonable to pursue pharmacologic stress testing. In addition, it would be worthwhile to obtain a preoperative resting electrocardiogram in this patient. Preoperative electrocardiogram is indicated in patients with known CAD, significant arrhythmias, PAD, cerebrovascular disease, or other significant structural heart disease, except for those undergoing low-​risk surgery.4

WH AT A R E T H E P R EO P E R AT I V E C O N C E R NS F O R T H I S PAT I E N T ?

Patients presenting for vascular surgery often have concurrent systemic diseases. This patient has generalized vascular disease that has not only affected his peripheral vessels but also those of his cardiac vessels. He has several risk factors for peripheral arterial disease (PAD) including DM and a positive smoking history. In fact, more than 50% of patients presenting for vascular surgery have advanced CAD and 25% of vascular surgery patients have unrecognized myocardial infarction (MI) that significantly affect long-​term morbidity and mortality.1 Furthermore, functional capacity is an important predictor of perioperative and long-​term cardiac events. This patient has poor functional capacity with = 0.2 ng/​mL. Of these patients, the 30-​day mortality rate was 1.9%.11 Thus, there is a significant interest in maintaining myocardial supply-​demand to prevent myocardial injury. This can be controlled with maintaining adequate hemodynamic parameters, with vasopressors or anti-​hypertensives if required. After vascular surgery, unlike coronary artery bypass grafting, the procedure has not corrected their

92  •   Vascu l ar

coronary disease. Yet, these patients experience the stresses of the postoperative period. There is evidence that heart rates of approximately 85 beats per minute can trigger ischemic ST changes. It is important to identify and treat the underlying cause of these hemodynamic perturbations.6,26 A less common mechanism for perioperative MI is plaque rupture. In patients with chest pain and/​or ST elevation with evidence of stent thrombosis requires emergent interventional cardiology consult for percutaneous coronary angiography in the cardiac catheterization laboratory. Importantly, thrombolytic therapy is contraindicated for management of MI as they are contraindicated in patients with recent surgery. Furthermore, achieving adequate fluid balance, avoiding severe anemia, and maintaining oxygenation is of utmost importance. Pain and anxiety are important factors to control as well to avoid stress response changes in myocardial supply-​ demand balance. Pain can be addressed with IV opioids, patient controlled analgesia, or epidural local anesthetics/​ opioids if applicable. R E VI EW Q U E S T I O N S 1. Which of the following is NOT part of the RCRI? A. Congestive heart failure B. Preoperative serum creatinine of 2.3 mg/​dl C. Prior stroke with no residual D. History of prior vascular surgery E. Coronary stent placed 5 years ago Answer: D RCRI has 6 components to estimate the risk of major adverse cardiac events (MACE) after noncardiac surgery. They include high-risk surgery (intrathoracic, intraperitoneal, or suprainguinal vascular surgery), history of ischemic heart disease, history of congestive heart failure, history of cerebrovascular disease, preoperative treatment with insulin, and preoperative creatinine >2 mg/dL. 2. Which of these symptoms are NOT expected in a patient with lower extremity claudication? A. Chronic  ulcers B. Cool extremity C. Pain with movement and relieved with rest D. Pain with movement and relieved by leaning forward E. Diminished pulses in an extremity Answer: D Pain that is relieved with rest, diminished pulses, chronic ulcers, and a cool extremity are all hallmarks of claudication. On the other hand, neurogenic claudication caused by lumbar spinal stenosis can be described as pain with movement that is relieved by leaning forward. 3. Which patient would benefit from intra-​arterial continuous blood pressure measurement? A. A patient with CAD

B. A patient that had a full breakfast the day of surgery C. A  patient that needs frequent intraoperative blood samples D. A patient that took warfarin the day of surgery E. A patient that has COPD Answer: C Indications for arterial line insertion include the need for continuous blood pressure management in surgeries that can have large fluid shifts. Furthermore, cases where the rapid detection of blood pressure shifts is important or if the use of vasopressors are projected are other indications for the this monitor. Also, in the case of this patient who would need frequent blood draws, an arterial line would be helpful. Finally, a patient in which an accurate noninvasive blood pressure cannot be obtained (obesity, LVAD, etc) are also indications. 4. Which of the following situations would ideal for neuraxial anesthesia for lower extremity bypass surgery? A. Surgery at a location where neuraxial anesthesia is rarely used B. A  patient that took his clopidogrel on the day of surgery C. Surgery that would require that patient to be lateral for several hours D. A patient with dementia E. A  cooperative patient who has not had any recent anticoagulation Answer: E There are several factors that are important to consider when determining whether a patient is a good candidate for neuraxial anesthesia. One of the most important is to determine if the patient has had any recent anticoagulation that may preclude from a safe procedure. Furthermore, operator skill and familiarity is important as is the comfort of the patient to remain on the operating table for several hours must be considered. 5. You are called to the bedside in postanesthesia care unit for a patient that just had a femoropopliteal bypass surgery. He is complaining of extreme leg pain and that he cannot move his foot. After calling for help and notifying the surgical team, what is your next step of management? A. Check compartment pressures B. Open the dressings C. Administer additional pain medications D. Perform surgical decompression E. Ask for a Doppler to check for pulses Answer: B In this scenario, we are concerned about not enough distal blood flow to the leg. The first thing to attempt is to relieve the pressure from a bandages that may be too tight. After this, further evaluation for compartment syndrome can be undertaken. 6. Which of the following is NOT a cause of PAD? A. Takayasu’s arteritis

10. Lower E xtremity C l audication  •   93

B. Thromboangiitis obliterans C. Radiation arteritis D. Fibromuscular dysplasia E. Homocystenemia Answer: A The most common cause of PAD is atherosclerosis. Other less common causes of PAD include embolism, thromboangiitis obliterans, radiation arteritis, fibromuscular dysplasia, and homocysteinemia. 7. Which of the following is considering high risk surgery per RCRI criteria? A. Open reduction and internal fixation of humerus fracture B. Mandibulectomy C. Tracheostomy D. Lower extremity bypass surgery E. Esophagectomy Answer: E High risk surgery per the RCRI include intrathoracic, intraperitoneal, or suprainguinal vascular surgery.

QUESTIONS AND ANSWER S This chapter has accompanying questions and answers which are available to subscribers as part of the Oxford eLearning platform. To access the questions, go to http://​ oxfordmedicine.com/​anesthesiologyPBL REFERENCES 1. Feringa HH, Karagiannis SE, Vidakovic R, et al. The prevalence and prognosis of unrecognized myocardial infarction and silent myocardial ischemia in patients undergoing major vascular surgery. Coron Artery Dis. 2007;18:571–​576. 2. Fleisher K, Fleishmann K, Auerbach A, et  al. 2014 ACC/​AHA guideline on perioperative cardiovascular evaluation and management of patients undergoing noncardiac surgery. J Am Col Cardiol. 2014;64(22):e77–​e137. 3. Botto F, Alonso-​Coello P, Chan MTV, et al. Myocardial injury after noncardiac surgery:  a large, international, prospective cohort study establishing diagnostic criteria, characteristics, predictors, and 30-​day outcomes. Anesthesiology. 2014;120:564–​578. 4. Jeger RV, Probst C, Arsenic R, et al. Long-​term prognostic value of the preoperative 12-​lead electrocardiogram before major noncardiac surgery in coronary artery disease. Am Heart J. 2006;151:508–​513. 5. American Society of Anesthesiologists. Standards for Basic Anesthetic Monitoring. Park Ridge, IL:  American Society of Anesthesiologists; 2015. 6. Miller R, Cohen N. Miller’s Anesthesia. 8th ed. Philadelphia: Elsevier Saunders; 2015. 7. Rosenfeld BA, Beattie C, Christopherson R, et  al. The effects of different anesthetic regimens on fibrinolysis and the

development of postoperative arterial thrombosis. Anesthesiology. 1993;79(3):435–​443. 8. Tuman KJ, McCarthy RJ, March RJ, Delaria GA, Patel RV, Ivankovich AD. Effects of epidural anesthesia and analgesia on coagulation and outcome after major vascular surgery. Anesth Analg. 1991;73(6):696–​704. 9. Horlocker TT, Wedel DJ, Rowlingson JC, Enneking FK. Executive summary:  regional anesthesia in the patient receiving antithrombotic or thrombolytic therapy. Reg Anesth Pain Med. 2010;35(1):102–​105. 10. Twine CP, Mclain AD. Graft type for femoro-​popliteal bypass surgery. Cochrane Database Syst Rev. 2010;2:CD00148. doi:10.1002/​ 14651858.cd001487.pub2 11. Devereaux PJ, Chan MT, Alonso-​Coello P, et al. Association between postoperative troponin levels and 30-​day mortality among patients undergoing noncardiac surgery. JAMA. 2012;307:2295–​2304. 12. Garner MR, Taylor SA, Gausden E, et  al. Compartment syndrome: diagnosis, management, and unique concerns in the twenty-​ first century. HSS J. 2014;10(2):143–​152. 13. Bah F, Bhimji S. Peripheral arterial disease. In: StatPearls [Internet]. Treasure Island, FL: StatPearls Publishing; 2017. https://​www.ncbi. nlm.nih.gov/​books/​NBK430745/​ 14. Patel S, Surowiec S. Intermittent Claudication. In:  StatPearls [Internet]. Treasure Island, FL: StatPearls Publishing; 2017. https://​ www.ncbi.nlm.nih.gov/​books/​NBK430778/​ 15. Hirsch AT, Haskal ZJ, Hertzer NR, et al. ACC/​AHA guidelines for the management of patients with peripheral arterial disease (lower extremity, renal, mesenteric, and abdominal aortic). J Vasc Interven Radiol. 2006;17(9):1383–​1398. 16. Schroeder T. The TASC supplement—​ international recommendations for management of peripheral arterial disease. Eur J Vasc Endovasc Surg. 2000;19(6):563. 17. Aquino R, Johnnides C, Makaroun M, et al. Natural history of claudication: long-​term serial follow-​up study of 1244 claudicants. J Vasc Surg. 2001;34(6):962–​970. 18. Parker SD, Brewlow MJ, Frank SM. Catecholamine and cortisol responses to lower extremity revascularization, correlation with outcome variables. J Cardiothorac Vasc Anesth. 1996;10(5):688. 19. Liu SS. Neuraxial anesthesia and analgesia in presence of standard heparin. Reg Anesth Pain Med. 28:172–​197. 20. Pierce E, Pomposelli FB, Stanley GD. Anesthesia type does not influence early graft patency or limb salvage rates of lower extremity arterial bypass. J Cardiothorac Vasc Anesth. 1997;11(6):803. 21. Cook PT, Davies MJ, Cronin KD. A prospective randomized control trial comparing spinal anaesthesia using hyperbaric cinchicaine with general anaesthesia for lower limb vascular surgery. Anaesth Intensive Care 1986;14:373–​380. 22. Wiis JT, Jensen-​Gadegaard P, Altintis Ü, et al. One-​week postoperative patency of lower extremity in situ bypass graft comparing epidural and general anesthesia: retrospective study of 822 patients. Ann Vasc Surg. 2014;28(2):295–​300. 23. Ghanami RJ, Hurie J, Andrews JS, et al. Anesthesia-​based evaluation of outcomes of lower-​extremity vascular bypass procedures. Ann Vasc Surg. 2013;27(2):199–​207. 24. Sessler, D. Perioperative thermoregulation and heat balance. Lancet. 2016;387:2655–​2664. 25. Rajagopalan S, Mascha E, Na J, Sessler DI. The effects of mild perioperative hypothermia on blood loss and transfusion requirement: a meta-​analysis. Anesthesiology. 2008;108:71–​77. 26. Frank SM, Beattie C, Christopherson R, et  al. Perioperative rate-​ related silent myocardial ischemia and postoperative death. J Clin Anesth. 1990;2:326–​331.

94  •   Vascu l ar

11. TEVAR—​T HORACIC ENDOVASCULAR SURGERY—​ CEREBRAL SPINAL FLUID DRAINAGE CATHETER S Oran Kremen and Kumar Vivek

S T E M C A S E A N D K EY Q U E S T I O N S

WH AT A R E T H E S I G NS A N D SY M P TO M S O F A N ACU T E D I S S EC T I O N ? S H O U L D T H E C A S E B E C A N C E L E D I N T H E EV E N T O F A D I S S E C T I O N ?

A 71-​year-​old man presents to the preoperative area from home before receiving a thoracic endovascular aortic aneurysm repair (TEVAR). He has a past medical history of hypertension, hyperlipidemia, coronary artery disease (CAD), chronic obstructive pulmonary disease (COPD) from years of smoking and diabetes type 2, for which he takes oral antihyperglycemic medication. He has a thoracic aortic aneurysm measuring 6 cm in diameter, beginning 1 cm below the left subclavian artery take-​off and extending 7 cm caudad. The aneurysm was discovered after a widened mediastinum was seen on chest X-​ray during an emergency room visit for shortness of breath. His preoperative vitals are as follows: heart rate 102 beats per minute, BP 183/​67 mm Hg, and SpO2 93% on room air.

Classically, the first presenting sign of an acute dissection is a “tearing” chest pain that radiates to the back. Tachycardia and hypertension in conjunction with chest pain in our patient should be very concerning for an acute dissection or a myocardial infarction. Hypotension can also be seen in a dissection if the dissection involves the coronary arteries or causes aortic valve insufficiency, pericardial effusion, or tamponade. The case should be delayed, and the patient should be emergently worked up for a myocardial infarction. The workup includes beta blockade, aspirin, supplemental oxygen, and obtaining a 12 lead EKG. If there is ST elevation on the EKG, the patient immediately goes to the catheterization (cath) lab. As previously mentioned, an aortic dissection can involve the coronary arteries causing symptoms of myocardial infarction. This can be demonstrated in the cath lab as well. If there are no ST changes, a computed tomography (CT) scan should be performed to rule out an ascending aortic dissection (Stanford type A), which is a surgical emergency necessitating immediate open heart surgery. If a descending aortic dissection (Stanford type B) is discovered, the patient should be admitted to an intensive care unit and medically optimized before proceeding to surgery. The exception to type B management is evidence of limb ischemia or end-​organ ischemia. In these settings, the patient should to be taken to the operation room by vascular surgery for immediate correction.

I S T H I S PAT I E N T O P T I M I Z E D TO P RO C E E D TO T H E O P E R AT I N G RO O M ? I S T H E R E OT H E R I N F O R M AT I O N WE S H O U L D O BTA I N FI R S T B E F O R E P RO C E E D I N G ?

The patient has some concerning vital signs. He is both hypertensive and tachycardic, which are risk factors for developing a dissection. It is important to know what the patient’s baseline blood pressure is and what heart rhythm he is in. Obtaining a 12 lead electrocardiogram (EKG) is important. The patient could have an arrhythmia that is causing tachycardia, or he could be in pain or anxious. His low oxygen saturation is acceptable given his COPD, but he should be preoxygenated as well in the operating room prior to induction of general anesthesia. Ideally, the patient should have a regular heart rate of 60 to 80 beats per minute with a more controlled systolic blood pressure. Once an intravenous (IV) line is established, a beta-​blocker can be given. Metoprolol is a good choice to provide heart rate control during the case. An afterload-​reducing agent like nicardipine or sodium nitroprusside can be used to lower the blood pressure intraoperatively to prevent excessive hemorrhage during surgical access of the femoral vessels, rupture of the aneurysm, or development of an acute dissection. In general, the larger the aneurysm diameter, the greater the risk of dissection.

WH AT I S A T EVA R ? A R E T H E O U TC O M E S B ET T E R T H A N A C L A S S I C A L O P E N T H O R AC I C AO RT I C A N EU RYS M S ( TA A) R E PA I R ?

A TEVAR is a minimally invasive procedure where a stent graft is inserted percutaneously from the femoral artery into the thoracic aorta. The graft is placed above and below the aneurysm sac to prevent further growth. Stent grafts can also be used in descending aortic dissections to cover the origin of false lumens to prevent further propagation of the dissection. TEVAR has shown better neurological outcomes than an open procedure and can be performed on more frail patients.

95

TEVAR patients also have shorter hospital stays. However, the long-​term outcomes are not different, and TEVAR has a high rate of endoleak, a complication unique to endovascular repair of aortic aneurysms. These procedures require long-​ term monitoring with radiologic imaging using contrast dye. WI L L T H E C A S E B E P E R F O R M E D U N D E R G E N E R A L A N E S T H E S I A O R US I N G R E G I O NA L A N E S T H E S I A ? WH AT MO N ITO R S A R E N E E D E D F O R T H I S  C A S E?

This case can be performed under general anesthesia or regional anesthesia. However, most centers perform this case under general anesthesia for several reasons. TEVAR procedures require the use of heparin during stent graft placement, which may be contraindicated with certain regional techniques. Breath holding for digital subtraction angiography is commonly needed for these cases, and it is very easy to perform with a secured airway. Tight hemodynamic control is needed during a TEVAR, and this is easier to obtain with a patient under general anesthesia. Finally, in the case of rupture, the airway is already secured. Standard American Society of Anesthesiologists monitors should be used in this case with a 5 lead EKG. A preinduction arterial line should be placed as well. A lumbar spinal drain should be placed to monitor cerebrospinal fluid (CSF) pressure and to drain CSF. Large bore vascular access is needed in the event of a rupture. This can be obtained with either large peripheral IVs or with an introducer central line. A  central line should strongly be considered, as maintaining spinal cord perfusion pressure postoperatively is important and can be accomplished with pressors such as norepinephrine. In patients with significant cardiac disease and left or right heart failure, a pulmonary artery catheter can be considered. It should be discussed with the surgical team if somatosensory-​evoked potential (SSEP) or motor-​evoked potential (MEP) monitoring will be used, since spinal cord ischemia is one of the main complications of both open and endovascular thoracic aortic aneurysm repair. WH AT I S A LU M BA R S P I NA L D R A I N ? WH Y I S I T P L AC E D ? WH AT A R E T H E C O M P L I C AT I O NS ?

A lumbar spinal drain is a catheter placed into the subarachnoid space usually in the L3–​L4 or L4–​L5 intervertebral space. It is placed with a large, usually 14 G Touhy needle. The drain is placed to allow CSF drainage during the operation and postoperatively to maintain spinal cord perfusion pressure. After the stent graft is deployed, blood flow to the spinal cord can be impaired resulting in spinal cord ischemia. By draining CSF, CSF pressure is reduced and blood flow to the spinal cord improves, thereby reducing ischemia. Placing a lumbar spinal drain is usually performed by the anesthesiologist and can be performed both pre-​and postinduction. It is not a benign procedure and can have significant complications. The most benign complication is a CSF leak after catheter removal, resulting in a postdural puncture headache. There has also been documentation of

the catheter fracture during removal, requiring emergent neurosurgery. Infection resulting in meningitis has also been described in the literature. The most severe complications involve bleeding. Similar to epidurals placed for other surgical procedures, there is a risk for epidural hematoma. While the drain is being placed, the anesthesiologist should be careful not to obtain a traumatic or “bloody” tap. During the TEVAR procedure, heparin will be given, which greatly increases the risk of an epidural hematoma. The American Society of Regional Anesthesia and Pain Medicine guidelines should be followed regarding anticoagulation and platelet levels. Unique to lumbar spinal drains is intracranial hemorrhage (ICH), which results from overdrainage of CSF. While infrequent, it carries a mortality of up to 40%. If the CSF becomes blood tinged, all drainage should be stopped, and immediate CT imaging of the head should be performed. H OW S H O U L D T H E LUM BA R S P I NA L D R A I N B E M A NAG E D I N T R AO P E R AT I V E LY ? P O S TO P E R AT I VE LY ?

After the catheter is placed, it should be attached to a sealed reservoir and a pressure monitor. CSF is drained at a rate of 10 to 15 mL/​hr to achieve a CSF pressure between 10 and 15 mm Hg. If the CSF pressure remains between 10 and 15 mm Hg without draining CSF, the CSF pressure can just be monitored. More aggressive drainage of CSF can be performed (up to 20 mL/​hr) if there are changes in SSEP and MEP monitoring or if the awake patient is complaining of paresthesia. However, as previously mentioned, if too much CSF is drained, an ICH can result. Postoperatively, most centers maintain the drain for 48 to 72 hours. CSF pressure should continue to be monitored. Similar to while in the operating room, CSF is drained up to 15 mL/​hr to maintain a CSF pressure of approximately 10 mm Hg while the patient is neurologically intact. If a neurologic deficit occurs, aggressively drain CSF. One study advocates going down to a CSF pressure of 5 mm Hg in the setting of new-​onset neurologic deficits. Delayed neurologic deficits have been associated with malfunctioning lumbar spinal drains. If the drain malfunctions, it should be replaced. DISCUSSION An aortic dissection was first described in 1760 by Dr. Nicholls, physician to King George II, after performing an autopsy.1 The incidence of TAA and dissection is increasing, likely from increasingly better diagnostic modalities and an aging population.2 A large study by Olsson et al.3 in 2006 showed that in 2002, the number of affected individuals was 16.3 per 100,000 per year for men and 9.1 per 100,000 per year for women. While detection is increasing and understanding of the dis­ ease process is improving, thoracic aortic disease still carries significant mortality and morbidity. Without surgical treatment, the 5-​year survival of a thoracic aneurysm is 54%, but even with surgery the incidence of morbidity and mortality from complications remains quite high.

96  •   Vascu l ar

A chest x-​ray of a widened mediastinum in a patient with a thoracic aortic aneurysm. Sternal wires are shown from previous ascending aortic dissection repair. (A) Patient chest x-​ray showing a large aortic knob and overall wide mediastinum. (B) Lateral chest x-​ray shows wide descending thoracic aorta. Figure 11.1 

TAAs are mostly silent, with >95% being asymptomatic until an acute event occurs. Sixty percent of TAAs occur in the aortic root and ascending aorta, 10% in the aortic arch, 40% in the descending aorta, and 10% in the thoracoabdominal aorta.2 Most TAAs are discovered incidentally through imaging for another medical issue (Fig. 11.1). However, in a setting of an acute event, the patient may feel a “tearing” chest pain that radiates to the back, which may be the first presenting sign of a TAA.

2%. At 6.0 cm, the rate increases to 7% annually. Past 6.0 cm, the rate of dissection or rupture is approximately 30% annually for ascending aortic aneurysms. A diameter greater than 7.0 cm, descending TAAs have a rate of rupture or dissection greater than 40% annually. Once an acute event occurs, short-​ and long-​term survival of these patients drastically decreases.2 Surgical correction of a TAA is commonly offered once an aneurysm is between 5 and 6 cm in diameter. D I AG N O S I S

E P I D E M I O L O GY A N D ET I O L O GY

TAAs have been found to be familial. Twenty-​one percent of patients with a TAA have a family member with an aneurysm of some type. Aneurysms will appear earlier in patients with a familial history, than those with no family history. The median age at diagnosis is approximately 70.3 Genetic disorders such as Marfan syndrome (most commonly), Ehlers-​Danlos syndrome, Loeys-​Dietz syndrome, and Turner syndrome are associated with TAAs as well. If a young patient presents with an aortic aneurysm, one should be very suspicious of a connective tissue disorder. Bicuspid aortic valves are also associated with TAA, and patients with bicuspid valves should be screened for an aneurysm.2 Smoking is the greatest risk factor for aortic aneurysms. Other risk factors include advanced age and male sex. Men are approximately twice as likely to develop a thoracic or abdominal aneurysms as compared to women. Uncontrolled hypertension is associated with aneurysm formation, and acute hypertensive events are a frequent cause of dissection.1,4 Once a TAA has formed, it grows indolently. Ascending aortic aneurysms expand by 0.10 cm annually while descending TAAs grow at a rate of 0.29 cm annually. Aneurysms from syndromic patients may grow at an even faster rate. Malignant Loeys-​Dietz syndrome can lead to a growth rate of 1.0 cm per year. As the aorta becomes more dilated, the rate of acute dissection or rupture increases. A TAA with a diameter between 4.0 and 4.9 cm has an annual rate for an acute event around

TAAs are very rarely diagnosed through physical exam or laboratory findings. If the aneurysm involves the aortic root, a murmur from aortic valve insufficiency may be heard. The main stay of diagnosis is imaging. While a routine chest X-​ray can sometimes find a TAA presenting as a widened mediastinum, definitive diagnosis is generally made with a CT or magnetic resonance imaging (MRI). Aneurysms are generally diagnosed when a diameter is found to be greater than 4 cm. Currently, CT angiography is the standard for imaging. It is widely available, and the exam can be performed rapidly. CT angiography is also the best choice for monitoring aneurysm growth. MRI can be used, but there is limited evidence supporting the accuracy of aortic aneurysm assessment. When contrast dye was used, MRI imaging was found to be comparable to CT. In younger patients, when radiation exposure should be limited, MRI is a good alternative.5 Echocardiography can be used, but it has several limitations. Transthoracic echocardiography can be used to image the ascending aorta arch and the very proximal portion of the descending aorta. However, the imaging windows are generally not ideal, and a large body habitus can severely worsen image quality. Transesophageal echocardiography (TEE) provides excellent images of the proximal aorta and of the descending thoracic aorta but does not image the aortic arch well. TEE is useful intraoperatively for open TAA repairs, but its invasiveness limits its usefulness for monitoring. Incidental

11. T E VA R —T h oracic E ndovascu l ar S ur g ery  •   97

aneurysms can be identified with TEE, if the TEE is used for other procedures.5 In acute dissections or rupture, CT is once again the main imaging modality due to its wide availability and rapid image acquisition. TEE is also excellent for identifying dissection flaps through the use of color-​flow Doppler. TEE is commonly used to confirm type A dissections in emergent surgery.6

it could be located in a radiology suite, catheterization lab, a regular operating room with a portable fluoroscope, or in a hybrid operating room. Anesthesiologists should take the location of the procedure into consideration when planning their intraoperative management.4 All patients should be monitored with American Society of Anesthesiologists standard monitors and have a preinduction arterial line placed. In TEVAR procedures, the brachial artery is commonly used by the surgical team for arterial access as well as the femoral arteries. It is important to discuss with members P R EO P E R AT I VE M A NAG E M E N T of the surgical team prior to the procedure what their vascular When preparing a TAA patient for the operating room, ide- access plans are. Obtaining arterial pressure monitoring on the ally the patient should be seen in a preoperative clinic to allow left arm is especially useful in TEVAR procedures because of time for all the appropriate studies, labs, and medical optimi- the potential of “jailing” or occluding the left subclavian artery zation to occur. A  thorough history and physical should be after the stent graft is deployed. If this occurs, left subclavian performed. Special attention should be paid to cardiovascular artery bypass surgery may be required.8 The arterial line will function and obtaining a baseline neurological exam. This pa- allow for early detection of the occlusion and allow assessment tient population commonly has a large number of comorbid of limb ischemia. Open TAA repairs are uniformly done under general anconditions including, but not limited to, CAD, heart failure, uncontrolled hypertension, peripheral vascular disease, esthesia and require lung isolation with a double lumen endotracheal tube and central venous access. Patients are also likely COPD, obesity, and diabetes. Even if a minimally invasive procedure is planned, an an- to be placed on partial bypass.9 TEVARs can be performed esthesiologist must be prepared to convert to an open pro- on a sedated patient using regional and/​or local anesthesia. cedure. Functional status of the patient should be assessed However, patient cooperation is needed for breath holding in accordance with American College of Cardiologists/​ in these cases, and hemodynamics can be more difficult to American Heart Association guidelines, and stress testing or control in the awake patient. Also, in the event of a rupture, catheterization should be ordered appropriately. A  baseline or need to convert to open surgery, the airway will not be EKG should be ordered, and a transthoracic echocardiogram secured.4 In addition, the use of epidural or spinal techniques should be ordered as well, if there is any concern about car- may conceal a postoperative neurologic deficit and delay treatdiac dysfunction. Preoperative labs should include a complete ment. For these reasons, TEVARs are typically performed blood count, basic metabolic panel, coagulation panel, and a under general anesthesia. Hemodynamic control is extremely important before and type and screen. Thoracic abdominal aneurysm patients are at risk for rupture and even TEVAR procedures can have signif- after placement of the stent graft in a TEVAR. Prior to graft icant blood loss. A basic metabolic panel is needed to assess insertion, blood pressure should be in the normal range to renal function. Acute renal injury is a significant complication prevent damage to arteries during vascular access and acute in both open and endovascular procedures. Open procedures events. However, after the stent graft is deployed, maintaining involve cross-​clamping the aorta and interrupting renal blood adequate spinal cord perfusion is critical. Target mean arsupply, and TEVARs use large amounts of contrast dye that terial pressures (MAPs) are normally 80 to 100s mm Hg damage intrinsic renal function.4,7 An arterial blood gas test postoperatively to protect the spinal cord. Spinal cord ischemia and resulting paraplegia is a major should be considered for patients with severe COPD or other complication of both open and endovascular TAA repair and lung disease. Medical optimization should include control of blood leads to significant morbidity and mortality. Lumbar spinal pressure, heart rate, and blood sugar if the patient is diabetic. drains have been found to help reduce the incidence of spinal Commonly, patients are continued on beta-​blockers, aspirin, cord ischemia. They are used to transduce CSF pressure and and a statin. However, beta-​blocker therapy has not been drain CSF as needed to maintain adequate spinal perfusion shown to have a clinical benefit in TAA patients, except in pressure (SCPP). SCPP is calculated with the following patients with Marfan syndrome. Statins have been used in ab- formula.10 dominal aortic aneurysm patients to prevent further growth SCPP = MAP –​[CSFP or CVP (whichever is greater)], of the aneurysm. Interestingly, statins have not been shown to have benefit in abdominal aortic aneurysms, but has been shown to prevent dissection, rupture, death, and need for sur- where CSFP is CSF pressure, and CVP is central venous pressure. gical intervention in TAA patients.2 In addition to monitoring CSF pressure, surgeons may use SSEP and MEP to monitor the spinal cord during surgery.9 As I N T R AO P E R AT I VE M A NAG E M E N T such, the anesthetic must be adjusted appropriately. It should TAAs repaired by TEVAR can occur in different locations in be noted that SSEP and MEP monitoring may be affected by a hospital. The procedure requires the use of intraoperative decreased blood flow to the lower extremities during TEVAR fluoroscopic imaging, and depending on hospital facilities, and open procedures. 98  •   Vascu l ar

S P I NA L C O R D I S C H E M I A A N D LUM BA R S P I NA L  D R A I N S

Preventing spinal cord ischemia has been a major focus of research in TAA repairs. The advent of TEVAR procedures has seen a significant drop in postoperative paraplegia, but the rate is still as high as 8%. The spinal cord receives its blood supply from a single anterior spinal artery and 2 posterolateral spinal arteries, all originating from the vertebral arteries. At each vertebral level, right and left posterior intercostal arteries branch from the aorta and form segmental arteries that feed the anterior and posterior spinal arteries. The largest of these is the artery of Adamkiewicz, which typically arises between T9 and T12 and is the main thoracic blood supply to the spine.11,12 With stent graft deployment in a TEVAR, blood flow through these arteries is interrupted, and blood flow to the spinal cord is diminished, causing ischemia. The lumbar spinal drain is placed to allow removal of CSF to maintain SCPP of at least 70 mm Hg. The goal is to maintain a CSFP of approximately 10 mm Hg. While higher pressures can be tolerated, it puts the spinal cord at increased risk of ischemia. Lower pressures increase the risk of ICH.13 CSF should be drained at regular increments at a rate of 10 mL/​hr. Once again, more or less CSF can be drained to maintain appropriate CSFP. ICH is the most serious complication of the lumbar spinal drain. It carries a mortality of approximately 40% but is rare, occurring in less than 1% of patients. It is thought to occur from overly aggressive drainage of CSF. This results in sagging of the brain and stretching of the dural veins resulting in hemorrhage. The majority of the ICHs were found in the cerebellar region.13 Other complications occurred at low rates as well. In 2009, Estrera et al.13 reported that meningitis occurs in 0.2% of patients. CSF leak and postdural puncture headaches also occurred in 0.2% of patients, and a fractured catheter requiring surgical removal occurred in 1 patient with lumbar spinal drain placement. Epidural hematoma is also a possible complication but did not occur in the study. Lumbar drain placement is not a benign procedure and carries a complication rate of up to 5% in some studies.13 However, Hnath et al.14 found that not placing a spinal drain in TEVARs resulted in an 8% incidence of spinal cord ischemia, while spinal drain placement resulted in a spinal cord ischemia rate of 0%. Other techniques to prevent spinal cord ischemia are also used. Under normothermic conditions, the brain and spinal cord can develop permanent injury from ischemia after just several minutes. Mild hypothermia (32°–​ 34°C) can provide some neurologic protection. There are also techniques for producing regional spinal cord hypothermia. The most common technique involves the infusion of cold saline (4°–​ 5°C) into the epidural space via an epidural catheter placed in a low thoracic vertebral interspace.15,16 There is very little evidence supporting pharmacologic interventions that provide spinal cord protection. Historically, methylprednisolone, mannitol, magnesium, lidocaine. or thiopental have been used.12,17

P O S TO P E R AT I V E M A NAG E M E N T

Immediately after emergence from anesthesia, a neurological exam should be performed. If the patient is neurologically intact, he or she typically go to the surgical intensive care unit for management. Neurologic monitoring and hemodynamic management are the main foci of postoperative care. The lumbar spinal drain should be maintained for 48 to 72 hours, and CSFP should be followed. Blood pressure should be maintained slightly elevated to maintain SCPP, but at the same time, controlled to maintain the graft. Blood pressure goals should be discussed with the surgical team in the postoperative period. If needed, vasopressors such as norepinephrine or phenylephrine can be used to increase MAP. Close attention should be paid to the CSF color. If blood is detected, drainage should be stopped, and a head CT should be immediately obtained. Blood in the CSF is believed to be a very sensitive indicator of ICH, even before neurologic symptoms appear.13 Delayed neurologic deficit has been documented as occurring several days postoperatively. Neurologic function should be monitored closely, and care should be taken so that the lumbar spinal drain does not become dislodged or obstructed, as this has been linked to delayed neurologic deficit.12,13,17 Renal function should also be monitored closely after a TEVAR. A  large amount of contrast dye is needed for the procedure, and acute kidney injury is not uncommon. Urine output should be monitored, as well as trending creatinine. In addition, IV fluid should be given to help maintain renal function.4,7,8 C O N C LUS I O N

The incidence of TAAs is increasing, which means more surgical corrections will occur in the future. Minimally invasive techniques such as TEVAR have shown good outcomes and will likely become more common. Lumbar spinal drains have been shown to be important in the prevention of spinal cord ischemia, one of the major risks of TAA surgical correction. At most institutions, the drain is placed and managed by anesthesiologists, so it is important to understand its purpose, how it prevents spinal cord ischemia, the complications from drain placement, and how best to avoid them. R E VI EW Q U E S T I O N S 1. During placement of a lumbar spinal drain, you get blood return from your Touhy needle. What should be your next step? A. Withdraw the Touhy needle and try a new placement at another level B. Delay the case C. Continue at the current level D. Abandon the lumbar spinal drain placement

11. T E VA R —T h oracic E ndovascu l ar S ur g ery  •   99

Answer: A You should try placement at another level. As long as the CSF is clear, you are OK to proceed with the case, since heparinization normally occurs more than an hour after drain placement. You should not continue at the same level, as the trauma to area could be made worse. Some centers will delay the case for 24 hours. Any traumatic tap should be discussed with the surgical team.13 2. Which of the following is NOT a common postoperative complication of a TEVAR procedure? A. Acute renal injury B. Pneumonia C. Transfusion-​related acute lung injury D. Lower extremity paresthesias Answer: C Transfusion-​related acute lung injury is associated with transfusion of blood products, particularly plasma and platelets. TEVARs are endovascular procedures and have relatively little blood loss. Pneumonia is actually the most common complication after TEVARs, occurring in almost 10% of patients, nearly twice the rate of open procedures. This may occur because typically patients undergoing TEVARs are more ill and have more comorbidities. Lower extremity paresthesias are becoming less common with the advent of lumbar spinal drains and other spinal cord protection techniques. Without protection, spinal cord ischemia occurs in approximately 10% of patients who receive TEVARs.8 3. Which of the following is a risk factor for developing a thoracic aortic aneurysm? A. Female  sex B. Bicuspid aortic valve C. Trisomy  21 D. Drinking alcohol Answer: B Bicuspid aorta valves are associated with aneurysms of the ascending aorta. Other risk factors include male sex, family history of aneurysms, genetic disorders such as Marfan, Ehlers-​ Danlos, Loeys-​Dietz, and Turner syndromes. The greatest risk factor is smoking. Uncontrolled hypertension is also associated with aneurysm formation and acute dissections.1,2–4 Trisomy 21 and drinking alcohol have not been associated with development of a TAA. 4. At what diameter of a thoracic aortic aneurysm is surgical correction commonly offered to a patient? A. 3 to 4 cm B. 4 to 5 cm C. 5 to 6 cm D. 6 to 7 cm Answer: C The upper limit of normal for the thoracic aorta past the aortic root is 4 cm. An aneurysm is diagnosed with a diameter >4 cm. Surgical correction is normally offered >5 cm. After an aneurysm grows to 6  cm, the rate of aneurysmal expansion rapidly increases, and the risk of acute dissection greatly

increases. Surgical correction should be offered before the aneurysm grows to this size.2,5 5. Which of the following is NOT a technique for preventing spinal cord ischemia? A. Regional spinal cord hypothermia B. Lumbar spinal drain C. Controlled hypertension D. SSEP Answer: D Regional cooling of the spinal cord has been shown to provide protection to the spinal cord. However, it is invasive and not commonly used.15,16 Lumbar spinal drains and controlled hypertension both attempt to maintain adequate spinal cord perfusion pressure to prevent ischemia.10 SSEP is a monitor and may alert operating room staff to the development of spinal cord ischemia. However, SSEP does not prevent spinal cord ischemia from developing. 6. A patient receiving a TEVAR operation has an arterial line and central line in place. The blood pressure is 123/​59 (80) with a CVP of 12 mm Hg. What is the spinal cord perfusion pressure? A. 59 mm  Hg B. 80 mm  Hg C. 111 mm  Hg D. 68 mm  Hg Answer: D SCPP = MAP –​[CSFP or CVP (whichever is greater)].10 We don’t have a CSF pressure, so we have to used CVP instead. 80 –​12 is 68 mm Hg. 7. A  patient presents to the emergency department with tearing back pain, chest pain, and a blood pressure of 184/​91. An EKG shows sinus tachycardia with no ST segment or T wave changes. CT scan shows an aortic dissection below the level of the left subclavian artery. What is the next appropriate course of action? A. Take the patient to the operating room for emergent surgery. B. Take the patient to the cath lab to rule out a myocardial infarction. C. Obtain a vascular surgery consult and begin medical management. D. Obtain a transesophageal echocardiogram looking for ascending aorta involvement. Answer: C CT scans are is the main imaging modality to identify aortic dissections.6 This CT scan shows a Stanford type B dissection, which is not a surgical emergency. The exception to this would be if there were signs of end-​organ damage. The EKG does not show ST segment or T wave changes, which suggests the chest pain is not from an ST-​elevation myocardial infarction (STEMI) or non-​ST-​elevation myocardial infarction (NSTEMI). With no signs of a STEMI, this patient should not go to the cath lab, as left heart catheterization may cause additional damage by placing wires into the false

100  •   Vascu l ar

lumen of the dissection. A TEE could confirm that there is no Stanford type A dissection, which is a surgical emergency, but would require subjecting the patient to sedation and a procedure that will not likely change management. The patient should have his heart rate and blood pressure optimized and be evaluated by vascular surgery for surgical correction at a future date. 8. Which physical exam finding is suggestive of a thoracic aortic aneurysm? A. A diastolic murmur at the right second interspace B. A bruit when auscultating the left neck while the patient holds his or her breath C. A pulsatile mass felt during and abdominal exam D. Different right and left upper extremity blood pressures taken with a manual cuff Answer: A TAAs are commonly silent and rarely diagnosed through physical exam or laboratory findings. Imaging is the mainstay of diagnosis of TAAs.5 If a TAA involves the aortic root, it can cause aortic valve insufficiency, and a murmur can be heard during auscultation of the heart. Bruits heard in the neck are suggestive of carotid artery stenosis. An abdominal, pulsatile mass is felt when a patient has a large abdominal aortic aneurysm. Different upper extremity blood pressures is suggestive of aortic coarctation or stenosis of one of the subclavian arteries. 9. Obstruction of which arteries by a TEVAR stent graft is the cause of spinal cord ischemia and spinal cord injury? A. Posterolateral spinal arteries B. Posterior intercostal arteries C. Vertebral arteries D. All of the above Answer: B Along with the single anterior vertebral artery, the posterolateral spinal arteries run the length of the spinal cord and supply it with blood. They originate from the vertebral arteries, which are branches off the right and left subclavian arteries, not the aorta. Right and left posterior intercostal arteries branch off the aorta at each vertebral level and supply segmental arteries that feed both and anterior and posterior spinal arteries. When a TEVAR stent graft is deployed, it reduces blood flow into the segmental arteries by obstructing or “jailing” the intercostal artery branches off of the aorta. This reduced blood flow to the spinal arteries is the causes of spinal cord ischemia and injury.11,12 10. A  patient is undergoing an elective TEVAR. A  lumbar spinal drain is placed without issue. You are given a break by a colleague, and when you return you notice that 30 mL of CSF was drained over 30 minutes. What complication of lumbar spinal drain placement should you be concerned about? A. ICH B. Postdural puncture headache C. Uncal herniation D. Spinal cord ischemia

Answer: A Excessive draining of CSF is thought to cause sagging of the brain and stretching of the dural veins, which results in ICH. ICH is not common, but in studies it has shown a mortality of 40%. Any signs of blood in the drained CSF should greatly raise your suspicious of ICH. Postdural puncture headaches are associated with CSF leak after removal of an intrathecal catheter or with inadvertent puncture of the subarachnoid membrane during epidural placement. While uncomfortable, it is not life-​threatening. Uncal herniation is possible but unlikely and has not been demonstrated to occur in studies. Draining CSF has been shown to prevent spinal cord ischemia, so for this reason choice D is incorrect.13

QUESTIONS AND ANSWER S This chapter has accompanying questions and answers which are available to subscribers as part of the Oxford eLearning platform. To access the questions, go to http://​ oxfordmedicine.com/​anesthesiologyPBL REFERENCES 1. Tsai TT, Nienaber CA, Eagle KA. Acute aortic syndromes. Circulation. 2005;112:3802–​3813. 2. Kuzmik GA, Sang AX, Elefteriades JA. Natural history of thoracic aortic aneurysms. J Vasc Surg. 2012;56:565–​571. 3. Olsson C, Thelin S, Ståhle E, Ekbom A, Granath F. Thoracic aortic aneurysm and dissection: increasing prevalence and improved outcomes reported in a nationwide population-​based study of more than 14,000 cases from 1987 to 2002. Circulation. 2006;11:2611–​2618. 4. Kothandan H, Haw Chieh GL, Khan SA, Karthekeyan RB, Sharad SS. Anesthetic considerations for endovascular abdominal aortic aneurysm repair. Ann Cardiac Anaesth 2016;19:132–​141. 5. Holloway BJ, Rosewarne D, Jones RG. Imaging of thoracic aortic disease. Br J Radiol. 2011;84:S338–​S354. 6. Evangelista A, Flachskampf FA, Erbel R, et al. Echocardiography in aortic diseases:  EAE recommendations for clinical practice. Eur J Echocardiography. 2010;11:645–​658. 7. Glade GJ, Vahl AV, Wisselink W, Lisen MAM, Balm R. Mid-​term survival and costs of treatment of patients with descending thoracic aortic aneurysms; endovascular vs. open repair: a case-​control study. Eur J Vasc Endovasc Surg. 2005;29:28–​34. 8. Piffaretti G, Tozzi M, Lomazzi C, Rivolta N, Caronno R, Castelli P. Complications after endovascular stent-​grafting of thoracic aortic diseases. J Cardiothorac Surg. 2006;1:26. 9. Safi JH, Miller CC, Huynh TT, et. al. Distal aortic perfusion and cerebrospinal fluid drainage for thoracoabdominal and descending thoracic aortic repair ten years of organ protection. Ann Surg. 2003;238:372–​381. 10. Mutch WA. Control of outflow pressure provides spinal cord protection during resection of descending thoracic aortic aneurysms. J Neurosurg Anesthes. 1995;7:133–​138. 11. Drinkwater SL, Goebells A, Haydar A, et  al. The incidence of spinal cord ischaemia following thoracic and thoracoabdominal aortic endovascular intervention. Eur J Vasc Endovasc Surg. 2010;40:729–​735. 12. Griepp RB, Griepp EB. Spinal cord perfusion and protection during descending thoracic and thoracoabdominal aortic surgery: the collateral network concept. Ann Thorac Surg. 2007;83:S865–​S869.

11. T E VA R —T h oracic E ndovascu l ar S ur g ery  •   101

13. Estrera AL, Sheinbaum R, Miller CC, et  al. Cerebrospinal fluid drainage during thoracic aortic repair:  safety and current management. Ann Thorac Surg. 2009;88:9–​15. 14. Hnath JC, Mehta M, Taggert JB, et al. Strategies to improve spinal cord ischemia in endovascular thoracic aortic repair:  outcomes of a prospective cerebrospinal fluid drainage protocol. J Vasc Surg. 2008;48:836–​840. 15. Dietrich WD, Levi AD, Wang M, Green BA. Hypothermic treatment for acute spinal cord injury. Neurotherapeutics. 2011;8:229–​239.

16. Davison JK, Cambria RP, Vierra DJ, Columbia MA, Koustas G. Epidural cooling for regional spinal cord hypothermia during thoracoabdominal aneurysm repair. J Vasc Surg. 1994;20:304–​310. 17. Cheung AT, Weiss SJ, McGarvey ML, et  al. Interventions for reversing delayed-​onset postoperative paraplegia after thoracic aortic reconstruction. Ann Thorac Surg. 2002;74:413–​421.

102  •   Vascu l ar

SECTION IV ABDOMINAL

12. ACTIVE PHEOCHROMACY TOMA Vilma Joseph and Chukwuemeka Okafor

S T E M C A S E A N D K EY Q U E S T I O N S You have just finished your five bariatric cases and look at the schedule to see what your assignment is for the following day. Tomorrow, you are scheduled to anesthetize a patient undergoing a right partial adrenalectomy. Your patient is a 45-​year-​old woman who was diagnosed with a pheochromocytoma a few months earlier. She presented to her primary doctor with the intermittent headache, sweating, and palpitations. She was then worked up by an endocrinologist and began seeing an endocrine surgeon. WH AT I S A P H EO C H RO MO C Y TO M A ? H OW D O E S I T P R E S E N T ?

A pheochromocytoma is a rare neuroendocrine, catecholamine-​ producing tumor derived from chromaffin cells. Although usually located in the adrenals, this catecholamine-​producing malignancy can manifest elsewhere but specifically along the peripheral nervous system. Those arising from nonadrenal sites are known as paragangliomas. Pheochromocytomas can present in a variety of ways. This woman presented to her primary care physician complaining of recurrent palpitations, anxiety attacks, and diaphoretic episodes. When vital signs were checked, her blood pressure was found to be 190/​100  mm Hg. Hypertension, either paroxysmal or consistent, is one of the most insidious symptoms. Palpitations, diaphoresis, and headache are some of the classical signs that prompt most patients to see a physician. Although uncommon, pheochromocytomas have also been seen to manifest with symptoms of weight loss and constipation. NA M E OT H E R D I S E A S E S TAT E S T H AT M AY P R E S E N T S I M I L A R LY. H OW S H O U L D A P H EO C H RO MO C Y TO M A B E WO R K E D  U P ?

Besides a history of depression and use of nortriptyline, the patient had no other medical history. Her history and presentation are not uncommon and can be attributed to a wide range of diagnoses. A  few of these include carcinoid syndrome, amphetamine and cocaine use, anxiety/​psychiatric issues, and primary hypertension. When a pheochromocytoma is highly suspected, biochemical testing is sought. The patient was told to collect her urine for 24 hours at home and return to the lab with the sample. A  24-​hour urine catecholamine and metanephrine

collection is typically one of the initial diagnostic tests performed due to its high sensitivity and specificity for pheochromocytoma/​paraganglioma. In some studies, specificity was found to be as high as 99.7%. Some institutions instead assess plasma metanepherine levels. Although its specificity isn’t traditionally as high as the 24-​hour urine test, its sensitivity is approximately 96%. Prior to testing, she was told to discontinue her nortriptyline, a commonly prescribed tricyclic antidepressant. Antidepressants, amphetamine-​containing agents, and labetalol are some of the more common medications that interfere with accurate testing of epinephrine, norepinephrine, and their metabolites. After biochemical testing confirms the diagnosis, radiological localization is done. Computed tomographic (CT) scan as well as magnetic resonance imaging (MRI) are used for this purpose and have similar sensitivity and specificity. Metaiodobenzylguanidine (MIBG) scanning can be done in cases where CT and MRI are unable to detect a tumor. MIBG scanning offers both a higher specificity and sensitivity than CT or MRI. WH AT M E D I C AT I O NS A R E T H E PAT I E N T T R E AT E D WI T H A S A N O U T PAT I E N T B E F O R E T H E S U RG E RY ?

When the patient’s plasma metanephrines returned positive, an MRI was used to localize the pheochromocytoma to the right adrenal gland. She was seen by an endocrine surgeon, and surgery was scheduled. The next appropriate step to both treat and prepare her for the surgery was to be started on a specific drug regimen. She began taking phenoxybenzamine, an alpha-​adrenergic blocking agent. The medication was used exclusively and titrated over the course of two weeks. When mild postural hypotension could be elicited, the medication was continued at that dose. Her presenting heart rate had improved on this regimen, but there was residual tachycardia her endocrinologist felt could be better controlled. She was then started on a beta-​adrenergic blocker, labetalol. WH AT A R E S O M E C R IT E R I A T H AT S H O U L D B E M ET 24 TO 4 8 H O U R S B E F O R E S U RG E RY ?

The patient was instructed to continue her medication regimen until the day of surgery. She was also told to keep a heart rate and blood pressure diary. The target blood pressure over

105

the remaining days prior to the surgery was less than 160/​ 90  mm Hg. Her electrocardiograms (EKGs) were checked daily to assess ST changes that may be indicative of myocardial ischemia. On the morning of surgery, she was kept on a monitor in the preoperative area. EKG criteria during this period showed no more than one premature ventricular contraction every five minutes.

of aorta.2 Pheochromocytomas and sympathetically derived paragangliomas are known to produce large, superfluous amounts of epinephrine, norepinephrine, and/​or dopamine. The adrenal medulla is well vascularized thereby allowing the products of the tumor to find its way into the systemic circulation and exerting its effects quite easily. Fortunately, these tumors do not always secrete catecholamines into the circulation. In some patients, they are in storage and are only released under particularly stressful circumstances (exercise, change in posture, infection, etc.) while in others an intermittent regular WH AT M E D I C AT I O NS S H O U L D B E AVO I D E D flow of these vasoactive substances are released. Quantifying I N T R AO P E R AT I VE LY ? exactly how much of each hormone is secreted from a tumor Any drug that has the potential to result in an increase in from either of those circumstances above continues to be a catecholamines or activation of the sympathetic nervous challenge for researchers.3 Approximately, 25% to 33% of persons found to have a disystem should generally be avoided as an anesthetic agent intraoperatively. Some of these drugs include ketamine, agnosis of pheochromocytoma have an inherited condition or atracurium, morphine, droperidol, halothane, desflurane, and germline mutation including those of the RET, VHL, NF1, SDHB, SDHC, and SDHD genes or in the more recently succinylcholine. recognized SDHA, SDHAF2, TMEM127, and MAX genes.4 Growth of a pheochromocytoma can indicate both multiple endocrine neoplasia type 2A and type 2b syndromes. Despite DISCUSSION this, a majority of pheochromocytomas will occur sporadically with no clear hereditary basis. The commonly referenced ET I O L O GY “10% rule” in describing pheochromocytomas (10% bilateral, A pheochromocytoma is a catecholamine-​producing tumor 10% malignant, 10% hereditary) has long been debunked as derived from sympathetic, specifically chromaffin, cells more research has come to light and a better understanding of 1 located in the adrenal medulla (see Figure 12.1). It is rare but the disease has been appreciated.5 Contemporary knowledge not uncommon for these tumors to be borne from an extra-​ and studies attribute a much greater percentage of those afadrenal site. Those that are, are referred to as paragangliomas. fected to having a hereditary component. Paragangliomas may or may not be chromaffin positive and The symptomatology of pheochromocytoma is based prican rise from either the sympathetic or parasympathetic marily on what, when, and how much of the catecholamines system. In general, those derived from peripheral parasymare released into the circulation. Hypertension, either sustained pathetic tissue will not actively produce catecholamines. The or sporadic, is the most common and dominant feature of the most common sites paragangliomas have been found to manidisease. Because elevated blood pressures rarely manifest themfest from include the organ of Zuckerkandl, chromaffin tissue selves as a particular, noticeable dysfunction to patients, it’s not surrounding the inferior mesenteric artery, and the bifurcation uncommon for paragangliomas to go undetected. The classic, appreciable presenting triad of symptoms one may present with to the emergency department or to his or her primary care physician include sporadic unprovoked headache, palpitations, and diaphoresis, which may present altogether or separately. Studies have shown 70% of pheochromocytomas present with the latter triad, and only 50% present with sustained hypertension.6 The urgency to treat pheochromocytomas is due to their high potential for adverse and sometimes fatal incidents. A  dangerously high catecholamine surge from one of these tumors can easily result in organ-​specific dysfunction and possible failure throughout the entire body. Pulmonary edema, stroke by way of intracranial hemorrhage, myocardial infarction, and heart failure are all possibilities from a large catecholamine output episode. Tachycardia, among other dangerous arrhythmias, is not uncommon in pheochromocytoma exacerbations in and out of the operating theater. Although most pheochromocytomas will not metastasize, Figure 12.1  400x high-power view of pheochromocytoma tumor showing this is another potential dangerous consequence of leaving characteristic histologic components. The chromaffin cells here have the these tumors untreated. Once one has spread to other organs, classic nested and trabeculated architecture. Other standard components the mortality and morbidity are adversely affected no matter of pheochromocytoma tumors are visible such as nuclear enlargement the management that has been enacted. In a recent retrospective intracellularly and hyperchromisia with cytoplams that is both oncoytic review including 152 patients with pheochromocytomas, no in some cells and basophilic in others.1 106  •   A bdominal

patients found with malignant disease were alive after 10 years after supposed complete surgical resection. This abysmal mortality rate is attributed to tumor burden at metastasized sites as well as the sequela of hypercatecholaminemia (i.e., heart failure).7 D I AG N O S I S

When pheochromocytoma is suspected in a patient, routine biochemical testing is performed. Twenty-​ four-​ hour urine catecholamines and metabolites (metanephrines and normetanephrines) are usually the first test sought. Subsequently, a confirmatory plasma assessment of the substances is obtained. Most institutions follow this algorithm, while others will first turn to plasma catecholamine metabolites to exclude a suspected pheochromocytoma. This test is then followed with urine catecholamines and metabolite testing only if the plasma values are three to four times the upper limit of normal.8 There are a variety of drugs (antipsychotics, antidepressants, and amphetamine-​ containing medications) as well as food (aged cheese, caffeine, cured meats, and chocolate) that may give you falsely elevated levels of catecholamines and their metabolites if they have recently been ingested (see Table 12.1). Patients should be made mindful of this before presenting to have a sample drawn. After biochemical testing is completed, radiological imaging is performed with the goal of localizing the malignancy. Both CT and MRI are utilized here. CT with contrast is usually ordered with a specific adrenal protocol to better visualize possible tumor tissue (see Figure 12.2).9 Both pheochromocytoma as well as paragangliomas should appear dense and would retain intravenous contrast. MRI will show a pheochromocytoma as hyperintense on T2-​weighted images and iso-​to hypointense to liver on T1-​weighted images. Hyperintense structures on MRI appear to be whiter or brighter than their reference/​surrounding structure, while hypointense findings are darker in color than a reference structure. Table 12.1  COMMON ITEMS THAT HAVE BEEN KNOWN TO ELEVATE URINARY AND PLASMA METANEPHRINES SUBSTANCES CAPAPBLE OF CAUSING FALSE-​P OSITIVE METANEPHRINE (URINARY/​P LASMA) RESULTS

Drugs Amphetamine containing substance Tricyclic antidepressents Serotonin–​norepinephrine reuptake inhibitors, selective serotonin reuptake inhibitors Monoamine oxidase inhibitors Levodopa Acetaminophen Food/​Beverages Bananas Caffeine Chocolate Vanilla Citrus fruits

Large pheochromocytoma located on the right adrenal gland, taken with contrast enhanced CT scan.9 Figure 12.2 

If either of these imaging techniques is unable to reveal the pheochromocytoma location, despite strong symptoms and positive biochemical markers, an MIBG is performed (see Figure 12.3).10 With an MIBG scan, metaiodobenzylguanidine is labeled to 1 of 2 iodine isotopes and injected into a peripheral vein. It is taken up by particular neural tissue associated with pheochromocytoma. After at least 24 hours, the patient is presented in front of a scanner, which displays particular areas of uptake of the radioactive material, possibly confirming the presence of a pheochromocytoma or paraganglioma. Of note, neuroblastomas can also be identified by MIBG. Most studies and meta-​analyses place the sensitivity of MIBG in detecting pheochromocytomas and paragangliomas in the range of 70% to 90%,11 qualifying it as a reliable way to differentiate them from any other disease and in confirming a diagnosis. O U T PAT I E N T M A NAG E M E N T

Pheochromocytomas can only be definitively treated by surgical removal. Until the patient is taken to the operating room, close outpatient follow-​up and pharmacological management are crucial to positive outcomes. Most outpatients with the confirmed diagnosis of active pheochromocytoma are started on a long-​acting nonselective alpha-​adrenergic blocking agent, the most common being phenoxybenzamine. Determining individual patients’ ideal dose is a challenge. Generally, clinicians carefully titrate the dose over the course of days to weeks until the symptoms of catecholamine excess are greatly diminished and/​or evidence of postural hypotension is evident. Shorter acting alpha blocking agents such as prazosin and doxazosin can be substituted closer to the operative day. This substitution allows tighter management of hemodynamics and to combat residual alpha blockade postoperatively when there is less baseline catecholamine in the circulation.

12. Active P h eoc h r omacytoma  •   107

Anterior rt

P E R I O P E R AT I V E A N E S T H ET I C M A NAG E M E N T

Posterior lt

lt

rt

Before the patient proceeds for surgery, there is a set of criteria most clinicians regularly abide by. They are known as the Roizen’s criteria which has remained consistently reliable overtime and include: 1. Blood pressure less than 160/​90 for 24 hours 2. Postural hypotension  >80/​45 3. EKG should be free of any ST changes for a week. 4. No premature ventricular contractions of more than 5 in 1 minute.14 Pheochromocytoma patients should have all standard American Society of Anesthesiologists monitors attached preinduction. One single 18 to 20 G intravenous line should be sufficient. Most experts would recommend a preinduction arterial line. This may be a prudent choice since these patients are typically volume contracted, and introduction of a hypnotic agent such as propofol, with known systemic vascular resistance-​lowering ability can result in hypotension and cardiovascular collapse. Etomidate may be a better agent to use here due to its hemodynamic-​stabilizing profile. Dynamic blood pressure monitoring may also be needed due to the posFigure 12.3  Whole-​body 123I MIBG demonstrating focal accumulation of sibility of sympathetic surge that may result with direct larradiopharmaceutical medially in the left upper quadrant.10 yngoscopy. Central access is usually sought after intubation. This allows the anesthesiologist to titrate vasoactive agents Beta-​blockers have been known to be added to the outpa- and insert a pulmonary artery catheter, if deemed necessary. tient regimen, but recent studies show that they should only Although the average blood loss is in the order of a few hunbe introduced when there appears to be objective evidence of dred milliliters,15 central access such as this is invaluable for continued excess catecholamine. Tachycardia, hypertension, the purpose of volume expansion in these patients whose vasand continued symptoms are some of the indications for beta cular tone has been decreased substantially. Transesophageal blockade. Labetalol and atenolol, nonselective beta-​blockers, echocardiography is not routinely used but should be placed are primarily used. These beta-​blockers should never be started in the appropriate patients. Pharmacological management of the patient during the before an alpha agent for fear of completely unopposed alpha-​ case is paramount in these surgeries. Vasodilators including adrenergic stimulation, which can have catastrophic results nicardipine and nitroprusside that can be titrated effectively secondary to a hypertensive crisis. should be ready and available. Agents that can be bolused such Although alpha-​and beta-​blockers are mainstay therapies, as nitroglycerine are invaluable. Epinephrine, norepinephthere are other drugs that have proven to be effective in the days rine, and dopamine infusions and boluses should be readily to weeks preceding operative management. Dihydropyridine available. Phenylephrine has proven to serve as an invaluable calcium channel blockers such as amlodipine and nicardipine agent when abrupt hypotension occurs throughout these may be used because of their relative ease of titration and low cases. Some studies have demonstrated the efficacy of high-​ occurrence of orthostatic hypotension. They work through potency opioid drips, such as remifentanil, in blunting the the inhibition of catecholamine-​induced calcium influx in overabundant sympathetic response a patient may produce smooth muscle. Studies have shown they are particularly 16 at various points during the surgery. A  few drugs are not useful in the subset of patients who have, or are more likely to 12 necessarily contraindicated but avoided during pheochromosuffer from, catecholamine-​induced vasospasm. Alpha methyltyrosine is another drug used in perioperative cytoma cases, such as ketamine, ephedrine, meperidine, and management of pheochromocytoma. It works by inhibiting desflurane, for fear of their intrinsic ability to cause catecholthe synthesis of norepinephrine. Its advantageous is that it amine release. In situations where hypotension is refractory to higher does not cause hypotension and can preserve tissue respondoses of vasopressors, methylene blue has been studied to siveness to adrenergic agents when titrated carefully. This drug be effective in restoring tone and raising blood pressure due is used sparingly and has for the most part fallen out of favor to cyclic guanosine monophosphate inhibition. Glucose because of its harsh side effect profile, which includes somno13 levels should be assessed and treated with insulin regularly lence, anxiety, depression, and tremor. 108  •   A bdominal

intraoperatively as hyperglycemia is not uncommon in these cases due to the wave of catecholamine surges. Most pheochromocytoma surgeries are now performed laparoscopically. Insufflation has been known to be particularly stimulating, and the anesthesiologist must be ready to either prophylax against and/​or treat the subsequent sympathetic surge. After careful dissection, the process of tumor removal begins, usually by dissecting out the adrenal vein. It is believed that the ligation of this drainage can help prevent strong hemodynamic swings and cardiac arrhythmias that may occur during tumor manipulation and removal.17 More recently, this thought is being challenged by researchers who believe neither early nor late takedown of the adrenal vein during removal has any benefit over the other.18 Keep in mind, it’s common practice to bolus a judicious amount of fluids before the patient’s adrenal vein is ligated to prevent robust hypotension from the new lack of significant catecholamines. The management of a patient with a pheochromocytoma is far from complete after skin closure. The immediate postoperative period has been proven to be yet another critical time in these patients. Patients can recover in the postanesthesia care unit or in the intensive care unit depending on the extent of their disease and intraoperative events. All invasive monitors and lines should be maintained. Hypotension is regularly seen postoperatively with the differential including residual anesthetic effects, under resuscitation, and prior or ongoing blood loss. Intraoperative fluid management, blood panel, and knowl­ edge of medications taken by the patient during and before surgery are necessary in guiding therapy. Long-​acting pharmacological agents used preoperatively, if not discontinued early enough, must remain high on the differential for possible culprits in consistently low blood pressures. Hypertension in the postanesthesia care unit should also be evaluated. It can be a sign of uncontrolled pain, fluid overload, or a return of autonomic reflexes, which should easily be addressed or something more sinister such as a residual tumor or inadvertent ligation of the renal artery.10 Hypoglycemia and its sequelae are a noted dangerous circumstance of resection. The etiology of hypoglycemia has been thought to be a consequence of catecholamine-​induced depletion of glycogen stores, overstimulation of insulin by alpha blockade medications, and/​or hyperinsulinemia after the stark drop in the catecholamine inhibitory effect on beta 2 receptors of the pancreatic islet cells. Glucose levels should be checked frequently over the next 24 hours, and insulin and dextrose-​ containing fluid should be readily available for the patient.19 Post resection, urinary catecholamines and plasma metanephrines must continue to be followed as an outpatient. Its recommended that they be measured as early as 2 weeks out from surgery and continue to be measured every 3 months for at least the first year. If no elevation in the pertinent biomarkers exists, then they can be checked annually.20 Patients who have sporadic disease should continue to be followed for a minimum of 10 years, while those with extra-​adrenal or familial pheochromocytoma should have lifetime follow-​ up. The 5 year survival rate in benign pheochromocytomas has been studied to be as high as 96% while that in the malignant

pheochromocytoma population is less than half of that at 44%21 but was seen to be as low as 20% in a 1999 study.20 C O N C LUS I O N

Successful discovery and management of a pheochromocytoma is quite the undertaking but by no means is an impossible task. It begins by recognizing the mostly vague nonspecific symptoms as well as the more telltale ones to facilitate an early diagnosis and workup. Perioperative pharmacotherapy demands close, careful follow-​up preoperatively. The intraoperative management from both the surgical and anesthetic side rests on preparation and communication. Postoperatively, providers must be vigilant for hemodynamic and electrolyte disarrangements. The unpredictable nature of these tumors will still test the most seasoned anesthesiologist, but the experience is ultimately invaluable. R E VI EW Q U E S T I O N S 1. A  30-​year-​old woman presents to her doctor’s office with a 3-​ month history of spontaneous diaphoretic episodes, anxiety, and palpitations. A  workup is undertaken, and she found to have a unilateral pheochromocytoma. Considering the etiology of her disease is genetic, which of the following conditions would least likely be responsible for her condition? A. Multiple endocrine neoplasia type 2A B. Multiple endocrine neoplasia type 2B C. Multiple endocrine neoplasia type 1 D. Succinate dehydrogenase subunit B Answer: C Multiple endocrine neoplasia types 1, 2A, and 2B are all closely linked to the development of both benign and malignant neoplasias in the endocrine system. Type 1 has been found to predispose patients to pancreatic, pituitary, and parathyroid disease. Those with either type 2A or 2B have been found to develop medullary carcinomas and pheochromocytomas.4 Keep in mind there is no multiple endocrine neoplasia type 2 gene but there is a type 1 gene. Multiple endocrine neoplasia type 2 syndromes are found in patients with mutations in the RET gene. Mutations in the succinate dehydrogenase subunit B gene have been linked with development of pheochromocytomas.4 2. Paragangliomas have been found to manifest from various places in the body. Which of the following is NOT a common place where paragangliomas have been found? A. Hypothalamus B. Organ of Zuckerkandl C. Inferior mesenteric artery D. Bifurcation of the aorta Answer: A Paragangliomas can give rise anywhere within the paraganglionic system. Paragangliomas are derived from chromaffin tissue complexes, located para-​aortic and para-​vertebral

12. Active P h eoc h r omacytoma  •   109

extending from the cervical region to the pelvis. Research has shown as much as 85% of paragangliomas are found retroperitoneally with most being para-​aortic in location.2 The organ of Zuckerkandl is the most common extra-​adrenal site to find aberrant paraganglia growth. It is a chromaffin body derived from early embryonic neural crest tissue that can be found between the inferior mesenteric artery and the bifurcation of the aorta. Paragangliomas have been shown to form at both the inferior mesenteric artery and bifurcation of the aorta independently.2 Finally, although some paragangliomas have been located in the skull, literature has not revealed a finding of the hypothalamus giving rise to one. 3. The symptomatic triad that is commonly seen with pheochromocytomas include A. flank pain, headache, and dyspnea. B. headache, dyspnea, and palpitations. C. headache, palpitations, and diaphoresis. D. hypertension, palpitations, and diaphoresis. Answer: C Flank pain is seldom a presenting factor of most pheo­ chromocytomas. Considering pheochromocytomas can grow to achieve a variety of sizes it can be a symptom but is not included in the triad.2 Headache, palpitations, and diaphoresis are the classic triad associated with this disease. These have nothing to do with the actual neoplasm growth but instead are a consequence of the catecholamines that are excreted from the tumor tissue.6 Elevated circulating catecholamines are able to act on peripheral receptors, raising blood pressure to the point of end organ dysfunction manifested by headache. These same catecholamines are able to act on cardiac receptors to cause noticeable tachyarrythmias. In paroxysmal surges of catecholamines, it is common for sufferers to have diaphoresis due to the surge of vasoactive agents. Unexplained hypertension, be it paroxysmal or persistent, is a common sign of pheochromocytoma but it should not be high on your differential if this is the 1 symptom. Hypertension can present for a variety of reasons and is neither specific nor sensitive for pheochromocytoma.

so one would definitely see both elevated in an assay. Studies have investigated the utility of sitting/​lying supine before venipuncture. Critical illness will activate the sympathetic arm and release similar catecholamines as you would appreciate in a stressful situation as those mentioned. 5. The most sensitive radiological imaging tool for the identification and localization of pheochromocytomas/​ paragangliomas is A. CT  scan. B. ultrasound imaging. C. MRI. D. MIBG  scan. Answer: D CT scan and MRI are both radiological methods used in the workup of pheochromocytoma or paraganglioma. Localization of the suspected neoplasm is usually performed using either of these methods in the context of a patient with a positive urinary/​plasma catecholamine and metabolite assay. In the event that neither is able to pinpoint the neoplasm, an MIBG scan is usually sought. I-​metaiodobenzylguanidine is injected intravenously, allowed to circulate, and then its uptake is analyzed via gamma camera to assess uptake. Both pheochromocytomas and paragangliomas will have significantly elevated uptake appearing darker than surrounding structures on the scan. Its sensitivity has been found to be as high as 90%.11 Ultrasound imaging is not used in the localization of pheochromocytoma or paraganglioma. 6. The initial drug that should be started to manage pheochromocytoma patients in an outpatient setting is

A. oral atenolol. B. phenoxybenzamine. C. hydralazine. D. prazosin. Answer: B Beta blockade should never be attempted before the introduction of an alpha-​adrenergic blocking agent. In the presence of such management, there will be unopposed alpha 4. Which one of the following is NOT a reason why a false-​ agonism systemically resulting in a hypertensive crisis, positive plasma catecholamine and metanephrine assay stroke, and so on. Beta blockade is commonly introduced in the face of tachyarrhythmias after alpha-​adrenergic drugs would occur? have already been started. A  long-​acting alpha-​adrenergic A. Tricyclic antidepressants blocking agent is the cornerstone and initial treatment of B. Marijuana  use pheochromocytoma/​paragangliomas. The most commonly C. Stress used is phenoxybenzamine. The dose must be carefully D. Critical illness/​infection titrated by an experienced endocrinologist before the paAnswer: B tient is deemed ready to go for his or her tumor resection. Urine and plasma can be found to be elevated under many Hydralazine is a commonly used vasodilator. It has been circumstances unrelated to tumors producing catecholamines. used as an adjunct treatment of outpatient pheochromocyIt has been well documented in the literature that breakdown toma, but it is not the initial medication these should be products and metabolites of catecholamines will be found started on. Prazosin is a selective short acting alpha-​1 adrenelevated in patients who are taking tricyclic antidepressants. ergic blocking agent. It, along with doxazosin and terazosin, These drugs decrease synaptic uptake of catecholamines at the have been traditionally used close to the patient’s resection synapse leaving them to be available for further breakdown. day. They all have both a higher and stronger incidence of Stress (surgery, exercise, etc.) is a state where catecholamines postural hypotension after use making them less ideal agents are appropriately elevated to meet the demands of the body for outpatients. 110  •   A bdominal

7. Roizen’s criteria in perioperative pheochromocytoma patients include all the following EXCEPT A. blood pressure less than 160/​90 for 24 hours or less. B. heart rate under 110 for 24 hours or less. C. postural hypotension. D. EKG should be free of any ST changes for 1 week. E. no more than 5 premature ventricular contractions in 1 minute on EKG. Answer: B Roizen’s criteria should be used in preoperative evaluation to assess the appropriateness of patient’s alpha blockade before surgery. These are

Mainstay vasopressors in these cases include epinephrine, norepinepherine, and vasopressin. Even with these agents, it is not uncommon to have residual hypotension. Methylene blue has been found to be useful in these situations. Its use is limited to vessels that are considered vasoplegic, secondary to chronic circulating substances including catecholamines and inflammatory mediators. In these cases, there is vessel endothelial dysfunction promoting the production of nitric oxide, contributing to difficulty of the vasculature to vasoconstrict. Methylene blue works by inhibiting nitric oxide synthase.22 10. Common postoperative management of pheochromocytoma resection patients may involve dealing with all the following EXCEPT

1. No in-​hospital blood pressure >160/​90 mm Hg for 24 A. hypotension. hours prior to surgery. B. hyperglycemia. 2. No orthostatic hypotension with blood pressure 4 METs)

Poor

Functional capacity

Surgical risk

(tracheal >intercostal >caudal > paracervical >epidural >brachial plexus >sciatic >subcutaneous. Local tissue perfusion and the extent of plasma protein binding also play a role in systemic absorption. This concept is important to understand because once a drug concentration reaches toxic levels in the blood, adverse cardiac and neurologic events may follow.26,27 L O C A L A N E S T H ET I C TOX I C IT Y A N D T R E AT M E N T

Local anesthetics also interfere with the function of organs in which the conduction of nerve impulses occurs. These undesired effects are dependent on systemic plasma concentrations of the local anesthetic. The most important factors determining plasma concentration of local anesthetics are the dose of drug

16. Patient wit h M o r bid O besity and  O S A   •   143

administered the rate of absorption of the drug, biotransformation/​elimination of the drug from circulation, site of injection, and use of vasoconstrictors. As previously discussed, the rate of absorption of local anesthetics is greater in some areas of the body as compared to others. Administration of equivalent doses of a local anesthetic at 2 different sites can result in markedly different peak plasma concentrations, which may lead to local anesthetic toxicity.27 The progression of signs that occur during local anesthetic toxicity are drowsiness, paresthesias in the mouth and tongue, auditory hallucinations, muscular spasms, seizures, coma, respiratory arrest, and cardiac arrest. Central nervous system (CNS) and cardiovascular (CV) toxicity cause the most concern during local anesthetic toxicity. In the CNS, the initial signs of toxicity are restlessness, disorientation, and tremor. This eventually progresses to convulsions. It is postulated that local anesthetics first interrupt transmission of inhibitory neurons in the CNS once toxic levels are reached, producing a state where excitatory neurons become unopposed.26 As the level of local anesthetic rises, CNS symptoms eventually progress to coma, respiratory failure, and death. Treatment for CNS toxicity involves the use of benzodiazepines and propofol to halt seizure activity. Typical doses of propofol used are 0.5 to 1 mg/​kg. CNS toxicity rapidly resolves because drugs are rapidly redistributed to other tissues in the body.27 In the CV system, local anesthetics primarily target the myocardium when plasma concentrations reach toxic levels. They decrease the force of myocardial contractions and conduction rate. These complications tend to occur at higher systemic concentrations when compared to CNS toxicity. The CV depressant effects are directly correlated with the potency of the local anesthetic, making bupivacaine more likely to cause CV toxicity than lidocaine. Prolonged cardiac nerve conduction eventually leads to widening of the QRS complex, followed by rapid onset of arrhythmias such as ventricular fibrillation. Maintenance of circulation is the most important step in treatment of CV toxicity of local anesthetics.27 This may require the use of chest compressions as per advanced cardiac life support protocol. Lipid emulsion therapy can also be used and has been shown to be effective in returning a patient to circulatory stability. However, the exact mechanism of action remains unknown. It is theorized that the emulsion functions as a lipid sink, which takes up local anesthetic and eventually allows cardiac nerve conduction to resume.27,28 The recommended dose for lipid emulsion (20%) therapy is 1.5 mL/​kg IV bolus over 2 to 3 minutes followed by an IV infusion of lipid emulsion 0.25 mL/​kg/​min until circulatory stability is achieve. The initial bolus may be repeated if CV collapse persists.29 According to the American Society for Regional Anesthesia guidelines on local anesthetic, the upper limit of lipid emulsion therapy is 10 mL/​kg over the first 30 minutes.30 C O N C LUS I O N

• Peripheral nerve block, specifically, interscalene brachial plexus block is an effective alternative to general anesthesia in obtaining anesthesia and analgesia for shoulder arthroscopy.

• Bezold-​Jarisch reflex (hypotension, bradycardia, coronary vasodilation) may occur after interscalene brachial plexus blockade, while the patient is in the sitting position for shoulder arthroscopy. • PNB can be performed for various surgical subspecialty procedures such as orthopedic, gastrointestinal, and vascular surgery. All parties (patient, surgeon, and anesthesiologist) involved must agree before PNB is performed. • The absolute contraindications to PNB include patient refusal, sepsis, infection at the injection site, allergy to local anesthetics, and an increased risk for local anesthetic toxicity. • Complications of performing a PNB include neurologic injury, bleeding, infection, intravascular injection of local anesthetic, pneumothorax, phrenic nerve blockade, Horner’s syndrome, local anesthetic toxicity, and block failure. • Adjuvants such as dexamethasone, epinephrine, clonidine, and sodium bicarbonate can be added to the local anesthetic to speed onset or prolong duration of nerve block. • The progression that occurs during local anesthetic toxicity are signs of drowsiness, paresthesias in the mouth and tongue, auditory hallucinations, muscular spasm, seizures, coma, respiratory arrest, and, finally, cardiac arrest. • Lipid emulsion is a treatment of local anesthetic toxicity. The recommended dose for lipid emulsion (20%) therapy is 1.5 mL/​kg IV bolus over 2 to 3 minutes followed by an IV infusion of lipid emulsion 0.25 mL/​kg/​min until after circulatory stability is achieve. The initial bolus may be repeated if CV collapse persists. R E VI EW Q U E S T I O N S 1. A 50-​year-​old man with history of chronic obstructive pulmonary disease, insulin-​dependent diabetes, hypertension, and chronic kidney disease is scheduled to have his rotator cuff repair in an ambulatory surgery center. Which of the following is true? A . A  supraclavicular block would be the best option since it effectively blocks the brachial plexus for the shoulder. B. The surgery can be performed with sedation and local infiltration of anesthesia provided by the surgeon. C. An interscalene block would be best to cover the shoulder and requires less local anesthetic volume. D. An infraclavicular block is performed at the distal trunks and can be used to provide adequate blockade for the shoulder. Answer: C “The interscalene technique of brachial plexus blockade is ideally suited for shoulder procedures.”12p324–356

144  •   O rt h opedics

2. Which brachial plexus nerve block is best at holding peripheral nerve catheters? A. Axillary brachial plexus block B. Supraclavicular brachial plexus block C. Infraclavicular brachial plexus block D. Interscalene brachial plexus block Answer: C “Infraclavicular block is well-​suited for catheter technique because the musculature of the chest wall helps stabilize the catheter and prevents its dislodgment compared with the more superficial location with the interscalene or supraclavicular approaches.”31

7. Which one is an absolute contraindication to PNB? A. Patient on coumadin, first dose was taken yesterday B. Pediatric patient C. History of peripheral neuropathy D. Infection at the site of injection Answer: D Infection at the site of the injection decreases the effectiveness of local anesthetic.12 8. What effect would infection/​inflammation at the site injection have on a local anesthetic?

A. Bradycardia, vasodilation, and hypotension B. Tachycardia, vasoconstriction, and hypertension C. Hypotension, miosis, and anhidrosis D. Hypertension, mydriasis, and vasodilation Answer: A The Bezold-​Jarisch reflex occurs when stimulation of inhibitory cardiac receptors by mechanical, chemical substances, or drugs lead to increase in parasympathetic activity (bradycardia, vasodilation, and hypotension).3

A. Decreased pH of local tissue would delay onset of action and efficacy of local anesthetics B. Decreased pH of local tissue would shorten the onset of action and increase the efficacy of local anesthetics C. The local infection/​inflammation would only hinder the action of certain local anesthetics while leaving others unaffected. D. Infection/​ inflammation has no effect on local anesthetics at the site of administration. Answer: A Infection at the site of the injection decreases the effectiveness of local anesthetic.12

4. What dermatome is not covered by an interscalene brachial plexus block?

9. What effect does epinephrine have on the local anesthetic mixture?

A. C5–​C6 B. C6–​C7 C. C7–​C8 D. C8–​T1 Answer: D C8–​ T1 dermatome covers the ulnar nerve and medial antebrachial cutaneous nerve. At the interscalene brachial plexus anatomical position, the C8–​T1 branch of the brachial plexus is not visualize thus local anesthetic cannot be distributed around the nerve.12

A. Epinephrine is vasodilator which will increase the uptake of local anesthetic and prolong its action. B. Epinephrine is a vasoconstrictor and it decreases the uptake of local anesthetic which prolongs the duration of action. C. Epinephrine reduces the pH of the local anesthetic solution and increases the proportion of uncharged, lipid soluble molecules of local anesthetic which decrease the onset time of the drug. D. Epinephrine has no effect on the local anesthetic mixture Answer: B Epinephrine is a vasoconstrictor and it decreases the uptake of local anesthetic which prolongs the duration of action.21

3. What are three common signs of Bezold-​Jarisch reflex?

5. What adjuvant speeds onset of action of local anesthetic? A. Dexamethasone B. Sodium Bicarbonate C. Clonidine D. Opioids Answer: B Sodium bicarbonate speeds up onset by alkalization of local anesthetic17 6. What PNB can be performed for ankle surgery? A. Femoral nerve block B. Popliteal nerve block C. Adductor canal block D. Lumbar plexus block Answer: B Ankle surgery can be performed solely on popliteal nerve block. However, if incision is on the medial aspect of the ankle, a saphenous nerve block should be combined with a popliteal nerve block.16

10. What makes bupivacaine more cardiotoxic compared to other local anesthetics? A. Bupivacaine has a decreased clearance from plasma relative to other local anesthetics. B. Bupivacaine causes direct damage to myocardial tissue. C. Bupivacaine preferentially binds to cardiac sodium channels and takes longer to dissociate from these channels when compared to other local anesthetics. D. Bupivacaine if the local anesthetic with the highest pKa which makes it the most lipid soluble and most potent. Answer: C Bupivacaine preferentially binds to cardiac sodium channels and takes longer to dissociate from these channels when compared to other local anesthetics.27

16. Patient wit h M o r bid O besity and  O S A   •   145

QUESTIONS AND ANSWER S This chapter has accompanying questions and answers which are available to subscribers as part of the Oxford eLearning platform. To access the questions, go to http://​ oxfordmedicine.com/​anesthesiologyPBL REFERENCES 1. Urmey WF, McDonald M. Hemidiaphragmatic paresis during interscalene brachial plexus block: effects on pulmonary function and chest wall mechanics. Anesth Analg. 1992;74(3):352–​357. 2. Thames M, Klopfenstein H, Abboud F, et  al. Preferential distribution of inhibitory cardiac receptors with vagal afferents to the inferoposterior wall of the dog. Circ Res. 1978;43:512–​519. 3. D’Alessio JG, Weller RS, Rosenblum M. Activation of the Bezold-​ Jarisch reflex in the sitting position for shoulder arthroscopy using interscalene block. Anesth Analg. 1995;80(6):1158–​1162. 4. Martin F, Martinez V, Mazoit JX, et al. Anti-​inflammatory effect of peripheral nerve blocks after knee surgery: clinical and biologic evaluation, Anesthesiology. 2008;109:484–​490. 5. Capdevila X, Barthelet Y, Biboulet P, Ryckwaert Y, Rubenovitch J, d’Athis F. Effects of perioperative analgesic technique on the surgical outcome and duration of rehabilitation after major knee surgery. Anesthesiology. 1999;91:8–​15. 6. Rodgers A, Walker N, Schug S, et al. Reduction of postoperative mortality and morbidity with epidural or spinal anaesthesia: results from overview of randomized trials. BMJ. 2000;321:1493. 7. Garnett RL, MacIntyre A, Lindsay P, et al. Perioperative ischemia in aortic surgery:  combined epidural/​general anesthesia and epidural analgesia vs. general anesthesia and I.V analgesia. Can J Anaesth. 1996;43:769–​777. 8. Freise H, Daudel F, Grosserichter C, et  al. Thoracic epidural anesthesia reverses sepsis-​induced hepatic hyperperfusion and reduces leukocyte adhesion in septic rats. Crit Care. 2009;13:R116. 9. Beilin B, Bessler H, Mayburd E, et  al. Effects of preemptive analgesia on pain and cytokine production in the postoperative period. Anesthesiology. 2003;98:151–​155. 10. Sessler DI. Does regional analgesia reduce the risk of cancer recurrence? A hypothesis. Eur J Cancer Prev. 2008;17:269–​272. 11. Exadaktylos AK, Buggy DJ, Moriarty DC, Moscha E, Sessler DI. Can anesthetic technique for primary breast cancer surgery affect recurrence or metastasis? Anesthesiology. 2006;105:660–​664. 12. Morgan, GE, Mikhail MS, Murray MJ. Peripheral nerve blocks. In: Butterworth J, Mackey DC, Wasnick JD, eds. Clinical Anesthesiology. 4th ed. New York: Lange Medical Books; 2006:324–​356. 13. Hogan QH. Pathophysiology of peripheral nerve injury during regional anesthesia. Reg Anesth Pain Med. 2008;33(5):435–​441.

14. Sorenson EJ. Neurological injuries associated with regional anesthesia. Reg Anesth Pain Med. 2008;33(5):442–​448. 15. Eichenberger U, Stockli S, Marhofer P, et al. Minimal local anesthetic volume for peripheral nerve block: a new ultrasound guided, nerve dimension-​based method. Reg Anesth Pain Med. 2009;34:242–​246. 16. Gadsden J. Indications for peripheral nerve blocks. August 26, 2013. NYSORA.com. Accessed April 1, 2017. 17. Ririe DG, Walker FO, James RL. Effect of alkalization of lidocaine on median nerve block. Br J Anaesth. 2000;84:163–​168. 18. Williams BA, Murinson BB, Grable BR, Orebaugh SL. Future considerations for pharmacologic adjuvants in single-​injection peripheral nerve blocks for patients with diabetes mellitus. Reg Anesth Pain Med. 2009;34:445–​457. 19. Cummings KC, Napierkowski DE, Parra-​Sanchez I, et  al. Effect of dexamethasone on the duration of interscalene nerve blocks with ropivacaine or bupivacaine. Br J Anaesth. 2011;107:446–​453. 20. Weber A, Fournier R, Van Gessel E, et al. Epinephrine does not prolong the analgesia of 20ml ropivacaine 0.5% or 0.2% in a femoral three-​in-​one block. Anesth Analg. 2001;93:1327–​1331. 21. Murphy DB, McCartney CJ, Chan VW. Novel analgesic adjuncts for brachial plexus block:  a systematic review. Anesth Analg. 2000;90:1122–​1128. 22. Popping DM, Elia N, Marret E, et al. Clonidine as an adjuvant to local anesthetics for peripheral nerve and plexus blocks: a meta-​analysis of randomized trials. Anesthesiology. 2009;111:406–​415. 23. Jadon A, Panigrahi MR, Parida SS, et  al. Buprenorphine improves the efficacy of bupivacaine in nerve plexus block:  a double blind randomized evaluation in subclaviane perivascular brachial block. J Anaesthesiol Clin Pharmacol. 2009;25(2):207–​210. 24. Karakaya D, Buyukgoz F, Baris S, et  al. Addition of fentanyl to bupivacaine prolongs anesthesia and analgesia in axillary brachial plexus block. Reg Anesth Pain Med. 2001;26(5):434–​438. 25. Becker DE, Reed KL. Essentials of local anesthetic pharmacology. Anesthesia Progress. 2006;53(3):98–​109. 26. Lin Y, Liu SS. Local anesthetics. In: Barash PG, Cullen BF, Stoelting RK, et al., eds. Clinical Anesthesia. 7th ed. Philadelphia: Lippincott; 2013:561–​582. 27. Gadsden J. Local anesthetics: clinical pharmacology and rational selection. October 14, 2013. NYSORA.com. Accessed April 13, 2017. 28. Ottokar S, Swamidoss CP. Anesthetic techniques and their clinical application for specific orthopedic procedures. In:  Mackenzie CR, Cornell CN, Stavros G, Memtsoudis CN, eds. Perioperative Care of the Orthopedic Patient. New York: Springer; 2014:81. 29. Fusco N. ToxTidbits:  antidote facts. Maryland Poison Center, University School of Pharmacy, May 2011 https://www.mdpoison. com/healthcareprofessionals/antidotes.html. 30. Neal JM, Bernards CM, Butterworth JF, et  al. ASRA practice advisory on local anesthetic systemic toxicity. Reg Anesth Pain Med. 2010;35:152–​161. 31. Ultrasound-​ g uided infraclavicular brachial plexus block. June 8, 2013. NYSORA.com. Accessed June 12, 2017.

146  •   O rt h opedics

SECTION VIII GENITOURINARY

17. ROB OTIC PROSTATECTOMY Terry Ann Chambers

S T E M C A S E A N D K EY Q U E S T I O N S A 63-​year-​old male from Australia, after several bouts with urinary incontinence, was found to have prostate cancer. His brother was also recently diagnosed with prostate cancer. Due to this strong family history, the surgeon decides to do a robotic-​assisted radical prostatectomy (RARP). He has hypertension, a 20-​pack/​year smoking history and is obese. The surgeon sends him to your preoperative clinic. WH AT A R E T H E R I S K FAC TO R S F O R P RO S TAT E C A N C E R ?

Prostate cancer is the most common nonskin cancer in the United States. Even though 60% of all prostate cancers are diagnosed after the age of 65, the rates of diagnosis increase at age 40, going from 1 in 10,000 to 1 in 39. Hence, age is the biggest risk factor. Family history also plays a significant role, with 1 family member doubling your risk of diagnosis and 2 family members quadrupling the risk. A family history of other cancers, such as ovarian, breast, or colon may suggest a genetic component such as BRCA mutation. African men are also 73% more likely to get diagnosed and twice as likely to die from the disease. Obesity, while not a risk factor for overall diagnosis, is associated with a more aggressive form of the disease. Other lifestyle risks factors for an aggressive form to prostate cancer include lack of vegetables in the diet and smoking.1

A  reasonable approach according to the National Cancer Institute suggests confirming an elevated level in an asymptomatic patient with a repeat test and following the patient with serial PSAs and digital rectal exams over time. Continued rise in PSA or a suspicious mass could then be evaluated with imaging such as a transrectal ultrasound and tests to rule out infection. If suspicion is high, a biopsy would be recommended. Other biomarkers, such as urinary prostate cancer antigen 3 RNA, may enable more accurate prostate cancer screening compared to PSA alone. Currently, this test is only approved as a diagnostic test after a negative biopsy. Researchers are still trying to define the ideal clinical parameters for its use.3 T H E PAT I E N T H A S S TO P P E D S MO K I N G F O R 1 Y E A R A N D TA K E S E NA L A P R I L F O R H I S H Y P E RT E NS I O N. WH AT WI L L YO U I NS T RU C T H I M TO D O ?

Smoking at the time of prostate cancer diagnosis is associated with increased prostate cancer-​specific mortality and recurrence, as well as cardiovascular disease mortality. Those who have quit smoking for 10  years have prostate cancer mortality similar to never-​smokers.4 This patient has stopped smoking, which will improve his prostate cancer specific prognosis over time.5 Smoking is an independent risk factor for postoperative morbidity. He has quit smoking for over a year and minimized the probability of perioperative pulmonary debilitation related to long-​ term smoking and eliminated the need for bridging therapy such as a nicotine H OW I S P RO S TAT E C A N C E R D I AG N O S E D ? patch while hospitalized.6,7 Hypertension may be associated with an increased risk To detect clinically significant tumors, a combination of prostate-​specific antigen (PSA), digital rectal exam, and bi- of prostate cancer. Its high prevalence in this population opsy is used. The use of PSA as a screening tool has facilitated is being investigated as a possible causative factor.8 Use of diagnosis at an earlier stage (localized disease) and at a younger antihypertensives has also been investigated for its effect on age. This facilitates a greater chance of clean margins at sur- prostate cancer and does not appear to be a factor.9 The use of gical resection. However, PSA level increases with age and is angiotensin converting enzyme inhibitors and angiotensin II higher with benign conditions such as benign prostatic hy- receptor blockers accentuate the hypotension caused by anperplasia, urinary tract infection, and prostatitis. A high false-​ esthesia. However, this may be a dose dependent effect, and positive exists, such that only 25% of men with an elevated withholding this drug may increase the incidence of postopPSA (>4 ng/​mL) are actually found to have prostate cancer erative hypertensive episodes. The most recent meta-​analysis on biopsy.2 There are varying expert opinions, but no clear did not find any increase in mortality or major cardiac event consensus exists on the PSA threshold to recommend biopsy. whether or not the drug was continued.10

149

T H E PAT I E N T ’S B O DY M A S S I N D E X I S 33; H OWEV E R , H I S A I RWAY E X A M I S R E A S S U R I N G. A R E T H E R E A N Y F U RT H E R C O N S I D E R AT I O NS S P E C I F I C TO T H I S P RO C E D U R E?

The robotic prostatectomy is accomplished in the lithotomy position with steep Trendelenburg—​around 45 degrees. The arms are usually tucked in. This requires attention to proper padding and restraints to keep the patient on the operating table and avoid injury. Restraints across the shoulders have been abandoned due to reports of nerve injury. Nonslip padding is secured to the bed and the patient restrained to this padding to prevent patient movement. The face should also be protected to avoid unintentional injury from the movement of the robotic arms and to avoid accidental dislodgement of the endotracheal tube. Access to the patient is limited once the robot is docked and in position. All lines and monitors should be checked and in place, and the patient positioned and padded well to accommodate surgery for >6 hours. Surgical time may be reduced in centers with more experienced operators; however, many centers still have a steep learning curve. After induction of anesthesia, line placement, and positioning, there is still docking time and console time prior to the beginning of actual meaningful surgery. Actual starting time may take up to 2 hours. Movement of the patient or manual cardiopulmonary resuscitation efforts must wait until the robot is undocked.11 Other nonsurgical risks for this procedure include subcutaneous emphysema, pneumothorax/​ pneumomediastinum, corneal abrasion, and venous gas embolism.12 In an obese patient, these issues may be more challenging. The arms may have to be in the less than 90-​degree adducted position, and the position of the robot and surgical team adjusted. While the operative time is increased in obese patients, the actual clinical outcomes appear to be similar.13 O N T H E DAY O F S U RG E RY, T H E PAT I E N T I S E X T R E M E LY A N X I O US B E C AUS E H E H A S F O U N D T H AT T H E R E I S A R I S K O F P E R I O P E R AT I V E V I S UA L L O S S F RO M H I S G O O G L E S E A RC H. H OW WI L L YO U R E A S S U R E  H I M ?

In the steep Trendelenburg position required for robotic surgery, there is a decrease in venous drainage that contributes to the significant global tissue edema of the upper thorax and head. There is not a definitive relationship between intraocular and intracranial pressure. However, there is a correlation between intraocular pressure and central venous pressure.14 In addition, increased intraocular pressure and transient visual field deficits have been documented in RALP procedures.15 These increased intraocular pressures have been correlated with increases in mean arterial pressure and peak inspiratory pressure.16 Case reports of posterior optic neuropathy after robotic prostatectomy have been reported.17 A consensus statement by the Anesthesia Patient Safety Foundation suggests that (i) the prone position in major spine surgery, (ii) steep Trendelenburg position in robotic pelvic surgery, (iii)

intra-​abdominal pressure from insufflation, (iv) low intravenous (IV) colloid–​crystalloid ratios, and (v) a long duration of such conditions could all increase the risk of retro-​orbital and/​ or optic nerve edema and compartment syndrome.18 They encourage anesthesia professionals and surgeons to discuss the risk, though remote, of partial or complete blindness. Possible interventions may include minimizing intraabdominal pressures, reducing the degree of Trendelenburg, increasing the colloid to crystalloid ratio, monitoring anemia, and consider staging the procedure to reduce operative time. T H E PAT I E N T I S U N D E R   G E N E R A L ANESTHESIA AND HAS BEEN POSITIONED A N D W E L L PA D D E D. T H E T R O C A R S A R E P L AC E D A N D P N EU M O P E R I TO N EU M I S AC H I EV E D. T H E S U R G E O N A S K S F O R   S T E E P T R E N D E L E N B U R G . H E A S K S F O R   E X T R A-​L O N G T R O C A R S A N D T E L L S YO U T H I S W I L L B E A D I F F I C U LT P R O C E D U R E A N D M AY L A S T 6 H O U R S . W H AT C A R D I O VA S C U L A R C H A N G E S D O YO U E X P E C T ? H OW W I L L YO U M A NAG E H I S P U L M O NA RY M E C H A N I C S ? W H AT A B O U T H I S F LU I D S ?

A small but extremely detailed and informative study was done by LeStar et  al.19 in 2011 in American Society of Anesthesiology physical status (ASA) I and II patients undergoing RALP to elucidate the hemodynamic changes that occurred during the procedure. Central venous pressure, mean pulmonary artery pressure, and pulmonary capillary wedge pressure saw a greater than twofold increase in the steep Trendelenburg position. The wedge pressures were of that seen in heart failure (>18 mm Hg). Pneumoperitoneum increased systemic vascular resistance by 20%; however, this normalized with Trendelenburg. Notably, right ventricle and left ventricle stroke work increased by 65%. (See Table 17.1). They used volume control ventilation to ensure a stable tidal volume. After pneumoperitoneum, they noted an increase in peak inspiratory pressure of 46% and in maximum inspiratory pressure of 28%. Trendelenburg added a further increase in peak inspiratory pressure of 20% that was not statistically significant. Lung compliance decreased by more than 50% with pneumoperitoneum and Trendelenburg. Pulmonary shunt was unaltered, and ventilation perfusion distribution remained the same. Current teaching with pneumoperitoneum sees diaphragmatic shift cephalad decreasing functional residual capacity by 50%. Pneumoperitoneum causes blood flow to mesenteric and renal arteries to fall, leading to a decrease in urinary output. Typically, an increase in preload with subsequent decrease over time via decreased venous return of preload and cardiac output may be seen, but augmentation by steep Trendelenburg may mitigate these changes.20 Postsurgical parameters after exsufflation saw a hyperdynamic pattern, including rises in ETCO2, but without a rise in PaCO2. A  more recent study larger study in ASA physical status II and III patients documented a fall in PaO2 and a rise in PaCO2 with steep Trendelenburg which is more in keeping with our clinical observations.21

150  •   G enitou r ina ry

Table 17.1 HEMODYNAMIC AND PARAMETER CHANGES IN ROBOTIC PROSTATECTOMY PNEUMOPERITONEUM

TRENDELENBURG 45º

POSTSURGERY EXSUFFLATION

HR

No change

No change

Increase

CO

No change

No change

Increase

MAP

Increase

Increase

Back to baseline

MPAP

Increase

Increase

Back to baseline

PVR

No change

No change

No change

ETCO2

No change

Increase

Increase

PaCO2

No change

No change

No change

POSITION a

Change in parameters compared to supine position; changes only if they were statistically significant.

a

HR = heart rate; CO = cardiac output; MAP = mean arterial pressure; MPAP = mean pulmonary artery pressure; PVR = pulmonary vascular resistance; ETCO2 = end-tidal carbon dioxide; PaCO2 = partial pressure of carbon dioxide Adapted from Lestar M, Gunnarsson L, Lagerstrand L, Wiklund P, Odeberg-​Wernerman S. Hemodynamic perturbations during robot assisted laparoscopic radical prostatectomy in 45 degree Trendelenburg position. Anesth Analg. 2011;113(5):1069–​1075.

It is reassuring that there were no changes in cardiac output or heart rate with the extreme changes in position. However, these were relatively healthy patients. Knowing that for pulmonary arterial and wedge pressures in the realm of heart failure stroke work is increased by more 50% and lung compliance decreases by more than 50% in the steep Trendelenburg position is useful for the anesthesiologist trying to make decisions regarding care of patients with decreased cardiac reserve. Even the exsufflation parameters warrant increased vigilance from the anesthesiologist for patients in whom an increased heart rate might precipitate a demand ischemia. Fluid management in a patient with pneumoperitoneum and steep Trendelenburg undergoing a long procedure is challenging. The steep Trendelenburg causes venous congestion, which can be exacerbated by excess fluids, and the pneumoperitoneum decreases renal blood flow such that you cannot use urinary output as a reliable indicator of volume status. The length of the procedure, of course, compounds the problem. The recommended practice is to limit fluids to 1 to 2 L during the first few hours until the anastomosis is complete. This allows the surgeon to complete the most difficult aspect of the procedure with little tissue edema. Once this is complete, fluids are then liberalized, and urinary output is checked after pneumoperitoneum is relieved. Due to the augmentation of blood flow by steep Trendelenburg, this strategy may be fine. However, for the procedure that may last more than 3 hours, a patient who is volume depleted may not tolerate fluid restriction. A  combination of colloid such as albumin and crystalloid may allow more adequate volume resuscitation without exacerbating edema. Increased colloids to crystalloid ratios have been suggested by the Anesthesia Patient Safety Foundation as 1 strategy to limit venous congestion and prevent perioperative visual loss.22

T H E PAT I E N T H A S FAC I A L A N D P E R I O R B ITA L E D E M A AT T H E E N D O F T H E P RO C E D U R E . B L O O D L O S S WA S M I N I M A L . WI L L YO U E X T U BAT E  H I M ?

The accepted standard criteria for extubation should be followed, including return of consciousness and spontaneous respiration, resolution of neuromuscular blockade, and the ability to follow simple commands while recognizing that significant tracheal edema may be present. Consider checking for an adequate leak around the endotracheal tube prior to extubation. Minimal blood loss and lack of wide hemodynamic perturbations during the case are all reassuring parameters when evaluating for extubation. Given that steep trendelenburg is a now a cited risk factor for postoperative visual loss, the patient’s concern, the lack of specific etiology and treatment, and the devastating impact on quality of life, the patient should be assessed as soon as they are awake and alert for any visual disturbance. If there are changes, the team should consider a postprocedure consult with an ophthalmologist to assess the patient’s visual field and acuity and investigate etiology.23 DISCUSSION Robotic assisted laparascopic prostatectomy is the most commonly performed robotic surgery. According to Intuitive Surgical, robotic-assisted surgery accounted for 87% of minimally invasive prostatectomies performed in 2013. Blood loss is typically 60 years of age to get a biopsy. 3. When positioning an obese patient for robotic prostatectomy, the following modifications may be necessary: A. Padding of shoulders, head, and neck B. Securing head with tape to the bed C. Abducting the arms 30 D. Glaucoma E. None of the above Answer: A While a less experienced surgeon may decide not to a robotic procedure on a patient with previoius abdominal or pelvic surgery, there are no absolute contraindications listed. 8. The anesthesiologist would require further studies and expert consultation prior to doing the case if: A. the patient has hypertension, diabetes and smokes 3 cigarettes/​day. B. the patient is on aspirin which he stopped 7 days ago. C. the patient has mild renal insufficiency. D. the patient has pulmonary hypertension diagnosed 2 years ago. Answer: D All the conditions listed can be evaluated adequately by a primary care physician, good history and physical and basic laboratory values. Pulmonary hypertension should have involvement of a cardiologist, with recent values and interventions given this procedure will augment the pulmonary artery pressures. QUESTIONS AND ANSWER S This chapter has accompanying questions and answers which are available to subscribers as part of the Oxford eLearning platform. To access the questions, go to http://​ oxfordmedicine.com/​anesthesiologyPBL REFERENCES

6. The risk of perioperative visual loss should be: A. Discussed with the patient by the surgeon or anesthesiologist and the precautions being taken to prevent its occurrence. B. Hidden from the patient to prevent preoperative anxiety. C. Minimal if it has not occurred at your institution. D. Assessed as soon as the patient is awake and alert. E. A and  D Answer: E The risk of postoperative visual loss is related to procedure and while rare is not known to be institution specific. Willfully hiding risks from patients is unethical. APSF encourages discussion of this rare risk due to how devastating it is and visual assessment should occur as soon as patient is able to participate. 7. The surgeon may cancel the case after being told which of the following: A. The patient had an exploratory laparotomy for bowel obstruction 6 years ago and a hernia repair 4 years ago.

1. Prostate Cancer Foundation. www.pcf.org/​c/​prostate-​cancer-​risk​factors 2. Barry MJ. Clinical practice: prostate-​specific-​antigen testing for early diagnosis of prostate cancer. N Engl J Med. 2001;344(18):1373–​1377. 3. Bhavna M, Feng FY. Long noncoding RNAs in prostate cancer:  overview and clinical implications. Asian J Androl. 2016 Jul-​Aug;18(4):568–​574. 4. Kenfield S, Stampfer M, Chan JM, et al. Smoking and prostate cancer survival and recurrence. JAMA. 2011 Jun 22;305(24):2548–​2555. 5. Rieken M, Shariat SF, Kluth LA, et  al. Association of Cigarette Smoking and Smoking Cessation with Biochemical Recurrence of Prostate Cancer in Patients Treated with Radical Prostatectomy. Eur Urol. 2015 Dec;68(6):949–​956. 6. Warner DO. Perioperative abstinence from cigarettes:  physiologic and clinical consequences. Anesthesiology. 2006;104:356–​367. 7. Theadom A, Cropley M. Effects of preoperative smoking cessation on the incidence and risk of intraoperative and postoperative complications in adult smokers:  a systematic review. Tob Control. 2006;15:352–​358. 8. Liang Z, Xie B, Li J, et al. Hypertension and risk of prostate cancer: a systematic review and metanalysis. Sci Rep. 2016;6:31358. 9. Pai PY, Hsieh VC, Wang CB, et al. Long term antihypertensive drug use and prostate cancer risk: a 9-​year population-​based cohort analysis. Int J Cardiol. 2015 Aug 15;193:1–​7.

17. Robotic P r ostatectomy  •   153

10. Holman C, Fernandes NL, Biccard BM. A systematic review of outcomes associated with withholding or continuing angiotensin-​ converting enzyme inhibitors and angiotensin receptor blockers before noncardiac surgery. Anesth Analg. 2018 Jan 29. 11. Baltayian S. A brief review: anesthesia for robotic surgery. J Robotic Surg. 2008;2:59–​66. 12. Hong JY, Kim WO, Kil HK. Detection of subclinical CO2 embolism by transesophageal echocardiography during laparoscopic radical prostatectomy. Urology. 2010;75:581–​584. 13. Gu X, Araki M, Wong C. Does elevated body mass index (BMI) affect the clinical outcomes of robot-​assisted laparoscopic prostatectomy (RALP)? A  prospective cohort study. Int J Surg. 2014 Oct; 12(10):1055–​1060. 14. Lee LA, Vavilala MS, Sires BS. Intraocular pressure is partially dependent on central venous pressure during prone spine surgery. Anesthesiology. 2003;99:A289. 15. Taketani Y, Mayama C, Suzuki N, et al.. Transient but significant visual field defects after robot-​assisted laparoscopic radical prostatectomy in deep Trendelenburg position. PLoS One. 2015;10(4):e0123361. 16. Blecha S, Harth M, Schlachetzki F, et al. Changes in intraocular pressure and optic nerve sheath diameter in patients undergoing robotic-​ assisted laparoscopic prostatectomy in steep 45° Trendelenburg position. BMC Anesthesiol. 2017 Mar 11;17(1):40. 17. Lee LA. Visual loss, venous congestion and robotic prostatectomies. ASA Newslett. 2011;75:26–​27. 18. LeStar M, Gunnarsson L, Lagerstrand L, et  al. Hemodynamic perturbations during robot assisted laparoscopic radical

prostatectomy in 45 degree Trendelenburg position. Anesth Analg. 2011;113(5):1069–​1075. 19. Haas S, Haese A, Goetz AE, et  al. Haemodynamics and cardiac function during robotic-​ assisted laparoscopic prostatectomy in steep Trendelenburg position. Int J Med Robot. 2011 Dec;7(4):408–​413. 20. Lebowitz P, Yedlin A, Hakimi AA, et  al. Respiratory gas exchange during robotic-​assisted laparoscopic radical prostatectomy. J Clin Anesth. 2015 Sep;27(6):470–​475. 21. Lee LA, Stoelting RK. APSF-​sponsored conference on perioperative visual loss develops consensus conclusions. APSF Newslett. 2013;27:52–​53. 22. Barash PG, Cullen BF, Stoelting RK, eds. Clinical Anesthesia. 5th ed. Philadelphia: Wolters Kluwer; 2006. 23. Lee L. “Blind Spot”. WebMM. PSNet Blind Spot AHRQ Patient Safety Network, June 2005. Web 25 May, 2018. 24. Ploussard G. Robotic surgery in urology: facts and reality. What are the real advantages of robotic approaches for prostate cancer patients? Curr Opinion in Urol. 2018;28(2):153–158. 25. Hatton KW, Kilinski LC, Chandrashekar R, Schell RM. Multiple failed external defibrillation attempts during robot-​assisted internal mammary harvest for myocardial revascularization. Anesth Analg. 2006;103:1113–​1114. 26. Tewari P. Cardioversion during closed chest robotic surgery:  relevance of pad position. Anesth Analg. 2007;105(2):542. 27. Huser AS, Müller D, Brunkhorst V, et al. Simulated life-​threatening emergency during robot-​ assisted surgery J Endourol. 2014 Jun;28(6):717–​721.

154  •   G enitou r ina ry

18. END-​S TAGE RENAL DISEASE—​PATIENT FOR KIDNEY TRANSPLANT Ferenc Gyulai

S T E M C A S E A N D K EY Q U E S T I O N S A 69-​year-​old man presents for preoperative evaluation prior to consideration of renal transplantation. He has a long-​ standing history of systemic hypertension, type 2 diabetes mellitus, obesity, and obstructive sleep apnea. He developed end-​stage renal disease (ESRD) as a result of diabetic nephropathy and has been on hemodialysis via right subclavian tunneled catheter for the last 3 years. Over the past few months, the patient reports worsening dyspnea with mild to moderate exertion and multiple episodes of paroxysmal nocturnal dyspnea over the past 6 months. He has also developed 2 pillow orthopnea and bilateral lower extremity. He denies chest discomfort, angina, palpitations, syncope, or presyncope. He has bilateral pleural effusions noted on chest X-​ray. Ventilation-​perfusion scan is low probability for pulmonary embolism. Lower extremity Doppler studies are negative for deep venous thrombosis. Chest computed tomography reveals no evidence of emphysema or interstitial lung disease but does reveal a large right pleural effusion. A screening pretransplant transthoracic echocardiogram showed normal left ventricular (LV) and right ventricular (RV) size and systolic function, with estimated pulmonary artery pressure of 60 mm Hg. He is therefore referred for further evaluation of his pulmonary hypertension in consideration of transplant.

is soft and nondistended. There is mild hepatojugular reflux noted. Extremities revealed 2 to 3+ bilateral pitting edema to the upper shins. WH AT D O E S T H E EC H O C A R D I O G R A M S H OW ?

A transthoracic echocardiogram is performed, and representative parasternal long and short axis views and apical 4 chamber views are shown. In addition, pulsed wave Doppler in the RV outflow tract and transmitral Doppler profiles are shown. LV systolic function is normal at 55%. Mitral regurgitation is noted as moderate. There is no mitral stenosis. The parasternal long axis view (Fig. 18.1, Panel A) confirms normal LV size with significant LV hypertrophy. The parasternal short axis views (Fig. 18.1, Panels B and C) reveal a notable absence of septal flattening in systole or diastole with a convexed septal profile. The apical 4-​chamber view (Fig. 18.1, Panel D) illustrates significant left atrial enlargement, while RV size is mildly dilated with RV:LV ratio of roughly 1.0. Notably, the RV apical angle is relatively acute and does not form or share the apex of the heart, and there is minimal RV hypertrophy, These 2D findings do not support the presence of peripheral vascular disease. On hemodynamic assessment, pulse wave Doppler in the RV outflow tract (RVOT) (Fig. 18.1, Panel E) reveals a normal parabolic profile without “notching.” The RVOT acceleration time is low-​normal at 110 ms. These findings together strongly suggest a normal pulmonary vascular resistance. The velocity time integral in the RVOT is in the normal range (14  cm), WH AT I S T H I S PAT I E N T ’S P R E S E N TAT I O N implying normal RV stroke volume. Transmitral pulse wave O N P H Y S I C A L E X A M I NAT I O N ? Doppler (Fig. 18.1, Panel F) shows a restrictive inflow pattern, On examination, blood pressure is 178/​60 mm Hg, pulse is consistent with high left atrial pressure. Lastly, tricuspid an64 beats per minute, respiratory rate is 12 breaths per minute, nular plane systolic excursion performed via M-​mode (Fig. and oxygen saturation is 90% on room air. There is a square-​ 18.1, Panel G) confirms normal RV function with tricuspid wave response in systolic blood pressure to Valsalva maneuver. annular plane systolic excursion of 23 mm. In general, the patient is a mildly obese, well-​appearing In this case, the findings on history and physical examinaman in no apparent distress. Jugular venous pressure is 12 cm tion combined with the echocardiographic findings strongly of water with normal venous contours. On cardiac examina- suggest a hemodynamic phenotype of heart failure with pretion, S1 and S2 are normal with normal P2 intensity. There is served ejection fraction and secondary pulmonary hypera 2/​6 holosystolic murmur at the right upper sternal border. tension, without concomitant peripheral vascular disease. LV apical impulse is normal and nondisplaced. There is no RV The patient presents with several symptoms consistent with heave. Lung auscultation showed diminished breath sounds at marked left heart congestion including orthopnea and paroxthe right base up through the mid-​right lung field. Abdomen ysmal nocturnal dyspnea.

155

(a)

(b)

(c)

(d)

(e)

(f )

(g)

Given the preserved RV function and a phenotype consistent with left-​sided diastolic heart failure, this patient would be considered an acceptable candidate for renal transplantation once volume status was optimized by volume removal with ultrafiltration and with blood pressure under better control; repeat evaluation with right heart catheterization may be needed. In the invasive hemodynamic assessment of such patients, it is not uncommon to find a top normal or only mildly elevated resting wedge pressure. In such cases, saline fluid challenge or exercise should be considered. From Raina A. Pulmonary hypertension in patients with chronic kidney disease: noninvasive strategies for patient phenotyping and risk assessment. Adv Pulm Hypertens. 2013;12(2):76–​81. Figure 18.1 

WH AT A R E T H E M O S T FR EQ U E N T C O MO R B I D I T I E S I N R E NA L FA I LU R E?

A large portion of deaths in patients with chronic kidney disease (CKD) are due to involvement of the cardiovascular system. Ailments start early in the form of dilated cardiomyopathy, heart failure, and pulmonary hypertension. Arteriosclerosis is facilitated by diabetes and dyslipidemias, while hypertension and cardiomyopathy is usually due to both volume and pressure overload and high levels of renin-​angiotensin. Anemia occurs due to impaired erythopoiesis secondary to decreased erythropoietin synthesis and release, decreased red cell life span, increased hemolysis and bleeding, repeated loss during hemodialysis, and uremia-​induced bone marrow suppression. Pulmonary congestion due to volume overload results in hypoxemia and hypocapnia. Intraperitoneal fluid used in peritoneal dialysis can cause diaphragmatic splinting with basal atelectasis and shunting. In addition, the endocrine, coagulation, and the central nervous system may be affected as well.

Most centers however use general anesthesia to provide stable hemodynamics, good muscle relaxation, and adequate depth of anesthesia. WH AT A R E S O M E O F T H E A N E S T H ET I C C O M P L I C AT I O NS A F T E R R E NA L T R A N S P L A N T ?

Some of the most relevant postoperative anesthetic complications after renal transplantation are vomiting, pulmonary aspiration, delayed respiratory depression, pulmonary edema, hemodynamic instability, and cardiac arrhythmias. Cardiovascular complications are responsible for a great majority untoward postanesthesia consequences. The strongest predictors of bad clinical outcome in recipients are age greater than 60  years, coronary artery disease, and diabetes mellitus.

DOES RENAL FAILURE INFLUENCE THE PHARMACOKINETICS AND PHARMACODYNAMICS OF ANESTHETIC AGENTS?

DISCUSSION I N T RO D U C T I O N

After progression to ESRD, renal transplantation is the treatment of choice and is the most commonly performed organ transplantation. Compared with maintenance dialysis, sucLow serum albumin levels increase the free fraction components cessful transplantation improves quality of life and reduces of drugs, and uremia-​related blood brain barrier damages can mortality risk for most patients. Successful transplantation to increase the entry of unbound drugs into the central nervous replace failed human organs was a daunting goal at the start of system (CNS). Therefore, the dose of induction agents may the 20th century. Investigators in Vienna initially attempted need to be adjusted according to the volume status, acidic pH, kidney transplantation in several animals in 1901. After 5 and increased exposure of the nervous system to these drugs. decades, Dr.  René Küss performed the first kidney transIn the opioid group, mostly morphine and meperidine are plantation that functioned in humans. The kidney worked affected, since they and their metabolites are excreted through without immunosuppression but was rejected 2  months the kidneys. later. It was in 1954 that Dr.  Joseph Murray performed the The clearance of muscle relaxants is also extended in this first successful kidney transplantation using a kidney from patient population. an identical twin. Further progress was made with advances in immunosuppression—​ the use of azathioprine in 1959 WH AT A R E T H E S O U RC E S O F D O N O R K I D N EY ? by Dr.  Roy Calne and its combination with steroids by Donor kidney sources can either be a living related donor, Dr. Thomas Starzl. The introduction of antilymphocyte globliving unrelated donor, or a deceased cadaveric donor. Initial ulin by Dr. Starzl in 1967 and development of organ presertesting is done to determine major blood group compatibility. vation solutions by Dr. Folkert Belzer (1968) and Dr. Jeffery Cadaveric kidney should be ABO identical to the recip- Collins (1969) enabled the use of allografts from remote ient whereas a live donor kidney may be either ABO identical organ donors and better outcome as compared to earlier 1 or compatible. However, it is possible to place an ABO-​ transplants. incompatible organ in a recipient using various strategies, such as plasmapheresis and immunoabsorption to overcome rejec- C O MO R B I D IT I E S I N R E NA L FA I LU R E R E L AT E D tion since the Rh system is not expressed in the graft tissue. TO A N E S T H E S I A The kidneys are essential for adjusting body fluid volumes, electrolyte composition, acid base balance and hemoglobin concentration. They receive about 25% of cardiac output and function If regional anesthesia is considered, ramifications from uremia-​ as filters for toxins and drugs in the circulation. Chronic renal related bleeding tendency, residual heparin given during di- failure or, more appropriately, CKD refers to a decline in the alysis, altered platelet function, and decrease in coagulation glomerular filtration rate (GFR) caused by a variety of diseases factors should be kept in mind. The advantages of combined such as diabetes mellitus (40%), hypertension (27%), chronic spinal–​epidural technique are rapid onset and good muscle re- glomerulonephritis (13%), cystic kidney disease (3.5%), inlaxation from spinal and supplemental analgesia through epi- terstitial nephritis (4%), and other diseases such as obstructive uropathy, lupus nephritis, and human immunodeficiency dural during and after surgery. WH AT A N E S T H ET I C T E C H N I Q U E I S MO S T P R EVA L E N T ?

18. E nd -S tage Renal D isease  •   157

virus.2p1654 CKD may be categorized as mild (GFR of 60–​89 mL/​min/​1.73 m2), moderate (GFR of 30–​59 mL/​min/​1.73 m2), severe (GFR of 15–​29 mL/​min/​1.73 m2), or ESRD. Hemodialysis or peritoneal dialysis is typically initiated as the GFR falls to less than 15 mL/​min/​1.73 m2. The progression of renal disease from 1 stage to the next results in deleterious effects on multiple organ systems.3

atelectasis and shunting. Uremic lung is a radiological entity characterized by perihilar congestion.

Electrolytes and acid-​base status

Inability to excrete water, electrolytes, and free acids results in metabolic acidosis, hyponatremia, hyperchloremia, and hyperkalemia. For every 0.1 unit change in pH, potassium increases by 0.6 mEq/​L. Severe hyperkalemia increases cardiac Cardiovascular system and skeletal muscle excitability. The electrocardiogram (EKG) Almost 50% of deaths in patients with CKD are due to in- shows peaked T waves, flat P waves, increased PR interval ,and volvement of the cardiovascular system. Damage starts in a wide QRS complex that can progress to sine wave and venearly stages and frequently in the form of ischemic heart tricular fibrillation. dis­ease, dilated cardiomyopathy, congestive cardiac failure, Treatment involves use of 10 mL of 10% calcium gluconate LV hypertrophy, and pulmonary hypertension. Accelerated intravenously, 1 mEq/​Kg sodium bicarbonate iv, beta agonists, arteriosclerosis is promoted by diabetes and dyslipidemias, hyperventilation in mechanically ventilated patients, furosewhile hypertension and cardiomyopathy is usually due to mide, and magnesium. However, hemodialysis or peritoneal both volume and pressure overload and high levels of renin-​ dialysis is the definite treatment. Hypermagnesemia usually angiotensin. Volume overload occurs due to expansion of accompanies hyperkalemia (GFR 7.2, and plasma calcium levels >1  mmol/​L optimizes conditions for clot formation.2 Acidosis reduces thrombin generation and increases clot lysis, while hypothermia reduces fibrin and clotting factor synthesis and impairs platelet function. Active patient warming with forced air blankets, heated mattresses, and ability to heat rapid infusions to >39°C should be standard practice.

Intraoperative cell salvage The use of autologous cell salvaged blood can reduce the need for allogenic blood transfusion. RBCs are salvaged from the operative field, adding anticoagulants to the aspirated blood. Citrate anticoagulation is used to avoid heparin accumulation. This blood is stored in a reservoir, before being centrifuged to separate out the components. The RBCs are washed and altered, removing biochemical debris including free hemoglobin, platelets, white blood cells, plasma, and heparin. Since the retransfused washed RBCs from cell salvage provide no plasma, clotting factors, or platelets, this must be factored in when assessing the need for replacement hemostatic therapy. Transfusion of potentially harmful fat-​micro emboli, free hemoglobin, denatured proteins and platelet, and leukocyte microaggregates have been postulated as causes of disseminated intravascular coagulopathy, acute respiratory distress syndrome, and acute renal failure in some case reports. Malignant cells are not washed from cell salvaged

184  •   B lood P r oducts

blood and cell salvage has been contraindicated in the presence of malignant disease due to the theoretical risk of metastasis.

Intraoperative normovolemic hemodilution Intraoperative normovolemic hemodilution involves venesection of the patient and volume replaced with other fluids. The venesected blood is then retransfused intra-​and postoperatively as required, usually post reperfusion. This preserves the integrity of the venesected RBCs and clotting factors and provides a reserve of whole blood to be transfused as required. Contraindications include coronary heart disease, significant anemia, and severe pulmonary hypertension.2 In practice, most patients are too anemic to make venesection a viable option. C O N C LUS I O N

The transfusion practices of liver transplantation have been transformed over the years.2 Transplantation without transfusion has become an attainable goal. Liver transplantation present a significant risk of coagulopathy and bleeding. This is multifactorial in nature, with recipient and donor factors and surgical and anesthetic techniques playing a role. The benefits of effectively monitoring and treating coagulopathy and using techniques to reduce bleeding have been demonstrated to reduce transfusion requirements and improve outcomes after OLT. This has been aided by the wider availability of POCT to monitor and correct severe coagulopathy with factor concentrates instead of blood products. Preoperative optimization of red cell mass can reduce transfusion. Perioperative coagulation management using restrictive transfusion strategies and clotting factor algorithms based upon TEG have been shown to be beneficial.2 Blood transfusion can be lifesaving, and preparations should be made to give blood and blood products when required during all transplantations. If practice can be improved to avoid unnecessary transfusions, outcomes from liver transplantation will continue to improve. R E VI EW Q U E S T I O N S 1. What is massive transfusion? A. >50% blood volume in 2 hours B. >50% volume in 4 hours C. >1 blood volume in 12 hours D. >1 blood volume in 48 hours Answer: B Massive transfusion is defined as replacement of >1 blood volume in 24 hours or >50% of blood volume in 4 hours (adult blood volume is approximately 70 mL/​kg) or, in children, transfusion of >40 mL/​kg (blood volume in children over 1 month old is approximately 80 mL/​kg).2 2. All of the following are risk factors for transfusion in OLT EXCEPT A. increased donor age.

B. hyperdynamic circulation. C. hemodilution. D. recipient >35 years. Answer: D Preoperative risk factors include these 7 variables: age >40 years, hemoglobin concentration 2.0, platelet count 100 μmol/​L for females and >120 μmol/​ L for males, and albumin 6% implies increased fibrinolysis and treated with antifibrinolytics

2 0. U pdate on B lood M anagement in L ive r T r ansplant  •   185

FFP and platelets. It is exacerbated by hypothermia and impaired liver metabolism. It is more likely if infusion of >50 cc/​min of blood products is administered. Patient needs to be recognized by the clinical signs and symptoms of hemodynamic compromise:  hypotension, and electrocardiogram changes including widening of QRS complex, QT prolongation, and electromechanically dissociation.2

MA α° 20 min

R-time Figure 20.1 

K

A30

6. A  65-​year-​old male is undergoing evaluation for liver transplant. He has a history of shortness of breath that worsens when he is sitting up. Blood gas reveals an A-​a gradient of 22. Which of the following is the most probable diagnosis? A. AV malformation B. Portopulmonary hypertension C. Hepatopulmonary syndrome D. Hepatic hydrothorax

30 min

TEG

Answer: C Refer to Figure 20.1 and Table 20.1. LY30 is a measure of fibrinolysis, if high needs to be treated with anti-fibrinolytics. 5. A  patient undergoing OLT received 10 units of RBCs, 5 FFP, 2 cryoprecipitate, 1 unit of pooled platelets in the last hour. Previously normal electrocardiogram now shows QT prolongation and wide QRS. Blood pressure on norepinephrine is 75/​50 mm Hg. What is the most appropriate next step in management? A. Insulin and dextrose B. Calcium chloride C. 5% albumin D. Start epinephrine infusion Answer: B The scenario represents hypocalcemia related to massive transfusion. Main cause of hypocalcemia in OLT is citrate toxicity from massive blood transfusion, related mainly to

Answer: C Hepatopulmonary syndrome is a complication of end-​stage liver disease, which is characterized by-​presence of liver disease; A-​a gradient >15 and >20 in patients 60 years old, respectively; and echocardigraphic evidence of pulmonary vascular dilation (double contrast echocardiogram/​99m-​TC macroaggregated albumin lung perfusion scan; microbubbles in left heart in 6%). Liver transplant is the definitive treatment in most of the patients. Portopulmonary hypertension refers to the development of pulmonary hypertension in setting of portal hypertension with or without liver disease. Diagnostic criteria include pulmonary artery pressure >25 with normal pulmonary artery occlusion pressure and elevated pulmonary vascular resistance.4 7. All of the following are treatment options in hypotension caused by carcinoid crisis EXCEPT A. IV fluid bolus. B. epinephrine. C. phenylephrine. D. somatostatin.

Table 20.1  THROMBOELASTOGRAPHY-​B ASED BLOOD PRODUCT THERAPY PARAMETER

INTERPRETATION

POSSIBLE TREATMENT

R-​time

Time taken from addition of reagent to initial fibrin formation. A prolonged R time implies a deficiency in clotting factors.

Administer FFP

K-​time

Time taken from initial fibrin formation to achieve a certain level of clot firmness. A prolonged K time implies low fibrinogen.

Administer cryoprecipitate

Alpha angle

Measures speed at which fibrin builds up and clot strengthening occurs. Similar concept to K time.

Administer cryoprecipitate

MA

Maximum amplitude/​clot strength. If high, can indicate hypercoagulation. If low, implies thrombocytopenia and/​or platelet dysfunction.

If low, administer platelets

LY30

Percent of thrombolysis 30 minutes after achieving maximum amplitude. This measures fibrinolysis. Normal atracurium > rocuronium > cisatracurium B. Rocuronium > atracurium > succinylcholine > cisatracurium C. Atracurium > rocuronium > cisatracurium > succinylcholine D. Cisatracurium > rocuronium > atracurium > succinylcholine Answer: A Among the NMBA, succinylcholine is the most frequently incriminated (61%), followed by atracurium (20%), cisatracurium (6%), vecuronium (5%), and rocuronium (4%). Proportion is in consideration of the market share.6 4. A 62-​year-​old otherwise healthy woman loses 1,000 mL of blood rapidly during a partial hepatectomy. After adequate volume resuscitation, including the initiation of packed red blood cell transfusion, she becomes increasingly hypotensive and tachycardic. Her vital signs are BP 64/​42  mm Hg, HR 136 bpm, SpO2 98%, temperature 38.4°C. What is the MOST appropriate next step in management? A. Continue packed red blood cell transfusion. B. Initiate fresh frozen plasma transfusion. C. Continue volume resuscitation with albumin. D. Discontinue transfusion. Answer: D This patient has signs suggestive of an acute transfusion reaction. Despite adequate volume resuscitation and control of surgical bleeding, she is hypotensive, tachycardic, and febrile after the initiation of packed red blood cell transfusion. The transfusion should be discontinued, and the blood sent back to the blood bank for testing. Febrile transfusion reactions occur in 0.5% of red blood cell transfusions and 30% of platelet transfusions and are thought to be due to recipient antibodies directed against human leukocyte antigens on donor white blood cells or platelets. Cytokines released from white blood cells in stored blood product (especially platelets) may also be a contributing factor. Patients experiencing a febrile reaction have increase in temperature of >1°C as well as headache and back pain (not obvious in patient under general anesthesia) in addition to signs similar to allergic reaction. The febrile reaction is usually delayed up to 2 hours after the transfusion but is treated successfully with acetaminophen and diphenhydramine. Leukoreduction helps to reduce febrile transfusion reactions. An acute hemolytic transfusion reaction is usually the result of ABO blood group incompatibility and may be fatal. Patients having acute hemolytic reaction present with fever, dyspnea, chest pain, low back pain, and sudden hypotension. Under general anesthesia, only hypotension and fever may be apparent. Acute renal failure may result; the transfusion should be stopped and volume resuscitation with addition of mannitol or furosemide should be considered. 5. In patients with severe anaphylactoid reaction, A. previous exposure to the triggering drug is necessary.

B. pulmonary edema is a common clinical finding. C. bronchospasm is the most dangerous feature. D. an aura may precede the acute reaction (if patient awake). Answer: D Patients don’t need prior exposure for an anaphylactoid reaction; primary exposure can trigger a reaction clinically indistinguishable form an anaphylaxis.2 Circulatory collapse is the most dangerous feature of an anaphylactoid/​anaphylactic reaction. Pulmonary edema is a rare event. If a patient is awake, symptoms similar to aura may precede the actual reaction they vary from gastrointestinal upset to anxiety.6 6. A 60-​year-​old, healthy gentleman is getting a redo total hip replacement. He is methicillin-​resistant Staphylococcus aureus positive, and pre-​procedure prophylaxis with vancomycin was initiated after uneventful induction and intubation. Within 20 minutes, it was noticed that the patient’s whole body had a red rash, with relatively stable cardio-​respiratory status. The next BEST step is A. hold vancomycin. B. draw tryptase level. C. hold vancomycin and give diphenhydramine. D. hold vancomycin and restart once the symptoms resolve. Answer: C Anaphylaxis with vancomycin is rare and should be distinguished from red man syndrome, a clinical entity resulting from nonspecific histamine release and observed when the drug is rapidly injected. Red man syndrome is the most common hypersensitivity reaction associated with vancomycin. The incidence varies between 3.7% and 47% in infected patients. Studies show that the most severe reactions occur in patients younger than the age of 40, particularly in children. The effects of red man syndrome can be relieved by antihistamines. Pretreatment with hydroxyzine can significantly reduce erythema and pruritus. Administration of diphenhydramine to patients before starting vancomycin infusion (1 g over 1 hour) can prevent the occurrence of red man syndrome with the first dose of vancomycin. Other studies have shown that combining an H1 receptor blocker with an H2 receptor blocker such as cimetidine may help to prevent or reduce the risk of red man syndrome. If red man syndrome appears, then the vancomycin infusion should be discontinued immediately. A dose of 50 mg IV diphenhydramine hydrochloride or orally can abort most of the reactions. Once the rash and itching dissipate, the infusion can be resumed at a slower rate and/​or at a lesser dosage. Hypotension will require IV fluids, and, if severe, vasopressors may be needed. Each IV dose of vancomycin should be administered over at least a 60 minutes interval to minimize the infusion-​related adverse effects. Longer infusion times should be used in patients receiving doses considerably larger than 1 g vancomycin. 7. Which of the following tests laboratory tests will help make a diagnosis of anaphylaxis? A. Tryptase drawn within 2 hours of the reaction

204  •   A llergy

B. Histamine drawn within 2 hours C. Both tryptase and histamine drawn within 2 hours of a severe reaction D. IgE assays drawn within 24 hours Answer: C Histamine and tryptase concentrations correlate with the severity of the allergic reaction; combined histamine and tryptase measurements have been recommended for the diagnosis of immediate reactions, due to short half-​lives. Blood samples may be drawn within 30 minutes in grade I or II reactions and within 30 minutes and 2 hours in grade III or IV reactions.2,3 8. A 15 year old boy scheduled for a urologic procedure, developed severe anaphylactic reaction intraoperatively. Which of the following food allergies could have given a clue to this outcome? A. Chestnuts B. Almonds C. Oranges D. Brocolli Answer: A Latex Fruit Allergy is a well known entity-certan tropical fruits and vegetables have a high cross reactivity with Latex and patient’s allergic to them can have a Latex allergy/anaphylaxis. High risk food include-Avocado, Banana, Chestnut, Kiwi; Moderate risk foods are-Apples, Carrots, Celery, Melons, Papaya, Tomato, Potato. 9. Which of the following will classify as Anaphylaxis in 40 year old? A. Hives and hypotension 1 hour after a procedure under MAC B. Systolic Blood Pressure < 90 mm Hg immediately after induction C. Bronchospasm and rash 15 minutes after antibiotic administration under GA D. All of the above Answer: D Anaphylaxis ranges from mild to moderate and can occour within minutes to hours after exposure to inciting allergen. Diagnostic criteria include: 1. Skin changes within minutes to hours and one of the following changes-respiratory or hemodynamic 2. 2 or more of the following-skin manifestestation, gastrointestinal symptoms, respiratory or hemodynamic compromise 3. Hypotention >30% of baseline or systolic 2 is used to aid in the recognition of organ dysfunction. A diagnosis of septic shock is made when MAP is 2 C. SIRS + Probable infection D. Requirement of vasopressors Answer: B According to Sepsis 3 criteria, sepsis is defined as an immunological and inflammatory response to infection resulting in end organ dysfunction defined by a SOFA score greater than 2. 5. Which of the following does NOT need to be done within the first 3 hours of suspecting sepsis? A. Draw blood cultures B. Start steroids C. Volume resuscitate D. Start antibiotics Answer: B The 3-​hour bundle in the management of sepsis as per the surviving sepsis guidelines includes the following:  send

blood cultures, start appropriate antibiotics, fluid resuscitation, and measure lactate. Steroids are initiated in vasopressor resistant septic shock and are not part of the 3-​hour bundle.

QUESTIONS AND ANSWER S This chapter has accompanying questions and answers which are available to subscribers as part of the Oxford eLearning platform. To access the questions, go to http://​ oxfordmedicine.com/​anesthesiologyPBL REFERENCES 1. Herborn J, Parukar S. Anesthetic considerations in transplant recipients for nontransplant surgery. Anesthesiol Clin. 2017;35:539–​553. 2. Moreno R, Berenguer M. Post-​ liver transplantation medical complications. Ann Hepatol. 2006;5(2):77–​85. 3. Fleisher L, Fleischmann, K, Auerbach A et  al. 2014 ACC/​AHA guideline in perioperative cardiovascular evaluation and management of patients undergoing non cardiac surgery. J Am Coll Cardiol. 2014;64(22):77–​137. 4. Holt C. Overview of immunosuppressive therapy in solid organ transplantation. Anesthesiol Clin. 2017;35:365–​380. 5. Rhodes A, Evans L, Alhanazzi W, et  al. Surviving Sepsis Campaign:  international guidelines for management of sepsis and septic shock: 2016. Crit Care Med. 2017; 45:486–​552. 6. Singer M et al. The Third International Consensus for Sepsis and Septic Shock (Sepsis 3). JAMA. 2016;315(8):801–​810.

23. L iver T ransplant R ecipient P resenting for S urgery  •   213

24. PATIENT WITH LIVER FAILURE PRESENTING FOR TRANSJUGULAR INTRAHEPATIC PORTOSYSTEMIC SHUNT Marina Moguilevitch

S T E M C A S E A N D K EY Q U E S T I O N S A 66-​year-​old man with a history of hepatitis C and cirrhosis complicated by intractable ascites presents for an elective transjugular intrahepatic portosystemic shunt (TIPS) procedure. His past medical history is also significant for hypertension and diabetes mellitus type 2. He was in his usual state of health until approximately 1 year ago when he slowly began to develop symptoms. Initially he had painless jaundice with easy bruising. These symptoms progressed to ascites with abdominal distension and progressive dyspnea on exertion. Previously he could walk a mile without dyspnea but now is unable to walk even a few blocks. He has had his ascites drained twice previously, which provided some symptomatic relief; however, after each drainage, the ascites returned. His physician recommended that he have a TIPS procedure secondary to his intractable ascites. He is currently on the liver transplant waiting list. He presents to the hospital on the day of the procedure when you meet him for the first time. WH AT Q U E S T I O N S WI L L YO U A S K T H E PAT I E N T ? WH AT A R E YO U L O O K I N G F O R I N T H E P R E O P E R AT I VE A S S E S S M E N T ?

He has had nothing by mouth for 10 hours. On exam, a large protruding abdomen is prominent. The airway exam is unremarkable. His labs are significant for an international normalized ratio (INR) of 1.8, creatinine of 1.6, and a bilirubin of 3.5. He is anemic with a hematocrit of 26.4, and his platelets are less than 100. WH AT I S H I S MO D E L F O R E N D -​S TAG E L I VE R D I S E A S E ( M E L D) S C O R E A N D C A N IT B E US E D TO P R E D I C T MO R B I D I T Y/​M O RTA L I T Y F O R T H E P RO C E D U R E?

D O E S T H E I N R N E E D TO B E C O R R EC T E D ? I S T H E I N R US E F U L I N P R E D I C T I N G B L E E D I N G R I S K I N PAT I E N TS WIT H C I R R H O S I S ?

The patient is brought to the room and placed on the procedure table, where routine American Society of Anesthesiologists monitors are placed. A 20 G intravenous (IV) line has been placed in the left arm by the preoperative nursing staff. As you begin to administer oxygen, the interventional radiologist walks in and tells you, “Sedation is fine for this case.” I S MO N ITO R E D A N E S T H E S I A C A R E A N AC C E P TA B L E A N E S T H ET I C T EC H N I Q U E F O R A T I P S P RO C E D U R E? WH E N WO U L D G E N E R A L E N D OT R AC H E A L A N E S T H E S I A B E PREFERRED ?

You ultimately decide on general endotracheal anesthesia for your anesthetic management. You preoxygenate the patient with 100% oxygen for 3 minutes via facemask. As you are getting your equipment ready, the circulating nurse asks if you will need help giving cricoid pressure when you intubate. I S R A P I D S EQ U E N C E I N D U C T I O N I N D I C AT E D F O R T H I S PAT I E N T ? I F S O,  WH Y ?

After intubating without difficulty and confirming proper tube placement, the peak airway pressures are elevated. Lung sounds are clear to auscultation. While troubleshooting the elevated peak airway pressures, the patient’s hepatologist enters the room and tells you she will be performing a paracentesis. The paracentesis drains 4 L of ascites from the abdomen. H OW WI L L T H I S A FFEC T YO U R M A NAG E M E N T ? WH AT C H A N G E S D O YO U E X P EC T TO S E E?

Prior to bringing the patient to the procedure room, the interventional radiologist tells you someone has gone to the blood bank to pick up 2 units of fresh frozen plasma (FFP) to correct the INR, as he is worried about bleeding risk.

Shortly after the start of the procedure, the patient becomes hypotensive. After adjusting the depth of your anesthetic, there is no improvement. The interventional radiologist reports that there has been no noticeable bleeding. Your presumptive diagnosis is hypovolemia.

215

H OW WI L L YO U C O R R EC T T H E VO LU M E S TAT US ? S H O U L D YO U R VO LUM E R E P L ET E WI T H C RY S TA L L O I D O R C O L L O I D ? I S T H E R E A N Y B E N E F I T F O R A D M I N I S T E R I N G E IT H E R C RYS TA L L O I D O R C O L L O I D ?

The next day after the procedure, the patient is noticed to have worsening of his mental status and increasing bilirubin levels. A R E YO U C O N C E R N E D ? WH AT WI L L B E YO U R D I F F E R E N T I A L D I AG N O S I S ? WH AT WO U L D YO U D O ? WH AT A R E T H E OT H E R P OT E N T I A L LY L I F E -​T H R E AT E N I N G C O M P L I C AT I O N S O F T I P S ? WH AT I S A T R A N S JU GU L A R I N T R A H E PAT I C P O RTO S Y S T E M I C S H U N T P RO C E D U R E?

TIPS is used to treat complications related to portal hypertension secondary to liver cirrhosis. It is performed in the interventional radiology suite with the creation of a channel between the hepatic vein and intrahepatic portion of portal vein. This tract is kept open by the metal stent being deployed through internal jugular vein. This intervention allows decompression of the high-​pressure portal system by bringing blood into the systemic circulation. The goal of the procedure is to decrease the intrahepatic venous pressure gradient to 5), and significant thrombocytopenia (platelets count 65 years, child’s score >12, prior hepatic encephalopathy, placement of the large diameter stent >10 mm, and low portal pressure gradient 35, age >50, neck >40 cms, and male gender are the criteria for an increased risk of OSA in the ambulatory setting. The STOP-​Bang form helps the anesthesiologist prepare for appropriate postoperative monitoring and care. Currently, mild to moderate OSA patients can be cared for safely in the outpatient location. Regional anesthesia whenever is possible, the

use of continuous airway pressure, adequate preoxygenation, and oral and nasal devices to prevent the airway obstruction in PACU are some of the recommendations. Severe OSA patients require a thorough preoperative evaluation and a discussion with the surgical team and nursing staff about whether the ambulatory setting is appropriate for them.1 PONV is essentially as old as the practice of the general anesthesia. Regardless of the progress in antiemetic development and multimodal tactics to fight PONV, there is a clear difference between 2 independent factors:  postoperative nausea (approximately 20%) and vomiting (approximately 4%). This serious problem can cause unanticipated hospital admission (post-​discharge nausea and vomiting [PDNV]), increasing the PACU stay and affecting the overall experience of the patient in the perioperative period. The simplified risk score for PONV include female gender, nonsmoker, history of PONV, and postoperative opioid use. When 0, 1, 2, 3, and 4 of the risk factors are present; the resultant risk for PONV is about 10%, 20%, 40%, 60%, and 80%, respectively. We expect a lower risk for PONV in ambulatory PACU, since procedures are typically shorter and less invasive than inpatient procedures. Nevertheless, PDNV represents a substantial risk to patients who has been discharged from medical observation and no longer have access to IV fluids or medication, making dehydration, nausea, and the ability to eat or take oral pain medication impossible. The reported incidence of PDNV is 37% in the first 48 hours after discharge. The identified 5 independent predictors of PDNV are female sex, age 30  kg/​m2 with awake alveolar hypoventilation (PaCO2 >45 mm Hg), which cannot be attributed to other conditions.2 To make this diagnosis, patients should be seen by a pulmonologist to not only rule out other coexisting pulmonary disease but also to evaluate the degree of cardiopulmonary compromise through tests such as serum bicarbonate, pulse oximetry, ABG, pulmonary function tests, polysomnography, chest radiography,

230  •   P ostanesthesia C are  U nit

Table 26.1  EFFECTS OF MORBID OBESITY ON VARIOUS ORGAN SYSTEMS AND POTENTIAL SURGICAL COMPLICATIONS Cardiac

Pulmonary hypertension, coronary artery disease, systemic hypertension, congestive heart failure

Pulmonary/​Airway

↓FRC, restrictive lung defects, airway obstructi on, difficult airway/​intubation

Vascular

Difficult IV access, ↑ risk of venous thromboembolism

Hematologic

Polycythemia

GastrointestinalI

More challenging laparotomy/​laparoscopy, ↑omental fat

Metabolic

↑ Risk of diabetes mellitus and insulin resistan ce, ↑systemic inflammation

Neurologic

↓ Central respiratory drive

Renal

↑ Risk of chronic kidney disease

Skin

Breakdown/​cellulitis

FRC = functional residual capacity; IV = intravenous; DM = diabetes mellitus Sources: 4.Chau EH, Lam D, Wong J, Mokhlesi B, Chung F. Obesity hypoventilation syndrome: a review of epidemiology, pathophysiology, and perioperative consideration. Anesthesiology. 2012;117(1):188–​205;Piper A, Yee B. Clinical manifestations and diagnosis of obesity hypoventilation syndrome. In: Post T, ed. UpToDate. Waltham, MA: UpToDate; 2017. www.uptodate.com. Accessed April 13, 2017.

electrocardiogram, and possible right heart catheterization by a cardiologist.2 When considering a patient such as this for surgery, it is prudent to remember the increased likelihood of perioperative

complications such as hypoxemia, myocardial ischemia/​infarction, unanticipated admission, and sudden death during the perioperative period.1 Many ambulatory surgical centers set maximal BMI guidelines to help delineate which patients they can safely care for, considering factors such as availability of bariatric equipment, staff to help with moving to and from a hospital bed, type of surgery to be performed, and expected speed of postoperative recovery to name a few. Mask ventilation and intubation are typically more challenging in the patient with OSA;1 their pharyngeal airway is narrower and more collapsible,3 they may have an exaggerated relaxation of these muscles in response to narcotics, and the presence of extra soft tissue as well as diminished functional residual capacity makes intubation more difficult. Prudent use of awake intubation when indicated is advised, and proper positioning for intubation is of elevated importance in this patient population (see Fig. 26.1.) The careful practitioner will have a wide array of airway adjuncts (i.e., multiple blades, videolaryngoscopy, fiberoptic bronchoscopy, laryngeal mask airways) available to assist in rapid securement of the airway. Dosing of anesthesia drugs should be adjusted in the morbidly obese patient so that they are titrated based on lean body weight, except for the neuromuscular blockers, which should be based on total body weight in the case of succinylcholine (morbidly obese patients have increased levels of plasma cholinesterase) or ideal body weight for the nondepolarizing muscular blockers.4 Short-​acting drugs are ideal for this patient population as metabolism may be impaired and fat-​soluble drugs (or their metabolites) may accumulate over time. Selection and placement of monitors in the morbidly obese can be challenging as well. Standard noninvasive blood pressure cuffs may not fit properly, and consideration should be given to placement of an arterial line, especially in patients with OHS, for emergency laparotomy, and for those who are possibly septic. IV access is typically also difficult so these

Suggested positioning of the morbidly obese patient for intubation (Agent for Healthcare Research and Quality website, https://​psnet.ahrq. gov/​webmm/​case/​221. Accessed April 16, 2017). Figure 26.1 

2 6. O bstructive S leep  A pnea  •   231

be beneficial. A side benefit of use of these agents (as well of avoidance of narcotics) is the potential for quicker recovery of bowel function postoperatively. Neuraxial and regional techniques have been shown to be opioid-​sparing and, in many cases, can eliminate the need for postoperative narcotics. A  functioning epidural catheter in this patient would cover much of his abdominal pain. If local anesthetic alone is not sufficient to cover pain, neuraxial opioids, particularly with lipophilic agents, such as fentanyl, have a lower incidence of respiratory depression compared with systemic IV administration. Special caution must be taken with hydrophilic narcotics, such as morphine, as the potential for delayed respiratory depression postoperatively can be especially dangerous in this patient population. The risks and benefits of a neuraxial technique in this patient with an elevated white blood cell count should be carefully considered, but most practitioners would regard potential bacteremia as a contraindication and would pursue alternative modes. Other potential options include transverse abdominis plane (TAP) blocks (and possibly TAP catheters) to cover some of the incisional pain. Upon the conclusion of surgery, the decision of whether Figure 26.2  An example of a transcutaneous CO2 monitor (SenTec or not to extubate the patient in the operating room is transcutaneous CO2 monitor). Used with permission, Sina Kohlbrenner, typically based on many things such as fluid shifts, airway SenTec Inc. March 31, 2017). edema, difficulty of intubation, preoperative lung function, and hemodynamic stability. In this patient, additional patients may benefit from central line placement to ensure re- things to consider will be his underlying moderate pulmoliable access as well as a conduit for potential vasoactive drug nary hypertension (likely to be exacerbated during the postadministration. In the patient with a significant history of con- operative period by an almost certainly worsened degree gestive heart failure, consideration should be given to place- of hypercapnia), potential for impending systemic inflamsepsis (underlying increased ment of a Swan-​Ganz catheter and/​or use of intraoperative matory response syndrome/​ transesophageal echocardiography. Whether or not to place inflammatory state from morbid obesity superimposed these additional monitors prior to the induction of anesthesia on infectious process), and the probable need for CPAP is up to the discretion of the anesthesiologist, bearing in mind postoperatively, which may result in abdominal distension the preoperative morbidity, cooperation of the patient, and in a patient with poor history of previous compliance with CPAP. Ideally, this patient would have brought in his CPAP ease of placement of the invasive monitors. Planning ahead for postoperative pain control, especially machine and mask from home, though studies have shown after an exploratory laparotomy, can be one of the most dif- that fewer than 20% of patients with a diagnosis of OSA ficult yet important considerations in the patient with OSA receive CPAP therapy or have a properly functioning and and OHS. The mainstay of postoperative analgesia, opioids, fitting mask.6 There is some evidence to suggest improved has been shown to increase both desaturation and episodes lung function at one and 24 hours from starting CPAP of central apnea in this group. This is believed to be due to therapy immediately on extubation in the operating room, both an increased sensitivity to opioid effect as well as alter- rather than waiting until arrival in the postoperative care ation of pain processing in OSA patients.5 Ideally, postop- unit or until a patient becomes symptomatic.7 If a patient erative pain control in this patient will be multimodal and has brought in his or her CPAP from home, it is not unminimize narcotics. Acetaminophen (IV if oral medication common that the mask will not have a proper fit. If the use is not tolerated), nonsteroidal anti-​inflammatory drugs or of CPAP is planned, respiratory therapy should be available selective COX-​2 inhibitors, and agents such as gabapentin/​ to assess the CPAP mask fit and function, and biomedical pregabalin are frequently given immediately prior to surgery engineering may need to “certify” the device before it can be in an attempt at “preemptive” analgesia. During the sur- plugged in at your institution. The choice of whether to inigical procedure, infusions of ketamine, dexmedetomidine, tiate CPAP postoperatively on a patient is an individualized and/​or lidocaine have all been shown to have opioid sparing decision that must take into account the patient’s level of reeffects and are often continued into the postoperative pe- sidual anesthesia, underlying conditions and comorbidities, riod, depending on the degree of pain, acuity of care that and the surgery. Studies have shown improved outcomes in will be provided, and how important it is to avoid narcotics terms of unplanned intensive care unit (ICU) admission, in a certain patient. The patient with OHS is certainly at length of hospital stay, and incidence of adverse cardiovasgreatly increased risk of perioperative morbidity/​mortality, cular and respiratory complications when CPAP is used and any attempts at reducing or eliminating narcotics would postoperatively.8 However, other studies have not shown 232  •   P ostanesthesia C are  U nit

clear benefits of positive airway pressure in all patients with OSA. The use of postoperative IV opioid patient controlled analgesia in patients with OSA is controversial. One study showed that use of capnometry combined with automatic pausing of patient-​controlled analgesia was more effective than pulse oximetry alone at preventing potentially dangerous outcomes in patients with sleep apnea.9 Accordingly, there are devices currently being marketed that not only deliver narcotics but that also monitor EtCO2 and use this to determine future dosing in an effort to prevent dangerous hypercapnia from developing.10 Most hospitals currently do not have access to this type of device and may instead be required to rely upon just pulse oximetry (if anything) as a sole monitor during patient-​controlled analgesia use. While institutional policy will vary from hospital to hospital, it is not uncommon to require continuous pulse oximetry in all patients being initiated on patient-​ controlled analgesia. Whether to recommend or require continuous pulse oximetry in a patient receiving opioids as needed (prn) is less clear. Certainly, most patients tolerate the addition of prn opioids for analgesia with only standard monitoring required. Patients with OSA are at an increased risk of hypoxemia and hypercapnia, with these risks being increased both postoperatively and with the addition of opioid analgesics. Somnolence in the postoperative care unit is frequently seen in OSA patients, often dangerous, and typically is a multifactorial condition. This patient has several risk factors for postoperative somnolence, including physiologic and surgical factors; however, complications of OSA would be the most likely cause. Significant upper airway obstruction can lead to hypercapnia and/​or hypoxemia, both of which contribute to somnolence, but while oxygenation is relatively easy to monitor with pulse oximetry, ventilation is more difficult to accurately monitor as respiratory efforts are subjective and chest wall rise against a partially obstructed upper airway might still appear to be normal. An ABG is a quick and reasonably inexpensive method to determine if hypoventilation and hypercapnia are contributing factors. It must be remembered that PaCO2 values need to be interpreted in the context of this patient’s underlying medical conditions, including long-​ standing sleep apnea. For this reason, some authors recommend a preoperative ABG to determine the patient’s baseline PaCO2 levels and aid in interpreting any postoperative blood gas values; in the absence of this, a higher than normal HCO3 on preoperative labs would usually raise the suspicion that this patient typically has a significant respiratory acidosis for which his kidneys have compensated (a preoperative serum bicarbonate of 27 mEq/​L has 82% sensitivity for identifying patients with OHS).2 If significant hypercarbia is detected on the ABG, noninvasive positive pressure ventilation should most likely be instituted while you attempt to determine if other factors besides OSA are also contributing. While many authors disagree on the automatic use of positive airway pressure in all patients with sleep apnea in the postoperative unit, the majority agree on its use when a patient becomes symptomatic or shows signs of either hypoxemia or hypercarbia. Additionally, this is a patient with pulmonary hypertension,

and any hypoxemia or associated hypercarbia will further worsen this condition. The use of CPAP in the postoperative care unit will require both respiratory therapy and nursing who are familiar with its use and can monitor its effectiveness. Newer noninvasive monitoring devices, including transcutaneous CO2 monitoring, will aid in the postoperative management of patients with OSA. Also included in the differential diagnosis of postoperative somnolence for this patient would be residual anesthetic effects, opioid CNS depressant effects, residual neuromuscular blockade, and the potential for systemic inflammatory response syndrome (SIRS)/​sepsis. An obese patient such as this one will have an increased volume of distribution for both volatile and IV anesthetics and will require increased time for complete clearance of the depressant effects of any anesthetics. The respiratory depressant effects of opioid analgesics have been shown to be more pronounced in a patient with OSA, which could contribute to increased somnolence.5 This can be quickly evaluated by assessing for the level of miosis, though the sensitivity of this evaluation is questionable. If suspected as a cause of oversedation, naloxone can be slowly titrated to effect. If residual neuromuscular blockade is suspected, acceleromyography will give an objective measure of residual blockade, and the patient can be treated appropriately. Alternatively, standard train of four monitoring and clinical signs can evaluate for inadequate reversal of neuromuscular blockade if acceleromyography is not available and additional reversal agent can be given as needed. In this patient with diverticulitis and free air in the abdomen, it is important to consider the possibility of SIRS/​ sepsis in the evaluation of somnolence. Any patient suspected of having SIRS/​sepsis should be treated according to the Surviving Sepsis Campaign, including having 2 sets of blood cultures drawn and empiric broad spectrum antibiotics started covering likely pathogens, which in this case would need to include any enteric gut flora. Hypoxia is another frequent complication encountered postoperatively in the morbidly obese patient with OSA and OHS. In a patient with known pulmonary hypertension, increasing pulmonary arterial vasoconstriction (from hypercarbia or other causes) leading to increasing shunt fraction needs to be considered. Other causes would again include residual anesthetic, narcotic, neuromuscular blockade, and pain. A  laparotomy is generally associated with a moderate amount of pain, and splinting secondary to inadequate pain control can contribute to hypoxemia. Postoperative ventilation–​perfusion mismatch is a common cause of hypoxemia in the PACU, with atelectasis being the most common cause of ventilation–​perfusion mismatch. Prolonged time in the supine position predisposes this patient to atelectasis and patients with OSA have been shown to have increased obstruction in the supine position. For this reason, some have recommended positioning patients in a semilateral recumbent or sitting position in the PACU, if the surgery allows that positioning.11 Less likely causes of hypoxemia in this patient would also include pulmonary edema, aspiration, and bronchospasm and should be ruled out by simple auscultation and/​or chest radiography.

2 6. O bstructive S leep  A pnea  •   233

While there is no set level of airway pressure that should be used for every patient, there are some guidelines available to help when starting CPAP therapy postoperatively. It is important to first know what level of airway pressure a patient uses at home, though it is possible that after a surgery, this level will not be adequate. Many factors can influence the amount of positive airway pressure that will be ideal for a patient, including body size, degree of underlying disease, and comorbid conditions. Immediately postoperatively, additional physiologic changes can result in an increased need for positive airway pressure, including fluid shifts associated with surgery, altered respiratory mechanics due to an often prolonged supine position, and the sedative effects that are seen with any remaining anesthetic or narcotic. Two studies evaluating the ideal level of positive airway pressure in newly diagnosed postsurgical OSA patients found 9 +/​–​2 cm H20 to be an optimal level.12 Other authors recommend using a CPAP machine that is capable of automatically altering flows to maintain a patent airway, as this is often tolerated better by patients than a set level of airway pressure.13 As previously mentioned, many patients at home use a CPAP that does not appropriately fit and will not be able to achieve an adequate level of positive airway pressure. Respiratory therapy will be able to determine if the patient’s home mask will be adequate or if a new one will need to be fitted for the hospital stay. As with hypoxemia and somnolence previously discussed, postoperative hypotension is a multifactorial condition that requires detailed knowledge of the individual patient and the surgery. While a full discussion of postoperative hypotension is beyond the scope of this chapter, there are some considerations unique to the patient with OSA and pulmonary hypertension, as the typical treatments for hypotension can occasionally worsen pulmonary hypertension.14 Ideally, bedside transthoracic echocardiogram to evaluate the patient’s fluid status and right ventricular function will help to elucidate the cause of the hypotension. While fluid boluses and continuous infusions are generally avoided in the patient with pulmonary hypertension for fear of worsening this condition, a postoperative patient who is clinically hypovolemic will still benefit from a judicious fluid bolus. It must also be remembered that in a patient with ruptured diverticulitis, hypotension could be the result of a developing sepsis, and the management of this will differ from that of other etiologies of hypotension. The decision on what type of care unit a patient will need after surgery is complex and must take into account all of the patient’s individual comorbidities and the surgery in an attempt to predict the level of care the patient will need. A patient with severe OSA is more likely to be sent to a critical care unit, and a higher number of comorbid conditions increases the likelihood a patient will be sent to an ICU postoperatively. This patient has multiple comorbidities in addition to his severe OSA, including pulmonary hypertension and diabetes mellitus, and is at risk of developing SIRS/​sepsis from ruptured diverticulitis and thus should likely be sent to an ICU postoperatively. It is important to note that the majority of postoperative complications in one study of patients with OSA occurred after the patient was transferred to the ward.15 A clear plan of care (ideally, made preoperatively) will

help to mitigate or prevent any unforeseen complications postoperatively, and it should always be remembered that patients with sleep apnea are at increased risk of unplanned ICU admissions and readmissions. The decision about whether or not to provide supplemental oxygen to these patients is shrouded in the traditionally taught belief that even 2 L of nasal cannula oxygen in OSA and chronic obstructive pulmonary disease patients will reduce the respiratory drive and potentially contribute to a worsening clinical picture including hypoxia, further hypercapnia, and potentially somnolence. In these patients, hypoxia may play a more critical role in respiratory arousal than in a typical patient, particularly postsurgically. This is also a group in which opioids are likely to be given, further depressing the respiratory drive. However, a study comparing patients with OSA who received supplemental oxygen via nasal cannula postoperatively with those who did not receive any supplemental oxygen showed that some of these concerns are unfounded. The group that received supplemental oxygen via nasal cannula was found to have higher average SaO2, lower oxygen desaturation index, decreased apnea/​ hypopnea index, and a shortened duration of apnea/​hypopnea. Transcutaneous CO2 was not found to be significantly different between the 2 groups.16 In the patient who is noncompliant with CPAP therapy, nasal cannula therapy with either oxygen or an air/​oxygen mix titrated to high-​flow levels (thought to produce a level of CPAP) might be a reasonable alternative with appropriate monitoring. High-​flow nasal cannula as an alternative to CPAP is emerging as a potential therapy for OSA, with several studies showing promising results including a decrease in hypopnea/​apnea episodes similar to that seen with the use of CPAP. This could potentially be better tolerated in a patient unaccustomed to a CPAP mask.17 R E VI EW Q U E S T I O N S 1. Which of the following is not one of the criteria considered in the STOP-​Bang questionnaire? A. Neck circumference B. Gender C. Presence of diabetes D. Presence of hypertension Answer: C The STOP-BANG scoring system uses snoring, daytime so­mnolence, observed apneas, hypertention, age weight, neck circumference and gender to compute a score (Low risk OSA 0–2, intermediate risk 3–4, High risk 5–8). The presence of diabetes is not considered in the STOP-BANG scoring system. 2. Which of the following criteria must be met to make a diagnosis of OHS? A. BMI >30 kg/​m2 B. Awake alveolar hypoventilation (PaCO2 >45 mm Hg), which cannot be attributed to other conditions C. Hypoxemia during the day D. Both A and B E. A, B, and C

234  •   P ostanesthesia C are  U nit

Answer: D Although hypoxemia during the day can certainly be seen in patients with OHS, it is not required to make the diagnosis of OHS. However, the presence of both a BMI>30 kg/m2 as well as awake alveolar hypoventilation (PaCO2 >45 mm Hg) (which cannot be attributed to other conditions), is required for the diagnosis. 3. The prevalence of OSA in the morbidly obese is believed to be greater than A. 20%. B. 40%. C. 70%. D. 90%. Answer: C OSA should always be suspected in a morbidly obese patient since the incidence is reported to be at least 70%. 4. Which of the following is NOT believed to contribute to the development of pulmonary hypertension in patients with OSA? A. Increased afterload on the left ventricle causing elevated left atrial pressures B. Recurrent upper airway obstruction C. Increased venous return leading to right ventricle overload D. Recurrent hypoxia causing pulmonary vasoconstriction Answer: A Recurrent upper airway obstruction, increased venous return leading to right ventricle overload, and recurrent hypoxia causing pulmonary vasoconstriction are all know contributors to the development of pulmonary hypertension. The presence of increased afterload on the left ventricle does not typically result in pulmonary hypertension. 5. Which of the following drugs should be dosed based on total body weight even in the morbidly obese patient? A. Rocuronium B. Succinylcholine C. Propofol D. Remfentanil Answer: B Because of increased levels of plasma cholinesterase in the morbidly obese, succinylcholine should be dosed based on total body weight. Non-depolarizing muscle relaxants such as Rocuronium should be dosed based on ideal body weight, and all other drugs should be dosed based on lean body weight. 6. When CPAP is used postoperatively, which of the following improved outcomes are typically seen in a patient with OSA? A. Fewer unplanned ICU admissions B. Shorter length of stay C. Reduced incidence of respiratory complications D. All of the above Answer: D CPAP use post-operatively has been shown in various studies to result in fewer unplanned ICU admissions, shorter length

of stay, and reduced incidence of respiratory complications. New evidence is emerging that high flow nasal cannulas may also serve as a “replacement” for CPAP in patients who do not tolerate this mode of non-invasive ventilation. 7. Which of the following is NOT typically used in the diagnosis of OHS? A. Serum bicarbonate B. Pulse oximetry C. Left heart catheterization D. Chest radiography Answer: C Serum bicarbonate, pulse oximetry, and chest radiography are all commonly used in evaluating a patient for OHS as well as right heart catheterization. Left heart catheterization would not typically be used. 8. Commonly seen comorbidities in people with morbid obesity include everything but: A. Hypokalemia B. Polycythemia C. Insulin resistance D. Increased risk of chronic kidney disease Answer: A Polycythemia, insulin resistance, and increased risk of chronic kidney disease are all commonly found results of morbid obesity. Hypokalemia is seen in patients with anorexic eating disorders. 9. When considering the factors which make mask ventilation and intubation of the morbidly obese patient more challenging, which is not a factor? A. The pharyngeal airway is narrower and more collapsible B. There is an exaggerated relaxation of pharyngeal muscles in response to narcotics C. The FRC is reduced in these patients D. Succinylcholine response is diminished in the morbidly obese. Answer: D A reduction in FRC lowers the time to desaturation, and narrowing of the pharyngeal airway makes mask ventilation more difficult. Succinylcholine response in these patients should be identical to other patients as long as they are dosed appropriately using total body weight. 10. Patients with OSA are believed to be more sensitive to narcotics as a result of: A. Changes in the metabolism of opioids in the morbidly obese B. An increased sensitivity to opioid effect C. Normal pain processing but diminished analgesic effect of the opioid D. A heightened hypercarbic drive. Answer: B An increased sensitivity to opioid effect as well as alteration of pain processing is believed to cause the increased sensitivity to narcotics seen in patients with OSA regardless of their weight.

2 6. O bstructive S leep  A pnea  •   235

There is no evidence to show that opioids are metabolized any differently in the morbidly obese. The analgesic effect of the opioid is not diminished but rather relatively enhanced in these patients. OSA patients tend to have a depressed hypercarbic drive.

QUESTIONS AND ANSWER S This chapter has accompanying questions and answers which are available to subscribers as part of the Oxford eLearning platform. To access the questions, go to http://​ oxfordmedicine.com/​anesthesiologyPBL REFERENCES 1. Adesanya AO, Lee W, Greilich N, Joshi G. Perioperative management of obstructive sleep apnea. Chest. 2010;138(6):1489–​1498. 2. Piper A, Yee B. Clinical manifestations and diagnosis of obesity hypoventilation syndrome. In:  Post T, ed. UpToDate. Waltham, MA: UpToDate; 2017. www.uptodate.com. Accessed April 13, 2017. 3. Sato S, Hasegawa M, Okuyama M, et al. Mask ventilation during induction of general anesthesia: Influences of sleep apnea. Anesthesiology. 2017;126(1):28–​38. 4. Ingrande J, Lemmens HJM. Dose adjustment of anaesthetics in the morbidly obese. Br J Anaesth. 2010;105(Suppl 1):i16–​i23. 5. Lam KK, Kunder S, Wong J, Doufas A, Chung F. Obstructive sleep apnea, pain and opioids: is the riddle solved? Curr Opin Anesthesiol, 2016;29:134–​140. 6. Chung F, Nagappa M, Singh M, Mokhlesi B. CPAP in the perioperative setting: evidence of support. Chest. 2016;149 (2):586–​597. 7. Neligan PJ, Malhotra G, Fraser M, et al. Continuous positive airway pressure via the Boussignac system immediately after extubation

improves lung function in morbidly obese patients with obstructive sleep apnea undergoing laparoscopic bariatric surgery. Anesthesiology. 2009;110(4):878–​884. 8. Gupta RM, Parvizi J, Hanssen AD, Gay PC. Postoperative complications in patients with obstructive sleep apnea syndrome undergoing hip or knee replacement; a case-​control study. Mayo Clin Proc. 2001;76(9):897–​905. 9. Kadam VR, Danesh M. Postoperative capnostream monitoring in patients with obstructive sleep apnoea symptoms—​case series. Sleep Science. 2016;9:142–​146. 10. Maddox RR, Oglesby H, Williams CK, et al. Continuous respiratory monitoring and a “smart” infusion system improve safety of patient-​ controlled analgesia in the postoperative period. Adv Patient Safety. 2008;4. https://​www.ncbi.nlm.nih.gov/​books/​NBK43753/​ 11. Practice guidelines for the perioperative management of patients with obstructive sleep apnea: an updated report by the American Society of Anesthesiologists Task Force on Perioperative Management of Patients with Obstructive Sleep Apnea. Anesthesiology. 2014;120:268–​286. 12. Liao P, Luo Q, Elsaid H, Kang W, Shapiro CM, Chung F. Perioperative auto-​titrated continuous positive airway pressure treatment in surgical patients with obstructive sleep apnea: a randomized controlled trial. Anesthesiology. 2013;119:837–​847. 13. Kushida CA, Berry RB, Blau A, et al. Positive airway pressure initiation: a randomized controlled trial to assess the impact of therapy mode and titration process on efficacy, adherence, and outcomes. Sleep. 2011;34:1083–​1092. 14. Dezube R, Housten T, Mathai S. Postoperative care of the patient with pulmonary hypertension. Adv Pulm Hypertens. 2013;12(1):24–​30. 15. Liao P, Yegneswaran B, Vairavanathan S, Zilberman P, Chung F. Postoperative complications in patients with obstructive sleep apnea:  a retrospective matched cohort study. Can J Anesth. 2009;56:819–​828. 16. Liao P, Wong J, Singh M, et  al. Postoperative oxygen therapy in patients with obstructive sleep apnea: a randomized controlled trial. Chest. 2017;151:591–​611. doi:10.1016/​j.chest.2016.12.005. 17. Nishimura M. High-​flow nasal cannula oxygen therapy in adults. J Intensive Care. 2015;3:15.

236  •   P ostanesthesia C are  U nit

27. RHEUMATOID ARTHRITIS PATIENT PRESENTING FOR A TOTAL KNEE REPLACEMENT Perin Kothari and Sree Kolli

S T E M C A S E A N D K EY Q U E S T I O N S A 68-​year-​old female presents to the operating room for an elective total knee arthroplasty for osteoarthritis. Her past medical history includes stable coronary artery disease, well-​ controlled diabetes, and hypertension. In addition, she suffers from rheumatoid arthritis (RA), which began in her wrists, but now has spread “all over her body.” Upon further questioning, she states that her neck, wrists, elbows, knees, and hips are all aching intermittently. She also has a drug-​eluting stent from a distant myocardial infarction and still takes daily aspirin and clopidogrel. She has not taken either medication in 16 days. WH AT A R E T H E P R EO P E R AT I V E C O N C E R NS F O R T H I S PAT I E N T ?

RA, although commonly presenting as pain in joints, can have several extra-​articular manifestations that are relevant to an anesthesiologist. RA is a type of arthritis that develops from chronic synovial tissue inflammation.1 It is 3 times more common in females compared to males. Over a period of time, this inflammation will result in loss of joint strength and bone erosion. Beyond joint pain, patients with RA will take a variety of medications that can have effects on the immune system or coagulation status. Furthermore, there can be significant airway involvement. In addition, cardiac and pulmonary organ may also be involved requiring further workup of these systems prior to surgery. YO U R PAT I E N T C O M P L A I N S O F J O I N T PA I N A S 1 O F H E R S Y M P TO M S. WH AT OT H E R S Y M P TO M S C A N B E P R E S E N T T H AT WO U L D C LU E YO U I N TO H E R D I AG N O S I S ?

The patient may also complain about systemic symptoms such as anorexia, fatigue, and weakness. She may also have lymphadenopathy on physical exam, especially in the head and neck region. Furthermore, joints that are involved may have stiffness and may have tenderness and warmth to the touch.

WH AT I S T H E P R I M A RY T R E AT M E N T O F R A , A N D WH AT A R E YO U R C O N C E R N S WI T H T H E S E M E D I C AT I O NS ?

Once the diagnosis of RA is made clinically and with laboratory studies, patients are started on medications that can aid in preserving function in those affected joints. These medications are known as disease-​modifying antirheumatic drugs. RA patients often are taking pain medications ranging from nonsteroidal anti-​ inflammatories to chronic opioid therapy for management of their symptoms. Notably, disease-​ modifying antirheumatic drug therapy can have a significant effect on immune function. These medications predispose patients for infection, which is always of concern to orthopedic surgeons who are planning for a joint replacement. Furthermore, nonsteroidal anti-​inflammatory therapy have commonly known renal and gastrointestinal side effects, which should be questioned on history. U P O N A I RWAY E X A M , S H E S TAT E S S H E H A S S O M E N EC K S T I FFN E S S. U P O N FL E X I O N, S H E S TAT E S T H AT H E R FI N G E R S B EC O M E NUM B. WH AT D O YO U S US P EC T ?

The patient states that she has a stiff neck lately with range of motion testing, indicating she may have cervical involvement of her RA. Instead of canal stenosis or other pathology, RA patients may suffer from an increased risk of atlantoaxial subluxation, usually anteriorly. This can occur in up to 40% of patients with RA. The symptoms include cervical neck pain and myelopathy.2 WH AT P R EO P E R AT I VE T E S T I N G I S R EQ U I R E D B E F O R E A I RWAY M A N I P U L AT I O N ?

In patients with neurological symptoms, flexion, extension cervical ,and open mouth odontoid X-​ray films should be examined prior to surgery even if a regional or neuraxial anesthetic is planned.

237

WH AT OT H E R C O N S E Q U E N C E S O F R A C A N A F F E C T A I RWAY M A NAG E M E N T ?

Since all synovial joints may be affected, there may be temporomandibular joint (TMJ) involvement. This manifests as small mouth opening and failure of an upper lip bite test. Furthermore, cricoarytenoid joints can also be involved. This would result in decreased movement of vocal cord, which may make inserting an endotracheal tube past the glottis difficult. U P O N C H A RT R EV I EW, T H E PAT I E N T H A D P R EV I O US LY P R E S E N T E D TO T H E E M E RG E N C Y D E PA RT M E N T WIT H S H O RT N E S S O F B R E AT H A N D F EV E R . S H E WA S D I AG N O S E D WI T H B RO N C H I T I S A N D D I S C H A RG E D WI T H A N T I B I OT I C S. WH AT I S A N A LT E R NAT I VE D I AG N O S I S ?

These symptoms can alert the anesthesiologist to pericarditis. Symptoms of pericarditis can include fever and dyspnea. Upon exam and testing, a friction rub can be heard and diffuse ST elevations may be seen. If there is pulsus paradoxus, this could be due to constrictive pericarditis. Since right heart failure and effusion can be a manifestation, echocardiography would be an important tool for perioperative evaluation of pericarditis. T H E PAT I E N T H A S ITC H Y A N D D RY EY E S A N D TA K E S EY E D RO P S DA I LY. WH AT A R E YO U R C O N C E R N S R EG A R D I N G  T H I S ?

Other connective tissue disorders can be present in patients with RA as well. Uncomplicated keratocojunctivitis sicca or Sjrogren’s syndrome can be present concurrently affecting mucous membranes. Patient are especially at risk for corneal abrasions due to dry eyes. Thus, ophthalmologic ointments should be included in their perioperative care. DISCUSSION P R EO P E R AT I VE A S S E S S M E N T

RA is a connective tissue disorder defined by synovial joint destruction. It is a common affliction that affects approximately about 1% of adults but affects 3 times more women than men with symptoms beginning between the ages of 30 and 50 years. Thus, it is a common disease that will present to an anesthesiologist for evaluation in the perioperative period. Because of its propensity to affect multiple organ symptoms, a thorough history and physical is necessary. Imaging and other investigative studies are often warranted. Symptoms of RA include pain and stiffness of multiple joints. This often present in the morning and persists after activity. Symptoms are progressive and often spread to other joints over several months. However, extra-​articular symptoms affect 40% of patients with rheumatoid arthritis.2 Constitutional symptoms are also present, characterized by generalized fatigue and weakness. On physical exam, cervical lymphadenopathy can be noted.1 RA

has a propensity to affect the pericardium by inducing restrictive pericarditis. Patients with pericarditis may present with symptoms of dyspnea and chest pain. Upon physical exam, a pericardial friction rub can be auscultated. Furthermore, a pericardial effusion can also occur, and valvular pathology such as mitral valve prolapse can be seen as well. A preoperative echocardiogram would help identify areas of concern. Respiratory system involvement includes pleural effusion, fibrosis, and pneumonitis.3 Subcutaneous rheumatoid nodules that can be found near joints as well. Finally, ketatoconjunctivitis sicca, either isolated or associated with Sjorgren’s syndrome, is common, increasing the risk of perioperative corneal abrasion. It is important to include perioperative eye lubrication and other careful ophthalmologic eye care in patients with RA. Treatment of RA involves early pharmacotherapy with disease modifying anti-​rheumatic drugs. Because of the immune modulating risks of these medications, these patients are at high risk for infection. This risk is magnified in the perioperative period, especially when implanting artificial joints. Furthermore, these patients often have a degree of chronic pain. They may often take long-​term nonsteroidal anti-​inflammatory medications. The renal, hematological, and gastrointestinal side effects of these medications should be reviewed. I N T R AO P E R AT I V E M A NAG E M E N T

Airway management in patients with RA can be challenging. A thorough workup and discussion is needed with both the patient and the surgeon to evaluate and plan for management of a difficult airway. Because of their peripheral joint involvement, patients with RA may present for orthopedic surgery that may be accomplished with regional or neuraxial anesthesia, Nevertheless, it is important to have a plan for a rescue airway in these patients. RA patients have multiple contributors that can make endotracheal intubation difficult. Cervical spine involvement is 1 of the most well-​known effects of RA. The cervical spine is the second most commonly affected joint by RA, only behind the metacarpophalangeal joints.3 Pain is the most common symptom identifying cervical spine pathology. However, neurological symptoms such as myelopathy can be present. The most common cervical pathology stems from anterior atlantoaxial subluxation (C1 on C2), occurring 43% to 86% of patients with RA.4 In patients with RA, the odontoid process is abnormally freed. Thus, with flexion of the head the odontoid can move most commonly anteriorly causing spinal cord and/​or vertebral artery compression leading to paresis, spinal shock, and death.1 With up to 31% patients with RA being affected, TMJ involvement can further complicate airway management. TMJ dysfunction can inhibit mouth opening and subluxation. Furthermore, upper airway obstruction is common in patients with TMJ.5 Cricoarytenoid arthritis can be seen as well, which would make passing of an endotracheal tube through the vocal cord challenging due to limited movement of the cords. Furthermore, these joints may be swollen and erythematous.1

238  •   P ostanesthesia C are  U nit

Due to these factors, airway management of patients with RA is often challenging. Not only are there risk factors for difficult intubation, but even with successful endotracheal intubation there is a risk for significant neurological injury from inadequate positioning. Before making an airway management plan, several considerations must be made. Some of these include a comprehensive airway assessment, comorbidities, type of surgery, positioning during surgery, patient cooperation, and skills of the anesthesiologist and equipment available to them. It is important to do a focused neurologic exam to document any preexisting neurological deficits on the day of surgery. Furthermore, due to the high prevalence of cervical spine involvement in RA, preoperative imaging has been recommended to determine possible instability. Complete X-​ray imaging with lateral flexion, extension and open-​mouth odontoid views are essential. The safest method to secure and airway in patients with neurological symptoms of radiographic evidence of cervical disease involvement is a fiber-​optic bronchoscopic intubation with uninterrupted spontaneous respiration with a cervical collar in place.1 Regardless of the method of airway management, the American Society of Anesthesiologists’ Difficult Airway Guidelines should be followed.6 Surgical positioning is another concern for patients with RA. Ideally, patient should be placed in surgical position while awake to ensure all joints are comfortably positioned and well padded. Appropriate positioning of the head is debated in literature, and, thus, awake positioning can help guide management.7 The use of laryngeal mask airways (LMA) has been proven to be safe in patients with RA.8 Placement of an LMA used as a conduit to place an endotracheal tube can be used as an alternative for patients who refuse an awake intubation. A technique to do this while continuing spontaneous respiration is using an inhaled induction technique, followed by LMA placement. After confirmation of placement, muscle relaxant can be given, and then an endotracheal tube can be passed with or without bronchoscopic guidance. After completion of the procedure, extubation must be done with care and planned well in these patients. Judicious use of opioids is necessary postoperatively because reintubation and even emergency surgical airways may be impossible. If appropriate, regional and neuraxial anesthesia can serve as a good alternative to general anesthesia if the surgery type is appropriate, no contraindications being present, and the patient is agreeable. However, the ability to rescue the airway is essential and must be planned for in advance. R E VI EW Q U E S T I O N S 1. Which of the following organ system is NOT involved in RA? A. Cardiovascular  system B. Respiratory  system C. Nervous  system D. Renal  system E. Skeletal  System

Answer: D Rhematoid arthritis is a disease that affects several different organ systems. Beyond the synovial joint destruction in the skeletal system, patients can be affected by restrictive pericarditis, pleural effusions/pneumonitis, ophthalmologic concerns, and spinal cord compression due to cervical joint instability. 2. Which of the following symptoms is NOT expected in a patient with RA? A. Neck  pain B. Wrist  pain C. Dyspnea D. Infertility E. Dry  eyes Answer: D In RA, polyarticular joint pain ranging from neck, wrists, elbows, knees and hips is seen. Furthermore, dyspnea can be seen with pulmonary involvement due to pleural effusions, fibrosis, or pneumonitis. Keratocojunctivitis sicca or Sjrogren’s syndrome can be present concurrently as well. Infertility has not been associated with RA. 3. Which valvular abnormality is most commonly associated with RA? A. Tricuspid valve endocarditis B. Mitral stenosis C. Mitral valve prolapse D. Aortic regurgitation E. Bicuspid aortic valve Answer: C Mitral valve prolapse has been associated with RA. While other cardiac cormorbdiities can be observed in patients with RA, only MVP has been linked to the disease. 4. Which of the following direction of subluxation of the atlanto-​axial joint is most commonly seen in patients with RA? A. Superior B. Inferior C. Posterior D. Lateral E. Anterior Answer: E Instead of canal stenosis or other pathology, RA patients may suffer from an increased risk of atlantoaxial subluxation, usually anteriorly. This can occur in up to 40% of patients with RA. The symptoms include cervical neck pain and myelopathy. 5. Which is NOT an associated airway abnormality in patients with RA? A. Pain with movement of neck B. Short thyromental distance C. Numbness of hand with neck flexion D. Cricoarytenoid joint arthritis E. Small mouth opening Answer: B

27. R A Patient P resenting for A Total K nee R eplacement  •   239

Since all synovial joints can be affected in RA, the affects on airway management are significant. There can be pain with movement of the neck and cord compression due to the anterior movement of the atlantoaxial joint. Since the TMJ can be involved, there can be small mouth opening and failure of the upper lip bite test. Furthermore, cricoarytenoid joints can also be involved. This would result in decreased movement of vocal cord, which may make inserting an endotracheal tube past the glottis difficult. 6. Which of the following is NOT good candidate for neuraxial anesthesia in a patient with RA? A. Patient that took aspirin on the day of surgery B. Patient undergoing total knee arthroplasty C. A claustrophobic patient D. Patient with a history of nausea and vomiting E. A patient with a pacemaker Answer: C Per ASRA guidelines, patients taking aspirin can still be candidates for neuraxial anesthesia. Lower extremity joint replacement patients are prime candidates for neuraxial anesthesia barring other contraindications. Avoiding volatile anesthetics and opioids by using neuraxial anesthesia may help prevent PONV in patients with a known history. Presence of a pacemaker is not a contraindication to neuraxial anesthesia, although the underlying disease must be considered. A claustrophobic patient who would not tolerate laying supine under drapes may not be ideal for neuraxial anesthesia.

vertebral artery compression leading to paresis, spinal shock, and death. 9. What x-ray views of the cervical spine need to be obtained in patients with RA prior to surgery? A. Lateral flexion B. Lateral extension C. Open-mouth odontoid D. All of the above Answer: D Due to the high prevalence of cervical spine involvement in RA, preoperative imaging has been recommended to determine possible instability. Complete X-ray imaging with lateral flexion, extension and open-mouth odontoid views are essential. 10. What joint would not be affected in patients with RA? A. Metacarpophalangeal joints B. Temporomandibular joint C. Pubic bone joint D. Cricoarytenoid joint E. Elbow Joint Answer: C The pubic bone is a nonsynovial amphiarthrosis. Since RA only affects synovial joints, the pubic bone is not involved. All of the other joints listed are synovial joints and can be commonly involved.

7. Which of the following technique is considered the safest way to manage the airway of a patient with RA?

QUESTIONS AND ANSWER S

A. Fiber-​ optic intubation after induction doses of propofol and rocuronium B. Video laryngoscope C. Surgical  airway D. Fiber-​optic intubation with spontaneous respiration E. Direct laryngoscopy with manual in line stabilization Answer: D The safest method to secure and airway in patients with neurological symptoms of radiographic evidence of cervical disease involvement is a fiber-optic bronchoscopic intubation with uninterrupted spontaneous respiration with a cervical collar in place. Regardless of the method of airway management, the American Society of Anesthesiologists’ Difficult Airway Guidelines should be followed.

This chapter has accompanying questions and answers which are available to subscribers as part of the Oxford eLearning platform. To access the questions, go to http://​ oxfordmedicine.com/​anesthesiologyPBL

8. With flexion of the head in patients with RA, there can be odontoin can move anteriorly can potentially cause all of these issues EXCEPT: A. Spinal shock B. Stroke C. Paresis D. Blindness E. Death In patients with RA, the odontoid process is abnormally freed. Thus, with flexion of the head the odontoid can move most commonly anteriorly causing spinal cord and/or

REFERENCES 1. Miller RD. Miller’s Anesthesia. 8th ed. Philadelphia: Elsevier; 2015. 2. Portnoy D, Hagbderg C. Management of the Difficult and Failed Airway. 3rd ed. New York: McGraw-​Hill; 2017. 3. Hines R., Marschall K. Stoelting’s Anesthesia and Coexisiting Disease. Philadelphia: Elsevier; 2015. 4. Tokunaga D, Hase H, Mikami Y, et  al. Atlantoaxial subluxation in different intraoperative head positions in patients with rheumatoid arthritis. Anesthesiology. 2006;104:675–​679. 5. Yoshida A, Higuchi Y, Kondo M, et al. Range of motion of the temporomandibular joint in rheumatoid arthritis: relationship to the severity of disease. Cranio. 1998;16:162–​167. 6. American Society of Anesthesiologists Task Force on Management of the Difficult Airway. Practice guidelines for management of the difficult airway:  an updated report by the American Society of Anesthesiologists Task Force on Management of the Difficult Airway. Anesthesiology. 2013;118:251–270. 7. Takenaka I, Urakami Y, Aoyama K, et al. Severe subluxation in the sniffing position in a rheumatoid patient with anterior atlantoaxial subluxation. Anesthesiology. 2004;101:1269–​1277. 8. Bilgin H, Bozkurt M. Tracheal intubation using the ILMA, C-​Trach, or McCoy laryngoscope in patients with a simulated cervical spine injury. Anesthesia. 2006;61:685–​691.

240  •   P ostanesthesia C are  U nit

SECTION XIII OBSTETRICS

28. ANESTHESIA IN A PREGNANT PATIENT PRESENTING FOR URGENT NONOBSTETRIC SURGERY Shobana Bharadwaj and Jessica Galey

S T E M C A S E A N D K EY Q U E S T I O N S A 24-​year-​old G2P0101 was admitted at 33-​weeks gestation with nausea, vomiting, and right upper quadrant pain. The fetal heart rate (FHR) tracing was category 1, and blood pressure was within normal limits. Workup was consistent with acute cholecystitis, and she is scheduled for a cholecystectomy. T H E PAT I E N T I S V E RY A N X I O US A N D WA N T S TO K N OW I F T H E R E A R E A N Y R I S K S O F A N E S T H E S I A A N D S U RG E RY O N H E R FET US A N D P R E G NA N C Y. WH AT A R E R I S K S T H AT YO U WO U L D D I S C US S WI T H T H I S PAT I E N T ?

Clinical studies suggest that anesthesia and surgery during pregnancy does not increase the risk of congenital anomalies.1 However, there are other possible risks including the intraoperative concerns of uteroplacental perfusion and/​or fetal oxygenation, preterm labor and delivery, the effects of the disease process itself ,and the teratogenicity of anesthetic agents or other drugs administered during the perioperative period.2 T H E S U RG E O N WA N T S TO P E R F O R M T H I S P RO C E D U R E L A PA RO S C O P I C A L LY. I S T H AT A P P RO P R I AT E?

It would be appropriate to perform a laparoscopic cholecystectomy given the potential benefits being shorter hospital stay, less postoperative pain and use of narcotics, faster return to normal activities, and decreased risk for thromboembolic complications; there are no differences in maternal and fetal outcomes. Laparoscopy can be safely performed during any trimester of pregnancy.3,4 WH AT D O E S T H E P R EO P E R AT I V E P R E PA R AT I O N A N D M A NAG E M E N T I N VO LVE I N T H I S PAT I E N T ?

Preoperative preparation starts with communication between the surgeon, anesthesiologist, obstetrician and neonatologist. A plan should be in place that addresses how to proceed in the event of persistent non-​reassuring fetal status. Premedication

with histamine receptor antagonist, metoclopramide, and sodium bicitrate is necessary for acid aspiration prophylaxis as the pregnant patient is at risk for aspiration. WH AT K I N D O F I N T R AO P E R AT I VE MO N ITO R S A R E US E D F O R T H I S L A PA RO S C O P I C P RO C E D U R E , A N D H OW WO U L D YO U P O S IT I O N T H I S PAT I E N T ?

In addition to the standard American Society of Anesthesiologists intraoperative monitors, intraoperative FHR monitoring should be considered when feasible.5 Regardless of the intraoperative feasibility, FHR and uterine contractions should be monitored in the preoperative and postoperative period. Intraoperative CO2 monitoring by capnography is key during laparoscopy in the pregnant patient to maintain PaCO2 within maternal physiologic range and avoid fetal acidosis. Left uterine displacement position should be maintained to avoid hypotension from aortocaval compression. WH AT A R E T H E I N T R AO P E R AT I V E C O N C E R NS , A N D WH AT WI L L B E T H E I N T R AO P E R AT I V E A N E S T H ET I C M A NAG E M E N T ?

Placement of trocar is adjusted according to the fundal height of the uterus to minimize the risk of injuring the uterus.6 Pneumoperitoneal pressures should be below 15  mm Hg to reduce the risk of decreased uteroplacental perfusion, maternal ventilatory challenges, and fetal acidosis. Deep vein thrombosis prophylaxis with pneumatic compression device is important due to the risk of venous stasis with pneumoperitoneum. Intraoperative anesthetic management includes preoxygenation and rapid sequence induction of intravenous anesthesia and controlled ventilation and maintenance with inhalation agents and opioid analgesics. Difficult laryngeal visualization should be anticipated due to airway changes in pregnancy. WH AT A R E T H E FETA L A N D M AT E R NA L E FFEC TS O F L A PA RO S C O P Y ?

Although fetal acidemia from absorbed carbon dioxide is a concern, direct fetal effects from insufflation and

243

pneumoperitoneum are limited. Maternal hypotension from decreased preload due to insufflation, use of reverse Trendelenburg position and aortocaval compression may necessitate the use of vasopressor to maintain uteroplacental perfusion. Increase in peak airway pressure occurs from decreased lung compliance from insufflating a gravid abdomen. WH AT D RU G S C A N B E US E D F O R I N D U C T I O N A N D M A I N T E NA N C E?

Propofol and succinylcholine are safe for use as rapid sequence induction agents. Maintenance with moderate concentration of volatile agent with an opioid and non-​depolarizing muscle relaxant and avoidance of nitrous oxide would prevent bowel distention and allow administration of higher FiO2. WH AT A R E T H E V E N T I L AT I O N G OA L S F O R T H I S PAT I E N T D U R I N G T H E P RO C E D U R E?

Positive pressure ventilation is required to avoid hypercarbia, and thereby fetal acidosis, by maintaining end-​tidal carbon dioxide between 30 mm Hg to 34 mm Hg.7 At least 50% inspired oxygen concentration should be used to help preserve fetal well-​being along with avoiding respiratory alkalosis and hypotension that can reduce uteroplacental perfusion. T H E S U RG E O N N OW WA N T S TO C O N V E RT TO O P E N C H O L E C YS T EC TO MY. H OW D O E S T H AT A F F E C T T H E A N E S T H ET I C M A NAG E M E N T ?

The ventilator parameters have to be adjusted to avoid hyperventilation and vasoconstriction by maintaining the EtCO2 between 28 and 34  mm Hg. Anticipate an increase in opioid requirements and fluid shifts. Reassess the feasibility to monitor the FHR and uterine activity after release of pneumoperitoneum with external monitors.

T H E FH R T R AC I N G D U R I N G T H E S U RG E RY I S S H OWN I N T H E F O L L OWI N G FI GU R E 28.1. WH AT I S T H E I N T E R P R ETAT I O N O F T H I S T R AC I N G ? WO U L D YO U M A K E A N Y C H A N G E S I N T H E A N E S T H ET I C M A NAG E M E N T ?

It is a category 2 tracing with a baseline of 150 beats per minute with minimal to no variability but without decelerations. There are no contractions detected by the tocodynamometer. The effect of opioids and inhalational agents is to decrease the beat-​to-​beat variability. Inhalational agents additionally can cause uterine quiescence from smooth muscle relaxation. It is critical to maintain a normal maternal PaO2, PaCO2, and uterine blood flow. The FHR tracing is anticipated under general anesthesia. T H E P RO C E D U R E WA S U N EV E N T F U L , A N D T H E PAT I E N T I S E M E RG I N G FRO M A N E S T H E S I A . H OW WO U L D YO U R EV E R S E T H E N EU RO MUS CU L A R B L O C K A D E , A N D WH AT A R E T H E C O N C E R NS ?

It is important to check neuromuscular blockade with train of four twitches during the procedure and for a complete reversal. Atropine is administered with neostigmine for reversal due to greater placental transfer of atropine compared to glycopyrrolate to prevent possible fetal bradycardia with neostigmine.8 Caution should be exercised when administering sugammadex to pregnant women as no clinical data on exposed pregnancies are available. H OW WO U L D YO U M A NAG E T H I S PAT I E N T P O S TO P E R AT I VE LY ?

FHR and uterine activity monitoring should be continued in the postanesthesia care unit. Initiation of regional analgesia—​neuraxial/​peripheral nerve block—​will aid with good pain control with minimal sedation. Avoid nonsteroidal anti-​inflammatory drugs because they may cause

Figure 28.1

244  •   O bstetrics

premature closure of the ductus arteriosus at this gestational progesterone cause an increase in minute ventilation primarily age. Thromboprophylaxis should be continued in the post-​ via increased tidal volume. This increased alveolar ventilation operative period. leads to a decrease in maternal PaCO2 to approximately 28–32 mm Hg. This respiratory alkalosis is compensated for by the increased renal excretion of bicarbonate leading to a blood DISCUSSION HCO3 concentration of 16–21 mEq/L. Functional residual capacity and closing volume also declines as the uterus grows resulting in decrease oxygen reserve. This combined with the I N C I D E N C E A N D P RO C E D U R E S increased rate of oxygen consumption, apnea during anesENCOUNTERED thesia leads to more rapid oxygen desaturation in pregnancy. Caring for the pregnant surgical patient presents the Additionally, estrogen mediates an increase in friability of Anesthesiologist with a unique challenge when needing to nasal/oral mucus membranes and facial/airway swelling may care for both maternal and fetal well-being. There is a need occur leading to a potential for a difficult airway scenario. The GI system is largely affected by the compressive effects to not only understand the surgical procedure the patient of the gravid uterus altering the gastric/pyloric anatomy will undergo but also how the altered physiology of pregleading to incompetence of the lower esophageal sphincter and nancy and fetoplacental physiology impacts the anesthetic increase incidence of reflux. It is unclear at what gestational age decision making during the preoperative, intraoperative, and this effect becomes clinically significant, however most anespostoperative management. It is generally accepted that only thesia providers treat pregnant women as increased risk of asthe procedures of urgent nature are performed during pregpiration of gastric contents by the second trimester onwards. nancy. Commonly encountered procedures involve acute Of particular interest to the Anesthesiologist is the inintraabdominal disease (appendectomy, cholecystectomy) alcrease in sensitivity of the nervous system to the effects of though gynecologic procedures and maternal trauma occur as anesthetics during pregnancy. There is a decrease in MAC well. Maternal trauma is a large category representing signiffor inhaled anesthetics up to 30–40% by term. This change is icant morbidity and mortality. There are also pregnancy rethought to be mediated by the increase in progesterone during lated need for surgeries including cervical cerclage placement pregnancy. Also important is the increase in neural sensitivity or ovarian cystectomy or torsion. to local anesthetics leading to a reduction in dose requirement for spinal/epidural anesthesia. P H YS I O L O G I C C H A N G E S O F P R EG NA N C Y I N R EVI EW 9

An understanding of the physiologic changes of pregnancy is of utmost importance when caring for the pregnant surgical patient. These changes are the result of the altered hormones of pregnancy, the addition of the low-resistance placental unit, mechanical effects from the growing uterus, and increased metabolic demands. Changes in the cardiovascular, pulmonary, gastrointestinal and neurologic systems are of great importance in the perioperative setting. The cardiovascular system has high demands placed upon it in the setting of pregnancy in order to ensure adequate perfusion to the uteroplacental vascular bed. During pregnancy cardiac output increases up to 50% above baseline via increase in both heart rate and stroke volume. There is also a drop in systemic vascular resistance with the addition of the low resistance placental bed. In addition, there is an increase in maternal plasma volume up to 50% at term along with 30% increase in red cell volume leading to a dilutional anemia. After approximately 20 weeks gestation, compression of the inferior vena cava by the growing uterus can occur particularly when the patient is placed in the supine position. This can compromise venous return and cardiac output as well as increase uterine venous pressure leading to a decrease in uteroplacental blood flow. Therefore, after approximately 20 weeks gestation, pregnant patients should always be positioned with uterine displacement to avoid the compressive effects of the uterus. The pulmonary system also has a high demand placed upon it during pregnancy and the physiologic changes that occur begin as early as the first trimester when the effects of

FETA L S A FET Y

Fetal oxygenation is directly dependent upon maternal oxygenation and circulation. Therefore, the maintenance of normal maternal physiology is of utmost importance to ensure fetal well-being. Mild reductions in maternal oxygenation can be tolerated by the fetus however severe maternal hypoxemia will result in fetal hypoxemia and intrauterine asphyxia if left untreated. Situations where there is severe maternal hypoxemia such as respiratory failure, difficult intubation, aspiration, etc. rapid correction is necessary to prevent harm to the fetus. Similarly, severe maternal hypotension can lead to a decrease in uteroplacental perfusion which left untreated will lead to fetal hypoxemia and intrauterine asphyxia. Hypotension may occur due to high spinal/epidural levels with sympathectomy or deep general anesthesia for which maternal hemodynamics can be supported with intravenous fluids or low dose vasopressors. Hypotension in the surgical setting may also be due to aortocaval compression, hemorrhage/hypovolemia or high laparoscopic insufflation pressures. Regardless of the cause, correction is necessary to ensure adequate uteroplacental perfusion and fetal wellbeing. Maternal hypercapnia is also of concern as it can contribute to fetal academia as the fetus is unable to offload carbon dioxide across the placenta. Maintenance of normal maternal PaCO2 (28–32 mmHg) will ensure proper acid base status to support gas exchange across the placenta. The increased risk to the fetus may be due to the condition that necessitated surgery in the first place, with the highest rate in gynecologic procedures.

2 8. A nesthesia in a P regnant Patient  •   245

F ETA L E F FE C TS O F A N E S T H E S I A A N D S YS T E M I C D RU G S

Many drugs are currently utilized during a routine anesthetic. Volatile anesthetics are widely used for the maintenance of general anesthesia. Being lipid soluble, volatile anesthetics are able to cross the placenta to some extent leading to fetal CNS depression as evidence by decrease fetal heart rate variability. Opioids and IV induction agents similarly cause decrease in fetal heart rate variability. However, decelerations in fetal heart rate would not occur and other causes (i.e. maternal hypotension, hypoxia, hypercarbia, hypovolemia) should be sought and corrected. P R EVE N T I O N O F P R ET E R M L A B O R

Elective procedures should never be performed during pregnancy. Surgical procedures that are performed during pregnancy are often urgent or emergent in nature and involve ensuring maternal wellbeing, which is necessary for fetal wellbeing. Many studies have shown a higher incidence of abortion and preterm labor/delivery in pregnant women undergoing non-obstetric surgical procedures.10 These results are confounded by the fact that it is not clear whether the primary disease process leading to the need for surgery, manipulation of the uterus during surgery, or the surgical procedure itself cause these findings. Likely it is a combination of these factors. Surgical procedures remote from the abdomen and those that can be done during the second trimester carry a lower risk. The greatest risk of preterm labor is during the first week postoperatively. During this time patients are often monitored for signs of preterm labor while they recover. Occasionally, combined cesarean delivery with another surgical procedure such as cardiopulmonary or neurosurgical cases may be done. This decision is based upon the gestational age of the fetus, the risk of the surgical procedure to the fetus and the risk to the mother for delivery at a later time. During these cases the cesarean delivery is performed prior to the second procedure. S P EC I A L C O N S I D E R AT I O N S F O R T H E P R E G NA N T S U RG I C A L PAT I E N T L A PA RO S C O P Y

Special concerns exist when laparoscopic surgical procedures are planned on the pregnant patient. These concerns include abdominal insufflation leading to decreased uteroplacental perfusion either directly via increased intraabdominal pressure or via reduction in maternal cardiac output, fetal acidosis from absorbed carbon dioxide used for abdominal insufflation, and direct uterine or fetal trauma from surgical instruments.11 Meticulous surgical technique and maintenance of normal maternal physiology are necessary to avoid problems related to laparoscopic surgical procedures. F ETA L M O N ITO R I N G D U R I N G S U RG E RY

The ACOG position regarding the use of intraoperative fetal monitoring has been that the use of it should be individualized

on a case by case basis and should utilize inter-disciplinary discussion between the surgical team, obstetric team, pediatric team, and anesthesiology team.5 Intraoperative fetal monitoring requires that someone is present in the operating room that is able to interpret the fetal heart rate tracing and also that a plan exists for if abnormalities arise. As stated previously, during general anesthesia a decrease in variability would be expected from an anesthetized fetus, however decelerations may indicate uteroplacental hypoperfusion and the need for maternal resuscitation or even delivery. First maternal wellbeing would be evaluated for the presence of hypotension, hypoxia, acid-base disturbance. If decelerations would continue or progress to fetal bradycardia, cesarean delivery may be warranted. M AT E R NA L AC L S

ACLS in the pregnant woman should proceed with the same indications and treatment as the non-pregnant woman with the exception of the necessity for left uterine displacement during chest compressions. Aortocaval compression during CPR may compromise the effectiveness of chest compressions and limit maternal resuscitative efforts. If resuscitative efforts are unsuccessful in achieving return of circulation within about 5 minutes, resuscitative hysterotomy should be considered both for increasing the chance of fetal survival and improving the possibility of success with maternal resuscitative efforts.12 A N E S T H ET I C M A NAG E M E N T O F T H E P R EG NA N T S U RG I C A L PAT I E N T P R EO P E R AT I VE A S S E S S M E N T

A thorough preoperative assessment of the pregnant surgical patient includes evaluation of the patients medical conditions, obstetric history as well as a physical exam with focus upon the patients airway parameters. As stated above, pregnant patients have increased risk of difficult airway, desaturation with apnea, and aspiration of gastric contents. Plans for airway management should be made as well as premedication with nonparticulate antacid (ie sodium citrate) or H2 receptor antagonist should be considered. I N T R AO P E R AT I V E M A NAG E M E N T

Anesthetic choice should be guided by maternal status and planned surgical procedure. When feasible, local or regional techniques are preferred as they have the highest safety profile. However, most intraabdominal procedures, especially laparoscopic procedures, require the use of general anesthesia. Of high importance during the intraoperative management is maintenance of normal maternal physiology via monitoring of heart rate, blood pressure, capnography and oxygen saturation as well as ensuring the prevention of aortocaval compression with proper positioning. For general anesthesia, increased risk of aspiration of gastric contents mandates endotracheal intubation. Aspiration precautions should be taken including the consideration of

246  •   O bstetrics

premedications as above, adequate preoxygenation, and rapid sequence induction with cricoid pressure. Vasopressors should be available to treat maternal hypotension.

maternal physiologic range to avoid hypercarbia causing fetal acidosis or hyperventilation causing vasoconstriction.

P O S TO P E R AT I VE C A R E

A. function residual capacity. B. tidal volume. C. dead  space. D. oxygen consumption. Answer: A Functional residual capacity and closing volume decrease in pregnancy. This, combined with increased rate of oxygen consumption, leads to rapid oxygen desaturation after onset of apnea in pregnancy. Increase in minute ventilation is primariyly via increase in tidal volume. The deadspace to tidal volume ratio remains unchanged.

Pregnant patients are monitored for uterine activity and fetal heart rate after the surgical procedure is complete. Analgesia should be provided as well as routine postoperative care. R E VI EW Q U E S T I O N S 1. Decelerations in FHR under maternal general anesthesia A. occur because induction agents and opioids cross the placenta. B. occur because of fetal central nervous system depression from Inhalational agent. C. are not concerning if general anesthesia is maintained with total intravenous anesthesia. D. occur because of maternal hypotension or hypoxia. Answer: D Maternal hypotension that causes placental hypoperfusion or maternal hypoxia can lead to fetal hypoxemia. Fetal hypoxemia manifests as fetal heart rate deceleration under general anesthesia. Induction agents and opioids cross the placenta and cause decrease in FHR variability due to the effect on fetal CNS, but should not cause decelerations. Same is the case with total intravenous anesthesia.

4. A parturient at term becomes hypoxemic more rapidly after the onset of apnea. This is MOST likely due to a decrease in her

5. Which of the following is recommended for nonobstetric surgery during pregnancy?

A. If the fetus is considered previable (24 weeks gestation), continuous intraoperative FHR monitoring must be performed regardless of the type of procedure. C. If possible, surgery is best performed in the second trimester of pregnancy. D. Midazolam should be avoided due to teratogenic effects in humans. Answer: C Pre and post procedure assessment of FHR with contrac2. Risk of preterm labor after nonobstetric surgery is higher tion monitoring is recommended for viable gestational age. Intraoperative FHR monitoring should be considered when A. during the second trimester of pregnancy. feasible. Ultimately, each case warrants a team approach (an B. during the first week postoperatively. C. with upper extremity procedures compared to intra-​ esthesia and obstetric care providers, surgeons, pediatricians, and nurses) for optimal safety of the woman and the fetus. abdominal procedures. D. during intraoperative period when inhalational agent is Surgery when performed in the second trimester carries a lower risk for preterm labor. Recent studies show that there is used for maintenance. no association between benzodiazepines and cleft palate. Answer: B The greatest risk of preterm labor is during the first week postoperatively. Surgical procedures remote from the abdomen and those that are done during second trimester carry a lower risk. Inhalation agents cause uterine smooth muscle relaxation.

3. Anesthetic management for laparoscopic procedures during pregnancy A. does not require rapid sequence induction. B. involves a higher narcotic requirement. C. involves maintaining end-​tidal carbon dioxide between 28 and 32 mm Hg. D. usually means a prolonged hospital stay. Answer: C Benefits of laparoscopic procedure includes shorter hospital stay, less postoperative pain and use of narcotics and faster return to normal activities. Rapid sequence induction is required in pregnant patients as they are at risk for aspiration. It is very cruicial to maintain end-tidal carbon dioxide within

QUESTIONS AND ANSWER S This chapter has accompanying questions and answers which are available to subscribers as part of the Oxford eLearning platform. To access the questions, go to http://​ oxfordmedicine.com/​anesthesiologyPBL REFERENCES 1. Mazze RI, Källén B. Reproductive outcome after anesthesia and operation during pregnancy: a registry study of 5405 cases. Am J Obstet Gynecol. 1989 Nov;161(5):1178–​1185. 2. Cohen-Kerem R, Railton C, Oren D, Lishner M, Koren G. Pregnancy outcome following non-obstetric surgical intervention. Am J Surg. 2005;190(3):467–473. 3. Fatum M, Rojansky N. Laparoscopic surgery during pregnancy. Obstet Gynecol Surv. 2001;56(1):50–​59.

2 8. A nesthesia in a P regnant Patient  •   247

4. O’Rourke N, Kodali BS. Laparoscopic surgery during pregnancy. Curr Opin Anaesthesiol. 2006;19(3):254–​259. 5. ACOG Committee on Obstetric Practice. ACOG committee opinion no. 474: Nonobstetric surgery during pregnancy. Obstet Gynecol. 2011;117(2 Pt 1):420–421. 6. Guidelines Committee of the Society of American Gastrointestinal and Endoscopic Surgeons, Yumi H. Guidelines for diagnosis, treatment, and use of laparoscopy for surgical problems during pregnancy: This statement was reviewed and approved by the board of governors of the society of american gastrointestinal and endoscopic surgeons (SAGES), september 2007. it was prepared by the SAGES guidelines committee. Surg Endosc. 2008;22(4):849–861. 7. Bhavani-​Shankar K, Steinbrook RA, Brooks DC, Datta S. Arterial to end-​tidal carbon dioxide pressure difference during laparoscopic surgery in pregnancy. Anesthesiology. 2000;93(2):370–​373.

8. Clark RB, Brown MA, Lattin DL. Neostigmine, atropine, and glycopyrrolate: does neostigmine cross the placenta? Anesthesiology. 1996;84(2):450–​452. 9. Chestnut DH, Wong CA, Tsen LC, et  al. Chestnut’s Obstetric Anesthesia:  Principles and Practice. 5th ed. Philadelphia:  Elsevier/​ Saunders; 2014. 10. Corneille MG, Gallup TM, Bening T, et al. The use of laparoscopic surgery in pregnancy:  evaluation of safety and efficacy. Am J Surg. 2010;200(3):363–​367. 11. Jackson H, Granger S, Price R, et al. Diagnosis and laparoscopic treatment of surgical diseases during pregnancy: an evidence-​based review. Surg Endosc. 2008;22(9):1917–​1927. 12. Eldridge AJ, Ford R. Perimortem caesarean deliveries. Int J Obstet Anesth. 2016;27:46–54.

248  •   O bstetrics

29. PREECLAMPSIA PROBLEM-​B ASED LEARNING DISCUSSION Shamantha Reddy and Yelena Spitzer

S T E M C A S E A N D K EY Q U E S T I O N S A 32-​year-​old female G2P1 at 39 weeks gestation presents to the labor floor with headache, blurry vision, and abdominal pain lasting 1 day. Patient had a history of gestational hypertension (HTN) in her last pregnancy. This pregnancy was significant for occasional mild range blood pressures (BP) noted during clinic visits but was otherwise uncomplicated. Vitals on presentation are noninvasive BP 165/​82  mm Hg, heart rate 100 beats per minute (bpm), SpO2 100% on room air, and temperature 36.5°C. No labs are available at this time. Fetal heart tracing (FHR) at the time of presentation was 150 bpm with moderate variability. WH AT I S T H E D E F I N IT I O N O F H T N I N P R EG NA N C Y ?

Hypertension in pregnancy is defined as a systolic BP >140 mm Hg or greater, a diastolic BP >90 mm Hg, or both. Hypertension is considered mild until systolic BP >160 mm Hg or diastolic BP >110 mm Hg. To make an accurate diagnosis, 2 BP readings should be taken at least 4 hours apart. In the setting of severe HTN, however, 2 readings taken minutes apart can be used to make a diagnosis in order to expediently facilitate antihypertensive treatment.1,2 WH AT I S T H E D I F F E R E N T I A L D I AG N O S I S F O R T H I S PAT I E N T ?

The differential diagnosis for HTN in pregnancy includes:1,2 1. Gestational  HTN. 2. Preeclampsia without severe features—​previously this was called mild preeclampsia. This was changed by American College of Gynecology (ACOG) in 2014. 3. Preeclampsia with severe features. 4. Eclampsia 5. Chronic  HTN 6. Chronic HTN with superimposed preeclampsia/​ eclampsia

WH AT I S T H E I N C I D E N C E O F H Y P E RT E NS I V E D I S O R D E R S I N P R EG NA N C Y ?

According to the ACOG task force, hypertensive disorders of pregnancy including preeclampsia complicate about 10% of pregnancies worldwide. The incidence has increased by 25% in United States in the past 2 decades. World Health Organization reports that approximately one-tenth of maternal deaths in Africa and Asia, and one-fourth of maternal deaths in Latin America are due to hypertensive disorders of pregnancy and related complications.2 WH AT I S T H E D I FFE R E N C E B ET WE E N C H RO N I C HT N A N D G E S TAT I O NA L  HT N ?

The timing of onset of elevated BP differentiates chronic from gestational HTN . Chronic HTN is diagnosed prepregnancy or before 20 weeks gestation. Gestational HTN is diagnosed after 20 weeks gestation, often toward the end of pregnancy and is not associated with proteinuria. Gestational HTN often resolves postpartum but, if persistent, may be diagnosed as chronic HTN. Gestational HTN may be a sign of future chronic HTN.1,2 H OW D O YO U D E FI N E P R E EC L A M P S I A ?

Preeclampsia is a pregnancy specific hypertensive disease that can have multisystem involvement. The symptoms typically manifest after 20 weeks gestation, most often near term. Preeclampsia may be superimposed on another hypertensive disorder, such as chronic HTN. Preeclampsia was previously categorized as mild or severe. Recent ACOG guidelines have amended the classification as preeclampsia with or without severe features. Preeclampsia without severe features is defined as a BP ≥140/​90 mm Hg with proteinuria. Preeclampsia with severe features is defined as a BP ≥160/​110 mm Hg with either proteinuria and/​or signs of end-​organ dysfunction. These include thrombocytopenia (platelets ≤100,000/​µL), impaired liver function, progressive renal insufficiency (creatinine ≥1.1 mg/​ dL or a doubling of the baseline serum creatinine), pulmonary edema, or new onset headache or visual disturbance.1,2,3

249

WH AT I S EC L A M P S I A ?

Eclampsia is defined as the new onset of grand mal seizures in a woman with preeclampsia. Eclampsia can occur before, during, or after delivery. Eclamptic seizures typically have an abrupt onset and are tonic-​clonic in nature. Most seizures occur intrapartum or within the first 48 hours postpartum.1,2 WH AT I S P ROT E I N U R I A ?

Proteinuria is defined by the excretion of 300 mg or more of protein in the urine in a 24-​hour urine collection. Alternatively, a urine protein:creatinine ratio of 0.3 (mg/​dL) or 1+ protein on urine dipstick can be used. Quantitative measurements are preferred, but dipstick can be used when these are not available.1.2 WH I C H O RG A N S Y S T E M S C A N B E A FF E C T E D I N S EV E R E P R E E C L A M P S I A ? D E S C R I B E T H E C H A N G E S I N E AC H O RG A N S Y S T E M ?

Every organ system can be affected in severe preeclampsia.

Cerebral changes Headache, visual disturbances, hyperexcitability, and hyperreflexia are the classic hallmarks of preeclampsia. Cerebrovascular accidents and intracranial hemorrhage are possible. This is thought to be secondary to loss of cerebrovascular auto regulation resulting in vascular barotrauma as evidenced by measurement of cerebral blood flow and resistance and neuroimaging.1,4

resulting in end-​organ damage secondary to microthrombi formation. There is consumption of platelets and coagulation factors, which can lead to severe hemorrhage.1

Renal changes Reduction in glomerular filtration rate occurs in preeclampsia from cellular injury and disruption of the filtration apparatus. This leads to subsequent proteinuria.1,6 Rise in blood urea nitrogen, creatinine, and serum uric acid levels can be seen.1,6 Oliguria can be a late finding of worsening renal status.1 There is a characteristic glomerular lesion—​ glomerular endotheliosis—​that is responsible for the damage.6 After delivery, glomerular changes usually reverse rapidly, coinciding with resolution of the HTN and proteinuria.6

Hepatic changes Periportal hemorrhage and fibrin deposition in hepatic sinusoids is seen in preeclampsia.1 Spectrum of changes are either mild hepatocellular necrosis to the severe features leading to HELLP syndrome.1 Subcapsular bleeding can occur and in severe cases there is a risk of hepatic rupture.1 Spontaneous hepatic rupture is rare but, if happens, carries a 32% maternal mortality risk. Symptoms include abdominal pain that worsens and becomes epigastric or right upper quadrant pain, nausea, vomiting, and headaches.1

Uteroplacental perfusion

Uteroplacental perfusion can be impaired in preeclamptic pregnancies.1 In normal pregnancies, systemic vascular reCardiovascular changes sistance decreases. But in preeclampsia the resistance in uteroplacental circulation increases.1,2 Diastolic flow velocity In addition to HTN, the majority of preeclamptic patients dedecreases and peak systolic to end diastolic flow velocity ratio velop an increased cardiac output, with a hyperdynamic left increases.1 These changes if severe can cause intrauterine fetal 1,5 ventricle and increased systemic vascular resistance. End-​ growth retardation.1 organ ischemia can be seen. WH AT I S H E L L P SY N D RO M E?

Respiratory changes Airway edema is the concern in severe preeclampsia.1 There is a potential for difficult intubation, and fluid overload leading to pulmonary edema.

Hematologic changes

HELLP syndrome may present during antepartum or postpartum period, but the onset is antepartum 70% of the time. The term HELLP syndrome is an acronym for the following.1,2,7 Hemolysis: bilirubin >1.2 mg/​dL; lactate dehydrogenase >600  IU/​L

Elevated liver enzymes: serum glutamic oxaloacetic Thrombocytopenia, defined as a platelet count of 70 IU/​L 3 mm is a concern in patients with preeclampsia. Platelet aggregation appears to account for a decrease in platelet count Low platelets: platelet counts 34 weeks gestation, assuming there is no other indication for cesarean section. Cesarean section is appropriate when either maternal or fetal condition mandates immediate delivery or other indications for cesarean section exist.1–​3 T R E AT M E N T O F HT N I N P R EG NA N C Y

Antihypertensive medication is strongly recommended for severe HTN in pregnancy, with systolic BP ≥160 mm Hg and/​ or diastolic BP ≥110 mm Hg. The goal of treatment is to prevent cardiovascular complications (congestive heart failure

252  •   O bstetrics

and myocardial ischemia), renal injury, or hemorrhagic stroke from the severe HTN. There is no consensus on the treatment of nonsevere HTN.1–​3 Data from a systematic review of large trials suggest that treatment of mild to moderate HTN decreases the incidence of severe HTN by 50% but does not affect the incidence of preeclampsia development.9 There is also an association between antihypertensive treatments and fetal growth impairment. Therefore, only treatment of severe HTN is recommended.1,2 A  recent Cochrane review found no difference in either efficacy or safety when comparing hydralazine and labetalol. There was also no difference between hydralazine and calcium channel blockers.10 As per ACOG, the choice of antihypertensive agent should be based on provider experience and availability of the medication.2

renal impairment.1,2 In cases of suspected magnesium toxicity, infusion must be discontinued, and 10 cc of 10% calcium gluconate should be administered. In the event of respiratory compromise, endotracheal intubation and mechanical ventilation may be necessary. A N E S T H ET I C M A NAG E M E N T O F T H E P R E EC L A M P T I C PAT I E N T

Thorough preanesthetic evaluation is important in patients with preeclampsia. Laboratory studies obtained should include complete blood count, platelet count with aggregation if available, chemistry, blood urea nitrogen, creatinine, uric acid assays, liver function, and urinalysis with 24-​hour protein and creatinine clearance, type, and screen. A crossmatch is needed if antibodies are present. EC L A M P S I A : P R EVE N T I O N Lumbar epidural is the preferred form of analgesia for labor A N D T R E AT M E N T and delivery in preeclamptic patients provided that there is no Eclamptic seizures contribute significantly to maternal mor- existing coagulopathy. An early epidural is recommended for bidity and mortality. They are often preceded by symptoms the preeclamptic patient as the patient may develop thromof persistent frontal or occipital headache, blurred vision, bocytopenia Preeclamptic women are at increased risk of cephotophobia, and epigastric or right upper quadrant pain. sarean delivery compared with normal parturients, and early Magnesium sulfate (MgSO4) is the most effective agent for administration of epidural analgesia for labor facilitates the both prevention and treatment of eclamptic seizures. The subsequent administration of epidural anesthesia for emerexact mechanism of action of MgSO4 for seizure prophylaxis gent cesarean delivery.1,11 Furthermore, analgesia provided by is unclear. Eclampsia was thought to be secondary to cerebral an epidural is superior compared to other modes of analgesia vasospasm. MgSO4, acting as a calcium antagonist, causes ar- as it reduces circulating levels of catecholamines and stress-​ terial vasodilation. Recent data suggest that abrupt HTN in related hormones. The sympathectomy from epidural analsevere preeclampsia leads to forced dilation of cerebral vessels, gesia attenuates the exaggerated sympathetic and hypertensive hyperperfusion, and cerebral edema. MgSO4 as a vasodilator response to pain and facilitates BP control.1,11,12 Epidural anshould worsen cerebral edema. Therefore, new theories for algesia may improve intervillous blood flow in preeclamptic MgSO4 mechanism of action include a decrease in blood–​ women and improved placental perfusion.1,11,12 brain barrier permeability to relieve cerebral edema or antagSpinal anesthesia or a combined spinal-epidural anesonism of the N-​methyl-​D-​aspartate receptor to raise seizure thesia can be used for cesarean section in preeclamptic women threshold.1–​3 with no coagulopathy or thrombocytopenia. Hyperbaric MgSO4 is administered as a 4 to 6 g loading dose and 1 bupivacaine 0.75% (10.5 to 12 mg), fentanyl (10 to 15 mcg), to 2 g/​hr maintenance infusion. MgSO4 loading and infu- and preservative-​free morphine (100 to 200 mcg) may be sion require careful monitoring for signs of toxicity. Figure given as the spinal dose. According to a study by Aya et al., risk 29.3 illustrates the signs of MgSO4 toxicity and the corre- of hypotension was almost 6 times less in severe preeclamptic sponding MgSO4 level. MgSO4 is eliminated almost entirely women than in healthy partururients receiving spinal for elecvia renal excretion, and care must be taken in patients with tive cesarean section.13

Magnesium dose and toxicity Serum conc (mg/dL)

Manifestation

1.5–3

Normal concentration

4–7

Therapeutic levels

5–10

ECG changes (prolonged PR, widened QRS)

8–12

Loss of patellar reflex

9–12

Warmth: flushing

10–12

Somnolence, slurred speech

15–17

Muscle paralysis; respiratory distress

>30

Cardiac arrest

Figure 29.3 

 MgSO4 dose and signs of toxicity.

I N VA S I VE MO N ITO R I N G I N T H E P R E EC L A M P T I C PAT I E N T

Routinely, noninvasive monitoring is sufficient for preeclamptic patients. Laboring patients should have their BP monitored every 15 minutes in labor. Invasive arterial BP monitoring can be used based on the patient’s medical comorbidities and severity of preeclampsia. Patients who require frequent blood sampling to assess for electrolytes, respiratory function, acid-​ base status, and hematologic abnormalities may additionally benefit from invasive intra-​ arterial monitoring.1,3 Central venous pressure catheters and pulmonary artery catheters are rarely utilized for hemodynamic monitoring of preeclamptic patients. Central monitoring may prove useful in the setting of persistent oliguria, difficulty in fluid

29. P reeclampsia P roblem-B ased L earning D iscussion  •   253

management or the presence of pulmonary edema.1,3 There are no randomized control trials that support the utility of placing pulmonary artery catheters in preeclamptic patients. Complications of obtaining central access in pregnant patients include line infections, superficial and deep vein thrombosis, hematoma, and ventricular tachycardia.3 M A NAG E M E N T O F H E L L P SY N D RO M E

HELLP syndrome is a variant of severe preeclampsia and is characterized by rapid progressive deterioration of maternal and fetal clinical status. According to ACOG, immediate delivery is recommended after 34 weeks gestation if HELLP syndrome is diagnosed. If the fetus is less than 34 weeks gestation, ACOG recommends delaying delivery for 24 to 48 hours to complete a course of antenatal steroids if maternal and fetal status are otherwise stable.1–​3 Platelet count can fall precipitously in HELLP syndrome and should be checked prior to neuraxial anesthesia placement. Early epidural placement is recommended if there is concern for impending thrombocytopenia. The trend inplatelet count is more important than a single platelet value. There is no exact cut-​off for platelet count for neuraxial anesthesia. Most anesthesiologists feel comfortable performing neuraxial anesthesia in patients with stable thrombocytopenia with platelet counts of 80,000/​m3 to 100,000/​m3 but these data have largely been obtained from patients with gestational thrombocytopenia.14 There is no good evidence for the lowest acceptable platelet count in patients with HELLP syndrome. GENERAL ANESTHESIA IN THE P R E E C L A M P T I C PAT I E N T

In preeclamptic women, general anesthesia should only be employed in emergent situations or when contraindications exist for neuraxial anesthesia . In the setting of preeclampsia, there is an exaggerated hypertensive reaction to laryngoscopy and endotracheal intubation, as well as potential for difficult intubation secondary to airway edema. Aspiration prophylaxis with nonparticulate antacid sodium citrate (30 mL) and/or intravenous H2 receptor blockers are reasonable prior to induction of general anesthesia in order to decrease the acidity of the gastric contents.1 After preoxygenation and denitrogenation with 100% O2 via facemask, a rapid sequence induction with cricoid pressure using propofol 2 to 2.8 mg/kg and succinylcholine 1 to 1.5 mg/kg is reasonable. Esmolol, labetalol, nitroglycerin, and remifentanil can be titrated to attenuate the hypertensive response to laryngoscopy and endotracheal intubation. A preinduction arterial line may be placed in preeclamptic patients with severe features, if necessary. Availability of difficult airway equipment is important during induction and intubation of these patients.1 If minimum alveolar concentration of volatile agents is kept less than 1, there is no subsequent increase in blood loss or decrease in uterine contractility.1,15 If the patient is receiving MgS04, there is a potential for uterine atony, especially when combined with general anesthesia.1

C A S E D I S CUS S I O N

The patient is presenting with signs of preeclampsia with severe features. This is based on the patient’s presenting BP and symptoms of headache, blurry vision, and abdominal pain. The FHR tracing is reassuring at this time. A discussion with the obstetrician about the management and delivery planning of this patient is recommended. If the plan is for an induction of labor, an early epidural can be offered to the patient after anesthetic evaluation and laboratory data are available. If the scenario was different and the patient presented with a nonreassuring FHR tracing and indication for immediate delivery, neuraxial anesthesia could be attempted if laboratory data are available. If labs are not available, the fetus needs to be delivered emergently, and cesarean section under general anesthesia may be indicated if coagulopathy is suspected. R E VI EW Q U E S T I O N S 1. What are the risk factors for developing preeclampsia? A. Preeclampsia in prior pregnancy B. Advanced maternal age C. Obesity D. Chronic  HTN E. All of the above Answer: E All of these are risk factors for preeclampsia. See Figure 29.1 for more details. 2. Are women with prior preeclampsia at higher risk for cardiovascular disease in later life? A. Yes B. No Answer: A Yes, patients with preeclampsia are at risk for HTN and cardiac disease later in life. 3. Which of the following statements is true about preeclampsia and related diseases? A. Current guidelines recommend delivery at 37 weeks gestation for preeclampsia without severe features. B. Spinal anesthesia or a combined spinal epidural anesthesia can be used for cesarean section in preeclamptic women with no coagulopathy or thrombocytopenia. C. Risk of hypotension was almost 6 times less in severe preeclamptic women than in healthy pregnant receiving spinal for elective cesarean section. D. All of the above Answer: D All these statements are true. 4. Which of the following statements about magnesium sulfate use in preeclampsia is FALSE? A. MgSO4 is eliminated almost entirely via renal excretion, and care must be taken in patients with renal impairment.

254  •   O bstetrics

B. Therapeutic level of magnesium for preeclampsia is recommended to be 4 to 7 mg/​dL. C. At 30 mg/​dL levels, magnesium causes cardiac arrest. D. None of the above Answer: D All these are true. The therapeutic level of MgSO4 is 4 to 7 mg/​ dL. MgSO4 is excreted renally and dosage should be decreased in patients with renal insufficiency. Toxicity can occur from MgSO4. See Figure 29.3 for a full list of effects from magnesium toxicity. 5. In a patient with HELLP syndrome, which of the following is true? A. Early epidural is not recommended. B. Platelet count only needs to be checked prior to epidural placement, not epidural removal. C. Preeclampsia is classified as not having severe features if systolic BP 180 ms, symptomatic nonsustained ventricular tachycardia, and LV dysfunction.1

UNCOMMON

Anterior displacement of infundibular septum causing malalignment VSD

Muscular VSD

Overriding aorta

Doubly committed subarterial VSD

RVOT obstruction with or without PV stenosis

Pulmonary atresia

Right ventricular hypertrophy

Anomalous coronaries or aorto-​ pulmonary collaterals

RVOT = right ventricular outflow tract; PV = pulmonic valve; VSD = ventricular septal defect.

P R EO P E R AT I VE C O NS I D E R AT I O NS

There is no routine protocol for anesthetic management of patients with repaired ToF for pregnancy and delivery. Each patient’s plan must be formulated individually. A  full preoperative evaluation should be done starting with thorough review of medical records. Special attention must be paid to type and timing of surgical correction, and all cardiology follow-​ up postsurgery up to and including period of pregnancy. Electrocardiogram should be obtained, which usually shows a right bundle branch block pattern. Most recent transthoracic echocardiography and cardiac magnetic resonance imaging results should be reviewed. Factors that increases cardiac risk

3 0. P regnancy M anagement in a Parturient with  TO F   •   259

(a)

(b)

Pulmonary atresia

RV

Ao

Left Blalock-Taussig shunt

(c)

Potts shunt

PA

LV VSD

(d)

(e)

Modified Blalock-Taussig shunt

(f )

Central shunt

Waterston shunt

(g) Valved conduit

PA

RV

VSD patch

(A) Unrepaired tetralogy of Fallot with pulmonary atresia. (B) Classic left Blalock-​Taussig shunt consisting of direct connection of the divided left subclavian artery to the left pulmonary artery. (C) Potts shunt consisting of a direct connection between the anterior wall of the descending Ao and left PA. D. Waterston shunt consisting of a direct connection of the posterior wall of the ascending Ao and the right PA. (E) Modified Blalock-​Taussig shunt consisting of a synthetic (Gore-​Tex) tube connection the left subclavian artery to the PA. (F) Central shunt consisting of a synthetic tubular connection from the ascending Ao to the PA. (G) Intracardiac repair consisting of VSD patch closure and placement of a valved conduit from the RV to the PA. Reproduced from Lin, Aboulhosn, Child. Congenital heart disease in adolescents and adults. In: Fuster, Harrington, Narula, Eapen, eds. Hurst’s the Heart. 14th ed. New York: McGraw-​Hill Education; 2017. Figure 30.2 

should be sought early, such as aforementioned severe pulmonic regurgitation and RV dilatation, RV or LV dysfunction, QRS prolongation, and history of tachyarrhythmias. Transthoracic echocardiography can be an excellent tool in assessing RV dilatation and dysfunction and can be used to serially follow patient’s progression through pregnancy (Figs. 30.3 and 30.4). If the patient had neglected interval follow-​up, history taking should focus on degree of congestive heart failure symptoms, current functional status, and episodes of syncope. Physical examination needs to elucidate evidence of heart failure such as elevated JVP, peripheral and pulmonary edema, and signs of cyanosis and clubbing. An early diastolic murmur may signify pulmonary regurgitation, holosystolic murmur a residual VSD, and loud P2 could denote presence of pulmonary hypertension. Lastly, presence of a weakened unilateral radial pulse and a decreased blood pressure in that arm could signify a history of Blalock-​Taussig shunt. A N E S T H ET I C M A NAG E M E N T

In a parturient with repaired ToF, vaginal delivery is typically preferred unless standard cesarean indications are present. There are no randomized controlled trials comparing efficacy and safety of neuraxial versus GA in this population. The largest case series to date examined 20 patients over a 14-​year

period and found all patients had successfully undergone delivery under neuraxial anesthesia.7 The majority of patients had received standard epidural infusions and 2 underwent CSE. Single-​shot spinal was performed in 2 elective cesarean Table 30.2  LONG-​T ERM COMPLICATIONS AFTER REPAIR OF TETRALOGY OF FALLOT AND ASSOCIATED PHYSICAL EXAMINATION FINDINGS COMPLICATION

PHYSICAL EXAM FINDING

Pulmonic regurgitation

Holodiastolic murmur at LUSB

RV overload and dilatation

Signs of RV failure such as elevated JVP, peripheral edema, RV heave

Congestive heart failure

Same as above for RV LV failure may see pulmonary edema, malperfusion to other organs

Arrhythmias and sudden death

Tachycardia, irregular pulse, syncope

Residual right to left shunt

Late or progressive systolic murmur

Pulmonary hypertension

Loud P2

RV = right ventricle; LV = left ventricle; LUSB =: left upper sternal border; JVP = jugular venous pressure.

260  •   O bstetrics

(a)

(b)

(c)

(d)

Echocardiographic evaluation of right ventricular (RV) size; images from a patient with operated tetralogy of Fallot (TOF) and dilated RV. (A) Parasternal long-​axis view; the proximal part of the right ventricular outflow tract (RVOT) can be measured (RVOT 1). (B) Short axis at the level of the great vessels—​both the RVOT 1 as well as the distal part of the RVOT (RVOT 2) can be measured. (C, D) Apical 4-​chamber view with focus on the RV—​the RVD (right ventricle dimension) 1 and 2 represent the basal and midcavitary linear dimension, respectively. Reproduced from Daraban, Jurcut, Badila et al. Following up adult patients with tetralogy of Fallot: the role of echocardiography. J Clin Ultrasound. 2017;45:79–​95. Figure 30.3 

Measurement of tricuspid annular plane systolic excursion (TAPSE) in 2 patients with repaired tetralogy of Fallot; on the left panel, the right ventricular (RV) function is preserved, TAPSE 18 mm; on the right panel, RV dysfunction, with TAPSE 13 mm. Reproduced from Daraban, Jurcut, Badila et al. Following up adult patients with tetralogy of Fallot: the role of echocardiography. J Clin Ultrasound. 2017;45:79–​95. Figure 30.4 

deliveries. Given the potential for profound sympathectomy and vasodilation, spinal is usually not the preferred method of neuraxial anesthesia in patients with CHD, although neither patient here developed refractory hypotension. CSE may offer an attractive solution with dense blockade and higher quality analgesia compared to epidurals, and if chosen then a small dose of intrathecal bupivacaine can be administered followed by careful titration of the epidural until sufficient level is achieved. When performing the epidural, some technical changes may warrant consideration. In all ToF patients but especially those with unrepaired lesions or residual shunts, intravenous solutions must be meticulously deaired and loss of resistance to saline rather than air should be performed. Test dose solutions containing epinephrine can be used, but the anesthesiologist must monitor for signs of intravascular injection, which can lead to arrhythmias and rapid deterioration in this population. Finally, even with a slowly titrated epidural, sympathectomy and drop in SVR can still occur, potentially worsening any R–​L shunt. The resulting increase in contractility can also precipitate RVOT obstruction, increase transpulmonic gradient, and worsen regurgitant fraction. Therefore, SVR and preload should be carefully maintained with volume co-​loading and alpha agonists. GA can be performed if necessary for C-​section, even in cases of unrepaired ToF.5 All standard risks of GA in the obstetric patients are applicable here, including increased incidence of difficult airway, greater aspiration risk, and so on. In addition, GA necessitates positive pressure ventilation, which can decrease preload, worsen pulmonary hypertension, and precipitate RV failure. Volatile induced drop in SVR necessitates the same management considerations as mentioned before, while nitrous oxide can potentially exacerbate pulmonary vasoconstriction. Currently, there are no studies that demonstrate improved outcomes with neuraxial technique. Nevertheless, given higher rates of complications associated with GA in the general obstetric population, as well as demonstrated safety of neuraxial anesthesia techniques, we feel that neuraxial anesthesia should be the first line option for parturients with ToF. Lastly, some clinicians have argued for a passive second stage of labor to avoid repeated Valsalva maneuvers and increased strain on the cardiovascular system. Any active maternal expulsive efforts are abolished, and delivery is accomplished through either forceps or vacuum assistance. The best course of management is unknown, although 1 retrospective study did show prolonged second stage of labor and more postpartum bleeding from assisted delivery.12 MO N I TO R I N G

All parturients with ToF should be monitored on continuous telemetry during labor and/​or C-​section. Arrhythmias can be common in both repaired and unrepaired population, and especially in those patients with history of tachyarrhythmias, recurrence during pregnancy can approach 50%. If patient is completely asymptomatic and devoid of any signs of cardiac dysfunction, then routine noninvasive monitoring should be adequate for labor. Arterial blood pressure monitoring

may be pursued if the patient exhibits cardiac risk factors as previously outlined. Again, one must be wary of history of Blalock-​Taussig shunt and monitor the correct arm for blood pressure. Central venous pressure monitoring is rarely helpful, given the altered dynamics and function of RV and presence of prosthetic valve/​conduits. If PA catheter is considered, one must anticipate potentially challenging placement given altered anatomy, and presence of prosthetic valve is a contraindication to its use. Newer noninvasive cardiac output devices have not been studied in this population and their accuracy cannot be ascertained currently. Lastly, if GA is conducted for C-​section, transesophageal echocardiography may be a useful tool for cardiac function monitoring intraoperatively. E N D O C A R D IT I S P RO P H Y L AX I S

Current guidelines for endocarditis prophylaxis is equivocal in regards to repaired ToF patients undergoing labor and delivery. If the patient has unrepaired lesion, palliated lesion but not definitive surgery, or prosthetic valves, the 2007 American Heart Association (AHA) guidelines recommends routine prophylaxis but only for dental procedures or those that break the skin barrier.13 The 2017 AHA scientific statement update on management of pregnancy in patients with complex CHD states that while not officially endorsed, it is not unreasonable to give antibiotic prophylaxis in patients with high-​risk CHD lesions.14 Lastly, the American Congress of Obstetrics and Gynecology (ACOG) does not recommend routine antibiotic prophylaxis for vaginal and cesarean deliveries.15 So, currently, it is up to the individual institutions as to whether routine antibiotic prophylaxis for infective endocarditis should be given in an unrepaired lesion or a repaired lesion with prosthetic graft or valve. If the repair is completed without any prosthetic material, then routine antibiotic prophylaxis is not indicated. C O N C LUS I O NS

• As patients with ToF often attain excellent long-​ term survival after definitive surgery, more will reach childbearing age and present to obstetric anesthesiologists for labor and delivery. • It is imperative to understand the pathophysiology and anatomic changes associated with this condition as well as long-​term complications that may arise. • The anesthesiology must in consult with obstetrician and cardiology obtain a thorough workup of the patient to look for risk factors that may precipitate decompensation during or after pregnancy. • In general, pregnancy and labor is tolerated well in asymptomatic patients with repaired ToF, and neuraxial anesthesia with epidural is the preferred method of management. • Monitoring for both vaginal and/​or cesarean delivery is tailored to individual and the severity of disease process,

262  •   O bstetrics

but for otherwise healthy parturient with uncomplicated pregnancy, noninvasive monitoring is usually adequate.

repaired ToF can have residual shunt of varying degrees. So even for repaired ToF patients, all intravenous lines should be meticulously deaired. When placing an epidural, there is a small risk of air embolizing into the epidural venous plexus R E VI EW Q U E S T I O N S and then traveling systemically. Therefore, it may be prudent to use saline as the loss of resistance technique rather than air. 1. If surgically corrected during infancy, what percentage of Epinephrine containing test dose solution is commonly used patients born with ToF can be expected to survive into early to test intravascular epidural catheter placement. While epiadulthood and therefore can potentially become pregnant? nephrine injected intravascularly can cause tachyarrhythmias, it is still not contraindicated in ToF patients. In fact, almost all A. 20% patients in the largest case series to date received epinephrine B. 50% as part of the test dose during their epidural placement.7 It is C. 70% prudent to place parturients with ToF on continuous telem D. 90% etry monitoring during epidural placement as arrhythmias Answer: D can occur. No study has demonstrated superiority/​efficacy ToF is the most common cyanotic congenital heart defect in between epidural or CSE for patients with ToF in labor. infants, and has an incidence of 1/​3,600 live birth or up to By expert opinion slow dosing epidural may avoid sudden 10% of those born with CHD.1 While in the era prior to sur- decreases in SVR and therefore better tolerated in ToF pagical repair the mortality was very high, the first palliative pro- tient population. Nevertheless, CSE has been performed cedure was invented by Drs. Blalock and Taussig in the 1950s, successfully for both labor analgesia and cesarean delivery which then allowed infant survival to adolescence for defini- as well.7 tive repairs. Over the past few decades, surgical technique has improved to the point that now the preferred choice is single-​ 4. Which of the following is a potential long-​term complicastage definitive repair in early childhood. Currently 30-​year tion after ToF surgery and must be identified and monitored survival in patients who had definitive surgical repair as a child during pregnancy? can be expected to be >95%.4 A. Pulmonic regurgitation B. Right ventricle dilatation 2. Offspring of parturients with ToF is not at increased risk C. Atrial tachyarrhythmias for developmental abnormalities compared to the general D. Congestive heart failure population. E. All of the above A. True Answer: E B. False After ToF repair surgery, long term complications can develop. Answer: B Pulmonic regurgitation is 1 of the most frequent, as many of Patients with ToF who become pregnant will have increased the initial surgery require annular patch and/​or pulmonic risk for their offsprings to develop congenital abnormalities. valve replacement to relieve right ventricular outflow obstrucIn a study published in Lancet, Burn et  al.6 examined 395 tion and/​or pulmonic stenosis. By the time the patient reaches patients with ToF who had surgical repair in the 1970s and early adulthood, he or she has frequently developed pulmonic subsequently gave birth to live infants. The rate of recurrence regurgitation.1 Pulmonic regurgitation leads to a RV volume for ToF in the infants was 3 times higher than in the general overload, which over time sees RV dilatation as a compensapopulation. In another retrospective study, Veldtman et al.10 tory mechanism. Further right-​sided volume overload will found that 6% of the 147 pregnancies examined had devel- cause right atrial stretch/​dilatation, leading to increased inoped congenital anomalies including other CHD, clubbed cidence of arrhythmias. One must routinely monitors the feet, cleft lip/​palate, and strabismus. presence/​degree of pulmonic regurgitation and RV dilata3. Which of the following are potential concerns during epi- tion, and patients may require pulmonic valve replacement in dural placement for parturients with ToF that may necessitate early adulthood prior to the onset of congenital heart failure. Multiple studies have found that while corrected ToF patients changes in technique? typically tolerate pregnancy well, adverse outcomes signifi A. Use saline loss of resistance technique instead of air cantly increase if certain risk factors are present such as severe may be prudent. pulmonic regurgitation and heart failure.8,11 B. Epinephrine is contraindicated in test solution. 5. Which of the following is NOT typically seen in ToF as a C. CSE is preferred over epidural. congenital heart defect? D. Both A and B E. All of the above A. Malalignment  VSD Answer: A B. Anomalous origin of the coronaries In all CHD patients, there is always a risk of air traversing C. Hypoplastic left ventricle through the heart defect causing systemic circulation embo D. Both B and C lism. Unrepaired ToF has an open right to left shunt, and a E. All of the above

3 0. P regnancy M anagement in a Parturient with  TO F   •   263

Answer: D ToF typically consist of 4 anatomical findings. It starts with anterior displacement of the infundibular septum, which causes an overriding aorta and malalignment VSD. There are varying degrees of RVOT obstruction and a compensatory hypertrophy of the RV as a result. There may or may not be pulmonic stenosis as well (see Table 30.1). Anomalous origin of the coronaries can be seen in some variants but are very uncommon. Hypoplastic left ventricle is not seen. While the RV hypertrophies due to RVOT obstruction, left ventricle is of normal size and function at birth. 6. Which of the following physiologic changes during pregnancy would more significantly impact a parturient with uncorrected ToF compared to a parturient with corrected ToF? A. Increased circulatory volume B. Decreased  SVR C. Increased heart rate D. Increased red cell mass Answer: B In a patient with uncorrected ToF, a significant right to left shunting is present through the malalignment VSD. Pregnancy cardiovascular changes include decreased SVR due to effects of circulating progesterone and the lower SVR will promote increased shunt flow resulting in hypoxemia, cyanosis, and dyspnea. Secondary polycythemias can occur and significantly increase the risk of fetal demise and miscarriage.5 In repaired ToF, the VSD is closed at time of surgery and shunt flow eliminated, so a drop in SVR will not lead to decreased pulmonary flow and cyanosis. Increased circulatory volume, red cell mass, and heart rate all occur as a natural part of pregnancy, but none of these should impact uncorrected ToF parturients more negatively than corrected ones. In fact, given RV dilatation and failure can be a long-​term consequence of ToF surgery secondary to pulmonic regurgitation, increased circulatory volume may precipitate heart failure, arrhythmias, and sudden cardiac death even in repaired ToF patients.1 7. Which of the following statements regarding labor/​delivery in a patient with repaired ToF is true? A. Endocarditis prophylaxis is indicated according to the 2007 American College of Cardiologists (ACC)/​ AHA guidelines. B. Pulmonary artery catheters are generally not indicated given lack of benefit. C. Telemetry monitoring during labor is not helpful given most incidence of arrhythmia occurs postpartum. D. Given potential for cardiac decompensation, all patients should be scheduled for elective C-​section delivery. Answer: B ACC/​AHA guidelines suggest routine antibiotic prophylaxis for uncorrected cyanotic CHD such as ToF but only for dental procedures.13 It does not make any recommendations

for vaginal deliveries. The most current ACOG guidelines also does not recommend routine antibiotic prophylaxis.15 Currently, it is up to the individual institutions whether they want to pursue antibiotics prophylaxis for an unrepaired ToF lesion or a repaired one with prosthetic graft or valve. If the patient had repaired ToF without any prosthetic material then antibiotics is not indicated. Telemetry monitoring should be used during labor/​delivery in this patient population as arrhythmias are common in both repaired and unrepaired population and recurrence is high during pregnancy. While no large prospective study has been done, small case series have shown that patients with corrected ToF tolerate pregnancy and labor fairly well with small incidence of adverse events.7–​10 Therefore, labor should be attempted in these patients without other obstetric indications for cesarean delivery. PA catheter is generally not used in this population. There is no demonstrated benefit, and in fact in patients with prior pulmonic valve replacement, PA catheter placement is contraindicated. QUESTIONS AND ANSWER S This chapter has accompanying questions and answers which are available to subscribers as part of the Oxford eLearning platform. To access the questions, go to http://​ oxfordmedicine.com/​anesthesiologyPBL REFERENCES 1. Downing TE, Kim YY. Tetralogy of Fallot general principles of management. Cardio Clin. 2015;33:531–​541. 2. Shinebourne EA, Babu-​ Narayan SV, Carvalho JS. Tetralogy of Fallot: from fetus to adult. Heart. 2006;92:1353–​1359. 3. Gibbs JL, Monro JL, Cunningham D, et al. Survival after surgery or therapeutic catheterization for congenital heart disease in children in the United Kingdom: analysis of the central cardiac audit database for 2000–​1. BMJ. 2004;328:611. 4. Murphy JG, Gersh BJ, Mair DD, et al. Long-​term outcome in patients undergoing surgical repair of tetralogy of Fallot. N Engl J Med. 1993;329:593–​599. 5. Juwarkar C, Bharne S. Anesthetic management of a parturient with uncorrected tetralogy of Fallot for Cesarean section. Anesth Essays Res. 2012;6(2):244–​246. 6. Burn J, Brennan P, Little J, et al. Recurrence risks in offspring of adults with major heart defects: results from first cohort of British collaborative study. Lancet. 1998;351:311–​316. 7. Arendt K, Fernandes S, Khairy P et al. A case series of the anesthetic management of parturients with surgically repaired tetralogy of Fallot. Anesth Analg. 2011;113:307–​317. 8. Pedersen LM, Pedersen TAL, Ravn HB, Hjortdal VE. Outcomes of pregnancy in women with tetralogy of Fallot. Cardiol Young. 2008;18:423–​429. 9. Gelson E, Gatzoulis M, Steer PJ, Lupton M, Johnson M. Tetralogy of Fallot: maternal and neonatal outcomes. BJOG. 2008;115:398–​402. 10. Veldtman GR, Connoly HM, Grogan M, Ammash NM, Warnes CA. Outcomes of pregnancy in women with tetralogy of Fallot. J Am Coll Cardiol. 2004;44:174–​180.

264  •   O bstetrics

11. Khairy P, Ouyang DW, Fernandes SM, Lee-​Parritz A, Economy KE, Landzberg MJ. Pregnancy outcomes in women with congenital heart disease. Circulation. 2006;113:517–​24. 12. Ouyang DW, Khairy P, Fernandes SM, Landzberg MJ, Economy KE. Obstetric outcomes in pregnant women with congenital heart disease. Int J Cardiol. 2010;144:195–​199. 13. Wilson W, Taubert KA, Gewitz M, et  al. Prevention of infective endocarditis:  guidelines from the American Heart Association:  a guideline from the American Heart Association Rheumatic Fever, Endocarditis, and Kawasaki Disease Committee, Council on

Cardiovascular Disease in the Young, and the Council on Clinical Cardiology, Council on Cardiovascular Surgery and Anesthesia, and the Quality of Care and Outcomes Research Interdisciplinary Working Group. Circulation. 2007;116:1736–​1754. 14. Canobbio M, Warnes C, Aboulhosn J, et  al. Management of pregnancy in patients with complex congenital heart disease: a scientific statement for healthcare professionals from the American Heart Association. Circulation. 2017;135:e50–​e87. 15. ACOG practice bulletin:  prophylactic antibiotics in labor and delivery. Int J Gynecol Obstet. 2004;84:300–​307.

3 0. P regnancy M anagement in a Parturient with  TO F   •   265

SECTION XIV  RENAL

31. INTRAOPERATIVE HYPERKALEMIA M. Angele Theard and Alexandra Bastien

S T E M C A S E A N D K EY Q U E S T I O N S A 75-​year-​old obese male (body mass index 35) with a history of hypertension and coronary artery disease (CAD); myocardial infarction (MI), congestive heart failure (CHF), and percutaneous transluminal coronary angioplasty 5 years prior; diabetes mellitus (DM); and chronic kidney disease (CKD) presents to the emergency department (ED) after a motor vehicle accident with open bilateral femur fractures and a stable pelvic fracture. His medications include lisinopril, hydrochlorothiazide, glipizide, and aspirin. His past surgical history includes a thoracolumbar fusion and endoscopic sinus surgery. He exercises at a nearby gym and walks about 3 miles per week without any problems. After a long extraction time (3–​4 hours), this patient, the driver with a Glasgow Coma Scale of 15, is brought to the ED and a decision is made to take him to the operating room for wash out of his right lower extremity together, external fixation of his femur fracture, and fasciotomy of his left lower extremity. This patient is dyspneic (R-​40s) and complains of pain. His blood pressure is 100/​65 and he has a heart rate of 130 bpm. His hematocrit is 30 and his electrocardiogram (EKG) is significant for normal sinus rhythm (NSR) with Q waves in the inferior leads consistent with his old MI. An echocardiogram 5 years ago was significant for a 40% ejection fraction and diastolic dysfunction (Grade I). His electrolytes are within normal limits except for a creatinine of 2.5 mg/​dl, and he has a glucose of 220. A type and cross has been ordered.

WO U L D A N A RT E R I A L B L O O D G A S B E H E L P F U L I N M A NAG I N G T H I S PAT I E N T ?

Yes. This patient’s renal insufficiency consequent to his associated comorbidities (hypertension and DM) together with his crush injury (multiple orthopedic injuries, compartment syndrome, and long extraction time) places him at risk for exacerbation of his kidney disease. Hyperglycemia, chronic treatment with HCTZ and presumed blood loss from trauma places him at risk for hypotension from volume contraction exacerbating end-​organ perfusion causing metabolic acidosis and an increased risk of hyperkalemia. Chronic angiotensin converting enzyme inhibitor (ACEI) therapy will exacerbate hypotension in this patient, and this together with his CKD places him at risk for hypoaldosterone-​induced hyperkalemia, which is especially troublesome in the setting of his traumatic injuries. Crush injury and/​or compartment syndrome leads to rhabdomyolysis and consequent high levels of potassium. A blood gas with electrolytes would be very helpful in determining this patient’s acid-​base balance and his glucose and potassium levels. Preoperative resuscitation with either colloids or crystalloids (normal saline is preferred over lactated Ringer’s, which contains potassium [K+]) to ensure high urine output will be key to helping correct metabolic acidosis and prevent continuing renal damage from myoglobin. Sodium bicarbonate (50–​100 mEq/​L of 0.9 normal saline at 150 cc/​ hr) may also be helpful to help treat hyperkalemia and acidosis as well as alkalinize urine to prevent renal tubule obstruction. WH AT I N T R AO P E R AT I VE MO N ITO R I N G WO U L D YO U P L A N F O R T H I S PAT I E N T ?

A R E T H E R E A N Y A D D I T I O NA L S T U D I E S YO U WO U L D L I K E B E F O R E P RO C E E D I N G TO T H E O P E R AT I N G RO O M WI T H T H I S PAT I E N T ?

This patient presents for urgent noncardiac surgery with a cardiac history:  a revascularization procedure within the last 5 years after an MI. His history of MI, CHF, DM, and CKD places him in the intermediate risk category for surgery. According to the American College of cardiology/​ American Heart Association practice guidelines, given his good functional status (>4 Mets) together with his having an intermediate surgical risk procedure (orthopedic surgery), it is probably safe to proceed with surgery without additional cardiac evaluation. A  chest x-​ray would be helpful in determining the need for a right-​sided chest tube in a patient who will require positive-​pressure ventilation as part of a general anesthetic for this surgical procedure.

In addition to routine American Society of Anesthesiologists monitors, a urinary catheter if not already inserted will be indispensable for assessing organ perfusion and monitoring the response to treatment in this patient. Direct arterial pressure monitoring will be very helpful in the management of this patient not only for continuous blood pressure monitoring but for blood sampling to ensure that K+ levels as well as the patient’s acid-​base balance is being well-​managed. While central venous access provides limited information regarding volume status in this patient, with presumed diastolic dysfunction, it will be useful in the event that this patient requires vasoactive and/or inotropic support from drugs like norepinephrine which is much safer to give centrally than peripherally. Noninvasive cardiac output monitoring with information

269

regarding stroke volume variation and pulse pressure variation may also be helpful in guiding this patient’s response to fluid administration. T H I S PAT I E N T ’S AC C I D E N T O C C U R R E D A P P ROX I M AT E LY 1 H O U R A F T E R LU N C H A N D T H E S U RG E O N WO U L D L I K E TO P RO C E E D WI T H S U RG E RY I M M E D I AT E LY I N L I G H T O F T H I S PAT I E N T ’S C O M PA RT M E N T S Y N D RO M E A N D O P E N F R AC T U R E S. IS SUCCINYLCHOLINE A GOOD C H O I C E F O R R A P I D LY S EC U R I N G T H I S PAT I E N T ’S A I RWAY ?

Succinylcholine is recommended for swift control of the airway, which is helpful in limiting the risk of aspiration. Less than 60 seconds after administration of 1 mg/​kg of succinylcholine, the patient will be ready for intubation. This depolarizing muscle relaxant increases potassium levels by 0.5 to 1.0  mmol/​L from depolarization of acetylcholine receptors which, while negligible in someone with a normal baseline serum K+, poses increased risk of untoward clinical sequelae in this patient at risk for dangerous elevations in K+ from his crush injury. Assuming this patient does not have a difficult to manage airway (Mallampati Class  I  or II, thyromental distance of at least 3 fingerbreadths, and normal mouth opening) a nondepolarizing neuromuscular blocking drug like rocuronium at a higher dose (1.2 mg/​kg) approximates succinylcholine in time to readiness for intubation and will be a safer option for securing the airway. A nerve stimulator will of course be essential for monitoring this patient’s degree of neuromuscular block and readiness for extubation at the end of surgery. In the case of a patient with a difficult appearing airway, consideration should be given to an awake intubation. A P P R OX I M AT E LY 1.5 H O U R S A F T E R T H E S TA RT O F   S U RG E RY WI T H T H E FA S C I OTO M Y P R O C E D U R E A L M O S T C O M P L ET E , T H I S PAT I E N T D EV E L O P S P E A K E D T WAV E S F O L L OWE D BY A L O S S O F P WAV E S , A P R O L O N G E D Q R S , A N D S T-​S E G M E N T E L EVAT I O N WI T H A F EW E C TO P I C B E AT S , A N D H I S B L O O D P R E S S U R E H A S D R O P P E D TO  9 0/​5 0. WH Y T H E S E E KG CHANGES?

While these EKG changes may be suggestive of myocardial ischemia in this patient with CAD, intraoperative hyperkalemia due to crush syndrome after crush injury should be strongly considered. Relief of ischemia in the leg from the high compartment pressures leads to the release of toxic substances, worsening metabolic acidosis and hyperkalemia. Anticipation of these metabolic derangements and prompt treatment is essential to help prevent severe and irreversible myocardial decompensation. While sending blood for analysis of the patient’s metabolic state, administering intravenous fuids and vasoactive agents are critical.

T H E PAT I E N T ’S K + I S 6.5 M EQ/​L . H OW WO U L D YO U M A NAG E T H I S P OTA S S IUM E L EVAT I O N ?

This patient’s intraoperative EKG changes are most likely due to his high potassium. As long as the patient is not hypercalcemic, 1 gm of calcium gluconate or calcium chloride intravenously while monitoring the EKG will help restore the normal gradient across the cell membrane, allowing cardiac myocytes to function normally. Insulin administration (10 units intravenously) will help drive potassium into cells, decreasing serum K+. Dextrose is added to protect the patient from hypoglycemia. The addition of a ß2 agonist like albuterol via nebulizer works synergistically with insulin to shift K+ intracellularly. This management should accompany copious volume replacement to maintain urine output >200 mL/​hr. The addition of sodium bicarbonate to intravenous fluids may help limit damage to kidneys. While exchange resins like sodium polystyrene sulfonate (Kayexlate) eliminates excess potassium from the body, they may be difficult to administer intraoperatively and these drugs take time to reduce K+ levels. Hemodialysis (HD) also takes time but can be conducted during surgery if necessary. DISCUSSION P OTA S S IUM P H YS I O L O GY

Potassium, our most abundant cation, is necessary for normal cardiac and neuromuscular function.1 Potassium is a major contributor to the gradient across the cell membrane necessary for maintaining cellular excitability. Mitochondrial derived Sodium/potassium adenosine triphosphatase (Na/K ATPase) functions to maintain a high intracellular K+ concentration inside the cells and high sodium cation (Na+) outside the cell (intracellular K+ is 30-​fold greater than extracellular K+), thereby preserving the gradient which facilitates the movement of these ions across cell membranes.1 Potassium’s movement along its concentration gradient will contribute to a negative resting membrane potential of about 90 mV. Changes from this resting membrane potential determine the excitability of cardiac and neuromuscular cells.2 P OTA S S I U M R EGU L AT I O N

Tight regulation of K+ is essential for helping to maintain the excitability of cells while avoiding significant and potentially dangerous alterations in this electrolyte.3 Oral intake of K+ in foods will stimulate insulin release from the pancreas so that potassium is transported into cells. Excretion of K+ then occurs via the distal nephron of the kidney.1,4–5 The distal nephron (connecting tubules and collecting duct) responds to the rate of urine flow (low urine flow stimulates release of renin), the amount of solute (Na and Cl) delivered, and aldosterone (stimulated by K+ and angiotensin II [Ag II]).4–5 Renin released from the cells of the juxtaglomerular apparatus of the kidney will act on angiotensinogen from the liver to produce angiotensin I (Ag I) and angiotensin converting enzyme from the lung will convert Ag I to Ag II. Ag II, a vasoconstrictor will

270  •   R enal

The renin–​angiotensin–​aldosterone system and regulation of potassium excretion in the kidney. Reproduced with permission from Palmer BF. Managing hyperkalemia caused by inhibitors of the renin-​angiotensin-​aldosterone system. N Engl J Med. 2004;351:588. Figure 31.1 

help to increase perfusion and stimulate the adrenal glands to release aldosterone which will increase NaCl reabsorption together with water while increasing K+ secretion from the kidney (Fig. 31.1).5 The kidney is responsible for elimination of 90% of this ion, and while compensatory increases in gastrointestinal (GI) elimination of K+ (capable of increasing its contribution in eliminating K+ from 10% to 25%) is helpful in controlling K+ levels in the face of renal insufficiency, this organ alone cannot replace the regulatory effect of adequate renal function.3 P R EO P E R AT I VE R I S K FAC TO R S F O R H Y P E R K A L E M I A

Hyperkalemia is a concern for patients with renal dysfunction requiring surgery. The preoperative evaluation of these patients’ renal function, which ranges from Stage I kidney disease with

glomerular filtration rate (GFR) >90 mL/​min to Stage V disease with GFR 5.5 mmol/​ L) can occur without any notable EKG changes, the following abnormalities are concerning:  peaked T waves, prolonged PR intervals, widened QRS, bundle branch block, asystole, ventricular fibrillation, and/or changes characteristic of MI (see Fig. 31.2).1,2,4 Patients with pacemakers may encounter increased ventricular and atrial pacing thresholds causing failure to capture, increased latency, and, in the case of cardioverter-​defibrillators, inappropriate shocks.10 I N T R AO P E R AT I VE R I S K FAC TO R S F O R H Y P E R K A L E M I A

Intraoperative hypovolemia and consequent decreased organ perfusion and metabolic acidosis places patients at risk for acute increases in K+, particularly those presenting with already compromised kidney function.1,11,12 Intraoperative hyperkalemia due to the buffering of excess hydrogen ions in cells causing movement of K+ extracellularly in order to maintain electroneutrality (intracellular K+ is exchanged for the high levels of extracellular H+ ions) merits close attention to acid-​ base balance during surgery.1,11,12 Respiratory acidosis due to hypoventilation in the setting of sepsis or iatrogenic from preoperative sedatives or pain therapy can also contribute to and exacerbate derangements in acid-​base balance, causing increased serum K+.1,5 In the case of the insulin-dependent diabetic patient presenting with diabetic ketoacidosis, hyperkalemia is due in part to an absence of insulin and hyperglycemia inducedhyperosmolality. Hyper-osmolality results in osmotic movement of water from the cells into the extra-cellular fluid, along with potassium. Caution is necessary in interpreting serum electrolyte levels as these patients may actually be K+ depleted due to potassium loss in the urine.1,10–11 Succinylcholine administration useful for prompt muscle relaxation for rapid sequence intubation will cause an increased release of K+ through membrane depolarization of acetylcholine receptors (from 0.5 to 1.0 mmol/​L increase in K+).13,14 While this increase is negligible in most patients including the renal failure patient with a normal preoperative potassium level, other nondepolarizing muscle relaxants may be preferable for patients with CKD or ESRD needing emergency surgery who present with already elevated K+ levels. Equally important in managing surgical patients is being mindful of the exaggerated increases in K+ from succinylcholine which may occur in the septic patient presenting for emergency abdominal surgery; the patient with acute spinal cord injury presenting for stabilization with upregulation of extrajunctional acetylcholine receptors due to denervation

272  •   R enal

(a)

(b) I

aVR

V1

V4

II

aVL

V2

V5

III

aVF

V3

V6

I

aVR

V1

V4

II

aVL

V2

V5

III

aVF

V3

V6

Speed: 25 mm/sec (c)

Speed: 25 mm/sec

Panel A: EKG demonstrating complete heart block with ventricular escape in a patient with end stage renal disease and K+8.6 mmol/​L. Panel B: Same patient after treatment with Calcium gluconate (2 ampules): EKG is significant for sinus tachycardia with prolongation of QRS interval, first-​degree artrioventricular block, and peaked T waves. Panel C: Sometime after K+ corrected, the EKG is significant for anteroseptal ST and T wave changes with negative cardiac exercise imaging. Reproduced with permission from Kuvin JT. Electrocardiographic changes of hyperkalemia. N Engl J Med. 1998;338:682. Figure 31.2 

injury beginning 24 hours after injury; burn patients also due to upregulation; and patients with neuromuscular disorders like Duchenne muscular dystrophy, a disease characterized by myofibril atrophy and necrosis due to rhabdomyolysis.13,14 Every anesthesiologist must also remain vigilant regarding the small but nevertheless potentially lethal consequence of the succinylcholine and/or volatile anesthetic-induced clinical syndrome of malignant hyperthermia. Rigidity after induction with succinylcholine followed by unexplained sinus tachycardia or ventricular arrhythmias together with increased end tidal CO2 and increased temperature should raise concern and lead to a search for metabolic acidosis due to the severe rhabdomyolysis which occurs. Discontinuing the triggering anesthetic and treatment with Dantrolene 2.5 mg/kg to a total of 10 mg/kg every 5-10 minutes is life-saving through its ability to reduce the concentration of sarcoplasmic Ca+ and help lower serum K+. Early

treatment with this agent has successfully reduced the mortality of this syndrome from 60% to 1.4%.16 Care of the trauma patient in the operating room may be particularly challenging due to the need for massive blood transfusion, which may predispose these patients who are already acidotic and hypovolemic to hyperkalemia. Rhabdomyolysis and myoglobinuria will place patients with crush injury at even greater risk of severe hyperkalemia.17 T I S S U E C ATA B O L I S M SY N D RO M E S

Crush syndrome known as traumatic rhabdomyolisis or Bywaters’ syndrome, happens when a force or compression is applied to the body for an extended period of time, over four to six hours, or less in susceptible already compromised areas of the body As the compression occurs, cells in the

31. I ntraoperative H yperkalemia  •   273

immediate area are quickly damaged. Within an hour, the pressure continues to decrease circulation to the area. This compromises oxygen delivery. Anaerobic metabolism results and this generates large amounts of lactic acid. Cellular hypoxia results in the cell membrane losing its ability containing cell contents, which begin to leak through the walls because of the increasing wall permeability. Contents can include potassium, myoglobin, purines and other toxic substances that are dumped from the cells into the surrounding tissues. Symptoms may also include numbness and difficulty moving the extremity and visible muscle bulging. Other cells are so compromised that they lyse and die causing shock and acute renal failure after a crushing injury to skeletal muscle. The progressive acute renal failure is because of acute tubular necrosis.14,15 I N T R AO P E R AT I VE M A NAG E M E N T O F C RUS H S Y N D RO M E

Management of this disease oftentimes will have begun in the field. However, vigilance, together with a proactive approach to anesthetic care in the operating room, will be critical to avoid the risk of hyperkalemia, the most important electrolyte abnormality in the management of this syndrome.17–19 Isotonic saline at a rate of 1 L/​hr should be alternated with hypotonic saline with NaHC03 (50–​200 mEq NaC03 added to every second or third liter) with the goal of keeping urine output >300 mL/​hr with a pH of 6.5. Mannitol (50 mL or 20% mannitol) may help to decrease the considerable amount of fluids accumulating in the muscle tissue. Monitoring central venous pressure, intraoperative echocardiogram, or other noninvasive means of assessing a patient’s response to this resuscitation is key to averting pulmonary edema or volume overload. This together with close attention to electrolytes, particularly K+, will help determine the need for intraoperative HD.19 T R E AT M E N T O F H Y P E R K A L E M I A

Life-​saving emergency treatment is typically instituted at K+ levels >6.5 mEq/​L with or without EKG abnormalities.4 The potential for serious cardiac morbidity and mortality from hyperkalemia requires a systematic plan of treatment which includes prompt curtailing of the effect of K+ on the cell membrane, lowering serum K+, as well as the more protracted elimination of excess K+.1,4 Continuous EKG in an intensive care setting is key to correcting potassium levels safely in these patients postoperatively. R E M E D I E S F O R H Y P E R K A L E M I A

Calcium, insulin, and ß2 agonists are quick remedies for hyperkalemia. Calcium will help restore the normal gradient across the cell membrane, restoring the resting membrane potential and facilitating normal excitability of cardiac myocytes.1,4 The recommended dose of 1 gm of calcium chloride (CaCl) (or calcium gluconate) will take about 3 minutes (CaCl works faster) to work, and its effect will last about 30

minutes. This dose may be repeated in 5 minutes if there is no improvement in the EKG. Insulin 10 units intravenously will drive K+ into cells by stimulating the Na/​K ATPase pump. Typically administered with dextrose 25 g of a 50% solution unless the glucose is >250, this treatment works in about 15 minutes and will decrease K+ by 0.6 mmol/​L. Like insulin, ß2 agonists will also facilitate entry of K+ into cells. In the United States, this drug is given via nebulizer—​10 to 20 mg (the typical dose for bronchoconstriction in asthma is 0.5 mg)—​and it works in minutes (K+ decrease of 0.62–​0.98 mEq/​L). An intravenous form is available for use outside the United States.1,4 Sodium bicarbonate can also be used but will take longer.4 While sodium bicarbonate may help decrease K+ levels when administered as a 4-​hour infusion, shorter term bolus administration (50–​100 mEq over 5–​10 minutes in an adult) does not seem to provide any benefit for hyperkalemia when used alone. Caution is advised in patients with CHF as the added bolus of sodium may strain the heart.1,4 Hemodialysis is the most efficient and effective way of eliminating K+ from the body (1  mmol/​L in 1 hour and 2  mmol/​ L in 3 hours).4 Close monitoring after HD is recommended for the risk of rebound to prior K+ levels with 35% of the reduction abolished in an hour and 70% in 6 hours.4 Regarding ion exchange resins, sodium polystyrene sulfonate or Kayexlate is typically administered (orally or rectally) with a laxative like sorbitol in order to promote the exodus of secreted K+ in the gut in exchange for Na+ released by the resin. This treatment takes about 6 hours when administered orally. Diarrhea, hypernatremia, hypokalemia, and intestinal necrosis, particularly in the critically ill, has limited the use of this agent as therapy for hyperkalemia.4 N EW T H E R A P I E S F O R H Y P E R K A L E M I A

Patrimor, a nonabsorbable polymer recently approved by the Food and Drug Administration which, while effective in lowering serum K+, takes time, and side effects include constipation, hypomagnesemia, diarrhea, nausea, and abdominal discomfort.16 Another new, not yet approved potassium binder is sodium zirconium cyclosilicate (ZS-​9), which is effective in lowering K+ by 0.5 to 1 mEq/​L but also takes time and can cause edema and hypokalemia.20 R E VI EW Q U E S T I O N S 1. Which of the following is true regarding K+? A. The Na/​K ATPase pump is the critical regulator of potassium excretion. B. Metabolic alkalosis favors movement of K+ out of cells. C. The total body K+ in a 70-​kg adult is approximately 4256 mEq, of which about 4200 mEq is in the extracellular component. D. The Na/​K ATPase pump keeps intracellular potassium greater than extracellular K+ levels which is important in maintaining the electrical potential across cell membranes.

274  •   R enal

Answer: D The Na/​K ATPase pump is key to maintaining high intracellular K+ (4200 mEq intracellular vs. 56 mEq extracellular—​C) relative to the extracellular K+, thereby facilitating movement of this ion down its concentration gradient and affecting a negative resting voltage across the cell membrane necessary for maintaining excitability of cells.1 A: The kidneys are the major regulators of K+ excretion from the body with a small contribution of excretion from the GI tract. The Na/​K ATPase pump helps maintain the sodium/​ potassium gradient across the cell membrane. B: Metabolic acidosis tends to shift K+ out of cells while metabolic alkalosis favors movement into cells in order to maintain electroneutrality across cells. This knowledge is important for anticipating the potential for intraoperative hyperkalemia in patients who are acidotic.1 2. Which of the following is false regarding potassium and electrophysiologic function of myocytes? A. Changes in extracellular potassium concentration influences myocyte electrophysiologic function. B. As potassium levels increase in the extracellular space, the magnitude of the concentration gradient for potassium across the myocyte decreases. C. As the resting membrane potential becomes less negative in the setting of progressive hyperkalemia, Vmax (phase “0” of the action potential) decreases causing a slowing of impulse conduction through the myocardium. D. As the resting membrane potential becomes less negative (to –70mV), the percentage of available Na+ channels increase. Answer: D The membrane potential at the onset of depolarization determines the number of sodium channels activated during depolarization which will determine the magnitude of the inward sodium current and the Vmax of the action potential (AP).2 As the resting membrane potential becomes less negative, in the setting of hyperkalemia for example (B), the percentage of Na+ channels decrease causing a decrease in the inward sodium current and therefore a decrease in the Vmax of the AP. The decreased Vmax will lead to a slowing of impulse conduction (C). Increases in extracellular K+ therefore sets the stage for changes in voltage from normal (–90mV) across the cell membrane thereby effecting the Vmax of the AP making this cation largely responsible for changes in the electrophysiologic function of myocytes. (A). 3. Which one of the following is a symptom of hyperkalemia? A. Brain herniation B. Nephrolithiasis C. Laryngospasm D. Paralysis Answer: D As hyperkalemia becomes more severe, weakness may progress to flaccid paralysis with decreased deep tendon reflexes.4 Brain herniation may occur with hyponatremia (A). Hypotonicity

will affect the osmotic gradient favoring movement of water into the brain causing cerebral edema, increased intracranial pressure, and finally brain herniation.7 Answer B is a symptom of chronic hypercalcemia.7 Answer C can be consequent to hypocalcemia, a concern after parathyroid resection.7 4. Which of the following is true regarding causes of hyperkalemia? A. Hyperkalemia is not a risk of NSAID’s administration. B. Monitoring K+ levels in patients taking cyclosporine is not necessary as these patients are not at risk for developing hyperkalemia C. Patients administered succinylcholine may develop Pseudohyperkalemia D. Transcellular shift of K+, decreased renal K+ excretion, and drugs may place some patients at risk for hyperkalemia. Answer: D Transcellular shift of potassium between the intracellular and extracellular compartments after administration of depolarizing muscle relaxants like succinylcholine; decreased renal K+ excretion; and certain drugs, may place some patients at risk for hyperkalemia.1,7 Drug-induced hyperkalemia from drugs like NSAID’s (A) most commonly occurs in patients with other predisposing factors like diabetes, renal insufficiency, advanced age, and hyporeninemic hypoaldosteronism.1,5 Cyclosporine for immunosuppression in transplant patients suppresses the release of renin and interferes with the secretion of K+ in the collecting duct and for this reason regular evaluation of renal function is a necessary part of the routine care of these patients5 (B). Pseudohyperkalemia is artificially elevated potassium (hemolysis of a blood specimen)1,7(C). 5. EKG signs of hyperkalemia include all of the following EXCEPT: A. U waves B. Widened QRS C. Peaked T waves D. Prolonged PR interval Answer: A A U wave which sometimes follows the T wave on the EKG may be associated with hypokalemia or hypomagnesemia. Narrow-based peaked T waves are an early sign of hyperkalemia on the EKG. (C). With continued increases in K+ , a widened QRS complex (B) and a prolonged PR interval (D) reflect a decreased rate of phase 0 of the AP leading to a longer AP.2 6. Which of the following statements regarding ACEIs is FALSE? A. These drugs may be a cause of potassium elevation in some patients. B. They stimulate the formation of Ag II. C. They interfere with aldosterone secretion. D. They are frequently prescribed to treat hypertension in patients with diastolic dysfunction.

31. I ntraoperative H yperkalemia  •   275

Answer: B Ag I is converted to Ag II by angiotensin converting enzyme from the lung. Inhibition of this enzyme by ACEI will block the formation of Ag II.5 A: ACEI, by blocking the production of Ag II and interfering with aldosterone secretion (C), will cause a decrease in K+ secretion which in patients with renal insufficiency may lead to increased serum K+.5 D:  ACEIs are often useful in treating hypertension by interfering with aldosterone secretion, thereby blocking the increased absorption of NaCl and water as well as the vasoconstrictor action of Ag II. A  number of long-​term studies evaluating ACEI in high-​risk patients with a history of MI or CHF reveal a 20% reduction in mortality over patients treated with placebo.21 7. What is the most effective way of eliminating excess potassium from the body? A. Intravenous calcium chloride administration B. Insulin C. Hemodialysis D. Albuterol Answer: C Hemodialysis is the most definitive way of eliminating K+ from the body. Unfortunately, this therapy takes time (1–​4 hours). Central venous access must be established for temporary hemodialysis catheter insertion for filtering the blood. Close monitoring after HD is recommended for the risk of rebound to prior K+ levels from intracellular K+ stores (HD filters extracellular K+).4 A:  Calcium chloride or calcium carbonate administered in a dose of 1 gm intravenously, while helpful in restoring the resting membrane potential and excitability of cardiac cells in the emergency setting, does not eliminate excess K+. This drug is helpful, however, in its ability to quickly reverse the negative impact of potassium on the electrical activity of the heart.4 B and D: While insulin and albuterol both cause a shift of K+ intracellularly, they do eliminate potassium from the body.4 8. All of the following are true regarding Sodium polystyrene sulfonate EXCEPT: A. This cation exchange resin works slowly to lower K+ B. This resin works in the blood C. It is given orally or by retention enema D. This drug may be toxic to patients Answer: B Sodium polystyrene sulfonate (Kayexalate) is a cation exchange resin which works in the intestine to reduce K (not the blood).4 Unfortunately, this drug works slowly (A) to lower K+: onset of action of oral administration is 2 hours and less pronounced but faster onset when administered as an enema (C). In addition to its slow onset of action, there are reports of intestinal necrosis from the use of this agent4(D). 9. Which of the following is false regarding hyperkalemia? A. Furosemide can help treat hyperkalemia

B. 9-a-fludrocortisone is not helpful in the treatment of hyperkalemia C. b-adrenergic blocking agents can predispose patients to hyperkalemia by blocking the stimulatory effect of the sympathetic nervous system on the release of renin. D. Caution is advised in the use of Calcium to help with hyperkalemia caused by Digoxin toxicity Answer: B 9-a-fludrocortisone, a mineralocorticoid is helpful in patients with adrenal insufficiency, a known cause of hyperkalemia.1 A: Furosemide, a loop diuretic which acts in the medullary and cortical aspects of the thick ascending limb and the distal tubule blocks absorption of sodium, chloride, and water from the filtered fluid in the kidney tubule prompting diuresis together with K+ excretion.1 C: In addition to blocking the effect of the sympathetic nervous system on renin release, patients on b-adrenergic blocking agents may be predisposed to developing hyperkalemia through these agents’ effect in decreasing activity of Na/K ATPase.5 D: Digoxin which increases intracellular Calcium in cardiac myocytes by opening membrane channels can inhibit Na/K ATPase at toxic levels causing hyperkalemia.1 Calcium administration in the face of digoxin toxicity may lead to a massive influx of calcium into myocytes causing myocardial failure and death.1,4 10. Which of the following is true regarding succinylcholine administration? A. The negligible increase in K+ with administration of this muscle relaxant is not a problem in acute spinal cord injury. B. Its use is contraindicated only in the ESRD patient. C. It is safe to administer succinylcholine to patients with Duchenne muscular dystrophy. D. Exaggerated increases in K+ from succinylcholine may occur in patients with burn injury. Answer: D Succinylcholine continues to be the drug of choice when rapid control of the airway is needed. However, one of the more serious adverse effects of this muscle relaxant in some patients is hyperkalemia.14 Succinylcholine should be avoided 48 to 72 hours after burn and/​or spinal cord injury as time for upregulation of nicotinic acetylcholine receptors will have had time to occur, leading to an exaggerated increase in plasma K+ in these patients. Once burn patients are healed, assuming they are able to resume previous activity levels, acetylcholine receptors return to normal levels.13 A: Succinylcholine causes an increase in K+ of 0.5 to 1.0 mmol/​ L due to depolarization of acetylcholine receptors.13,14 This increase, while negligible, may be a problem in patients with already elevated K+ levels preoperatively. In denervated muscle, there is up-​regulation of nicotinic acetylcholine receptors as well as an isoform, α7acetylcholine receptor (α7AchR). Succinylcholine acts on both receptors to cause depolarization and leakage of intracellular K+. Additionally, choline, a metabolite of

276  •   R enal

succinylcholine, also acts on the up-​regulated α7AchRs, resulting in continued leakage of intracellular potassium, exacerbating the increased K+. In the renal failure patient with normal K+ levels, succinylcholine may be used safely as this patient population does not demonstrate an exaggerated response to succinylcholine induced increased in serum potassium (B).13,14 C:  Duchenne muscular dystrophy is a common childhood muscular disease characterized by progressive skeletal muscle degeneration leading to progressive weakness.12 These patients are at risk for rhabdomyolysis causing hyperkalemia and potential cardiac arrest after succinylcholine:  patients with Duchenne muscular dystrophy lack dystrophin necessary for additional support to the sarcolemma of muscle cells. Succinylcholine administration stresses the already weak sarcolemma, increasing its permeability and causing intracellular Ca+ to increase while K+ leaks out. Nondepolarizing muscle relaxants are better options for this patient population.15 11. Reasons for hyperkalemia after crush syndrome include all of the following EXCEPT: A. rhabdomyolysis. B. entrapped muscle. C. myoglobin induced renal failure. D. alkalinizing the urine. Answer: D After aggressive hydration of patients with crush syndrome, alkalinizing the urine may help with reducing the risk of renal failure. Alkalinizing the urine helps to decrease precipitation of myoglobin and may help improve the clearance of this protein released from skeletal cell membrane breakdown and reduce renal damage.17–​19 A: Rhabdomyolysis refers to the release of cellular components, like K+, from muscle cells as a result of damage due to crush injury which causes stretching and leaking of the cellular membrane.17,18 B:  In the setting of trauma, entrapped muscle (typically lower extremities) may be the initial inciting event leading to ischemia-​ induced damage of the skeletal muscle and inciting a cascade of events leading to acidosis, ARF, and hyperkalemia.17–​19 C:  Excess serum myoglobin in the setting of hypovolemia and metabolic acidosis associated with crush injury in trauma patients effects a lowered urine pH, which contributes to myoglobin induced obstruction of the distal nephron, worsening renal function and exacerbating hyperkalemia.17 12. Which of the following statements is true regarding CKD? A. Risk factors include hypertension and DM. B. There are five stages of CKD with stage V (GFR 90 mL/​min/​1.732); Stage 2 (60–​89 mL/​min/​1.73m2); Stage 3 (30–​59 mL/​min/​1.73m2 ); Stage 4 (15–​29 mL/​min/​ 1.73m2); and Stage 5 (250

SEVERE

Plasma glucose (mg/​dL)

>250

>250

Arterial pH

7.25–​7.30 7.00–​< 7.24

7.3, and a calculated anion gap 7.3 D. Sodium level 143 mEq/​L

Answer: C Resolution of DKA is determined by a blood glucose 15 mEq/​L, a venous pH >7.3, and a calculated anion gap 6 years (not 60 months), and vital capacity 70% of patients) •  Chronic pain syndrome (only in a small subset of patients) •  Opioid tolerance •  Proliferative retinopathy, blindness

Cardiovascular

•  Cor pulmonale •  Systemic hypertension (due to renal disease) •  Left or right ventricular dysfunction •  Conduction abnormalities due to iron overload •  Lower extremity ulcers

Pulmonary

•  Pulmonary infarction •  Airway hyperreactivity /​bronchospasm, •  Obstructive or restrictive lung disease •  Pulmonary fibrosis •  Pulmonary hypertension (30%–​40%) •  Acute chest syndrome (up to 40%)

Genitourinary

•  Hyposthenuria (inability to concentrate urine, predisposing to dehydration) due to renal papillary necrosis •  Hematuria •  Proteinuria, nephrotic syndrome •  Chronic renal insufficiency (present in 5%–​ 20% of adults) •  Priapism (10%–​40% of men)

Gastrointestinal

•  Cholelithiasis, choledocholithiasis, cholecystitis, intrahepatic cholestasis •  Acute hepatic sequestration crisis •  Cirrhosis (16%–​29%) •  Dyspepsia (due to gastric mucosal ischemia, not acid production) •  Ischemic colitis

Musculoskeletal

•  Osteomyelitis (especially due to Staphylococcus or Salmonella spp.) •  Dactylitis •  Avascular necrosis (present in up to 50% of adults)

Hematologic /​ Immunologic

•  Chronic hemolytic anemia •  Complications related to chronic transfusions: iron overload, hemolytic and non-​hemolytic transfusion reactions, infectious transfusion complications, alloimmunization •  Hypercoagulability •  Venous thromboembolism •  Acute splenic sequestration crisis •  Hemolytic  crisis •  Aplastic  crisis •  Immune dysfunction •  Autosplenism, susceptibility to infection with encapsulated organisms

CVA = cerebrovascular accident; CNS = central nervous system.

Two primary problems with HbS exist when compared to normal Hemoglobin-alpha1 (HbA1): its insolubility and its instability.

Insolubility When HbS undergoes the normal conformational changes that occur upon deoxygenation, valine is able to form hydrophobic bonds with adjacent Hb molecules. Because of this, the deoxygenated form of HbS will tend to polymerize and precipitate within the RBC. HbS polymers form a gelatinous network within the RBC that stiffens the cell membrane, increases viscosity, and produces the characteristic “sickle” shape.6 Sickled RBCs lose their pliability and become unable to traverse small capillaries, in turn provoking localized ischemia and/​or a painful crisis. These changes also lead to chronic hemolysis as the spleen takes up and destroys the abnormal RBCs.

Instability Normally, the iron-​containing heme is protected within a globin pocket in the RBC. Due to the instability of oxygenated HbS, there is accelerated denaturation of globin, leading to the release of large quantities of toxic iron and heme compounds within the RBC.15 Iron generates oxygen radicals which cause widespread oxidative damage to the cell membrane, leading to impairment of normal membrane function.4 The abnormal RBCs become more adherent to the vascular endothelium, causing endothelial damage and inflammation.16 The consequences of HbS extend well beyond just the structure and shape of the RBC; the entire vascular milieu is drastically altered. It is these widespread and diverse biochemical changes that produce the clinical features of SCD. As evidence of this, patients with SCD have upregulation of endothelial cell adhesion molecules, deranged nitric oxide physiology, activated coagulation pathways, elevated markers of platelet activation, and elevated cytokine levels.4,16,17 Because the vascular endothelium plays an important role as a regulator of vascular tone, coagulation, fibrinolysis, inflammation, lipid transport, and permeability, the consequences of HbS are widespread. It has even been proposed that VOC are triggered by changes in endothelium homeostasis and that sickling is a secondary event, rather than the trigger of vasoocclusion.4 P R EO P E R AT I V E EVA LUAT I O N

Patients with SCD are at risk for both SCD-​specific and nonspecific perioperative complications, and the goal of the preoperative assessment should be to determine the patient’s risk factors for potential complications in order to minimize them during surgery.5 SCD-​specific complications include VOC and ACS, which may occur up to 30% to 40% of cases. There is no consensus as to whether SCD patients are at higher risk for nonspecific perioperative complications than the general population.10,11

35. S ickle C ell D isease and C holecystectomy  •   307

History and physical It is important to assess the frequency, pattern, and severity of recent SCD events to obtain an understanding of the current disease state. A  history of frequent hospitalizations is an independent predictor of perioperative SCD-​related complications, as are advanced age, pregnant status, recent or current infection, and history of pulmonary disease.10,18 Other risk factors can be seen (see Table 35.2) and should be optimized when possible. Additionally, it is important to assess for chronic organ dysfunction or failure, with a focus on the neurologic, pulmonary, and renal systems in particular.

Labs and imaging Preoperative labs should include a complete blood count, type and screen (or crossmatch), and basic metabolic panel. It may also be helpful to check a reticulocyte count and HbS level; however, it is interesting to note that higher HbS levels are not associated with complications.18 Pulse oximetry and chest radiograph are useful in assessing for underlying pulmonary disease. Pulmonary function tests may also be appropriate if the patient reports a history of worsening or paradoxical dyspnea. Other studies should be guided by the individual history, physical, and nature of the specific surgical procedure. A N E S T H ET I C M A NAG E M E N T

Even with meticulous care, complications in patients with SCD are unfortunately frequent. Understanding of SCD has evolved since the anesthetic implications of the disease were initially described in 1955, yet many recommended management strategies, such as preemptive RBC transfusion, aggressive hydration, and avoidance of hypoxia, hypothermia, and acidosis, still persist in current practice.19 Clearly some of these are fundamental standards of anesthetic care and should be followed, but certain strategies have since come into question.

Hydration

up to 2 hours prior to surgery).7 However, there is no evidence that excessive hydration is beneficial, and the practice of preadmitting patients solely for hydration may be excessive, costly, and inconvenient.

Premedication Heavy premedication may potentially lead to respiratory depression and hypoxia. Due to concerns of hypoxia precipitating a sickle event, traditional practice has been to avoid premedication entirely or to only give small doses of anxiolytics preoperatively to patients with SCD. However, there is some evidence that patients with SCD may actually better withstand hypoxia and that hypoxia per se may not be a cause of acute complications.20,21 Patients with SCD often have significant psychologic distress due to their chronic conditions, and anxiety can be common in the perioperative period; withholding anxiolytic medications may lead to undue suffering.4 If premedication is given, it would be judicious to monitor the patient with pulse oximetry and provide supplemental oxygen if needed.

Transfusion/​blood management There are multiple indications for transfusion in SCD patients:  acute symptomatic anemia, aplastic crises, acute Table 35.2  PREDICTORS OF POSTOPERATIVE COMPLICATIONS IN SCD PREDICTORS OF POSTOPERATIVE COMPLICATIONS

Risk of the surgical procedure

Low, moderate, or high risk

Increased age

Related to disease progression

Previous hospitalizations for SCD-​ Risk is elevated for >5 prior related events hospitalizations History of pulmonary disease

Careful attention to fluid balance is important in this patient population. Some SCD patients may develop renal papillary necrosis and hyposthenuria (the inability to concentrate urine), predisposing them to dehydration. Others may develop renal insufficiency and are prone to volume overload with excessive fluid administration. Intravascular dehydration has long been implicated as a potential cause of perioperative SCD-​related complications, based on studies showing it precipitates sickling in vitro. For this reason, protocols commonly exist in which patients are admitted for intravenous (IV) hydration the night prior to surgery, with recommended fluid rates set at 1.5 times the maintenance requirement. However, there is a lack of definitive clinical evidence to support this assumption, and perioperative SCD events can be common despite generous hydration.4,5 Clearly, avoiding dehydration is prudent, and steps should be taken to ensure the patient does not become significantly hypovolemic (e.g., allowing the consumption of clear liquids

NOTES

Increases the risk of ACS

History of CNS disease Pregnancy

Increases the risk of maternal complications

Infection

May be related to elevated inflammatory markers, increases risk for ACS

FACTORS THAT ARE NOT PREDICTIVE OF POSTOPERATIVE COM PLICATIONS

Steady-​state Hb concentration Preoperative hemoglobin S level (% of total Hb) Number of prior blood transfusions History of cardiac or renal disease ACS = acute coronary syndrome; SCD = sickle cell disease; CNS = central nervous system; Hb = hemoglobin.

308  •   H ematology

splenic or hepatic sequestration, acute stroke, ACS, multiorgan failure, or in preparation for surgery.22,23 The topic of preoperative transfusion in SCD patients is a complicated and controversial one, with no clear consensus as to what constitutes optimal management. Several transfusion regimens have been described. The traditional (i.e. “aggressive”) approach is to transfuse blood until HbS concentration is reduced to below 30%, either by simple or exchange transfusion. The “conservative” approach is to perform simple transfusions until a target Hb concentration of 10 g/​dL or Hct of 30% is achieved. A landmark study in 1995 showed that the conservative strategy was as effective as the aggressive approach in preventing SCD-​related complications.10 However, more recent evidence is mixed, and the superiority of either approach is still debated.24 In general, exchange transfusion to reduce the HbS fraction to below 30% is recommended for high-​risk surgical procedures (e.g., cardiac surgery) or for patients with pulmonary disease anticipating prolonged anesthesia.25,26 The advantages of exchange transfusion over simple transfusion include the ability to rapidly adjust the Hct or HbS fraction with less risk of volume overload.22 However, it increases blood use, requires special equipment, and may not be available at all centers. Several studies have also examined whether withholding transfusion would be beneficial, with some showing worse outcomes while others suggesting no additional harm.18,27–​29 Due to the heterogeneity of evidence, it is difficult to make universal recommendations for preoperative transfusion, but for a moderate-​risk surgery like a cholecystectomy, it seems reasonable to transfuse preoperatively to achieve Hb of 10 g/​ dL (i.e., the conservative approach). However, this strategy must be individualized to the specific patient and surgical procedure. There is also no high-​ quality evidence to guide intraoperative decisions regarding transfusion, target Hb concentration, or target HbS fraction. Again, the decision should be individualized based on the patient, the surgical procedure, and the operative blood loss. Of note, intraoperative cellular salvage (e.g., Cell Saver) is typically not recommended for patients with SCD, or even sickle cell trait, due to concerns about it provoking severe sickling in the collected RBCs.30,31 One other factor complicating the issue of transfusion is that of alloimmunization, or the development of antibodies against non-​ABO blood groups. SCD patients are more likely to develop alloantibodies than the general population, with anti-​E, anti-​ K, and anti-​C being the most common antibodies formed.22,32 Blood banks typically have protocols for crossmatching SCD patients to help reduce the risk of alloimmunization, but it can still occur and complicate future crossmatching, making it harder to find potentially life-​saving blood. A N E S T H ET I C T EC H N I Q U E

The choice of anesthetic technique does not appear to significantly alter the risk of SCD-​related complications.7 General anesthesia is beneficial for improved ventilation and oxygenation control and is associated with less frequent and severe hypotension than neuraxial anesthesia. Common IV and/​or

volatile anesthetic agents may all be used, including nitrous oxide. Potential disadvantages of general anesthesia include inadequate postoperative analgesia and hypoventilation due to the respiratory depressant effect of anesthetic agents. Although there are concerns that regional anesthesia may increase the incidence of SCD-​related complications, it is not contraindicated and may actually provide benefits in postoperative pain control, especially in patients with opioid tolerance.5,7,11 Some authors have advocated for neuraxial anesthesia with the theoretical benefit of decreasing vaso-​occlusive events, in part due to sympathectomy-​mediated vasodilatory effect.33 Regardless of the anesthetic technique chosen, it is important to ensure optimal oxygenation, normothermia, adequate hydration, and tissue perfusion, as well as administer appropriate antibiotic prophylaxis and provide adequate postoperative analgesia.34 Certain drugs may be associated with methemoglobinemia (e.g., benzocaine, prilocaine, dapsone, and metoclopramide) or drug-​ induced hemolytic anemia (e.g., cephalosporins and penicillins); it would be judicious to avoid these drugs entirely or use them with extreme caution in the perioperative setting when caring for SCD patients. P O S TO P E R AT I VE A NA L G E S I A

Postoperative pain is experienced by all patients but requires especially careful assessment in patients with SCD. It is important to distinguish between normal postsurgical pain from the pain of an SCD-​related complication (e.g., VOC or ACS), a nonspecific postoperative complication (e.g., deep vein thrombosis [DVT] or pulmonary embolism [PE]), or a chronic, pre-​ existing pain condition.34,35 If the patient develops new-​onset pain after surgery, it should be carefully assessed and a diagnosis should be pursued while the pain is treated. Opioid tolerance is common in patients with SCD, and this may complicate perioperative management. Postoperative opioid requirements in SCD patients may be up to twice that of the general population.36 For this reason, a multimodal approach is beneficial, including the use of nonsteroidal anti-​ inflammatory drugs (NSAIDs), acetaminophen, gabapentinoids, NMDA antagonists, α2-​ agonists, and regional anesthesia techniques. Dexmedetomidine in particular is emerging as an attractive adjuvant that has opioid-​sparing effects and minimal respiratory depression.37 M A NAG E M E N T O F P O S TO P E R AT I V E C O M P L I C AT I O NS

Patients with SCD are at risk of developing both disease-​ related complications and other nonspecific postsurgical complications. The management of nonspecific complications is beyond the scope of this chapter. The 2 most important SCD-​related complications that warrant discussion here are VOC and ACH.

VOC VOC can occur anywhere in the body, though it appears most commonly in the long bones, ribs, vertebrae (typically in the

35. S ickle C ell D isease and C holecystectomy  •   309

lumbar spine), and abdomen.4 The pain from a VOC may be worse than expected postsurgical pain. No tests are definitive for its diagnosis, so clinical judgement is essential.6 When it occurs, treatment is primarily supportive and involves aggressive pain control and liberal hydration. Opioids are the mainstay of treatment for VOC, and PCA system is the management of choice.5,38,39 Adjuncts such as acetaminophen and NSAIDs may be useful as well; NSAIDs such as ketorolac may be particularly effective for bone-​related pain. Regional anesthesia may also be highly effective. Interestingly, transfusions do not shorten the duration of VOC and are not indicated for uncomplicated VOC.6,19

ACS ACS is a medical emergency that may require management in an ICU.6 It typically presents on postoperative day 2 or 3 with chest pain, tachypnea, and oxygen desaturation and lasts an average of 8 days. Mortality can approach 1% to 5%.5 The differential diagnosis includes pneumonia and PE, which can be common postoperatively. The diagnostic evaluation should be particularly thorough to eliminate these as the cause of the patient’s symptoms. Early ambulation, administration of humidified oxygen, and incentive spirometry may help to prevent ACS from developing, and there is some low-​level evidence that preoperative blood transfusion may help prevent ACS.13,24 If ACS does occur, the treatment includes both transfusion and supportive therapy. Supplemental oxygenation, incentive spirometry, and bronchodilator therapy are recommended.5 Transfusion is indicated to maintain a Hct of 30%, and emergency exchange transfusion may be required for hypoxia (SpO2 < 90%).6 Fluid status is important and should be carefully monitored to avoid the development of pulmonary edema. Pain control should be aggressive and is often best managed with a PCA. C O N C LUS I O N S

R E VI EW Q U E S T I O N S 1. What is the approximate rate of perioperative complications in patients with SCD? A. 30% B. 1% C. 50% D. 10% Answer: A Patients with SCD represent a high-​risk surgical population. They are susceptible to both SCD-​specific complications (e.g., painful crisis or ACS) and nonspecific complications (e.g., atelectasis, hospital-​acquired pneumonia, DVT, PE, and surgical site infection) in the perioperative period. The overall perioperative complication rate for SCD patients has been estimated to be as high as 30% to 40%, with a 30-​day mortality as high as 1.1%. 2. A patient with SCD with cholelithiasis presents for an elective cholecystectomy. Her preoperative labs are significant for a Hb concentration of 8.6 g/​dL, a Hct of 24.2%, and a HbS level of 68%. Which of the following preoperative transfusion strategies would be best for this patient? A. This patient does not need transfusion preoperatively, but a type and crossmatch should be performed. B. Simple transfusion should be used with a goal Hct of 30%. C. Simple transfusion should be used for a goal HbS 94%. B. Transfer the patient from the floor to a monitored unit (e.g., the surgical ICU). C. Obtain a stat chest film. D. Transfuse 1 unit packed RBC and recheck labs. E. Perform a Duplex ultrasound of the lower extremities. F. All of the above Answer: F Based on this presentation, the most likely diagnosis in this patient is either a PE or ACS. This patient has risk factors for both PE and ACS, and supportive therapy should be aggressively administered while a diagnosis is actively pursued. The initial steps in management should include escalating the level of care and administering oxygen, analgesics, and IV fluids. Transfusion would be indicated to decrease the fraction of HbS. A chest film is expected to be negative in PE and may show a new infiltrate involving at least 1 lung segment in ACS. An ultrasound of the lower extremity may show evidence of a DVT; however, if it is negative, DVT/​PE is not completely ruled out, as patients may have DVT involving the deep pelvic veins as well as the legs. There should be a low threshold for checking a computed tomography of the chest.

A. The preoperative HbS level B. A history of restrictive lung disease C. A  recent upper respiratory tract infection within the past week D. Age Answer: A Independent risk factors for perioperative SCD-​ related complications include increased patient age (which may relate to the progression of disease and organ dysfunction as patients get older), pregnancy, the nature of the surgical procedure (high, moderate, or low risk), previous hospitalizations for SCD events, and current or recent infection (e.g., urinary tract infection or upper respiratory infection). Interestingly, neither the steady-​state Hb nor the preoperative HbS level are predictive of the perioperative risk. See Table 35.2 for more details. 5. There is clear-​cut evidence that all patients with SCD should be transfused preoperatively if their Hb concentration is below 10 g/​dL. A. True B. False Answer: B The topic of preoperative transfusion is controversial, and different authors disagree about both the need for preoperative transfusion and the transfusion goals. In patients undergoing high-​risk surgery (e.g., on-​pump cardiac surgery), the aggressive strategy is typically used, and either simple or exchange transfusion is performed to decrease the HbS level to less than 30%. For very low-​risk surgeries (e.g., carpal tunnel release), studies have shown that patients can do well without transfusion. For moderate-​ risk procedures (e.g., cholecystectomy), the trend has been to follow the conservative approach and transfuse to a target Hb concentration of 10 g/​dL (or a Hct of 30%; these are used interchangeably). However,

6. A  patient with SCD is admitted to the floor following a left hip arthroplasty for avascular necrosis of the hip. On postoperative day 3, he begins to develop severe left-​sided pleuritic chest pain. His postsurgical pain has previously been well-​controlled but he has been fairly inactive. His most recent vitals show a heart rate of 125 bpm, blood pressure 105/​ 65, respiratory rate 22, and SpO2 86% on room air. His most recent labs showed a Hb of 9.2 g/​dL and HbS of 40%. An electrocardiogram shows sinus tachycardia with right axis deviation. Which of the following constitutes appropriate management?

7. Patients with sickle cell trait (SCT) are at elevated risk of perioperative complications compared with the general population. A. True B. False Answer: B Explanation: Sickle cell trait is the heterozygous carrier state of SCD (i.e., patients produce both HbA and HbS, denoted HbAS). Interestingly, there is some evidence that patients with sickle cell trait have a survival advantage against malaria infection compared with the general population. Sickle cell trait does not cause a marked increase in perioperative morbidity and mortality.

312  •   H ematology

QUESTIONS AND ANSWER S This chapter has accompanying questions and answers which are available to subscribers as part of the Oxford eLearning platform. To access the questions, go to http://​ oxfordmedicine.com/​anesthesiologyPBL REFERENCES 1. Herrick JB. Peculiar elongated and sickle-​shaped red blood corpuscles in a case of severe anemia. Trans Assoc Am Physicians. 1910;25:553. 2. Masom VR. Sickle cell anemia. JAMA. 1922;79:1318–1320. 3. Shapiro ND, Poe MF. Sickle-​ cell disease:  an anesthesiological problem. Anesthesiology. 1955;16:771–780. 4. Firth PG. Anaesthesia for peculiar cells—​a century of sickle cell disease. Br J Anaesth. 2005;95:287–299. 5. Firth PG, Head CA. Sickle cell disease and anesthesia. Anesthesiology. 2004;101:766–785. 6. Benz EJ. Disorders of Hemoglobin. In:  Fauci AS, Jameson JL, eds. Harrison’s Principles of Internal Medicine. 17th ed. New York: McGraw-​Hill; 2008:635–​639. 7. Oprea AD. Hematologic disorders. In:  Stoelting RK, Hines RL, Marschall KE, eds. Stoelting’s Anesthesia and Co-​Existing Disease. 6th ed. Philadelphia: Saunders Elsevier; 2012:411–​412. 8. Platt OS, Brambilla DJ, Rosse WF, et al. Mortality in sickle cell disease: life expectancy and risk factors for early death. N Engl J Med. 1994;330:1639–1644. 9. Serjeant GR, Ceulaer CD, Lethbridge R, et  al. The painful crisis of homozygous sickle cell disease:  clinical features. Br J Haematol. 1994;87:586–591. 10. Vichinsky EP, Haberkern CM, Neumayr L, et  al. A comparison of conservative and aggressive transfusion regimens in the perioperative management of sickle cell disease. N Engl J Med. 1995;333:206–213. 11. Koshy M, Weiner SJ, Miller St, et al. Surgery and anesthesia in sickle cell disease. Blood. 1995;86:3676–3684. 12. Rennels MB, Dunne MG, Grossman NJ, Schwartz AD. Cholelithiasis in patients with major sickle hemoglobinopathies. Am J Dis Child. 1984;138:66–67. 13. Ebert EC, Nagar M, Hagspiel KD. Gastrointestinal and hepatic complications of sickle cell disease. Clin Gastroenterol Hepatol. 2010;8:483–489. 14. Adam S, Jonassaint J, Kruger H, et al. Surgical and obstetric outcomes in adults with sickle cell disease. Am J Med. 2008;121:916–921. 15. Asakura T, Ohnishi T, Schwartz E. Abnormal precipitation of oxyhemoglobin S by mechanical shaking. Proc Natl Acad Sci U S A. 1974;71:1594–1598. 16. Hebbel RP, Vercelloti GM. The endothelial biology of sickle cell disease. J Lab Clin Med. 1997;129:288–293. 17. Platt OS. Sickle cell anemia as an inflammatory disease. J Clin Invest. 2000;106:337–338. 18. Haberkern CM, Neumayr LD, Orringer EP, et al. Cholecystectomy in sickle cell anemia patients:  perioperative outcome of 364 cases from the National Preoperative Transfusion Study. Blood. 1997;89:1533–1542. 19. Steinberg MH. Management of sickle cell disease. N Engl J Med. 1999;340:1021–1030.

20. Henderson AB, Thornell HE. Observations on the effect of lowered oxygen tension on sicklemia and sickle cell anemia among military flying personnel. J Lab Clin Med. 1946;31:769–776. 21. Klinefelter HF. The heart in sickle cell anemia. Am J Med Sci. 1942;203:34. 22. Josephson CD, Su LL, Hillyer KL, Hillyer CD. Transfusion in the patient with sickle cell disease: a critical review of the literature and transfusion guidelines. Transfus Med Rev. 2012;21:118–133. 23. Wanko SO, Telen MJ. Transfusion management in sickle cell disease. Hematol Oncol Clin N Am. 2005;19:803–826. 24. Estcourt LJ, Fortin PM, Trivella M, Hopewell S. Preoperative blood transfusions for sickle cell disease. Cochrane Database Syst Rev. 2016;4:CD003149. doi:10.1002/​14651858.CD003149.pub3 25. Liumbruno G, Bennardello F, Lattanzio A, Piccoli P, Rossetti G. Recommendations for the transfusion of red blood cells. Blood Transfus. 2009;7:49–64. 26. Lottenberg R, Hassell KL. An evidence-​based approach to the treatment of adults with sickle cell disease. Hematol Am Soc Hematol Educ Book. 2005;2005:58–65. 27. Alotaibi GS, Alsaleh K, Bolster L, et  al. Preoperative transfusion in patients with sickle cell disease to prevent perioperative complications: a systematic review and meta-​analysis. Hematology. 2014;19:463–471. 28. Aziz AM, Meshikhes AW. Blood transfusion in patients with sickle cell disease requiring laparoscopic cholecystectomy. J Soc Laparoendosc Surg. 2011;15:480–485. 29. Leff DR, Kaura T, Agarwal T, Davies SC, Howard J, Chang AC. A nontransfusional perioperative management regimen for patients with sickle cell disease undergoing laparoscopic cholecystectomy. Surg. Endosc. 2007;21:1117–1121. 30. Brajtbord D, Johnson D, Ramsey M, et  al. Use of the cell saver in patients with sickle cell trait. Anesthesiology. 1989;70:878–879. 31. You D, Peiro-Garcia A, Ferri-de-Barros F, Parsons D. Hemolysis following intraoperative cell salvage replacement in a scoliosis patient with sickle cell trait: a case report. Spine. 2017; 42:E1331–E1333. 32. Cox JV, Steane E, Cunningham G, Frenkel EP. Risk of alloimmunization and delayed hemolytic transfusion reactions in patients with sickle cell disease. Arch Intern Med. 1988;148:2485–2489. 33. Bakri MH, Ismail EA, Ghanem G, Shokry M. Spinal versus general anesthesia for cesarean section in patients with sickle cell anemia. Korean J Anesthesiol. 2015;68:469–475. 34. Khurmi N, Gorlin A, Misra L. Perioperative considerations for patients with sickle cell disease:  a narrative review. Can J Anaesth. 2017;64:860–869. 35. Buck J, Davies SC. Surgery in sickle cell disease. Hematol Oncol Clin N Am. 2005;19:897–902. 36. Crawford MW, Galton S, Naser B. Postoperative morphine consumption in children with sickle-​ cell disease. Pediatr Anesth. 2006;16:152–157. 37. Dhansura T, Kapadia S, Gandhi S. Dexmedetomidine as sedative and analgesic in a patient of sickle cell disease for total hip replacement. Indian J Anaesth. 2013;57:425–426. 38. Zempsky WT. Evaluation and treatment of sickle cell pain in the emergency department: paths to a better future. Clin Pediatr Emerg Med. 2010;11:265–273. 39. Santos J, Jones S, Wakefield D, Grady J, Andemariam B. Patient controlled analgesia for adults with sickle cell disease awaiting admission from the emergency department. Pain Res Manage. 2016;2016:3218186. doi:10.1155/2016/3218186.

35. S ickle C ell D isease and C holecystectomy  •   313

36. ANESTHESIA IN A PATIENT ON ANTICOAGULANT DABIGATRAN PRESENTING FOR EMERGENCY SURGERY Indu Kapoor, Charu Mahajan, and Hemanshu Prabhakar

S T E M C A S E A N D K EY Q U E S T I O N S A 38-​year-​old male presents to the emergency department at a tertiary trauma center with severe abdominal pain associated with nausea, vomiting, and hematemesis following blunt trauma abdomen in a road traffic accident. There was no history of loss of consciousness, seizures, or ear, nose, and throat bleed. On examination, the abdomen is rigid and tender along with presence of rebound tenderness. There are no other associated injuries present. His computerized tomography brain and cervical spine scans are normal. Chest x-​ray chest is also normal. His abdomen x-​ray shows air in the abdominal cavity. His focused assessment sonography for trauma shows hemoperitoneum and dilatation of the large bowel. He has a history of palpitation, breathlessness, and fatigue in the past. His relative gives a detailed history of his medical condition, which includes being diagnosed as having an irregular heart rate for which he has been receiving dabigatran 150 mg once a day for the past 1 year. His last dose of dabigatran was taken 4 hours before the accident. His vital signs on admission are stable except for tachycardia (heart rate of 110/​ min). His blood investigations are within normal limits except the coagulation profile, which is deranged. The prothrombin time (PT) ratio is 0.8, activated partial thromboplastin time ratio is 0.9, and dabigatran diluted thrombin time ratio of 7 with plasma concentration of 95 ng/​mL. Immediately after receiving the reports, the practitioner administers idarucizumab 5 gm (2.5 gm/​50 mL) through intravenous (IV) route over 30 minutes. A second blood sample is sent to the laboratory, and patient is shifted to the emergency operating room for exploratory laparotomy. WH AT I S DA B I G AT R A N ? WH E R E I S I T US E D F O R ? WH AT A R E OT H E R D I R EC T O R A L A N T I C OAGU L A N TS ( D OAC S )?

Dabigatran etexilate, which is a DOAC, belongs to the newer generation of oral anticoagulant. These drugs directly act by inhibiting a single activated coagulation factor. Dabigatran acts by inhibiting activated factor II (thrombin). Dabigatran is the first Food and Drug Administration (FDA) approved DOAC used for patients with atrial fibrillation to prevent stroke and systemic embolism and other prothrombotic conditions. Its peak plasma level reaches in 1 to 4 hours with half-​life of 7 to 15 hours and renal clearance of 80% to

85%. Other DOACs, including apixaban, rivaroxaban, and edoxaban, act by inhibiting activated factor X (Fig. 36.1). H OW S H O U L D WE MO N ITO R T H E C OAGU L AT I O N S TAT US I N T H I S PAT I E N T ? WH AT A R E T H E T E S TS T H AT C A N B E US E D F O R T H E R A P EU T I C MO N ITO R I N G F O R PAT I E N TS O N D OAC S ?

DOACs usually do not require monitoring for anticoagulation status. However, some clinical scenarios and conditions may warrant the assessment of anticoagulation status that can help in decision-​making, for example in patients on DOACs posted for emergency surgery and clinical conditions like thrombotic events. Routine coagulation tests like PT and activated partial prothrombin time (aPTT) are not reliable measures of the anticoagulant status in patients on DOACs. Few specific tests like diluted thrombin time, ecarin clotting time (ECT) for dabigatran, and specific chromogenic anti-​Xa assays for other DOACs are used to measure anticoagulation status in these patients. However, to date these tests are not FDA approved. S I N C E T H I S I S A N E M E RG E N C Y C A S E A N D T H E PAT I E N T I S S T I L L O N DA B I G AT R A N, T H E R E A R E H I G H E R C H A N C E S O F B L E E D I N G I N T H E P E R I O P E R AT I VE P E R I O D. H OW WI L L YO U P RO C E E D I N M A NAG I N G T H I S S IT UAT I O N ?

Dabigatran (Dabigatran etexilate), also known as reversible anticoagulant, is the only DOAC which has the specific antidote idarucizumab. Idarucizumab is a monoclonal antibody that binds to dabigatran with a 350 times higher affinity than of dabigatran to thrombin. Idarucizumab is useful in patients who are on long-​term treatment with dabigatran who present with life-​threatening bleeding risk following invasive procedure or surgery. It is a valuable drug in an emergent situation like this one. Idarucizumab is available as 2.5 gm/​50 mL concentration given in the dose of 5 gm over 30 minutes. If the patient still bleeds after administration of idarucizumab, the added treatment with hemostatic agents such as factor 3 or 4 prothrombin concentrate complex (3F-​or 4F-​PCC) can be used to control the bleeding. However, the administration of

315

Steps in coagulation

Coagulation pathway

Initiation

Drugs

TF/VIIa

IX

X IXa VIIIa

Rivaroxaban Apixaban Edoxaban YM150 Betrixaban TAK-442

Va Propagation

Xa

Fibrinogen Figure 36.1 

Dabigatran etexilate AZD0837

IIa

Fibrin formation

Fibrin

Sites of action of new anticoagulants in the coagulation cascade.

fresh frozen plasma to reverse dabigatran-​induced bleeding is not recommended. I F T H E S A M E PAT I E N T C O M E S F O R E L E C T I VE S U RG E RY, WH E N S H O U L D WE S TO P DA B I G AT R A N P R E O P E R AT I VE LY ? I S B R I D G I N G WI T H A PA R E N T E R A L A N T I C OAGU L A N T REQUIRED ?

Due to the fact that dabigatran is mainly excreted through the kidney, preoperative interruption of the drug depends on the creatinine clearance (CrCl) of the patient. If CrCl is 50 to 80 mL/​min or >80 mL/​min, the drug should be stopped at least 24 hours before the procedure. If CrCl is between 30 and 50 mL/​min, it should be stopped 48 hours prior, and if CrCl is 40 kg m−2 or a weight of >120 kg, check for a drug-​specific peak and trough level (anti-​FXa for apixaban, edoxaban, and rivaroxaban; ecarin time or diluted thrombin time with appropriate calibrators for dabigatran; or mass spectrometry drug level for any of the DOACs). If the level falls within the expected range, continuation of the DOAC seems reasonable. However, if the drug-​specific level is found to be below the expected range, change to a vitamin K antagonists rather than adjusting the dose of the DOAC. Carcinoma. Patients with cancer-​ associated VTE are at high risk for bleeding and VTE recurrence. Parenteral anticoagulants are the standard of treatment for initial first 3 to 6 months without increasing the risk of bleeding compared to warfarin. DOACs have been used in cancer patients to treat VTE, and it has been suggested that they could be safe and efficacious in this patient population. However, the populations of cancer patients included in the DOAC and LMWH trials are not comparable with regard to mortality and VTE risk, and no specific data from direct head-​to-​head comparisons of DOACs with LMWHs are currently available. Thus by clinical practice, guidelines do not recommend to use of DOACs for the management of VTE in cancer.13 Geriatric. In elderly patients (≥75  years), evidence of dabigatran efficacy is lacking and increased vigilance is warranted. It has been observed that dabigatran was associated with lower rates of intracranial hemorrhage and higher rates of gastrointestinal bleeding when compared to warfarin in elderly patients. Thus dabigatran can offer an alternative to warfarin in elderly patients, with fewer intracranial bleeding events; however, caution is warranted for gastrointestinal bleeding.14 As per the RE-​LY trial, dabigatran is associated with an increased risk of major bleeding and particularly extracranial bleeding in patients ≥75 years.15 According to Ruff et al.,16 pooled NOACs significantly reduced the risk of stroke or systemic embolic events and major bleeding compared to warfarin in patients aged ≥75 years. Pediatric. In recent years, DOACs have gained interest in their potential usage in pediatric patients.17 Limited data is available on the use of DOACs in pediatric patients, and none of them have FDA-​approved pediatric labeling. Off-​ label use of these DOACs in pediatrics is largely extrapolated from adult dosing guidelines. Fortunately, many clinical trials currently are recruiting pediatric patients and are underway to assess DOACs’ effects and efficacy in this special

population. To date, rivaroxaban and dabigatran have the most published data and ongoing trials in pediatric patients compared to edoxaban and apixaban. Dabigatran binds to thrombin directly and is not dependent on the availability of antithrombin, which can vary among patients, specifically pediatric patients.18 Dietrich et  al.19 concluded that diluted thrombin time is the best assay for measuring dabigatran concentration in pediatric patients. The authors concluded that this assay may be necessary to measure dabigatran concentrations until there are more studies looking into the pharmacodynamic and pharmacokinetic effects of this anticoagulant in the pediatric population. Currently, there are 3 phase II trials being conducted on the use of an oral dabigatran liquid formulation in the pediatric population. These studies aim to evaluate the pharmacokinetic/​pharmacodynamic data, safety, and tolerability of this solution in children who are at the end of their standard anticoagulation course for a VTE diagnosis.20–​22 However, it is still too early to make any definitive recommendations on DOAC usage in this special population. Based on the current literature, treatment of VTE in the pediatric population should always be initiated with UFH, LMWH, or warfarin. It is still too early to recommend one of the DOACs as an initial alternative for such indication due to the lack of published clinical trials. More data are needed to determine the appropriate dosing of these agents in different age groups. C O N C LUS I O NS

• At present, dabigatran is the only oral medication in the direct thrombin inhibitor class of anticoagulant. • Dabigatran provides multiple benefits over warfarin as well as over other DOACs in terms of the availability of specific antidote. • Dabigatran use is still in early stages in specific patient populations like patients with cancer or obesity, geriatrics, and pediatrics. DOACs’ efficacy and safety in such patients is still doubtful, so on the basis of current available literature recommendations for their use cannot be made. • Though there is limited clinical trial experience regarding the dosing and effectiveness of these agents, dabigatran-​ associated life-​threatening bleeding should be managed with specific antidote, activated charcoal, and various blood products like 3F-​or 4F-​PCC. • Efficacy data on the specific antidote for dabigatran, idarucizumab, does not exist at present in pediatric patients. It would be important to study this agent in this patient population in the near future. R E VI EW Q U E S T I O N S 1. A  36-​year-​old female weighing 110  kg presents to the emergency department with a history of transient loss of

3 6. Patient on A nticoagulant Dabigatran  •   319

consciousness while sitting on a chair at home. She gives a history of surgery for fracture femur 2 months back under regional anesthesia following which she was bedridden. She has started moving for last few days. On admission her vitals and a computerized tomography imaging of head are normal. All other investigations are within normal limits. Her Doppler study of the bilateral leg shows decreased blood flow. In view of suspected thromboembolism, she is started on dabigatran. What is the therapeutic dose of dabigatran for DVT in this patient? A. 150 mg twice a day B. 150 mg once a day C. 75 mg twice a day D. 75 mg once a day Answer: A As per current available evidence and guidelines provided by the Scientific and Standardization Committee and the International Society on Thrombosis and Haemostasis, in patients weighing £120 kg standard dosing of DOACs should be used. In patients >120 kg, DOACs should not be used as they may decrease their peak concentration and might shorten half-​ lives, which could raise concerns about underdosing in the extremely overweight population. A  literature search showed that no dose adjustment according to body weight has been mentioned. 2. Life threatening bleeding following the use of dabigatran can be managed many ways, which include the specific antidote idarucizumab, activated charcoal, factor 3F-​or 4F-​PCC, and hemodialysis. Which of the methods would NOT be effective to reverse the effect of other DOACs like rivaroxaban, apixaban, and edoxaban? A. Activated charcoal B. Idarucizumab C. 3F-​ or  4F-​PCC D. Hemodialysis E. Both B and D Answer: E Dabigatran is the only DOAC currently which has a specific antidote, idarucizumab; thus this particular antidote cannot be used to reverse the effect of other DOACs. Although literature is lacking in data, hemodialysis can also be considered for removal of dabigatran, and it is usually reserved for life-​threatening bleeding events such as intracranial haemorrhage.5–​7 Hemodialysis, however, is not effective for removing the factor Xa inhibitors like rivaroxaban, edoxaban, and apixaban, which are highly protein bound. 3. A 32-​year-​old man presents to the emergency department with a complaint of light-​headedness. On examination he is found to have an irregular pulse, which is later diagnosed as atrial fibrillation on electrocardiography. A computerized tomography scan of his head shows no significant finding. He is started on dabigatran 150 mg twice a day. Dabigatran is preferred over warfarin as an oral anticoagulant for stroke prevention or systemic embolism in patients with atrial fibrillation for all of the following reasons EXCEPT

A. fewer side effects compared to warfarin. B. minimal drug interactions compared to warfarin. C. more predictable pharmacodynamics and pharmacokinetics. D. dose adjustment is not required in patients with renal disease. Answer: D Eighty-​five percent of dabigatran is excreted through the kidney. CrCl is very much required for dose adjustment, especially if the patient is a known case of renal disease. Dose adjustment of dabigatran for stroke prevention or systemic embolism in a patient with atrial fibrillation is done based on the value of CrCl. If CrCl >30 mL/​min, then the dose is 150 mg twice a day; if CrCl is 15 to 30 mL/​min, the dose is 75 mg twice a day; and if CrCl 85 years old), and increased BMI (30-​35) are all predictors. No specific comorbid illness was associated with an increased likelihood of unanticipated admission. These findings support continued use of the ASA classification as a marker of patient perioperative risk rather than attributing risk to a specific disease process.33 Many common practices in ACSs may have influenced the findings in identifying predictors to unanticipated admissions. BMI >35 patients not being selected for most ambulatory centers may have reduced the sample size of BMI >40 patients. Scheduling parameters placing a limit on the anticipated operative time meant that cases running past 3 hours could be a result of intraoperative surgical complications. Time of day (after 3 pm) was not found to be a predictor because many

4 2 . A nesthesia for C arpa l T unne l S urgery  •   377

ASCs will schedule more complex cases earlier and ahead of shorter, minor procedures. Regardless, the evidence reinforces the importance of patient selection via preoperative screening. Advanced screening will also allow an opportunity to further medically optimize patients for outpatient surgery. M AC

MAC is a common anesthesia technique requested in ambulatory anesthesia because typical outpatient cases are low-​risk surgeries performed on relatively healthy patients (ASA I or II). However, familiarity with this technique can potentially lull the anesthesia provider and surgeon into complacency and decreased vigilance. As the number of ASCs increase, the number of ambulatory surgery procedures also continues to increase. This has resulted in a similar trend; as claims for chronic and acute claim management have increased from the 1980s to the 1990s and 2000s, claims for MAC have also shown an increase over the decades. MAC was the primary anesthetic technique in only 2% of claims for injuries in the 1980s, increasing to 5% in the 1990s and to a full 10% of claims in 2000 and later. Regional anesthesia has continued to represent nearly 20% to 25% of claims in each decade.32 An analysis of surgical anesthesia claims for events from 1990 to 2007 shows a profile of injuries related to MAC that is distinct from general and regional anesthesia claims. Death was the most common outcome in claims associated with MAC, representing 38% of surgical anesthesia claims associated with MAC in 1990–​2007.32 Respiratory depression as a direct consequence of anesthetic overdose was the most specific mechanism of injury and accounted for 21% of MAC claims.32 Inadequate oxygenation/​ ventilation related to sedative/​analgesic overdose contributes to the majority of untoward events. Drug combinations of propofol plus benzodiazepines or opioids were involved in over half of the cases of oversedation. Many of the patients involved were elderly, ASA physical status III–​V, and/​or obese. The care was judged as substandard in the majority of cases and preventable with better monitoring, including pulse oximetry, end-​tidal capnography, or both. Continuous monitoring of ventilation and oxygenation, and vigilance in recognizing imminent respiratory adverse events, are mandatory during MAC cases. Use of end-​tidal capnography to monitor ventilation is particularly important, as oxygen saturation is slow to decrease in the presence of supplemental oxygen. C O N C LUS I O N S

• Carpal tunnel surgery is a common procedure completed in the ambulatory setting. • There are several options available from an anesthetic management standpoint; however, in practice, the best option is one that will achieve the highest patient satisfaction safely, provide optimal surgical conditions for the surgeon, and expedite recovery to home discharge.

• Monitored anesthesia care IV sedation for the injection of local anesthesia by the surgeon and tourniquet discomfort is the predominate technique currently utilized. • Wide-​awake hand surgery is becoming more prevalent; however, this technique would be better performed in a procedure room in an ASC or an office-​based practice versus the required resources in an operating room of an ASC. • Regional anesthesia is an important technique in ambulatory anesthesia that directly contributes to perioperative efficiency, cost-​effectiveness, and overall safety. • Comprehensive patient selection leads to minimizing medical and surgical risk. • Ambulatory anesthesia needs to be adaptable, efficient, and ultimately safe for patients sent home after surgery.

R E VI EW Q U E S T I O N S 1. Which of the following is a characteristic of reperfusion syndrome following tourniquet release? A. Increase in peripheral vascular resistance B. Increase in ETCO2 C. Increase in temperature D. Increase in blood pressure Answer: B Deflation leads to a decrease in mean arterial blood pressure significantly, partly due to the release of metabolites from the ischemic limb into the circulation and the decrease in peripheral vascular resistance. A transient increase in ETCO2, a decrease in temperature, and central venous oxygen tension are seen.13 2. Emergency reversal of digital ischemia secondary to epinephrine overdose and restoring vascular flow is best performed with A. warm water immersion. B. amyl nitrite inhalations. C. topical nitroglycerin paste. D. phentolamine. Answer: D Phentolamine is an alpha adrenergic reversing agent.25 3. Transient neurologic syndrome (TNS) presents which of the following clinical presentations following an intrathecal injection of lidocaine? A. 12–​24 hours following surgery B. Bladder dysfunction C. Motor weakness D. Sensory  loss Answer: A Symptoms of TNS generally manifest within the first 12 to 24 hours after surgery, most often resolve within 3 days, and rarely persist beyond a week. Although self-​limited, the pain

378  •  A mbu l atory

can be quite severe, often exceeding that induced by the surgical procedure, and on rare occasions requiring hospitalization for pain control. Nonsteroidal anti-​inflammatory drugs are often fairly effective and should be used as first-​line treatment. TNS is not associated with sensory loss, motor weakness, or bowel and bladder dysfunction.19

Answer: C Patients at high risk for cardiac events (exclusive of coronary stents) are instructed to continue aspirin throughout the perioperative period, discontinue clopidogrel at least 5  days (and preferably 10  days) before surgery, and resume clopidogrel 24 hours postoperatively.37

4. A patient is in the block bay receiving an ultrasound-​g uided interscalene nerve block for shoulder arthroscopy. He has received midazolam 2 mg and fentanyl 100 mcg for the sedation. Immediately following a 20 mL injection of ropivacaine 0.5%, the patient becomes unresponsive and begins to shake. The first line medication to administer is

8. Following laparoscopic cholecystectomy, a patient with moderate OSA compliant with CPAP should be instructed to continue to use CPAP postoperatively because the risk of apnea is highest

A. propofol. B. lipid emulsion (20%). C. midazolam. D. epinephrine. Answer: B The initial focus is airway management:  ventilate with 100% oxygen ventilation; for seizure suppression, benzodiazepines are preferred; avoid propofol in patients with signs of cardiovascular instability. Alert the nearest facility that has cardiopulmonary bypass capability.27 Epinephrine can impair resuscitation from LAST and reduce the efficacy of lipid rescue. 5. Ultrasound-​g uided brachial nerve blocks, when compared to anatomic landmark and needle stimulation, has resulted in A. reduced vascular puncture and hematoma formation. B. reduced incidence of neurological complication. C. identical block success. D. slower sensory onset. Answer: A The evidence for ultrasound guidance includes faster sensory onset, greater block success, and decreased injected volume of local anesthetics versus the landmark-​based approach. There is no current evidence to suggest that ultrasound reduced the incidence of neurological complications. However, there is a reduction in the incidence of vascular puncture.26 6. All of the following are included in the Post Anesthesia Discharge Scoring System EXCEPT A. ambulation. B. surgical bleeding. C. pain  level. D. voiding. Answer: D Voiding and tolerating of per oral intake are no longer required for discharge from outpatient surgery.19 7. In the perioperative management of patients undergoing antiplatelet therapy, which of the following is an appropriate recommendation? A. Discontinue aspirin. B. Bridge with low molecular weight heparin. C. Discontinue clopidogrel at least 5  days (preferably 10 days). D. Resume clopidogrel 48 hours postoperatively.

A. during the first 12 hours. B. on postoperative day 1. C. on postoperative day 3. D. on postoperative day 7. Answer: C Patients should use the CPAP device whenever they are sleeping for several days as the risk of apnea is highest on the third postoperative day in patients with moderate to severe OSA.38 9. Which of the following is an absolute contraindication to having the procedure performed at an outpatient facility versus a hospital? A. Difficult  airway B. Implantable cardiac defibrillator C. Anticipated >5000 cc liposuction aspirate D. Malignant hyperthermia Answer: C Communication with the surgeon on fluid management is critical, and it is imperative to monitor input and output throughout the case and afterwards. If a large volume liposuction (>5000 cc of aspirate) is planned, the procedure should be performed as an inpatient instead of an ambulatory procedure.39 10. Which of the following medications is the most effective rescue antiemetic? A. 5-​HT3 antagonist B. Anticholinergic C. Steroids D. Antihistamines Answer: D Treatment strategies for breakthrough PONV is important. An antiemetic from a different drug class not previously given should be administered. Additionally, evidence proves that combination therapy is more efficacious than single-​ agent therapy. If no prior antiemetic prophylaxis is given, then 5-​ HT3 antagonist is the first-​line treatment. Second-​line treatment should utilize droperidol, promethazine, small doses of propofol, dimenhydrinate, or dexamethasone.38

QUESTIONS AND ANSWER S This chapter has accompanying questions and answers which are available to subscribers as part of the Oxford

4 2 . A nesthesia for C arpa l T unne l S urgery  •   379

eLearning platform. To access the questions, go to oxfordmedicine.com/​anesthesiologyPBL

http://​ 18. Lodenius Å, Ebberyd A, Hårdemark Cedborg A, et  al. Sedation

with Dexmedetomidine or Propofol Impairs Hypoxic Control of Breathing in Healthy Male Volunteers: A Nonblinded, Randomized Crossover Study. Anesthesiology. 2016;125(4):700–​715. 19. Collins AB, Gray AT, Miller RD, Pardo MC. Basics of Anesthesia. 6th ed. Philadelphia: Elsevier Saunders; 2011:284–​299. REFERENCES 20. Hoffman J. Going Under the Knife, With Eyes and Ears Wide Open. New York Times. 2017:A1. 1. Fleisher LA, Pasternak LR, Lyles A. A novel index of elevated risk of 21. Lalonde D, Martin A. Tumescent local anesthesia for hand surinpatient hospital admission immediately following outpatient surgery: improved results, cost effectiveness, and wide-​awake patient satgery. Arch Surg. 2007;142(3):263–​268. isfaction. Arch Plast Surg. 2014;41(4):312–​316. 2. White PF, White LM, Monk T, et  al. Perioperative care for the 22. Rozanski M, Neuhaus V, Reddy R, Jupiter JB, Rathmell JP, Ring DC. older outpatient undergoing ambulatory surgery. Anesth Analg. An open-​label comparison of local anesthesia with or without seda2012;114(6):1190–​1215. tion for minor hand surgery. Hand. 2014;9(4):399–​405. 3. Joshi GP, Ahmad S, Riad W, Eckert S, Chung F. Selection of obese 23. Davison PG, Cobb T, Lalonde DH. The patient’s perspective on patients undergoing ambulatory surgery: A systematic review of the carpal tunnel surgery related to the type of anesthesia: A prospective literature. Anesth Analg. 2013;117(5):1082–​1091. cohort study. Hand. 2013;8(1):47–​53. 4. Chung F, Abdullah HR, Liao P. STOP-​ bang questionnaire a 24. Ralte P, Selvan D, Morapudi S, Kumar G, Waseem M. Haemostasis in practical approach to screen for obstructive sleep apnea. Chest. Open Carpal Tunnel Release: Tourniquet vs Local Anaesthetic and 2016;149(3):631–​638. Adrenaline. Open Orthop J. 2010;4(0):234–​236. 5. McEvoy RD, Antic NA, Heeley E, et  al. CPAP for Prevention of 25. Lalonde D. Wide Awake Hand Surgery. New York: Thieme Medical Cardiovascular Events in Obstructive Sleep Apnea. N Engl J Med. Publishers; 2011. 2016;375(10):919–​931. 26. Neal JM, Gerancher CJ, Hebl JR, et  al. Upper Extremity Regional 6. Fleisher LA, Fleischmann KE, Auerbach AD, et  al. 2014 ACC/​ Anesthesia. Reg Anesth Pain Med. 2009;34(2):134–​170. AHA Guideline on Perioperative Cardiovascular Evaluation and 27. Neal JM, Bernards CM, Butterworth JF, et  al. ASRA practice adManagement of Patients Undergoing Noncardiac Surgery:  Executive visory on local anesthetic systemic toxicity. Reg Anesth Pain Med. Summary a Report of the American College of Cardiology/​american 2010;35(2):152–​161. Heart Association Task Force on Practice Guidelines. Vol 130; 2014. 28. Capek A, Dolan J. Ultrasound-​g uided peripheral nerve blocks of the 7. Cardiovascular P, Eapci I, France JC, et  al. 2014 ESC/​EACTS upper limb. BJA Educ. 2015;15(3):160–​165. Guidelines on myocardial revascularization. Eur Heart J. 29. Gray A, Laur J. Regional anesthesia for ambulatory surgery: where ul2014;35(37):2541–​2619. trasound has made a difference. Int Anesth Clin. 2011;49(4):13–​21. 8. Jørgensen ME, Torp-​ Pedersen C, Gislason GH, et  al. Time 30. Chan VW, Peng PW, Kaszas Z, et al. A comparative study of general elapsed after ischemic stroke and risk of adverse cardiovascular anesthesia, intravenous regional anesthesia, and axillary block for events and mortality following elective noncardiac surgery. Jama. outpatient hand surgery: clinical outcome and cost analysis. Anesth 2014;312(3):269–​277. Analg. 2001;93:1181–​1184. 9. Committee for Ambulatory Surgical Care. Guidelines for 31. Mariano ER, Chu LF, Peinado CR, Mazzei WJ. Anesthesia-​ Ambulatory Anesthesia and Surgery. 2013:1–​2. controlled time and turnover time for ambulatory upper extremity 10. Arslanian B, Kim DC, Mehrzad R, Kramer T. Forearm Bier Block: A surgery performed with regional versus general anesthesia. J Clin New Regional Anesthetic Technique for Upper Extremity Surgery. Anesth. 2009;21(4):253–​257. Ann Plast Surg. 2013;73(2):1–​2. 32. Metzner J, Posner KL, Lam MS, Domino KB. Closed claims’ analysis. 11. Foster BD, Sivasundaram L, Heckmann N, et al. Surgical Approach Best Pract Res Clin Anaesthesiol. 2011;25(2):263–​276. and Anesthetic Modality for Carpal Tunnel Release. Hand. 33. Whippey A, Kostandoff G, Paul J, Ma J, Thabane L, Ma HK. Predictors 2017;12(2):162–​167. of unanticipated admission following ambulatory surgery:  A retro12. Nielsen KC, Guller U, Steele SM, Klein SM, Greengrass R a, spective case-​control study. Can J Anesth. 2013;60(7):675–​683. Pietrobon R. Influence of obesity on surgical regional anesthesia in 34. O’Donnell BD, Iohom G. Regional anesthesia techniques the ambulatory setting:  an analysis of 9,038 blocks. Anesthesiology. for ambulatory orthopedic surgery. Curr Opin Anaesthesiol. 2005;102(1):181–​187. 2008;21(6):723–​728. 13. Sharma JP, Salhotra R. Tourniquets in orthopedic surgery. Indian J 35. Mulroy MF, Salinas F V, Larkin KL, Polissar NL. Ambulatory surgery Orthop. 2012;46(4):377–​383. patients may be discharged before voiding after short-​acting spinal 14. Vasu TS, Grewal R, Doghramji K. Obstructive sleep apnea syndrome and epidural anesthesia. Anesthesiology. 2002;97(2):315–​319. and perioperative complications:  A systematic review of the litera- 36. Shapiro FE, Punwani N, Rosenberg NM, Valedon A, Twersky R, ture. J Clin Sleep Med. 2012;8(2):199–​207. Urman RD. Office-​based anesthesia:  Safety and outcomes. Anesth 15. American Society of Anesthesiologists. Continuum of Depth of Analg. 2014;119(2):276–​285. Sedation:  Definition of General Anesthesia and Levels of Sedation/​ 37. Horlocker TT, Wedel DJ, Rowlingson JC, et al. Regional Anesthesia Analgesia.; 2014. in the Patient Receiving Antithrombotic or Thrombolytic Therapy. 1 6. American Society of Anesthesiologists. Position on Monitored Reg Anesth Pain Med. 2010;35(1):64–​101. Anesthesia Care.; 2013. 38. Raedar J, Urman RD. Practical Ambulatory Anesthesia. Cambridge: 17. De Oliveira GS, Fitzgerald PC, Hansen N, Ahmad S, McCarthy RJ. Cambridge University Press; 2015. The effect of ketamine on hypoventilation during deep sedation with 39. Hug CC. MAC should stand for Maximum Anesthesia Caution, not midazolam and propofol:  A randomised, double-​blind, placebo-​ Minimal Anesthesiology Care. Anesthesiology. 2006;104(2):221–​223. controlled trial. Eur J Anaesthesiol. 2013:1–​9.

380  •  A mbu l atory

SECTION XIX BARIATRIC ANESTHESIA

43. PERIOPERATIVE CARE OF THE BARIATRIC SURGERY PATIENT John N. Afthinos and Karen E. Gibbs

S T E M C A S E A N D K EY Q U E S T I O N S A 46-​ year-​ old, morbidly obese man (body mass index [BMI]:  53  kg/​m2) with rheumatoid arthritis and obstructive sleep apnea (OSA) on bilevel positive airway pressure (BiPAP) therapy (20/​15  cm H20) presents for elective laparoscopic sleeve gastrectomy for weight loss. He completed the appropriate preoperative nutritional and medical evaluations and was deemed an appropriate candidate. Specifically, he underwent a sleep study diagnosing his OSA and a subsequent titration study demonstrating near resolution of sleep-​ disordered breathing while on noninvasive ventilation.

preoxygenated for more than 4 minutes. After fentanyl and midazolam are given, he undergoes rapid sequence induction with propofol and succinylcholine in anticipation of difficult mask ventilation and possibly a full stomach. A  Bullard laryngoscope is used to visualize the larynx and a 7.0 endotracheal tube is placed without incident. Position is confirmed by auscultation, visible chest rise, and the presence of end-​tidal carbon dioxide on capnography. W H AT I S A B U L L A R D L A RY N G O S C O P E ? W H AT FA C T O R S L E D T O   T H E DECISION OF USING A BULLARD L A RY N G O S C O P E A S   T H E I N T U B AT I N G D E V I C E O F   C H O I C E F O R   T H I S PAT I E N T ? W H AT OT H E R I N T U B AT I N G ADJUNCTS OR APPROACHES COULD BE USED ?

WH AT I S T H E S I G N I F I C A N C E O F O S A I N T H E P E R I O P E R AT I VE M A NAG E M E N T O F A M O R B I D LY O B E S E PAT I E N T U N D E RG O I N G A G E N E R A L A N E S T H ET I C ?

In the preoperative holding area, he is found to have difficult intravenous (IV) access. After several attempts, the decision is made to insert a midline catheter in the left (nondominant) arm under ultrasound guidance. WH AT O P T I O N S E X I S T F O R I V AC C E S S I N E L E C T I V E C A S E S ? WH AT D EVI C E S A R E AVA I L A B L E TO A S S I S T I N O BTA I N I N G AC C E S S I N D I F FI C U LT C I RC UM S TA N C E S ?

On preoperative evaluation, the patient is 61.5  inches (156 cm) and weighed 287 lbs. (130.5 kg). His neck circumference is 16.5 inches (42 cm) and he has a full beard. He has a Mallampati IV airway, with a thyromental distance greater than three finger breadths. His teeth are protuberant and he exhibits trismus with a resultant small mouth opening of 2 cm. WH AT FE AT U R E S I N T H E PAT I E N T ’S E X A M I NAT I O N A R E C O N C E R N I N G F O R A P OT E N T I A L LY D I F F I C U LT A I RWAY ? WH AT P R E C AU T I O N S S H O U L D B E TA K E N I N M A NAG E M E N T ?

In the operating room, the patient is prepared for induction by creating a ramp to position his upper torso. He is

An esophageal stethoscope is placed in the axilla to monitor intraoperative temperature. Careful communication is maintained between the operating surgeon and anesthesiologist to identify the bougie before and during stapling of the stomach to avoid inadvertent transection. Any movement of the bougie is performed with both the surgeon and the anesthesiologist visualizing it on the monitor. The procedure is uneventful. WH AT P OT E N T I A L I S S U E S C A N A R I S E D U R I N G T H E G A S T R EC TO MY ? WH Y I S VI G I L A N C E D U R I N G T H I S T I M E BY T H E A N E S T H E S I O L O G I S T C R IT I C A L ?

Given his challenging airway, emergence and extubation criteria include alertness, following verbal commands, sustained 5 second head lift, and the presence of 4 twitches on the twitch monitor. In addition, the endotracheal cuff is deflated and audible respirations are heard from the pharynx (a positive cuff-​leak test). The patient is successfully extubated and taken to the postanesthesia care unit with BiPAP at bedside. He is placed in a monitored setting, as is an institutional protocol for such patients, and discharged the next day as expected.

383

WH AT S T R AT EG I E S E X I S T F O R A S A FE E M E RG E N C E I N T H E S E PAT I E N T S ? WH AT A P P ROAC H E S A R E AVA I L A B L E TO R E D U C E P O S TO P E R AT I VE NAUS E A A N D VO M IT I N G ( P O N V ) I N T H E S E PAT I E N TS ?

DISCUSSION M O R B I D O B E S IT Y A N D  O S A

Morbid obesity is an increasingly prevalent condition. Typically, obesity is expressed as BMI where weight in kilograms is divided by height in meters squared. It is estimated that nearly 7% of the United States population is morbidly obese.1 There are multiple comorbid conditions which are directly related to obesity that decrease life expectancy and quality of life. Bariatric surgery has become commonplace due to its ability to significantly ameliorate obesity-​related diseases, improving mortality rates and quality of life. In 2015 alone, about 193,000 bariatric surgeries were performed in the United States.2 OSA is prevalent in up to 80% of morbidly obese patients.3,4 OSA significantly increases the risk of difficult mask ventilation due to disordered pharyngeal airway tone and altered anatomy from increased adipose tissue.4,5 These patients also desaturate quickly when apneic. Rapid desaturation occurs secondary to underlying decreased chest wall and lung compliance.5,6 OSA is diagnosed via sleep study, and the apnea hypopnea index (AHI) is used to for stratification into mild, moderate, and severe. AHI is defined as the number of apnea (of more than 10 seconds) or hypopnea events per hour of sleep. Mild OSA is characterized by up to 5 AHI episodes; moderate OSA is characterized by 6 to 15 events; and greater than 15 events is considered severe OSA. Treatment can involve a mouth guard to adjust jaw position Table 43.1  STOP-​B ANG QUESTIONNAIRE STOP Do you Snore loudly (loud enough to be heard through closed doors)? Do you often feel Tired, sleepy, or fatigued during the day? Has anyone Observed you stop breathing while sleeping Do you have or are you being treated for high blood Pressure? Bang BMI ≥35kg/​m2? Age ≥50? Neck circumference>16 inches(40 cm)? Gender: Male? BMI = body mass index. Adapted from Nagappa M, Wong J, Singh M, Wong DT, Chung F. An update on the various practical applications of the STOP-​Bang Questionnaire in anesthesia, surgery, and perioperative medicine. Curr Opin Anesthesiol. 2017;30:118–​125 with permission.

and maintain patency of the airway or noninvasive positive pressure ventilation.2 If concern exists for OSA, and there has been no sleep study to make the diagnosis, the patient can be screened using the STOP-​Bang questionnaire. This questionnaire enables the rapid screening of patients who are at risk for OSA and can enable the anesthesiologist to appropriately care for the patient in the perioperative period (Table 43.1).7 A meta-​analysis of multiple studies evaluating the questionnaire reported a sensitivity of 91% in surgical patients but only 32% specificity.8 OSA is a significant risk factor for difficult mask ventilation, and advanced preparation for airway management is important to maintain patient safety.9 Strategies to achieve a safe and successful intubation with this condition include preoxygenation, proper positioning and use of airway adjuncts. Administering 100% oxygen via a tight-​fitting mask for at least 4 minutes can increase apnea tolerance.4 Proper positioning is also paramount. Placing the patient in a “beach-​ chair” position or ramping improves the respiratory dynamics during induction of general anesthesia and can improve the Cormack-​Lehane grade that can be achieved during direct laryngoscopy.10 Ramping aligns the external auditory meatus with the sternal notch. By doing so, the anesthesiologist can align the laryngeal, pharyngeal, and oral axes, which is desirable for successful intubation.11 Lastly, if patients are to receive a patient-​ controlled analgesia device postoperatively, it is recommended that they are in a monitored setting, preferably with their own BiPAP device at bedside for use. I V AC C E S S

Intravenous (IV) access in the morbidly obese patient can present a challenge. Studies evaluating difficult IV access among patients have identified obesity (BMI >30  kg/​m2), underweight (BMI 90°

0

~90°

1

5 cm or Slux >0

0

IO 250 beats/​min and VF are defibrillated. Unlike external defibrillation which often requires 200 joules of energy, AICDs tend to deliver approximately 30 joules for internal defibrillation. Defibrillation is delivered by shock coils. AICDs can have a single right ventricle lead, dual chamber leads (Right Atrium and right ventricle), and integration of biventricular pacing (right atrium, right ventricle, and coronary sinus leads). In addition to the often encountered transvenous AICD, wearable and subcutaneous cardio defibrillators are also available.5 P E R I O P E R AT I VE M A NAG E M E N T O F A I C D S

The generic code of AICDs was developed by the North American Society of Pacing and Electrophysiology and the British Pacing and Electrophysiology Group. This is outlined in Table 44.4.6 Preoperative evaluation and planning should necessarily include:

Figure 44.1 

AICD components.

1. Date of last interrogation: within 6 months for AICDs and 12 months for pacemakers is acceptable

4 4 . Patients with Automatic I mp l antab l e C ardio D efibri l l ator  •   395

Table 44.4  CLASSIFICATION OF DEFIBRILLATORS POSITION I: SHOCK CHAMBERS

POSITION II: ANTITACHYCARDIA PACING CHAMBER

O: None, A: Atrium, V: Ventricle, D: Dual; A + V

O: None, A: Atrium, V: Ventricle, D: Dual; A + V

2. Indication for placement

POSITION III: TACHYCARDIA DETECTION

E:Electrocardiogram H: Hemodynamic

POSITION IV: ANTIBRADYCARDIA PACING CHAMBER

O: None, A: Atrium, V: Ventricle, D: Dual; A + V

4. Does the AICD need to be disabled? If yes, how is this done?

3. Manufacturer, model, and type

5. What kind of monitoring does the patient need?

4. Battery longevity

6. Do we need to have a plan for external defibrillation?

5. Any leads placed within the last 3 months

7. Does the patient need the AICD checked postoperatively?

6. Current programming 7. Pacemaker dependency 8. Last pacing threshold if applicable 9. Magnet response 10. Surgical considerations such as site of surgery (above or below the umbilicus), generation of electromagnetic interference (EMI) from use of electrosurgical unit (ESU) or “bovie,” and site of placement of the dispersive pad or bovie pad. T H E I M P O RTA N C E O F E M I

Electromagnetic fields generated by external sources in the operating room can interfere or disrupt the normal functioning of AICDs. EMI is most commonly generated by ESUs. Both monopolar and bipolar electrosurgery can cause EMI, but the risk is highest with the use of monopolar instruments. EMI is also generated in the presence of radiofrequency ablation devices, lithotripsy, and magnetic resonance imaging. EMI is detrimental as it can lead to inappropriate shocks or overdrive pacing. This happens when the EMI is misinterpreted by the AICD as a tachyarrhythmia. Direct damage to the pulse generator disrupting its ability to deliver any treatment is also possible. Additionally, if the patient is pacemaker dependent, the misinterpretation of the EMI as tachycardia leads to underpacing and can culminate in profound bradycardia or asystole. Thus it is important to minimize the effects of EMI on AICDs in the perioperative period.4,6–10 I N T R AO P E R AT I V E M A NAG E M E N T O F A I C D S

The following questions need to be addressed in the intraoperative period: 1. Where is the surgical site? 2. Will there be EMI from electrosurgery? 3. What can be done to minimize the effects of EMI?

Surgical site helps determine if the AICD needs to be deactivated. Deactivation of the AICD refers to switching off the tachycardia detection algorithm and antitachycardia function to avoid inappropriate shocks. Such deactivation can be achieved by reprogramming by the CIED team. In an emergency, placement of a magnet over an AICD may result in deactivation. However, it should be noted that magnet placement does not alter pacemaker function of the AICD and will not result in asynchronous pacing. Typically, surgeries that are performed below the umbilicus do not need reprogramming or deactivation of the AICD. Deactivation is not recommended during such surgeries as risk of AICD malfunction is minimal. That said, it is still imperative to place the electrocautery grounding pad or the electrosurgery receiving plate in such a manner that the current path does not pass near or through the AICD. If possible the distance between the AICD and the receiving plate (grounding pad) should be at least 15 cm (6 inches) or more. For surgeries that take place above the umbilicus, deactivation of the AICD is reasonable, but the Cardiovascular implantable electronic device (CIED) team should be contacted for a perioperative strategy for device management as a single generalized recommendation may not be appropriate for all patients. Additionally, if the patient is pacemaker dependent and is undergoing a supraumbilical surgery that entails use of monopolar cautery, consideration should be given to reprogramming the pacemaker to an asynchronous mode. If EMI from monopolar cautery is expected, the surgeons should be encouraged to use it in short, intermittent, and irregular bursts at the lowest feasible energy. This helps minimize detrimental effects on functioning of the AICD. Additionally, as mentioned, the “grounding pad” should be appropriately situated so that the current path does not include the AICD. Consideration to use of bipolar cautery or ultrasonic scalpel as an alternative to using monopolar cautery is reasonable. Oftentimes this is not possible, as monopolar cautery can be employed for both cutting and coagulation while bipolar cautery typically works best for achieving coagulation limiting its utility. In monopolar cautery, the current is delivered by the instrument’s electrode to the tissue and then

396  •   M isce l l aneous

the current exits the body along the path of least resistance to the ground or receiving plate. Tissue effects of the current are maximum closest to the delivering electrode and diminish distally as current density decreases distally. In bipolar cautery, the current is confined to both the electrodes incorporated into the instrument. Thus EMI is typically not a problem with bipolar cautery use. Intraoperatively, anesthesiologists should focus on 3 goals in patients with AICDs. 1. Establish continuous electrocardiographic monitoring. 2. Establish monitoring of mechanical output of the heart: This is done by continuous palpation of the pulse and plethysmography from pulse oximetry or invasive arterial monitoring. Continual monitoring of blood pressure via noninvasive or invasive means is also emphasized. 3. Prepare for AICD failure or malfunction. Despite best efforts, sometimes things do not go as planned. In case the AICD fails, there should be availability of back-​up means for external defibrillation or cardioversion. Three scenarios are possible: Scenario 1: The AICD was never deactivated. Scenario 2: Deactivation was achieved by placing a magnet over the AICD. Scenario 3: Deactivation was achieved by reprogramming the device. If the patient develops an unstable rhythm needing cardioversion or defibrillation, the surgery team should be instructed to cease using all sources that can generate EMI. If Scenario 1 is applicable, and the AICD does not deliver the shock as it is supposed to, external pads are placed in an anteroposterior or right infraclavicular-​left apical location for delivering appropriate therapy. As far as possible, external pads should not be placed directly over the pulse generator of the device. In Scenario 2, the magnet is removed, and, if the AICD fails to deliver appropriate therapy, consider external cardioversion or defibrillation as appropriate. In Scenario 3, if immediate reprogramming or activation of AICD is not possible, it is prudent to rapidly move onto external cardioversion or defibrillation.4,6–10 P O S TO P E R AT I V E M A NAG E M E N T O F A I C Ds

Postoperatively, patients with AICDs should be monitored with continuous electrocardiography and pulse oximetry. Back-​up pacing and defibrillation should be available in postoperative recovery areas. All antitachyarrythmia functions should be restored. CIED team should interrogate the device postoperatively if it was deactivated intraoperatively or EMI interference led to significant intraoperative events such as need for cardiopulmonary resuscitation, external pacing, cardioversion, or defibrillation. If the AICD was never deactivated or there were no intraoperative events with respect to the AICD, it is prudent that patients have their AICD

interrogated within 1 month especially if monopolar electrosurgery was employed during the procedure.4,6 R E VI EW Q U E S T I O N S 1. A patient presents for aortofemoral bypass surgery. He has severe chronic obstructive pulmonary disease and is on home oxygen 2l/​min round the clock. Echocardiogram is suggestive of moderate pulmonary hypertension and his LVEF is 25%. He has an AICD in place. He also has a history of recall under general anesthesia. Which of the following is the LEAST appropriate intraoperative intervention? A. I:E ratio of 1: 3 B. Tidal volume of 7 mL/​kg predicted body weight C. Maintenance of normothermia D. Disabling antibradycardia function of the AICD E. Use of bispectral index (BIS) or monitor in addition to using clinical signs to assess depth of anesthesia Answer: D An AICD has 2 components: a defibrillator and a pacemaker. Antitachycardia function is disabled for surgeries above the umbilicus that require the use of monopolar cautery as electrical interference from cautery can cause the defibrillator to shock inappropriately or malfunction. Antibradycardia is a function of the pacemaker and is never turned off. In pacer-​ dependent patients it is reasonable to reprogram the pacemaker to an asynchronous mode in supraumbilical surgeries. Use of tidal volumes 6 to 8 mL/​kg and achieving normothermia intraoperatively are reasonable goals. Although use of BIS is not mandatory, it may be reasonable to employ it in patients with history of recall under anesthesia. Importantly, BIS monitoring does not replace assessment of clinical signs to gauge depth of anesthesia. 2. Which of the following is NOT an indication for placing an AICD? A. Arrhythmogenic RV dysplasia B. High-​ risk patients with long Q-​ T syndrome:  QT >500 ms C. Hypertrophic obstructive cardiomyopathy D. Brugada syndrome E. Sick sinus syndrome Answer: E While options A  to D are established indications, patients with sick sinus syndrome are candidates for pacemakers due to symptomatic bradycardia but do not need an AICD. 3. A  patient has severe aortic stenosis. He has never been hospitalized due to his HF. His EF is 60%. He complains of shortness of breath with ordinary activity. He is currently on beta-​blockers and diuretics. Left heart catheterization done last month was suggestive of nonobstructive coronary artery disease. Which of the following is true? A. His NYHA class is I. B. His HF can be classified as HFrEF. C. He would benefit from oral hydralazine.

4 4 . Patients with Automatic I mp l antab l e C ardio D efibri l l ator  •   397

D. He has Stage C HF. E. He needs coronary artery bypass grafting. Answer: D His NYHA class is II. His HF would be classified as HFpEF. Hydralazine as a vasodilator may be harmful in a patient with aortic stenosis. He does not need bypass grafting as he has nonobstructive coronary artery disease. His HF is best staged as Stage C. 4. A patient with an AICD-​pacemaker is coming in for bilateral mastectomy. The surgeon wants to use monopolar electrosurgery/​cautery. Which of the following is FALSE? A. The surgeon should be encouraged to use the monopolar cautery in short bursts. B. Back-​up pacing and defibrillation equipment should be ensured prior to proceeding for surgery. C. Antitachycardia function of the AICD should be deactivated. D. Rate responsiveness of the pacemaker should be deactivated. E. A magnet should be placed on the AICD to convert pacing mode to asynchronous pacing. Answer: E Monopolar cautery carries a high risk of generating EMI and disrupting AICD function. It should be used in short, intermittent, and irregular bursts. Back-​up pacing and defibrillation should be ensured when EMI is anticipated. AICD deactivation and turning off rate responsiveness of the pacemaker are recommended when monopolar cautery is going to be used in the vicinity of the AICD during supraumbilical procedures. Magnet placement over AICD deactivates the antitachycardia function and does not affect the pacemaker at all. 5. Which of the following statements about modern AICDs is FALSE? A. Modern AICDs have pacing capabilities. B. Modern AICDs cannot be configured to deliver cardiac resynchronization therapy. C. Subcutaneous AICDs can be placed in patients at high risk for bacteremia. D. Wearable cardioverter defibrillator vests are available. E. A  pulse generator is an important component of an AICD. Answer: B Modern AICDs, in addition to providing antitachycardia therapy, can deliver pacing and resynchronization therapy. Patients at high risk for bacteremia (chronic venous indwelling catheters such as hemodialysis catheters) may benefit from use of a subcutaneous AICD. Wearable cardioverter defibrillators are typically used to provide short-​term protection from ventricular fibrillation or sudden cardiac death in patients awaiting a cardiac transplant or transvenous AICD implantation. AICDs typically consist of electrodes and a pulse generator. 6. The patient had his AICD deactivated prior to thoracic surgery by the CIED team. Standard ASA monitors are connected and a single lumen endotracheal cuffed tube is

placed uneventfully. He develops sudden cardiac arrest from VF prior to incision. Which of the following is true? A. Chest compressions are contraindicated in the presence of an AICD as it may damage the AICD. B. He should be ventilated at 10 to 12 breaths/​min and provided with high positive end-​expiratory pressure of 15 cm of water. C. Two breaths should be delivered for every 30 chest compressions. D. A magnet should be placed over the AICD to activate its antitachycardia functions. E. External defibrillation should be provided as soon as possible. Answer: E In patients with AICDs who arrest due to VF, chest compressions can be safely performed. Patients with an endotracheal tube in place should be ventilated at a rate of 10 to 12 breaths/​min irrespective of number of chest compressions delivered. Chest compressions should be continuous and delivered at a rate of 100/​ min. External defibrillation is recommended as soon as possible to restore cardiac rhythm if the AICD’s antitachycardia functions cannot be restored immediately. Magnet placement over the AICD deactivates its antitachycardia response. 7. The patient is Question 6 is successfully resuscitated and extubated. He is now in the postanesthesia recovery unit (PACU) and doing well. Which of the following is FALSE? A. His AICD needs to be interrogated by the CIED team. B. The CIED team needs to restore antitachycardia function of the AICD. C. Back-​up pacing and defibrillation equipment should be available for this patient until the CIED team restores AICD functions. D. Standard ASA monitoring including electrocardiography, pulse oximetry, and noninvasive blood pressure monitoring are reasonable to perform in the PACU. E. General anesthesia can affect functioning of the AICD and lead to AICD failure. Answer: E Type of anesthesia does not influence AICD function. Patients who have had their AICD deactivated preoperatively need reactivation and interrogation of their device postoperatively before discharge. The same holds true for patients who experience a significant intraoperative event such as a shock or arrest, irrespective of whether their AICD was deactivated preoperatively. AICD patients should have standard ASA monitoring in the PACU. 8. Which of the following is safest for use in patients with AICDs? A. Computed tomography B. Monopolar cautery C. Lithotripsy D. Magnetic resonance imaging

398  •   M isce l l aneous

E. Radiofrequency ablation Answer: A While options B to E typically generate EMI that may disrupt AICD function, computed tomography is considered safe for use in these patients. 9. AICDs are indicated in all of the following EXCEPT A. Cardiac arrest resulting from VT/​VF not resulting from a transient or reversible cause. B. Patient with incessant VT or VF. C. Patients with a prior MI and LVEF ≤30%. D. Spontaneous sustained VT with structural heart disease. E. Any cardiomyopathy (EF ≤35%) with NYHA Class II or III HF symptoms. Answer: B Option B is not an indication for AICD as these patients typically need catheter ablation for termination of incessant VT or VF. 10. A  patient presents for elective cystoscopy under general anesthesia. He has an AICD and is pacer dependent. His last AICD check was 9 months ago. The surgeon is not planning to use monopolar cautery. Which of the following is true? A. The AICD should be disabled. B. The pacemaker should be programmed for asynchronous pacing. C. AICD interrogation should be performed prior to surgery. D. Continuous intraoperative electrocardiogram monitoring is not needed in this patient. E. Continuous intraoperative pulse oximetry monitoring is not needed in this patient. Answer: B As the last AICD check was 9  months ago, it should be interrogated prior to surgery. AICDs should be interrogated every 6 months and pacemakers every 12 months.

QUESTIONS AND ANSWER S This chapter has accompanying questions and answers which are available to subscribers as part of the Oxford eLearning platform. To access the questions, go to http://​ oxfordmedicine.com/​anesthesiologyPBL REFERENCES 1. Yancy CW, Jessup M, Bozkurt B, et al. 2013 ACCF/​AHA guideline for the management of heart failure: a report of the American College of Cardiology Foundation/​American Heart Association Task Force on Practice Guidelines. Circulation.2013;128:e240–​e327. 2. Moss AJ, Zareba W, Hall WJ, et al. Prophylactic implantation of a defibrillator in patients with myocardial infarction and reduced ejection fraction. N Engl J Med. 2002;346:877–​883. 3. Cleland JG, Daubert JC, Erdmann E, et  al. The effect of cardiac resynchronization on morbidity and mortality in heart failure. N Engl J Med. 2005;352:1539–​1549. 4. Crossley GH, Poole JE, Rozner MA, et al. The Heart Rhythm Society (HRS)/​American Society of Anesthesiologists (ASA) expert consensus statement on the perioperative management of patients with implantable defibrillators, pacemakers and arrhythmia monitors: facilities and patient management. Heart Rhythm.2011;8:1114–​1154. 5. DiMarco JP. Implantable Cardioverter-defibrillaor. N Engl J Med. 2003;349:1836–1847. 6. Practice Advisory for the Perioperative Management of Patients with Cardiac Implantable Electronic Devices: Pacemakers and Implantable Cardioverter-Defibrillators: An Updated Report by the American Society of Anesthesiologists Task Force on Perioperative Management of Patients with Cardiac Implantable Electronic Devices. Anesthesiology. 2011;114(2):247–261. 7. Godin JF, Petitot JC. STIMAREC report. Pacemaker failures due to electrocautery and external electric shock. Pacing Clin Electrophysiol. 1989;12:1011. 8. Mangar D, Atlas GM, Kane PB. Electrocautery-​induced pacemaker malfunction during surgery. Br J Anaesth. 1991;38:616–​618. 9. Wong DT, Middleton W. Electrocautery-​induced tachycardia in rate-​ adaptive pacemaker. Anesthesiology. 2001;94:710–​711. 10. Rozner M. Pacemaker misinformation in the perioperative pe riod:  programming around the problem [comment]. Anesth Analg. 2004;99:1582–​1584.

4 4 . Patients with Automatic I mp l antab l e C ardio D efibri l l ator  •   399

45. ANESTHETIC CONSIDERATIONS IN AUTONOMIC HYPEREFLEXIA Roshan Patel and Suzette Singh

S T E M C A S E A N D K EY Q U E S T I O N S An 83-​year-​old man from a nursing home with a medical history of type 2 diabetes, spinal cord injury at T6 after a motor vehicle accident resulting in paraplegia, benign prostatic hypertrophy, and frequent kidney stones presents for cystoscopy and lithotripsy. He has a history of constipation requiring laxatives as needed and enemas. He weighs 86 kg and his basic chemistry panel reveals a creatinine of 1.4. WH AT A R E YO U R A N E S T H ET I C C O N S I D E R AT I O N S ?

Anesthetic management of patients with spinal cord lesions should include consideration of autonomic hypereflexia (AH), as it can be triggered by a surgical stimulus. Regional anesthesia and general anesthesia are effective in the prevention of AH. Regional anesthesia, such as neuraxial procedures, should be considered in patients that appear to be insensate below the level of the lesion. While noninvasive blood pressure measurement can be used, direct arterial measurement should be prioritized and arterial vasodilators and alpha adrenergic blocking agents should be readily available. Body temperature should be monitored carefully as chronic vasodilation and loss of normal cutaneous vasoconstriction can predispose patients to hypothermia. In general, when caring for patients with spinal cord injury within the past 6 months, succinylcholine should be avoided because of the risk of hyperkalemia (even outside the setting of AH).

stimuli from lower segments of the spinal cord can result in hypertension and an elevated heart rate via the sympathetic nervous system and spinal reflexes. The central nervous system then is able to modulate this physiologic response via peripheral baroreceptors and an intact parasympathetic vagus nerve. Intact disinhibition from the central nervous system transmits parasympathetic outflow to the vagus nerve and inhibitory spinal cord nerves to permit the reduction in heart rate as well as peripheral vasodilation as a response to systemic hypertension.1,3 In essence, a sympathetic discharge will be controlled by a central disinhibition via activation of the parasympathetic nervous system (see Figs. 45.1 and 45.2). The interruption of this communication is precisely what results in AH. Autonomic hypereflexia is a condition that results when a strong sensory input is transmitted to the spinal cord via peripheral nerves (typically from visceral organs such as bowel or bladder). The noxious stimulus input is transmitted up the spinal cord resulting in a sympathetic surge from the thoracolumbar sympathetic chain and leads to an initial widespread vasoconstriction and peripheral arterial hypertension. Brain

Carotid & aortic baroreceptors Vagus nerve (CN X) Heart Spinal cord injury

WH AT I S T H E N EU RO PAT H I C ET I O L O GY O F A H ?

The autonomic nervous system is divided by the sympathetic and parasympathetic nervous systems, which carry motor and sensory information to the visceral organs and glands of the body. In short, the sympathetic system diverts perfusion from the digestive organs to the skeletal muscles and lungs while also causing bronchodilation and increased myocyte contractility to facilitate the “fight or flight” response. The parasympathetic system opposes the sympathetic system via negative feedback and results in a collective facilitation of energy conservation and nutrient absorption.1 In an individual with an intact central and peripheral nervous system, noxious

Diaphragm BP Skin breakdown

Fecal impaction Figure 45.1 

Splanchnic blood vessels Full bladder

Lack of central inhibition of sympathetic nervous system below spinal cord injury. (http://​emedicine.medscape.com/​article/​322809-​ overview)  Have your received permission to use the copyrights of these images/​

401

SYMPATHETIC Sympathetic Ganglia

PARASYMPATHETIC Constricts Pupils

Dilates Pupils Inhibits Salivation

Stimulates Salivation Decreases Heart Rate Bronchial Dilation Increases Heart Rate

Decreases Contractility

Inhibits Digestion Increases Contractility

Stimulates Glucose Release by Liver

Bronchial Constriction Stimulates Digestion

Stimulates Epinephrine & Norepinephrine Release

Stimulates Gallbladder Contracts Bladder Relaxes Rectum

Relaxes Bladder Contracts Rectum Orgasm Ejaculation

Vaginal Lubrication Erection

Peripheral Vasoconstriction

Peripheral Vasodilation

The autonomic nervous system. (https://​www.healthrising.org/​blog/​2013/​10/​31/​dysautonomia-​101-​lauren-​stiles-​chronic-​fatigue-​syndrome-​ fibromyalgia-​2/​) Figure 45.2 

Next, the brain detects the hypertensive crisis via intact carotid baroreceptors and attempts to modulate the blood pressure by vasodilating via descending inhibition; however, this central inhibitory communication is never received below the lesion due to an interruption at the level of the spinal cord lesion. (Note that there may be hypotension above the level of the lesion because these segments of the spinal cord are still intact.) The brain also attempts a second maneuver to lower the blood pressure via a reflexive bradycardia (see Fig. 45.3) through an intact parasympathetic vagus nerve; however, this resulting bradycardia is typically inadequate to lower the systemic blood pressure.2,4,5 Cardiac arrhythmias have also been reported. In short, the parasympathetic input is maintained above the lesion, while the sympathetic input remains unregulated below the level of the spinal cord lesion due to lack of inhibition from above (see Fig. 45.1). The patient in the stem case in this chapter may have a high, normal, or even low direct blood pressure reading if the blood pressure cuff is on an upper extremity as his lesion is T6. Remember to be mindful of bronchospasm and bradyarrhythmias from the predominance of the parasympathetic system above the lesion. The intact baroreceptors in the carotid sinus and the aortic arch convey appropriate responses to systemic hypertension through the petrosal ganglion to the

central nucleus ambiguous resulting in vagal outflow, bradycardia, and vasodilation above the spinal cord injury.2,3 In essence, the central nervous system is blind to the strong noxious stimuli below the spinal cord injury and the descending inhibition only makes it as far as the neurologic lesion will allow. In this patient, the systemic hypertension will prevail below his spinal cord injury at the level of T6. Other signs indicative of AH are flushing of the skin and diaphoresis above the injury, pupillary constriction, and nasal congestion as a result of unopposed parasympathetic outflow. Below the lesion, his skin will be pale, cold, and have piloerection due to the predominance of sympathetic tone and lack of parasympathetic negative feedback.5,7 This patient’s thoracic cord lesion at level T6 is of particular significance in the pathogenesis of AH since the splanchnic vascular system is one of the largest reserves of intravascular volume and mediated primarily by the splanchnic nerve, which is derived from T5–​T9. Therefore, lesions to the spinal cord at or above T6 permit strong unopposed sympathetic tone to the splanchnic vascular bed resulting in severe systemic hypertension. Had the lesion been below the level of T9, this would generally permit enough descending parasympathetic communication to modulate the splanchnic tone and control the peripheral hypertensive response.6,7

402  •   M isce l l aneous

Cerebral Cortex / Insula

Hypothalamus

Medullary Nuclei Involved in Cardiovascular Control

Vasculature

T1-T5

Descending sympathetic pathways

Vagus nerve Heart

T1-L2

Schematic diagram of central nervous system on the cardiovascular system. (http://​www.nature.com/​sc/​journal/​v44/​n6/​ full/​3101855a.html). The central nervous system (cerebral cortex and hypothalamus) communicate to the various nuclei within the medulla oblongata, where cardiovascular autonomic control is mediated. The parasympathetic pathways are transmitted to the peripheral nervous system via the vagus nerve to the cardiovascular system. The preganglionic fibers of the vagus nerve synapse with postganglionic parasympathetic neurons in ganglia on or near the target organ (the heart and the systemic vasculature) in order to mediate reflex bradycardia as a response to systemic hypertension. Descending sympathetic pathways provide tonic control to sympathetic pregangiolic neurons within spinal cord segments T1–L2. They then synapse with postganglionic neurons located in the sympathetic chain (paravertebaral ganglia). Finally, the sympathetic postganglionic neurons synapse with the target organs, the heart, and blood vessels to exhibit their target effects. Figure 45.3 

WH AT OT H E R C O N S I D E R AT I O N S S H O U L D B E M A D E A L O N G WIT H T H E D I F F E R E N T I A L D I AG N O S I S O F AU TO N O M I C DYS R E F L E X I A ?

Carcinoid syndrome, pheochromocytoma, thyroid storm, renal artery stenosis, and familial hypertension are other possibilities that should be considered in the setting of life-​ threatening hypertension. It should also be noted that, if left untreated, AH may result in seizures, retinal hemorrhage, fulminant pulmonary edema, intracranial hemorrhage, myocardial ischemia, renal insufficiency, and death.1,6 WH AT A R E T H E T R I G G E R S O F A H ?

AH is thought to be triggered by afferent stimuli that originate below the level of the spinal cord lesion. It is believed that this afferent input initiates and sustains an elevation in systemic vascular resistance via sympathetically mediated vasoconstrictors in muscle, skin, and splanchnic vascular beds.

The obstetric patient in labor is also at risk for developing AH in the setting of spinal cord injury. Essentially, any irritation or pain below a spinal cord lesion can result in AH. Bladder distention or irritation has been reported in 75% to 85% of cases and is a major consideration in a patient receiving cystoscopy in the setting of a T6 spinal cord lesion. Other known triggers include fecal impaction, bladder distention, urinary tract infection, renal calculus, scrotal compromise, bowel distention, gastritis, hemorrhoids, pregnancy, vaginitis, sexual intercourse, burns, ulcers, surgical stimulus, and orthopedic injury or fracture. Cystoscopy and lithotripsy procedures have been identified as triggers of AH in patients with spinal cord injury above T6. Blood pressure monitoring should be routinely performed in patients with spinal cord injury undergoing urologic or gynecologic procedures.1,2 WH AT I S T H E L O N G -​T E R M O U TC O M E O F A H ?

The cause of dysreflexia can be life-​threatening and investigation must be undertaken to identify and appropriately treat the underlying cause in order to avoid unnecessary morbidity and mortality. AH can become recurrent and even chronic in response to longstanding comorbidities like hemorrhoids or soft tissue ulcerations. Long-​term therapy includes calcium channel blockers or alpha blockade. Complications from AH are a result of sustained, severe peripheral hypertension and must be treated in order to avoid cerebral hemorrhage, myocardial ischemia, pulmonary edema, neurologic sequelae, and mortality.1,2 C A N A H B E P R EV E N T E D I N PAT I E N T S WI T H S P I NA L C O R D I N JU RY ?

Given that bowel and bladder pathology is the most common trigger for AH, preventing fecal impaction, bowel distention, and bladder distention is important in this patient population. Routine Foley catheterization and fecal disimpaction should be considered.7 Patient education and counseling can prove to be very beneficial in allowing patients to understand the etiology and prevention of this disease. DISCUSSION D E FI N IT I O NS O F K EY WO R D S

Dysautonomia:  Pathology of the autonomic nervous system that can result in the lack of modulation of constant temperature, breathing pattern regulation, blood pressure regulation, myocyte contractility, pupillary dilation, and excretory systems. Transected spinal cord injury: Pathology of the spinal cord that results in interruption of neuronal input from the periphery to the central nervous system and vice versa. Autonomic dysreflexia:  Autonomic dysreflexia is a condition in people who have sustained injuries to their spinal cord. It normally occurs when an irritation occurs the region below the level of a patient’s injury. The patient’s damaged spinal cord prevents communication of peripheral pain messages from reaching the central nervous system. This lack of central

45. A nesthetic C onsiderations in Autonomic H yperef l exia  •   403

descending inhibition results in severe peripheral hypertension below the level of spinal cord injury. Hypertensive crisis:  This term characterizes the 2 conditions of hypertensive urgency and hypertensive emergency. Hypertensive urgency occurs when blood pressure readings are 180/​110 or higher but there is no sign of organ damage. Hypertensive emergency means blood pressure is so high that organ damage can occur, resulting in signs of heart failure, cerebral hemorrhage, unstable angina, renal injury, and eclampsia in pregnancy. ET I O L O GY A N D PAT H O G E N E S I S

AH is caused by a noxious stimulus below the level of a spinal cord, typically related to bowel or bladder. The anesthesiologist should consider bladder wall irritation, Foley catheter obstruction, overdistended bowel or bladder, hemorrhoids, anal lesions, decubitis ulcer, burns, restrictive clothing, sexual activity, menstruation, labor, orthopedic injury, fracture, and intra-​abdominal pathology as potential stimuli.2 AH results when the noxious stimulus sends an impulse to the spinal cord but it is blocked at the level of the injury. Since the lesion has impeded this transmission, a reflex is activated that results in overstimulation of the sympathetic nervous system resulting in an exaggerated elevation in systemic vascular resistance below the level of the lesion. Although the heart is able to respond with reflexive bradycardia, the nerves below the lesion are unable to receive central input, resulting in the inability to regulate blood pressure below the spinal cord lesion. C L I N I C A L M A N I F E S TAT I O NS

This condition is characterized by episodes of severe hypertension with systolic blood pressure over 200  mm Hg (yet any elevation over 40 mm Hg systolic from the baseline pressure should raise suspicion), bounding headaches, diaphoresis, erythema of the skin, nasal congestion, diplopia, and even feelings of apprehension or impending doom.6,7 D I AG N O S I S

AH requires immediate identification and action. The most typical findings are hypertension typically over 20% of baseline, elevation of intracranial pressure resulting in intense headaches, facial flushing, diaphoresis above the level of the lesion, nausea, nasal congestion, piloerection, and sustained bradycardia. C O M P L I C AT I O N S

Complications of severe acute hypertension include pulmonary edema, seizures, myocardial infarction, or intracranial hemorrhage. Retinal and renal parenchymal damage can also occur.

bowel or bladder, and frequent straight catheterization or fecal disimpaction should be considered. Any surgical noxious stimulus must be removed from the patient, as it may be the precipitating trigger. It may be necessary to use rapidly acting vasodilators such as nitrates, clonidine, or ganglionic blockade to impede sympathetic outflow. The administration of neuraxial anesthesia or general anesthesia can temporarily abolish autonomic dysreflexia, and these are often considerations in the obstetric patient. Sitting a patient up or simply elevating the head of the patient to 90 degrees while letting the legs dangle off the edge of the bed can assist in blood pressure control. Removal of tight fitting clothing may also be necessary.2 E P I D E M I O L O GY

AH in the United States is reported to occur in 48% to 90% of individuals who have sustained a spinal cord injury at T6 or above. There is a much higher incidence of autonomic dysreflexia in patients who have a complete spinal cord injury (91%) with no motor or sensory input below the level of the lesion as compared to an incomplete injury (27%). AH can result in two-​thirds of obstetrical patients in labor with a spinal cord lesion at or above T6.1 C O N C LUS I O NS

• AH is a life -​threatening condition that affects individuals who have sustained spinal cord injuries, typically with lesions at or above the sixth thoracic vertebrae. • Injury to the spinal cord above T6 results in an interruption of neuronal input below the lesion resulting in an imbalanced reflex sympathetic surge causing life-​ threatening hypertension below the level of the spinal cord lesion (and hypotension and bradycardia above the lesion). • AH is thought to be triggered by afferent stimuli that originate below the level of the spinal cord lesion, most commonly the bowel or bladder. • The treatment of AH involves immediate determination and removal of the triggering stimuli along with antihypertensive administration. • If left untreated, AH can result in complications from severe acute hypertension including pulmonary edema, seizures, myocardial infarction, or intracranial hemorrhage. • Cystoscopy and lithotripsy procedures have been identified as triggers for AH in patients with spinal cord injury above T6. •-​ Blood pressure monitoring should be routinely performed in patients with spinal cord injury undergoing urologic or gynecologic procedures.

T R E AT M E N T

The treatment of AH involves immediate determination and removal of the triggering stimuli along with antihypertensive administration. Many patients are triggered by overdistended

R E VI EW Q U E S T I O N S 1. What percentage of patients with complete cord transection above T5 are seen to exhibit AH?

404  •   M isce l l aneous

A. 10 B. 25 C. 50 D. 85 D. 98 Answer: D Eighty-​five percent of patients with a complete spinal cord transection at T5 or above are seen to exhibit AH.8 2. When considering a diagnosis of AH, which of the following findings are NOT found in the patient? A. Tumors of the small intestine, colon, appendix, and bronchi B. Taking a combination of medications leading to muscle stiffness, high fever, and drowsiness C. Tumor of the adrenal gland D. Life-​threatening condition from an overactive thyroid E. Drug reaction causing the body to release large amounts of serotonin F Hypertension and proteinuria during pregnancy Answer: F Preeclampsia does not necessarily cause reflex bradycardia and usually leads to an increase in heart rate by 10 to 15 bpm. The other answers are typically included in the differential diagnosis for AH: (A) carcinoid syndrome, (B) neuroleptic malignant syndrome, (C) pheochromocytoma, (D) thyroid storm, (E) serotonin syndrome. (http://​www.umm.edu/​health/​medical/​ency/​articles/​ autonomic-​hyperreflexia) 3. What is the first step in treatment of AH? A. Telling the surgeon to remove the stimulus B. Direct-​acting vasodilators to treat a blood pressure over 200 mm Hg systolic C. Transcutaneous pacing to treat symptomatic bradycardia D. Increasing sevoflurane to deepen anesthetic levels E. Anticholinergic administration with atropine to treat symptomatic bradycardia Answer: A The first step in management of AH is removal of the stimulus, whether being caused by surgical stimulation or a bowel/​ bladder stimulus. The other choices are included as potential treatment of AH, once the stimulus is removed. 4. What is the most common cause of AH? A. Guillain-​Barré syndrome B. Direct spinal cord injury C. Multiple sclerosis D. Illegal stimulants, including cocaine and amphetamines E. Subarachnoid hemorrhage Answer: B While all of the answer choices are potential causes of AH, direct trauma to the spinal cord is the most common cause (http://​ w ww.umm.edu/ ​ h ealth/ ​ m edical/ ​ e ncy/​ a rticles/​ autonomic-​hyperreflexia). 5. Although it may be infrequent, what is the lowest level of spinal cord injury that has been reported to lead to AH?

A. T5 B. T8 C. T10 D. T12 E. L2 Answer: C Although rare, AH has been reported in patients with injury as low as T9–​T10. More commonly injury above T5–​T6 leads to AH.9 6. Reflex bradycardia seen in AH is primarily mediated by which of the following structures? A. The globus pallidus B. Chemoreceptors in the aorta and carotid body C. Baroreceptors in the aorta and carotid sinus D. The tractus solitaries E. The nucleus accumbens Answer: C An increase in blood pressure is initially sensed by stretch baroreceptors in the aortic arch and carotid sinus, eventually leading to vagal reflex bradycardia (http://​www.nature.com/​ sc/​journal/​v44/​n6/​full/​3101855a.html). 7. Which of the following are signs/​symptoms leading to a diagnosis of AH? A. Tachycardia B. Diaphoresis below the level of spinal cord injury C. Skin pallor and piloerection above the level of spinal cord injury D. Flushed skin above the level of spinal cord injury E. Dilation of pupils Answer: D Signs/​symptoms of AH include bradycardia (A), diaphoresis above the level of injury (B), skin pallor and piloerection below the level of injury (C), flushed or blotched skin above the level of injury (D), and pupillary constriction with blurred vision (E) (https://​www.rah.sa.gov.au/​hampstead/​downloads/​Auto_ ​Dysreflexia2.pdf ). 8. A 75-​year-​old male with T4 spinal cord injury from a motor vehicle collision and subsequent bowel/​bladder dysfunction and erectile dysfunction presents for cystoscopy. Which of the following antihypertensives should be avoided in the event of acute AH? A. Nitroprusside B. Hydralazine C. Nifedipine D. Phenoxybenzamine E. Prazosin Answer: A Men with spinal cord injury often experience sexual dysfunction and frequently use cGMP-​specific phosphodiesterase type 5 inhibitors, such as sildenafil or tadalafil. The use of nitrates is contraindicated in this situation. Other direct acting vasodilators should be employed in the treatment of AH (http://​emedicine.medscape.com/​article/​ 322809-​overview).

45. A nesthetic C onsiderations in Autonomic H yperef l exia  •   405

9. A  female patient with history of multiple sclerosis and prior episodes of AH presents for an outpatient gynecologic procedure. What blood pressure might the anesthesiologist see if the patient experiences another episode of AH intraoperatively? A. Systolic 115 mm Hg B. Systolic 125 mm Hg C. Diastolic 60 mm Hg D. Diastolic 70 mm Hg E. Diastolic 80 mm Hg Answer: B Patients with spinal cord injury typically have lower baseline blood pressures, ranging from systolic 90 to 110 mm Hg. An increase in blood pressure of 20 mm Hg systolic or 10 mm Hg diastolic is typically required to meet the diagnosis of AH. Oftentimes the patient may exhibit no other signs/​symptoms of AH aside from hypertension (http://​emedicine.medscape. com/​article/​322809-​overview). 10. Why do injuries above T5-​T6 typically lead to AH, as opposed to other spinal cord levels? A. Cardioaccelerator fibers originate from T1–​T4. B. The lumbrosacral plexus originates from T12–​L4. C. The greater splanchnic nerve derives its innervation from  T5–​T9. D. Bowel and bladder innervation primarily comes from  S2–​S3. E. The brachial plexus originates from C5–​T1. Answer: C Lesions below the origination and innervation of the greater splanchnic nerve allow enough descending inhibitory parasympathetic control to modulate splanchnic tone and prevent severe hypertension. Above this level, sympathetic tone will be uninhibited, leading to constriction of the splanchnic vascular bed and systemic hypertension (http://​emedicine.medscape. com/​article/​322809-​overview). 11. Which of the following techniques has NOT been shown to decrease the incidence of AH? A. Spinal anesthesia during labor and delivery B. Intrathecal baclofen pump in traumatic spinal cord injury

C. Intermittent catheterization in patients with bladder dysfunction D. Transverse abdominis plane block in patients presenting for circumcision E. Anal block in patients with bowel dysfunction Answer: D A transverse abdominis plane block would not cover the innervation leading to the operative site for a circumcision. All of the other answer choices have been shown to decrease the incidence of AH in patients at risk (http://​emedicine. medscape.com/​article/​322809-​overview).

QUESTIONS AND ANSWER S This chapter has accompanying questions and answers which are available to subscribers as part of the Oxford eLearning platform. To access the questions, go to http://​ oxfordmedicine.com/​anesthesiologyPBL REFERENCES 1. Autonomic Dysreflexia:  What You Should Know. Washington, DC: Consortium for Spinal Cord Medicine; 1997. 2. Acute Management of Autonomic Dysreflexia: Individuals with Spinal Cord Injury Presenting to Health-​Care Facilities. 2nd ed. Washington, DC: Consortium for Spinal Cord Medicine; 2001. 3. Milligan J, Lee J, McMillan C, Klassen H. Autonomic dysreflexia: recognizing a common serious condition in patients with spinal cord injury. Can Fam Physician. 2012 Aug;58(8):831–​835. 4. Roatta S, Farina D. Sympathetic actions on the skeletal muscle. Exerc Sport Sci Rev. 2010 Jan;38(1):31–​35. 5. Brown R, Burton A, Macefield VG. Input-​output relationships of a somatosympathetic reflex in human spinal injury. Clin Auton Res. 2009;19(4):213–​220. 6. Lindan R, Joiner F, Freechafer A, Hazel C. Incidence and clinical features of autonomic dysreflexia in patients with spinal cord injury. Paraplegia. 1980;18:285–​292. 7. Curt A, Nitsche B, Rodic B, Schurch B, Dietz V. Assessment of autonomic dysreflexia in patients with spinal cord injury. J Neurol Neurosurg Psychiatry. 1997 May;62(5):473–​477. 8. Barash et al. Clinical Anesthesia 6th ed. 9. Vallès M, Benito J, Portell E, Vidal J. Cerebral hemorrhage due to autonomic dysreflexia in a spinal cord injury patient. Spinal Cord. 2005;43(12):738–​740.

406  •   M isce l l aneous

46. NPO GUIDELINES Glenn Mann and Dustin Liebling

S T E M C A S E A N D K EY Q U E S T I O N S A 48 year-​old female with painful uterine fibroids and menorrhagia presents for an elective total abdominal hysterectomy. Her past medical history is significant for hypertension, type 2 diabetes mellitus, obesity, and daily tobacco use. She takes lisinopril, acetaminophen, and metformin daily. The patient drank black coffee at 4 am and has not had anything else to eat or drink since 11 pm yesterday. She took her medication with 3 small sips of water 45 minutes ago.

D O WE N E E D TO D E L AY T H E S U RG E RY B EC AUS E T H E PAT I E N T H A D 3 S I P S O F WAT E R WIT H M E D I C I N E 45 M I NU T E S  P R I O R ?

WH AT A R E T H E C U R R E N T R E C O M M E N DAT I O NS O N P R EO P E R AT I VE FA S T I N G ?

Water is considered a clear liquid. According to the American Society of Anesthesiologists (ASA) Preoperative Fasting Guidelines for Healthy Patients of All Ages, it is recommended that all patients abstain from drinking clear liquids 2 hours prior to elective surgery.1 Clear liquids include water, tea, black coffee, pulp-​free juice, and carbohydrate-​rich drinks. The ASA guidelines do not mention what to do when a patient has recently had a few sips of water with his or her medication. Practitioners must weigh the risks and benefits of delaying surgery for recent oral medication administration with sips of water.

The recommendations make a distinction between clear liquids, breast milk, non-​breast milk, light meal, and solids. Patients can have clear liquids up to 2 hours before a procedure, breast milk up to 4 hours before, nonhuman breast milk (formula, cow) up to 6 hours before, a light meal up to 6 hours before, and solids up to 8 hours before.

S H O U L D WE P R E M E D I C AT E T H E PAT I E N T WI T H A N H2-​R EC E P TO R AG O N I S T, P ROTO N P UM P I N H I B ITO R , G A S T RO I N T E S T I NA L S T I MU L A N T, A N D/​O R O R A L A N TAC I D P R I O R TO I N D U C T I O N ?

WH Y I S P R EO P E R AT I VE FA S T I N G I M P O RTA N T ?

Preoperative fasting is important because it minimizes the incidence of emesis and pulmonary aspiration when a patient is receiving anesthesia. Pulmonary aspiration of gastric contents places patients at significant risk for morbidity and mortality. D O T H I S PAT I E N T ’S C O MO R B I D IT I E S P U T H E R AT R I S K F O R H AV I N G A F U L L S TO M AC H ?

It is important to take into account each patient’s comorbidities when assessing the risks and benefits of keeping patients nil per os (NPO). Certain prescription medications are important for patients to continue during the perioperative period, as they may improve outcomes and reduce morbidity and mortality in the perioperative period. However, other medications may put patients at risk for pulmonary aspiration. Therefore, it is important that all patients consult with their surgeon, an anesthesiologist, and the prescribing physician prior to surgery in order to detail the medications that should be continued and the medications that should be held prior to surgery.

There is insufficient evidence that use of H2-​receptor blockers, proton pump inhibitors, gastrointestinal stimulants, and/​or oral antacids before elective surgery in healthy, nonobstetric patients provides any clear clinical benefit. For this reason, the use of these premedications to reduce the risk of emesis and pulmonary aspiration is not routinely recommended.2 However, pregnant patients or patients with gastroesophageal reflux, esophageal disease, hiatal hernia, or raised intra-​ abdominal pressure may have these premedications if the potential benefit outweighs the risk.3 Patients with altered consciousness or anticipated difficult airways may also be considered for premedication.3 DISCUSSION H I S TO RY  O F N P O

The practice of preoperative fasting was first established in the 1800s when physicians first recognized the risks associated with a recently ingested meal.3 Preoperative fasting was first introduced as a way to minimize the unpleasantness of emesis caused by chloroform; it was later that the complications of emesis became the driving force for keeping patients without

407

oral intake prior to surgery.3 In the mid-​1800s, physicians began encouraging their patients to avoid all solid foods before surgery; however, clear liquids could be ingested up to a few hours prior. While the first textbook on anesthesia published in 1847 did not make mention of preoperative fasting guidelines, physicians soon after began to acknowledge that there was a difference in rate of emesis in patients who ingested clears and liquids versus patients who did not. In 1853, an autopsy completed on a Burmese soldier who suffered a gunshot wound to the leg showed vomitus in the trachea after he had vomited during surgery.4 It was not until 1883 that the British surgeon Joseph Lister published the first preoperative fasting guidelines, advising surgeons that “while it is desirable that there should be no solid matter in the stomach when chloroform is administered, it will be found very salutary to give a cup of tea or beef-​tea about two hours previously.”3 For the next century, the practice of holding solids and/​or liquids prior to surgery was adopted sporadically. It was not until 1970, in Cohen and Dillon’s textbook Anesthesia for Outpatient Surgery, that the recommendation of nothing to eat or drink after midnight before surgery was suggested. Principles of preoperative fasting continued to advance during this time and studies emphasized surrogate markers for aspiration risk, namely, residual gastric volume and gastric pH. In 1974, a study completed on a Rhesus monkey by investigators Roberts and Shirley suggested that even healthy patients should be considered high risk for acid aspiration.5 This led to fear as well as widespread adoption of “NPO after midnight” in order to minimize the risk of pulmonary aspiration.3 With only surrogate markers available on the effects of oral intake on pulmonary aspiration, subsequent large-​scale studies including randomized control trials and meta-​analyses sought evidence to either confirm or refute the dogma that NPO after midnight was appropriate and safe for patients about to undergo surgery the following day. While many of these studies refuted Roberts and Shirley’s claims, adoption of evidence-​ based guidelines were cautious and slow.3 In short, guidelines on preoperative fasting varied until the late 1990s when the ASA set forth practice guidelines for all physicians to follow. A S A P R AC T I C E GU I D E L I N E S F O R P R E O P E R AT I VE FA S T I N G

In 1999, a task force of physicians created by the ASA established guidelines regarding preoperative ingestion of fluids and solids prior to surgery. These guidelines have since been adapted in 20111 and most recently in 2016.6 The ASA noted that the guidelines were meant for patients about to undergo elective surgery, as data on the effects of clear liquids and/​or solids on patients undergoing emergency surgery suggested that these patients had delayed gastric emptying times. The ASA also noted that the guidelines were only applicable to “healthy” patients, and not patients with comorbid diseases who may have altered gastric emptying time and altered gastric volumes following general anesthesia. These comorbidities include diabetes, gastric reflux disease, obesity, pregnancy, women in labor, ileus, bowel obstruction, and others that have slowed gastric emptying. The guidelines, based on a

thorough review of randomized clinical trials, recommended less restrictive regulations on preoperative fasting and made recommendations for patients of all ages. The guidelines made clear recommendations delineating when clear liquids, milk, a light meal, and solid foods could be safely ingested before elective surgery or sedation. The guidelines also concluded that routine pharmacologic prophylaxis was not indicated.1 This call was counter to the belief that all patients should be held NPO after midnight. The guidelines also noted that recommendations may need to be modified for patients in which difficult airway management is expected1 and that the benefits must be weighed against the risks in all patients, including patients undergoing elective surgery. The major risk that the ASA was referring to was pulmonary aspiration of gastric contents. R I S K O F P U L MO NA RY A S P I R AT I O N

Pulmonary aspiration is defined as “bilious secretions or solid matter in the tracheobronchial tree, or the presence of an infiltrate on postoperative chest x-​ray that was not identified on a preoperative chest x-​ray or physical exam.”7 Although many studies have concluded that the risk of pulmonary aspiration of gastric contents during general anesthesia is a rare event in patients undergoing elective surgery,7 concern for this complication was the major driving force for the development of preoperative fasting guidelines. Roberts and Shirley’s study on a Rhesus monkey was one such study that created concern for the risk of pulmonary aspiration after surgery with anesthesia. The study was based on the premise that surrogates could be used to determine whether or not ingestion of fluids and/​or solids increases the risk of regurgitation and pulmonary aspiration. After injecting liquids directly into the right main bronchus, the investigators extrapolated that patients with > 25mL of gastric volume with a pH